Sunteți pe pagina 1din 478

SOLUTION MANUAL

for

SEPARATION PROCESS ENGINEERING.


Includes Mass Transfer Analysis

3rd Edition

(Formerly published as Equilibrium-Staged Separations)

by

Phillip C. Wankat
SPE 3rd Edition Solution Manual Chapter 1

New Problems and new solutions are listed as new immediately after the solution number. These new
problems in chapter 1 are: 1A3, 1A4, 1B2-1B4, 1D1.

A2. Answers are in the text.

A3. New problem for 3rd edition. Answer is d.

B1. Everything except some food products has undergone some separation operations. Even the
water in bottles has been purified (either by reverse osmosis or by distillation).

B2. New problem for 3rd edition. Many homes have a water softener (ion exchange), or a filter, or a
carbon water “filter” (actually adsorption), or a reverse osmosis system.

B3. New problem for 3rd edition. For example: the lungs are a gas permeation system, the intestines
and kidney are liquid permeation or dialysis systems.

B4. New problem for 3rd edition. You probably used some of the following: chromatography,
crystallization, distillation, extraction, filtration and ultrafiltration.

D1. New problem for 3rd edition. Basis 1kmol feed.


.4 kmole E .4 MW 46 18.4 kg
10.8 kg
.6 kmol Water .6 MW 18
total 29.2 kg
Weight fraction ethanol = 18.4/29.2 = 0.630

Flow rate = (1500 kmol/hr)[(29.2kg)/(1 kmol)] = 43,800 kg/hr.

17
Chapter 2

New Problems and new solutions are listed as new immediately after the solution number. These new
problems are: 2A6, 2A9 to 2A16, 2C4, 2C8, 2C9, 2D1.g, 2.D4, 2D10, 2D13, 2D24 to 2D30, 2E1, 2F4,
2G4 to 2G6, 2H1 to 2H3.

2.A1. Feed to flash drum is a liquid at high pressure. At this pressure its enthalpy can be calculated
as a liquid. eg. h TF,Phigh c p LIQ TF Tref . When pressure is dropped the mixture is above
its bubble point and is a two-phase mixture (It “flashes”). In the flash mixture enthalpy is
unchanged but temperature changes. Feed location cannot be found from T F and z on the
graph because equilibrium data is at a lower pressure on the graph used for this calculation.

2.A2. Yes.

2.A4.

1.0

Equilibrium
yw zw = 0.965
(pure water)
Flash
.5 operating
line

2.A4
0
0 .5 xw 1.0

2.A6. New Problem. In a flash drum separating a multicomponent mixture, raising the pressure will:
i. Decrease the drum diameter and decrease the relative volatilities. Answer is i.

2.A8. a. K increases as T increases


b. K decreases as P increases
c. K stays same as mole fraction changes (T, p constant)
-Assumption is no concentration effect in DePriester charts
d. K decreases as molecular weight increases

2.A9. New Problem. The answer is 0.22

2.A10. New Problem. The answer is b.

2.A11. New Problem. The answer is c.

18
2.A12. New Problem. The answer is b.

2.A13. New Problem. The answer is c.

2.A14. New Problem. The answer is a.

2.A15. New Problem. a. The answer is 3.5 to 3.6


b. The answer is 36ºC

2.A16. New Problem. The liquid is superheated when the pressure drops, and the energy comes from the
amount of superheat.

2.B1. Must be sure you don’t violate Gibbs phase rule for intensive variables in equilibrium.
Examples:
F, z, Tdrum , Pdrum F, TF , z, p F, h F , z, p
F, z, y, Pdrum F, TF , z, y F, h F , z, y
F, z, x, p drum F, TF , z, x etc.
F, z, y, p drum F, TF , z, Tdrum , p drum
F, z, x, Tdrum F, TF , y, p
Drum dimensions, z, Fdrum , p drum F, TF , y, Tdrum
Drum dimensions, z, y, p drum F, TF , x, p
etc. F, TF , x, Tdrum
F, TF , y, x

2.B2. This is essentially the same problem (disguised) as problem 2-D1c and e but with an existing
(larger) drum and a higher flow rate.
With y = 0.58, x = 0.20, and V/F = 0.25 which corresponds to 2-D1c.
lb mole
If F 1000 , D .98 and L 2.95 ft from Problem 2-D1e .
hr
Since D α V and for constant V/F, V α F, we have D α F.
With F = 25,000:
Fnew Fold = 5, Dnew = 5 Dold = 4.90, and Lnew = 3 Dnew = 14.7 .
Existing drum is too small.
2 2
2
Fexisting D exist 4
Feed rate drum can handle: F α D. gives
1000 .98 .98
Fexisting 16,660 lbmol/h
Alternatives
a) Do drums in parallel. Add a second drum which can handle remaining 8340 lbmol/h.
b) Bypass with liquid mixing

19
y = .58,

V = .25 (16660) = 4150


16,660
25,000

LTotal
8340 x
Since x is not specified, use bypass. This produces less vapor.
c) Look at Eq. (2-62), which becomes
V MWv
D
3K drum 3600 L v v

Bypass reduces V

c1) Kdrum is already 0.35. Perhaps small improvements can be made with a better demister
→ Talk to the manufacturers.
c2) ρv can be increased by increasing pressure. Thus operate at higher pressure. Note this
will change the equilibrium data and raise temperature. Thus a complete new
calculation needs to be done.
d) Try bypass with vapor mixing.
e) Other alternatives are possible.

V zA zB
2.C2.
F KB 1 KA 1

Fz i
2.C5. a. Start with xi and let V F L
L VK i
Fz i zi
xi or x i
L F L Ki L L
1 Ki
F F
K i zi
Then y i Kixi
L L
1 Ki
F F

K i 1 zi
From yi xi 0 we obtain 0
L L
1 Ki
F F

20
zi V
2.C7. 1 f From data in Example 2-2 obtain:
V F
1 Ki 1
F
V/F 0 .1 .2 .3 .4 .5 .6 .7 .8 .9 1.0
f 0 -.09 -.1 -.09 -.06 -.007 .07 .16 .3 .49 .77

2.C8. New Problem.

p drum
F=L+V

Fz Lx Vy
Tdrum
x y
Solve for L & V

Or use lever arm-rule

21
2.C9. New Problem. Derivation of Eqs. (2-62) and (2-63). Overall and component mass balances are,

F V L1 L2
Fzi L1x i,L1 L2 x i,L2 Vyi

Substituting in equilibrium Eqs. (2-60b) and 2-60c)

Fz i L1K i,L1 L2 x i,L2 L 2 x i,L2 VK iV L2 x i,L2


Solving,
Fz i Fz i
x i,L 2
L1K i,L 2 L2 VK i,V L2 L1K i,L1 L2 F V L1 VK i,V L2

Dividing numerator and denominator by F and collecting terms.

zi
x i,liq 2
L V
1 K i,L1 L2 1 1 K i,V L2 1
F F

K i,V L 2 zi
Since yi K i,V L2 x i,L2 , y i
L V
1 K i,L1 L2 1 1 K i,V L2 1
F F
C C C C
Stoichiometric equations, x i,L 2 1, yi 1 , thus, yi x i,L 2 0
i 1 i 1 i 1 i 1

C K i,V L2 1 zi
which becomes 0 (2-62)
i 1 L V
1 K i,L1 L2 1 1 K i,V L2 1
F F

K i,L1 L 2 z i
Since x i,liq1 K i,L1 L2 x i,liq 2 , we have x i,liq1
L V
1 K i,L1 L 2 1 1 K i,V L2 1
F F
C K i,L1 L 2 1 z i
In addition, x i,liq1 x i,liq 2 0 (2-63)
i 1 L V
1 K i,L1 L2 1 1 K i,V L2 1
F F
2.D1. a. V 0.4 100 40 and L F V 60 kmol/h
Slope op. line L V 3 2, y x z 0.6
See graph. y 0.77 and x 0.48
b. V 0.4 1500 600 and L 900 . Rest same as part a.
c. Plot x 0.2 on equil. Diagram and y x z 0.3. yint ercept zF V 1.2
V F z 1.2 0.25 . From equil y 0.58 .
d. Plot x 0.45 on equilibrium curve.

22
L F V 1 V F .8
Slope 4
V V V F .2
Plot operating line, y x z at z 0.51 . From mass balance F 37.5 kmol/h.
e. Find Liquid Density.
MW L x m MWm x w MWw .2 32.04 .8 18.01 20.82
MWm MWw 32.04 18.01
Then, VL xm xw .2 .8 22.51 ml/mol
m w .7914 1.00

L MW L VL 20.82 22.51 0.925 g/ml


P MW
v
Find Vapor Density. v (Need temperature of the drum)
RT
MW v y m MW m y w MW w .58 32.04 .42 18.01 26.15 g/mol
Find Temperature of the Drum T:
From Table 3-3 find T corresponding to
y .58, x 20, T=81.7 C 354.7K
ml atm 4
v 1 atm 26.15 g/mol 82.0575 354.7 K 8.98 10 g/ml
mol K

Find Permissible velocity:

23
u perm K drum L v v

2 3 4
2-60
K drum exp A B nFlv C nFlv D nFlv E nFlv
V
Since V F 0.25 1000 250 lbmol/h,
F
lb
Wv V MW v 250 26.15 6537.5 lb / h
lbmol

L F V 1000 250 750 lbmol/h, and WL L MW L 750 20.82 15, 615 lb/h,
4
WL V 15615 8.89 10
Flv 0.0744, and n Flv 2.598
WV L 6537.5 .925
4
.925 8.98 10
Then K drum .442, and u perm .442 4
14.19 ft/s
8.98 10
V MW v 250 26.15 454 g/lb
A cs 4 3
2.28 ft 2 .
u perm 3600 v 14.19 3600 8.98 10 g/ml 28316.85 ml/ft
Thus, D 4A cs 1.705 ft. Use 2 ft diameter.
L ranges from 3 D 6 ft to 5 D=10 ft
Note that this design is conservative if a demister is used.
f. Plot T vs x from Table 3-3. When T 77 C, x 0.34, y 0.69. This problem is now
very similar to 3-D1c. Can calculate V/F from mass balance, Fz Lx Vy. This is
V z y 0.4 0.34
Fz F V x Vy or 0.17
F y x 0.69 0.34
g. Part g is a new problem. V = 16.18 mol/h, L = 33.82, y= 0.892, x = 0.756.

2-D2. Work backwards. Starting with x 2, find y2 = 0.62 from equilibrium. From equilibrium point
V
plot op. line of slope L V 2 1 V F 2 3 7. This gives
F 2
z2 0.51 x1 (see Figure). Then from equilibrium, y1 0.78 .
V z1 x1 0.55 0.51
For stage 1, 0.148 .
F y1 x1 0.78 0.51

2.D3. a. z 0.4V F 0.6 V 6.0 k mol h, L 4.0


L
y x z V F
V
Op. eq.
2
y x 2 3
3
See graph: y 0.55 x M 0.18 T ~ 82.8 C linear interpretation on Table 2-7 .

24
b. Product 78.0 C x 0.30, y 0.665,
Mass Bal: Fz Lx Vy F V x Vy
or 4.0 10 V 0.3 0.665V
V 2.985 and V F 0.2985
Can also calculate V/F from slope.

V
c. F 10, 0.3 V 3&L 7
F
L z 7 z
y x x
V V F z 0.3
If y 0.8, x 0.545 @ equil
7
Then z 0.3 0.8 0.545 0.6215.
3
7
Can also draw line of slope through equil point.
3

25
2.D4. New problem in 3rd edition. Highest temperature is dew point V F 0
Set zi yi . Ki yi x i
Want xi yi K i 1.0
K ref TNew K ref TOld yi K i
If pick C4 as reference: First guess K bu tan e 1.0, T 41 C : K C3 3.1, K C6 0.125
yi .2 .35 .45
4.0145 T too low
K i 3.1 1.0 .125
Guess for reference
K C4 4.014 T 118 C : K C3 8.8, K C6 .9
yi .2 .35 .45
0.6099
Ki 8.8 4.0145 .9
K C4,NEW 4.0145 .6099 2.45, T 85 : K C2 6.0, K C6 0.44
yi .2 .35 .45
1.20
Ki 6 2.45 .44
K C4,NEW 2.45 1.2 2.94, T 96 C : K C3 6.9, K C6 0.56
yi .2 .35 .45
0.804 Gives 84 C
Ki 6.9 2.94 .56

26
Use 90.5º → Avg last two T K C4 2.7, K C3 6.5, K C6 0.49
.2 .35 .45
yi K i 1.079
6.5 2.7 .49
T ~ 87 88º C
Note: hexane probably better choice as reference.
2.D5. a)

v1 = F2
v2
y1 = z2
z1 = 0.55 y2
p1,2 = 1 atm
1 2
F1 = 1000 V
0.25
F 2

x2
x1 = 0.30

L F
b) y1 x1 z Plot 1st Op line. y1 = 0.66 = z2
V1 V1
y = x = z = 0.55
to x1 = 0.3 on eq. curve (see graph)
L 0.55 0.80 .25
Slope 0.454545
V .55 0 .55
L1
0.45454 & L1 V1 F1 1000
V1

V 687.5
V1 = 687.5 kmol/h = F2 0.6875
F 1 1000

V L 0.75F
c) Stage 2 0.25 , 3, y x z2 0.66. Plot op line
F V 0.25F
0.66 F z 0.66
At x 0, y z V F 2.64. At y 0, x 2 z 0.88
0.25 L L F 0.75
From graph y 2 0.82, x 2 0.63 .
V
V2 F2 0.25 687.5 171.875 kmol/h
F 2

27
2.D6. V

F = 1.0 kmol/min Kc4 = 2.4


T = 50ºC Kc5 = 0.80
zc4 = 0.45 P = 200 kPa Kc6 = 0.30
zc5 = 0.35
Zc6 = 0.20 L
K i 1 zi
RR eq., 0 , First guess V/F = 0.6
1 Ki 1 V F
1.4 .45 0.2 .35 0.7 .2
f1 0.0215
1 1.4 .6 1 0.2 0.6 1 0.7 0.6
Use Newtonian Convergence
2
df k c
K i 1 zi
2
d V F i 1 1 Ki 1 V F
V V fk
F k 1 Fk df
d V F

28
2 2
df1 1.4 2 0.45 0.2 0.35 0.7 0.20
2 2 2
0.570
V 1 1.4 .6 1 0.2 0.6 1 0.7 0.6
d
F
V 0215
0.6 0.6377
F2 0.570
1.4 .45 0.2 .35 0.7 0.2
f2 0.00028
1 1.4 0.6377 1 0.2 0.6377 1 0.7 0.6377
Which is close enough.
zi 0.45 yi K i x i
x c4 0.2377,
V 1 1.4 .6377 yc4 2.4 0.2377 0.5705
1 Ki 1
F
0.35
x c5 0.4012, y c5 0.8 0.4012 0.3210
1 0.2 0.6377
0.20 0.3613 0.1084
x c6 , y8 0.30 0.4012
1 0.7 0.6377 xi 1.0002 yi 0.9998

V zA zB
2.D7.
F KB 1 KA 1
KM 5.6 and K P 0.21
V 0.3 0.7
0.2276
F 0.21 1 5.6 1
zM 0.3
Eq. (2-38) x M 0.1466
V 1 4.6 0.2276
1 KM 1
F
xP 1 xM 0.8534 , y M K M x M 5.6 0.1466 0.8208
yP 1 yM 0.1792
V K i 1 zi
2.D8. Use Rachford-Rice eqn: f 0 . Note that 2 atm = 203 kPa.
F 1 Ki 1 V / F
Find K i from DePriester Chart: K1 73, K 2 4.1 K 3 .115
Converge on V F .076, V F V F 152 kmol/h, L F V 1848 kmol/h .
zi
From x i we obtain x1 .0077, x 2 .0809, x 3 .9113
V
1 Ki 1
F
From yi K i x i , we obtain y1 .5621, y 2 .3649, y3 .1048
2.D9. Need hF to plot on diagram. Since pressure is high, feed remains a liquid
h F CPL TF Tref , Tref 0 from chart
CPL CPEtOH x EtOH CPw x w

29
Where x EtOH and x w are mole fractions. Convert weight to mole fractions.
Basis: 100 kg mixture
30
30 kg EtOH 0.651 kmol
46.07
70 kg water 70 18.016 3.885
Total = 4.536 kmol
100 0.6512
Avg. MW 22.046 Mole fracs: x E 0.1435, x w 0.8565 .
4.536 4.536
Use CPL at 100 C as an average CP value.
EtOH

kcal
C PL 37.96 .1435 18.0 .8565 20.86
kmol C
C PL 20.86 kcal
Per kg this is 0.946
MWavg 22.046 kg C
h F 0.946 2000 189.2 kcal/kg
which can now be plotted on the enthalpy composition diagram.
Obtain Tdrum 88.2 C, x E 0.146, and y E 0.617 .
For F 1000 find L and V from F = L + V and Fz Lx Vy
which gives V = 326.9, and L = 673.1

Note: If use wt. fracs. CPL 23.99 & CPL MWavg 1.088 and h F 217.6 . All wrong.

30
2.D.10 New Problem. Solution 400 kPa, 70ºC z C4 35 Mole % n-butane x C6 0.7
From DePriester chart K C3 5, K C4 1.9, K C6 0.3
zi
Know y i Kixi , xi , xi yi 1 zi
V
1 Ki 1
F
K i 1 zi
R.R. 0 z C3 1 z C6 z C 4 .65 z C6
V
1 Ki 1
F
zC6 zC6 V
For C6 0.7 z C 6 0.7 1 0.7
V V F
1 K C6 1 1 0.7
F F
V
z C6 0.7 0.49
F
4 .65 z C 6 0.9 .35 0.7z C 6
RR Eq: 0
V V V
1 4 1 0.9 1 0.7
F F F
2 equations & 2 unknowns. Substitute in for z C6 . Do in Spreadsheet.
Use Goal – Seek to find V F.
V
0.594 when R.R. equation 0.000881 .
F
V
z C6 0.7 0.49 0.7 (0.49)(0.594) 0.40894
F
2.D11. L F 0.6 V F 0.4 & L V 1.5
Operating line: Slope 1.5, through y x z 0.4

31
2.D12. For problem 2.D1c, plot x = 0.2 on equilibrium diagram with feed composition of 0.3. The
resulting operating line has a y intercept z V / F 1.2 . Thus V F 0.25 (see figure in
Solution to 2.D1) Vapor mole fraction is y = 0.58.
Find Liquid Density.
MW L x m MWm x w MWw .2 32.04 .8 18.01 20.82
MWm MWw 32.04 18.01
Then, VL xm xw .2 .8 22.51 ml/mol
m w .7914 1.00

L MW L VL 20.82 22.51 0.925 g/ml


p MW
v
Find Vapor Density. v (Need temperature of the drum)
RT
MW v y m MW m y w MW w .58 32.04 .42 18.01 26.15 g/mol
Find Temperature of the Drum T:
From Table 2-7 find T corresponding to y .58, x 20, T=81.7 C 354.7K

32
ml atm 4
v 1 atm 26.15 g/mol 82.0575 354.7 K 8.98 10 g/ml
mol K
Find Permissible velocity:

u perm K drum L v v

2 3 4
K drum,horizontal 1.25 K drum,vertical exp A B nFlv C nFlv D nFlv E nFlv 1.25

Since V V F 0.25 1000 250 lbmol/h,


Wv V MW v 250 26.15 lb lbmol 6537.5 lb / h

L F V 1000 250 750 lbmol/h, and WL L MW L 750 20.82 15, 615 lb/h,
4
WL V 15615 8.98 10
Flv 0.0744, and n Flv 2.598
WV L 6537.5 .925
K drum ,vertical 0.442, and K drum ,horiz 0.5525
4
0.925 8.98 10
u perm 0.5525 4
17.74 ft/s
8.98 10
V MW v 250 26.15 454 g/lbm
A cs 4
u perm 3600 v 17.74 3600 8.98 10 g/ml 28316.85 ml/ft 3
A Cs 1.824 ft 2 , AT A Cs 0.2 9.12 ft 2

With L/D = 4, D 4A T 3.41 ft and L 13.6 ft

2.D13. New Problem. The answer is ycresol = 0.19582


xp 1.76 .7
Since x c 0.3, x p 0.7, y p 0.80418
1 1 xp 1 .76 .7
yc 1 yp 0.19582
cp xc 1
Or y c , cp 0.5682
1 cp 1 xc pc

VPC 4
2.D14. Raoult’s Law: K C 4
PTot
log 10 VPC 4 4.04615 , VPC 4 11121 mm Hg
log 10 VPC 6 3.2658 , VPC 6 1844.36 mm Hg
zi
xi 1.0 1.0
1 Ki 1 V F

33
0.3 0.7
1
11121 1844.36
1 1 0.4 1 1 0.4
P P
Solve for Pdrum = 3260 mmHg
zi
xi
V
1 Ki 1
F
.3 11121
x C4 0.1527, y C 4 K C 4 x C 4 0.1527 0.52091
11121 3260
1 1 .4
3260
1844.36
x C6 1 x C 4 0.84715, y C6 0.84715 0.47928
3260
Check 1.00019

2.D15. This is an unusual way of stating problem. However, if we count specified variables we see
that problem is not over or under specified. Usually V/F would be the variable, but here it
isn’t. We can still write R-R eqn. Will have three variables: zC2, ziC4, znC4. Need two other
eqns: z iC4 z nC4 constant, and z C2 z iC4 z nC4 1.0
Thus, solve three equations and three unknowns simultaneously.

Do It. Rachford-Rice equation is,


K C2 1 zC2 K iC 4 1 z iC 4 K nC 4 1 z nC 2
0
V V V
1 K C2 1 1 K iC 4 1 1 K nC 4 1
F F F
Can solve for zC2 = 1 – ziC4 and ziC4 = (.8) znC4. Thus zC2 = 1 – 1.8 znC4
Substitute for ziC4 and zC2 into R-R eqn.
K C2 1 .8 K iC 4 1 K nC 4 1
1 1.8 z nC4 z nC 4 z nC 4 0
V V V
1 K C2 1 1 K iC 4 1 1 K nC 4 1
F F F
K C2 1
V
1 K C2 1
Thus, z nC 4 F
K C2 1 .8 K iC 4 1 K nC 4 1
1.8
V V V
1 K C2 1 1 K iC 4 1 1 K nC 4 1
F F F
Can now find K values and plug away. KC2 = 2.92, KiC4 = .375, KnC4 = .26.
Solution is znC4 = 0.2957, ziC4 = .8 (.2957) = 0.2366, and zC2 = 0.4677

2.D16. z C1 0.5, z C4 0.1, z C5 0.15, z C6 0.25, K C1 50, K C4 .6, K C5 .17, K C6 0.05


1st guess. Can assume all C1 in vapor, ~ 1/3 C4 in vapor, C5 & C6 in bottom
V / F 1 .5 .1 / 3 .53 This first guess is not critical.

34
V K i 1 zi
R.R. eq. f 0
F 1 Ki 1 V F
49 .5 .4 .1 .83 .15 .95 .25
0.157
1 49 .53 1 .4 .53 1 .83 .53 1 .95 .53
V V f V F 1
Eq. 3.33 2
F 2 F 1 zi K i 1
2
V
1 Ki 1
F
where V/F 1
0.53 and f V / F 1
0.157 .
calculate V/F 2
.53 0.157 2.92 0.584
V .584 150 87.6 kmol/h and L 150 87.6 62.4
z C1 .5
x C1 0.016883
1 K C1 1 (V / F) 1 49 .584
y C1 K C1 x C1 50 0.016883 0.844
Similar for other components.

2-D17. a. V 0.4F 400, L 600 Slope L F 1.5


Intercepts y = x = z = 0.70. Plot line and find xA = 0.65, yA = 0.77 (see graph)
b. V = 2000, L = 3000. Rest identical to part a.
c. Lowest xA is horizontal op line (L = 0). xA = 0.12
Highest yA is vertical op line (V = 0). yA = 0.52. See graph

35
d. V = 600, L = 400, -L/V = -0.667.
Find xA = 0.40 on equilibrium curve. Plot op line & find intersection point with
y = x line. zA = 0.52
zh
1
zi V xh
2.D18. From x i , we obtain
V F Kh 1
1 Ki 1
F
Guess Tdrum , calculate K h , K b and K p , and then determine V F .
K1 1 z i
Check: 0 ?
1 K1 1 V F
Initial guess: If x h .85 then Tdrum must be less than temperature to boil pure hexane
Kh 1.0, T 94 C . On this basis 85° to 90° would be reasonable. Try 85°C.
K h =0.8, K b 4.8, K p =11.7 .
0.6
1
V 0.85 0.6
1.471 . Not possible. Must have K h 0.706
F 0.8 1 0.85
Try T 73 C where K h 0.6 . Then K b 3.8, K p 9.9 .
0.6
1
V .85 0.735
F .6 1
Check:
K i 1 zi 8.9 .1 2.8 .3 .4 .6
0.05276
1 Ki 1 V F 1 8.9 .735 1 2.8 .735 1 .4 735
Converge on T ~ 65.6 C and V F ~ 0.57 .

2.D19. 90% recovery n-hexane means 0.9 Fz C6 L x C6


Substitute in L F V to obtain z C6 .9 1 V F x C6
C 8 balance: z C6 F Lx C6 Vy C6 F V x C6 K C6 Vx C6
or z C6 1 V F x C6 x C6 K C6 V F
Two equations and two unknowns. Remove x C6 and solve
.9 z C6 KV F
z C6 .93C 6
1 V F
V .1
Solve for V F. . Trial and error scheme.
F .9K C6 .1
Pick T, Calc K C6 , Calc V F, and Check f V F 0?
K ref Told
If not K ref new
1 df T

36
Try T 70 C. K C4 3.1, K C5 .93, K C6 .37 K ref
V .1
0.231 .
F .9 .37 .1
Rachford Rice equation
2.1 .4 .08 .25 .63 .35
f .28719
1 2.1 .231 1 .08 .231 1 .63 .231
.37
K ref Tnew 0.28745 use .28
1 0.28719
Converge on TNew ~ 57 C. Then K C4 2.50, K C8 .67, and V F 0.293 .

2.D20. New Problem. The K values are: K E 8.7 , K B 0.54 , K P 0.14


Can use Eq. (2-40), (2-41) or (2-42). If we use (2-42) the R – R eqn

V K i 1 zi
f 0 Use z F 1 zB zE .8 z B
F V
1 Ki 1
F
7.7 .2 .46 z B .86 .8 z B
Then RR eq =
1+7.7 25 1 .46 .25 1 0.86 .25
0 0.5265 0.51977 z B 0.8764 1.0955z B
0 .5757z B 0.3499 zB 0.6078

2.D21. a.) K C2 4.8 K C5 0.153


V zA zB
Soln to Binary R.R. eq.
F KB 1 KA 1
V 0.55 0.45
0.5309
F .153 1 4.8 1
zC2 0.55
x C2 0.1823, yC2 0.8749
V 1 3.8 .5309
1 K C2 1
F
x C5 0.8177 , y C5 0.1251
Need to convert F to kmol.
Avg MW 0.55 30.07 0.45 72.15 49.17
kg kmol
F 100, 000 2033.7 kmol/h
hr 49.17 kg
V V F F 1079.7, L F V 954.0 kmol/h

L v
b.) u Perm K drum
v

To find MW L 0.1823 30.07 0.8177 72.15 64.48

37
MW V 0.8749 30.07 0.1251 72.15 35.33
For liquid assume ideal mixture:
MW C2 MW C5
V1 x C2 VC2,liq x C5 VC5,liq x C2 x C5
C2,liq C5,liq

30.07 72.15
VL 0.1823 0.8177 103.797 ml/mol
0.54 0.63
MW L 64.48
L 0.621 g/ml
VL 103.797
MW v
For vapor: ideal gas: v
RT
atm g
700 kPa 35.33
101.3 kPa mol
v 0.009814 g / ml
ml atm
82.0575 303.16K
mol K
WL v
K drum : Use Eq. (2-60) with FlV
WV L

kmol 64.48 kg
WV 997.7 6, 4331.7 kg/h
h kmol
WV 881.5 35.33 31,143.4 kg/h
64331.7 0.009814
FlV 0.2597
31,143.3 0.621

2
K drum exp 1.877478 0.81458 n .2597 0.18707 n 0.2597
3 4
0.0145229 n 0.2597 0.0010149 n 0.2597 0.3372

0.621 0.009814
u Perm 0.3372 2.6612 ft/s
0.009814
ft 1.0 m
2.6612 0.8111 m/s
s 3.2808 ft

kmol kg
1079.7 35.33
V MWV h kmol
AC 6 3
1.392 m 2
u Perm 3600 v m s g kg 10 cm
0.8111 3600 0.009814
s h cm 3 1000g m3

D 4A C 1.33 m
Arbitrarily
L D 4, L 5.32 m

38
V K i 1 zi K iP 1 z iP K NP 1 z NP
2.D22. f 0
F 1 Ki 1 V F V V
1 K iP 1 1 K NP 1
F F
Solve for V/F.
V V
1 K NP 1 K iP 1 z iP 1 K iP 1 K NP 1 z NP 0
F F
V K iP 1 z iP K NP 1 z NP
where z iP z NP 1.0
F K NP 1 K iP 1
p drum p tot 760 mm Hg, T 90 C
1499.2
log10 VP NP
7.84767
2.75943
90 204.64
VPNP 574.68 mm Hg , K NP 574.68 p tot 0.75616
1580.9
log10 VPiP 8.117783.011679
20 219.61
VPiP 1027.256 mm Hg , K iP 1027.256 760 1.35165
Note: MWiP MWNP . z iP 0.5 in both wt & mol frac., as does z NP .
V 0.35165 0.5 0.24384 0.5
0.629
F 0.24384 0.35165
z iP
x iP 0.4095
V
1 K iP 1
F
x NP 1 x iP 0.5905; yip K iP x iP 0.55347 y NP 0.44653

2.D23. 5. 0°C, 2500 kPa


Fig 2.12: K M 5.7, K ethylene 1.43, K Ethane 0.98, K C6 0.007
V
First, try 0.6 (equal split ethylene and ethane)
F 1

V .47 .4 0.43 .05 .02 0.35 0.993 0.2


f 0.0108
F1 1 4.7 .6 1 .43 .6 1 .02 .6 1 .993 6
V
f
V V F 1
Eq. (2-46) 2
0.6059
F 2 F 1 zi K i 1
2
V
1 Ki 1
F
zi
Then Eq. (2-38), xi . xM 0.104, x ethylene 0.040
1 Ki 1 V F
x ethane 0.354, x C6 0.502, 1.0001 OK
Find yi Ki xi

39
2.D24. New Problem. p = 300 kPa
At any T. K C3 y C3 x C3
K’s are known. K C6 y C6 x C6 1 yC3 1 x C3

Substitute 1st equation into 2nd K C6 1 K C3 x C3 1 x C3


Solve for xC3, 1 x C3 K C6 1 K C3 x C3
x C3 K C3 K C6 1 K C6
1 K C6 K C3 1 K C6
x C3 & y C3
K C3 K C6 K C3 K C6
At 300 kPa pure propane K C3 1.0 boils at -14°C (Fig. 2-11)
At 300 kPa pure n-hexane K C6 1.0 boils at 110°C

1 K C6 1 1 K C6
Check: at -14°C x C3 1, y C3 1.0
1 K C6 1 K C6
0 K C3 0
at 110°C x C3 0, y C3 0
K C3 K C3

Pick intermediate temperatures, find K C3 & K C6 , calculate x C3 & y C3 .

T K C3 K C6 x C3 yC3 K C3 x C3
1- 0.027
= 0.684 0.9915
0ºC 1.45 0.027 1.45 - 0.027
10ºC 2.1 0.044 0.465 0.976 See
20ºC 2.6 0.069 0.368 0.956 Graph
30ºC 3.3 0.105 0.280 0.924
40ºC 3.9 0.15 0.227 0.884
50ºC 4.7 0.21 0.176 0.827
60ºC 5.5 0.29 0.136 0.75
70ºC 6.4 0.38 0.103 0.659

40
41
b. x C3 0.3 , V F 0.4, L V 0.6 0.4 1.5
Operating line intersects y x 0.3, Slope 1.5
L F
y x z
V V
F 0.3
at x 0, y z 0.75
V 0.4
Find yc3 = 0.63 and xC3 = 0.062
Check with operating line: 0.63 1.5 .062 0.75 0.657 OK within accuracy of the graph.
c. Drum T: K C3 y C3 x C3 0.63 0.062 10.2 , DePriester Chart T = 109ºC
L y .8 .6 1 f
d. y .8, x ~ .16 Slope 0.45 .45
V x .16 .6 f
V 1
f 0.69
F 1.45

2.D25. New Problem. 20% Methane and 80% n-butane


V
Tdrum .50 ºC , 0.40 , Find p drum
F
V K A 1 zA K B 1 zB
0 f
F V V
1 KA 1 1 KB 1
F F

Pick p drum 1500 kPa: K C4 13 K nC4 0.4


(Any pressure with K C1 1 and K C4 1.0 is OK)
12 .2 .6 .8
Trial 1 f1 0.2178 Need lower p drum
1 12 .4 1 .6 .4
K C 4 Pold 0.4
K C 4 Pnew 0.511
1 d f Pold 1 .2138

1.0
Pnew 1160 K C1 16.5
15.5 .2 .489 .8
f2 0.4305 .4863 0.055769
1 15.5 .4 1 .489 .4
0.511
K C 4 Pnew 0.541
1 0.055769
Pnew 1100 K C1 17.4
16.4 .2 .459 .8
f3 0.0159 , OK. Drum pressure = 1100 kPa
1 16.4 .4 1 .459 .4

42
zi 0.2
b.) xi , x C1 0.02645
V 1 16.4 .4
1 Ki 1
F
yC1 K C1x C1 17.4 0.02645 0.4603

2.D26. New Problem. a) Can solve for L and V from M.B. 100 = F = V + L
45 Fz 0.8V 0.2162L
Find: L = 59.95 and V = 40.05
y C3 0.8
b) Stage is equil. K C3 3.700
x C3 0.2162
0.2
K C5 .2552
0.7838
These K values are at same T, P. Find these 2 K values on DePriester chart.
Draw straight line between them. Extend to Tdrum , p drum . Find 10ºC, 160 kPa.
1064.8
2.D27. New Problem. a.) VPC5 : log10 VP 6.853 2.2832 ,
0 233.01
VP 191.97 mmHg
b.) VP 3 760 2280 mmHg ,
log10 VP 6.853 1064.8 / T 233.01
Solve for T = 71.65ºC
c.) Ptot 191.97 mm Hg [at boiling for pure component Ptot VP ]
1064.8
d.) C5: log10 VP 6.853 2.8045
30 233.01
VP 637.51 mm Hg
K C5 VPC5 Ptot 637.51 500 1.2750

1171.17
C6: log10 VPC6 6.876 2.2725
30 224.41
VPC6 187.29 mm Hg
K C6 187.29 500 0.3746
e.) KA yA x A KB yB x B (1 y A ) / (1 x A )
If K A & K B are known, two eqns. with 2 unknowns K A & y A Solve.
1 K C6 1 0.3746
x C5 0.6946
K C5 K C6 1.2750 0.3746
y C5 K C5 x C5 1.2750 0.6946 0.8856
f.) Overall, M.B., F = L + V or 1 = L + V
C5 : Fx F Lx Vy .75 0.6946 L + 0.8856 V
Solve for L & V: L = 0.7099 & V = 0.2901 mol
g.) Same as part f, except units are mol/min.

43
2.D28. New Problem. V
h

L
From example 2-4, x H 0.19, Tdrum 378K, V F 0.51, y H 0.6, z H 0.40
V MWv
With h D C, D
u perm 3600 v C

C=4, MWv = 97.39 lbm/lbmole (Example 2-4)


1 28316.85ml lbm
v 3.14 10 3 g mol 3
0.198
454g lbm ft ft 3
Example 2.4

L v
u perm K drum , K horiz 1.25 K vertical
V

From Example 2-4, K vertical 0.4433 , K horiz 1.25 0.4433 0.5541


12
0.6960 0.00314
u perm 0.5541 8.231 ft s [densities from Example 2-4]
0.00314
V lbmol
V F 0.51 3000 1530 lbmol hr
F hr
lbmol lbm
1530 97.39
h lbmol
D 5.067 ft
ft s lbm
8.231 3600 0.1958 3
s h ft
h 4D 20.27 ft
1
Use 5 20 or 5 22 ft drum.
2

2.D29. New Problem. The stream tables in Aspen Plus include a line stating the fraction vapor in a given
stream. Change the feed pressure until the feed stream is all liquid (fraction vapor = 0). For the Peng-
Robinson correlation the appropriate pressure is 74 atm.
The feed mole fractions are: methane = 0.4569, propane = 0.3087, n-butane = 0.1441, i-butane = 0.0661,
and n-pentane = 0.0242.
b. At 74 atm, the Aspen Plus results are; L = 10169.84 kg/h = 201.636 kmol/h, V = 4830.16 kg/h =
228.098 kmol/h, and Tdrum = -40.22 oC.

44
The vapor mole fractions are: methane = 0.8296, propane = 0.1458, n-butane = 0.0143, i-butane = 0.0097,
and n-pentane = 0.0006.
The liquid mole fractions are: methane = 0.0353, propane = 0.4930, n-butane = 0.2910, i-butane =
0.1298, and n-pentane = 0.0509.
c. Aspen Plus gives the liquid density = 0.60786 g/cc, liquid avg MW = 50.4367, vapor density =
0.004578 g/cc = 4.578 kg/m3, and vapor avg MW = 21.17579 g/mol = kg/kmol.

Since the flow area for vapor = LD and L = 4D, the area for flow = 4D 2. Then the equation for the drum
diameter is

D = {[(MWV) V]/[ρV uperm (L/D)]}0.5 = {[(21.17579 kg/kmol)(228.098 kmol/h)]/[(4.578 kg/m3)(uperm


ft/s)(1 m/3.281 ft)(3600 s/h)(4)]0.5

where the unit conversions are used to give D in meters. The value of u perm (in ft/s) can be determined by
combining Eqs. (2-59) and (2-60) for vertical drums with Eq. (2-64a).

Flv = (WL/WV)[ρV/ ρL]0.5 = (10169.84/4830.16)[0.004578/0.60786]0.5 = 0.18272

Resulting Kvertical = 0.378887 , Khorizontal = 0.473608, and uperm = 5.436779 ft/s, and D = 0.4896 m and L =
1.9585 m. Appropriate standard size would be used.

2.D30. New Problem. a. From the equilibrium data if yA = .40 mole fraction water, then x A = 0.09 mole
fraction water.
Can find LA and VA by solving the two mass balances for stage A simultaneously.
LA + VA = FA = 100 and LA (.09) + VA (.40) = (100) (.20). The results are VA = 35.48 and LA = 64.52.

b. In chamber B, since 40 % of the vapor is condensed, (V/F)B = 0.6. The operating line for this flash
chamber is,
y = -(L/V)x + FB/V) zB where zB = yA = 0.4 and L/V + .4FB/.6FB = 2/3. This operating line goes through
the point y = x = zB = 0.4 with a slope of -2/3. This is shown on the graph. Obtain xB = 0.18 & yB = 0.54.
LB = (fraction condensed)(feed to B) = 0.4(35.48) = 14.19 kmol/h and VB = FB – LB = 21.29.

c. From the equilibrium if xB = 0.20, yB = 0.57. Then solving the mass balances in the same way as for
part a with FB = 35.48 and zB = 0.4, LB = 16.30 and VB = 19.18. Because xB = zA, recycling LB does not
change yB = 0.57 or xA = 0.09, but it changes the flow rates VB,new and LA,new. With recycle these can be
found from the overall mass balances: F = VB,new + LA,new and FzA = VB,newyB + LA,new xA. Then VB,new =
22.92 and LA,new = 77.08.

45
Graph for problem 2.D30.

46
2.E1. New Problem. From Aspen Plus run with 1000 kmol/h at 1 bar, L = V = 500 kmol/h, WL = 9212.78
kg/h, WV = 13010.57 kg/h, liquid density = 916.14 kg/m3 , liquid avg MW = 18.43, vapor density = 0.85
kg/m3 , and vapor avg MW = 26.02, Tdrum = 94.1 oC, and Q = 6240.85 kW.

The diameter of the vertical drum in meters (with u perm in ft/s) is

D = {[4(MWV) V]/[3600 π ρV uperm (1 m/3.281 ft)]}0.5 =


{[4(26.02)(500)]/[3600(3.14159)(0.85)(1/3.281)uperm]}0.5

Flv = (WL/WV)[ρV/ ρL]0.5 = (9212.78/13010.57)[0.85/916.14]0.5 = 0.02157

Resulting Kvertical = 0.404299, and uperm = 13.2699 ft/s, and D = 1.16 m. Appropriate standard size would
be used. Mole fractions isopropanol: liquid = 0.00975, vapor = 0.1903

b. Ran with feed at 9 bar and pdrum at 8.9 bar with V/F = 0.5. Obtain WL = 9155.07 kg/h, WV = 13068.27,
density liquid = 836.89, density vapor = 6.37 kg/m3

D = {[4(MWV) V]/[3600 π ρV uperm (1 m/3.281 ft)]}0.5 =


{[4(26.14)(500)]/[3600(3.14159)(6.37)(1/3.281)uperm]}0.5

Flv = (WL/WV)[ρV/ ρL]0.5 = (9155.07/13068.27)[6.37/836.89]0.5 = 0.06112

Resulting Kvertical = .446199, uperm = 5.094885 ft/s, and D = 0.684 m. Thus, the method is feasible.

c. Finding a pressure to match the diameter of the existing drum is trial and error.

If we do a linear interpolation between the two simulations to find a pressure that will give us D = 1.0 m
(if linear), we find p = 3.66. Running this simulation we obtain, WL = 9173.91 kg/h, WV = 13049.43,
density liquid = 874.58, density vapor = 2.83 kg/m3, MWv = 26.10

D = {[4(MWV) V]/[3600 π ρV uperm (1 m/3.281 ft)]}0.5 =


{[4(26.10)(500)]/[3600(3.14159)(2.83)(1/3.281)uperm]}0.5

Flv = (WL/WV)[ρV/ ρL]0.5 = (9173.91/13049.43)[2.83/874.58]0.5 = 0.0400

Resulting Kvertical = .441162, uperm = 7.742851 ft/s, and D = 0.831 m.

Plotting the curve of D versus pdrum and setting D = 1.0, we interpolate pdrum = 2.1 bar

At pdrum = 2.1 bar simulation gives, WL = 9188.82 kg/h, WV = 13034.53, density liquid = 893.99 , density
vapor = 1.69 kg/m3, MWv = 26.07.

D = {[4(MWV) V]/[3600 π ρV uperm (1 m/3.281 ft)]}0.5 =


{[4(26.07)(500)]/[3600(3.14159)(1.69)(1/3.281)uperm]}0.5

Flv = (WL/WV)[ρV/ ρL]0.5 = (9188.82/13034.53)[1.69/893.99]0.5 = 0.0307

Resulting Kvertical = .42933, uperm = 9.865175ft/s, and D = 0.953 m.

This is reasonably close and will work OK. T drum = 115.42 oC, Q = 6630.39 kW,

47
Mole fractions isopropanol: liquid = 0.00861, vapor = 0.1914

In this case there is an advantage operating at a somewhat elevated pressure.

2.E2. This problem was 2.D13 in the 2nd edition of SPE.


a. Will show graphical solution as a binary flash distillation. Can also use R-R equation. To
generate equil. data can use
x C6 x C8 1.0, and yC6 yC8 K C6 x C6 K C8 x C8 1.0
1 K C8
Substitute for xC6 x C6
K C6 K C8
Pick T, find KC6 and KC8 (e.g. from DePriester charts), solve for xC6. Then yC6 = KC6xC6

T°C KC6 KC8 xC6 yC6 = KC6 xC6


125 4 1.0 0 0
120 3.7 .90 .0357 .321
110 3.0 .68 .1379 .141
100 2.37 .52 .2595 .615
90 1.8 .37 .4406 .793
80 1.4 .26 .650 .909
66.5 1.0 .17 1.0 1.0

L 1 V F .6
Op Line Slope 1.5 , Intersection y = x = z = 0.65.
V V F .4
See Figure.
yC6 = 0.85 and xC6 = 0.52. Thus KC6 = .85/.52 = 1.63.
This corresponds to T = 86°C = 359K

b. Follows Example 2-4.

48
MW L x C8 MW C6
x C8 MW C8
.52 86.17 .48 114.22 99.63
MW C6
MW C8 86.17 114.22
VL x C6 x C8 .52 .48 145.98 ml/mol
C6 C8 .659 .703
MW L 99.63 28316 ml/ft 3 lbm
L .682 g/ml 42.57
VL 145.98 454 g/lbm ft 3
MW v yC6 MWC6 yC8 MWC8 .85 86.17 .15 114.22 90.38
pMW v 1.0 90.38 g/mol
v 0.00307 g/ml 0.19135 lbm/ft 3
RT ml atm
82.0575 359K
mol K
Now we can determine flow rates
V
V F .4 10, 000 4000 lbmol/h
F
Wv V MW v 4000 90.38 361, 520 lb/h

L F V 6000 lbmol/h, WL L MW L 6000 99.63 597, 780 lb/h

WL v 597, 780 0.19135


Flv 0.111, nFlv 2.1995
Wv L 361, 520 42.57
2
K drum exp 1.87748 .81458 2.1995 .18707 2.1995
3 4
0.01452 2.1995 0.00101 2.1995 0.423

u Perm K drum L v v 0.423 42.57 19135 .19135 6.30 ft/s

V MW v 4000 90.38
A Cs 83.33 ft 2
u Perm 3600 v 6.3 3600 0.19135
D 4A Cs 4 83.33 10.3 ft. Use 10.5 ft.
L ranges from 3 × 10.5 = 31.5 ft to 5 × 10.5 = 52.5 ft.

Note: This uPerm is at 85% of flood. If we want to operate at lower % flood (say 75%)
have
u Perm75% 0.75 0.85 u Perm85% 0.75 0.85 .63 5.56
Then at 75% of flood, ACs = 94.44 which is D = 10.96 or 11.0 ft.

2.F1
xB 0 .1 .2 .3 .4 .5 .6 .7 .8 .9 1
yB 0 .22 .38 .52 .62 .71 .79 .85 .91 .96 1
Benzene-toluene equilibrium is plotted in Figure 13-8 of Perry’s Chemical Engineers
Handbook, 6th ed.

2.F2. See Graph. Data is from Perry’s Chemical Engineers Handbook, 6th ed., p. 13-12.

49
23
Stage 1) z F1 .4 f 13 Slope 2,
13
.4
Intercept 1.2 y1 .872 x1 .164 z2
13
13
Stage 2) z F2 .164 f 23 Slope 12
23
.164
Intercept .246 x2 .01 y2 .240 z3
23
Stage 3) z F3 .240 f 12 Slope 1
.240
Intercept .480 x3 .022 y3 .461
12

2.F3. Bubble Pt. At P = 250 kPa. Want K1z1 1


Guess T 18 C, K1 1, K2 .043, K3 .00095, .52
Converge to T 0C
z1
Dew Pt. Calc. Want 1.0
K1
Try T 0 C, K1 1.93, K2 0.11, K3 0.0033, 120.26

50
Converge to T 124 C . This is a wide boiling feed.
Tdrum must be lower than 95°C since that is feed temperature.
First Trial: Guess Td,1 70 C : K1 7.8, K 2 1.07, K 3 .083
Guess V F 0.5 . Rachford Rice Eq.
7.8 1 .517 .07 .091 .083 1 .392
fV F .14
1 6.8 .5 1 .07 .5 1 .083 1 .5
V F .6 gives f .6 .101
By linear interpolation V F .56. f 0.56 .0016 which is close enough for first
trial.
V V F F 56, L 44
zi
xi and y i Kixi
1 Ki 1 V F
x1 .1075 x2 .088 x3 .806 x 1.001
y1 .839 y2 .094 y3 .067 y .9999
Data: Pick Tref 25 C . (Perry’s 6 ed; p. 3-127), and (Perry’s 6th ed; p. 3-138)
th

1 81.76 cal/g 44 3597.44 kcal/kmol


2 87.54 cal/g 72 6302.88 kcal/kmol
3 86.80 cal/g 114 9895.2 kcal/kmol
at T 0 C, CpL1 0.576 cal / (g C) 44 25.34 kcal/(kmol C) .
For T 20 to 123 C, CpL3 65.89 kcal/(kmol C)
at T 75 C, CpL2 39.66 kcal/(kmol C) . (Himmelblau/Appendix E-7)
Cpv a bT cT 2
propane a = 16.26 b = 5.398 × 10-2 c = -3.134 × 10-5
-2
n-pentane a = 27.45 b = 8.148 × 10 c = -4.538 × 10-5
-3
**n-octane a = 8.163 b = 140.217 × 10 c = -44.127 × 10-6
** Smith & Van Ness p. 106
Energy Balance: E(Td) = VHv + LhL – FhF = 0
Fh F 100 .577 25.34 .091 39.66 .392 65.89 95.25 297, 773 kcal/h
Lh L 44 .1075 25.34 .088 39.66 .806 65.89 70.25 117, 450
2
VH v 56 .839 3597.4 16.26 5.398 10 45
2
0.94 6302.88 27.45 8.148 10 45
3
0.67 9895.3 8.163 140.217 10 45 240, 423

E Tdrum 60,101 Thus, Tdrum is too high.


Converge on Tdrum 57.2 C : K1 6.4, K 2 .8, K 3 .054
For V F 0.513, f 0.513 0.0027. V 51.3, L 48.7

51
x1 .137, x 2 .101, x 3 .762, x1 1.0000
y1 .878, y 2 .081, y3 .041, y1 1.0000
Fh F 297, 773; Lh L 90, 459; VH v 209,999; E Tdrum 2685
Thus Tdrum must be very close to 57.3°C.
x1 .136, x 2 .101, x 3 .762
y1 .328, y 2 .081, y 3 .041
V 51.3 kmol/h, L 48.7 kmol/h
Note: With different data T drum may vary significantly.

L F
2.F4. New Problem. yV Lx Fz or y x z
V V
L
V F 0.4, V 4kmol h , L 6, 1.5 slope
V
F
x 0, y z 2.5 .25 0.625
V
Find: V = 4 kmol/h, L = 6 kmol/h.

From the graph, x = 0.19 y = 0.34

Equilibrium is from NRTL on Aspen Plus.

52
FIGURE 2.F.4.

2.G1. Used Peng-Robinson for hydrocarbons.

Find Tdrum 33.13 C, L 34.82 and V 65.18 kmol/h


In order ethylene, ethane, propane, propylene, n-butane, xi (yi) are:
0.0122 0.0748 , 0.0866 0.3005 , 0.3318 0.3781 , 0.0306 0.0404 , 0.5388 0.2062.

2.G2. Used Peng-Robinson. Find Tdrum 30.11 C, L 31.348, V 68.66 kmol/h.


In same order as 2.G1,
x i yi are: 0.0189 0.1123 , 0.0906 0.3023 , 0.3255 0.3495 , 0.0402 0.0501 , 0.5248 0.1858 .

53
2.G3. Used NRTL-2. Tdrum 79.97 C , xM 0.2475, y M 0.6287 . Compares to graph with
xM 0.18 and y M 0.55 . Different equilibrium data.

2.G4. New Problem.

COMP x(I) y(I)


METHANE 0.12053E-01 0.84824
BUTANE 0.12978 0.78744E-01
PENTANE 0.29304 0.47918E-01
HEXANE 0.56513 0.25101E-01
V/F = 0.58354

2.G5. New Problem. Used NRTL. T = 368.07, Q = 14889 kW, 1st liquid/total liquid = 0.4221,
Comp Liquid 1, x1 Liquid 2, x2 Vapor, y
Furfural 0.630 0.0226 0.0815
Water 0.346 0.965 0.820
Ethanol 0.0241 0.0125 0.0989

2.G6. New Problem. Used Peng Robinson. Feed pressure = 10.6216 atm, Feed temperature = 81.14oC,
V/F = 0.40001, Qdrum =0. Note there are very small differences in feed temperature with
different versions of Aspen Plus.

COMP x(I) y(I)


METHANE 0.000273 0.04959
BUTANE 0.18015 0.47976
PENTANE 0.51681 0.39979
HEXANE 0.30276 0.07086
V/F = 0.40001

2.H1. New Problem. The spreadsheet with equations for problem 2.D16 is shown in Appendix B of
Chapter 2. The spreadsheet with numbers for i-butane replacing n-butane is below.

MC flash, HW 2.G.b., MC flash with i-


butane
K const. aT1 aT2 aT6 ap1 ap2 ap3
M -292860 0 8.2445 -0.8951 59.8465 0
-
iB -1166846 0 7.72668 0.92213 0 0
-
nPentane -1524891 0 7.33129 0.89143 0 0
-
nHex -1778901 0 6.96783 0.84634 0 0
p
T deg R 509.688 psia 36.258 F 150
zM 0.5 z iB 0.1 z np 0.15 znhex
V/F 0.602698586 0.25

54
guess
KM 51.86751896
KiB 0.926804057
KnPen 0.175621816
KnHex 0.05400053
Use goal seek for cell B24 to = 1.0 change B9
xM 0.015793905
xib 0.104615105
xnPen 0.29812276
xnHex 0.581601672
Sum 1.000133443
RR M 0.803396766
RR nB -0.007657401
RRnP -0.2457659
RRnHex -0.550194874
sum RR -0.000221409

2.H3. New Problem. Use the same spreadsheet as for problem 2H1, but with methane feed mole fraction
= 0.
Answer: V/F = 0.8625,
xib 0.08596648
xnPen 0.203540261
xnHex 0.710481125
KiB 3.886544834
KnPen 1.264637936
KnHex 0.574940847
yib = xib Kib = .33411 and so forth

55
Chapter 3

New Problems and new solutions are listed as new immediately after the solution number. These new
problems are: 3A7, 3A10, 3A11, 3C3, 3C4, 3D4, 3D8, 3G2.

3.A7. Simultaneous solution is likely when one of the key variables can be found only from the
energy balances. For example, if only 1 of x D , x B , D, B, FR A dist are given energy balances
will be required. This is case for most of the simulation problems and for a few design
problems. In some simulation problems the internal equations have to be solved also.

3.B1. a. x D , x B , opt feed, Q Re b


x D , x B , opt feed, Q C
x D , x B , opt feed, S (open steam), sat’d vapor steam
All of above with fractional recoveries set instead of x D , x B
D, x B , opt feed, L/D

b. N, N F , col diameter, frac. recoveries both comp.


N, N F , col diameter, FR A dist, Lo D
N, N F , col diameter, FR A dist, QR
N, N F , col diameter, FR A dist, QC
N, N F , col diameter, x D , QC or x B
N, N F , col diameter, S (sat’d steam), sat’d vapor steam, x D or x B

Many other situations are possible [e.g., 2 feeds, side streams, intermediate condensers or
reboilers etc.]

3.C1. See solution to problem 3-D2.

3.C2. See solution to problem 3-D3.

3.C3. New Problem in 3rd Edition. Fmix F1 F2 D B


Fmix z mix F1z1 F2 z 2 Dx D Bx B (Mole frac. MVC)

F1z1 F2 z 2
z mix
Fmix
Now solve like 1 Feed Column Fmix & z mix . From Eq. (3-3),
z mix x B
D Fmix kmol/h.
xD xB
B Fmix D kmol/h.

3.C4. New Problem in 3rd Edition. See solution to 3D4, Part b.

56
3.D1.
F1 F2 B D xB 0.0001 0.01%
QC
V1 F1z1 F2 z 2 Bx B D xD xD 0.85

1 D, x D z avg xB
F1 Lo
Solve D Ftotal
xD xB
F2 Ftotal F1 F2 1500 kg/h

F1z1 F2 z 2 1000 .60 500 0.10


z avg 0.43333
Ftotal 1500

L V 0.43333 0.0001
QR D 1500 764.62 kg/h
0.85 0.0001
kg
B, x B B Ftotal D 1500 764.62 735.38
h
Mass balance calculation is valid for parts a & b for problem 3G1.
Lo
a) 3, Eq 3-14 QC 1 L0 / D D h D H1
D
h D is a saturated liquid at x D 0.85 wt. frac. From Fig. 2-4, h D ~ 45 kcal/kg
H1 is saturated vapor at x D y1 0.85, H1 ~ 310 kcal/kg
QC 1 3 764.62 45 310 810, 497 kcal/hour
EB around column. F1h F1 F2 h F2 Qcol QC QR Dh D Bh B
h F1 81 C, 60 wt% ethanol ~ 190kcal / kg; h F2 20 C, 10 wt% ethanol ~ 10kcal / kg
h B (sat’d liquid – leaves equil contact, ~ 0 wt% ethanol) ~ 100 kcal/kg, Qcol = 0 (adiabatic)
QR 764.62 45 735.38 100 1000 190 500 10 810, 497 657, 259 kcal/kg
(b) V B 2.5 mass.

57
V (Satd vapor)
Lh L QR VH V Bh B

Reboiler QR L V B L 1838.45 735.38 2573.83


L
Sat’d liqd
V B 2.5 or V 2.5B 1838.45
B, x B =0.0001
Sat’d liqd

Approximately x B ~ yV ~ x L . Thus h L h B 100 . HV 640 kcal kg


QR 1838.45 640 735.38 100 2573.83 100 992, 763 kcal / h

EB. QC Dh D Bh B Qcol F1h F1 F2 h F2 QR


QC 34407.9 7353.8 190, 000 5000 992, 763 1,146, 001 kcal/h

3.D2. Column: mass bal: F + S = D + B (1)


MVC: Fz + SyS Dx D Bx B (2)
Note: yS 0
energy bal: Fh f SHs QC Dh D Bh B (3)
Condenser: mass bal. : V1 Lo D (4)
energy bal.: V1H1 QC Lo D h D (5)
Solve Eqs. (1) and (2) to get:
Fz Fx B Sx B 100 .3 100 .05 100 .05
D 36.4
xD xB .6 .05
Note: Not Eq. (3-3). Solve Eqs. (4) and (5) to get:
QC D 1 L D h D H1
Substitute Q C into Eq. (3):
L Dh D F S D h B Fh F SH S
1
D D h D H1
From Figure 2-4: h F 8, h D 65, h B 92, HS 638, H1 608 kcal/kg.
36.4 65 163.6 92 100 8 100 638
L D 1 2.77
36.4 65 408

3.D3. External balances: F + C = B + D (1)


Fz Cx C Bx B DyD (2)
QR Fh F Ch C Bh B DH D (3)

58
F = 2000, C = 1000, z = .4, x C 1.0, x B .05, yD .80, h F 20 C 30.7,
h C sat 'd liquid 50, h B sat 'd liquid 92, H D sat 'd vapor 327 kcal/kg
Around reboiler: L V B
Lx N Vy Re b Bx B
Lh N QR VH reb Bh B
For a total reboiler: x N x B , yN xN xB , hN hB 92
M.B.: V B hN QR VH reb Bh B
QR
or V since h B hN
H reb hN
HReb
617 (saturated vapor at y N 1 0.05 )
Fz Cx C Fy D Cy D
Solve Eqs. (1) and (2) for B: B
x B yD
800 1000 1600 800
Thus B 800 and D 2200
.05 .8
From Eq. (3), QR Bh B DH D Fh F Ch C

QR 800 92 2200 327 2000 30.75 1000 50 804,500 cal/h


QR 804500
V 1532.4 kg/h
H reb hN 617 92

3.D4. New Problem in 3rd Edition. F B D


MVC Fz Bx B Dy D
But given recoveries. Thus, use:
Fz F,M (Frac Rec Methanol in distillate) Dy D,M
and Fz F,W (Frac Rec water in bottom) Bx B,W
z F,M 0.3, z F,W 1 z F,M 0.7
yD,M unknown, x B,W unknown.
Methanol 29.7 100 0.3 .99 Dy D,M
If 99% methanol recovered in distillate, 1% is in bottoms 0.3 100 0.3 0.01 Bx B,M
Water 68.6 100 0.7 0.98 Bx B,W 2% water in distillate
1.4 100 0.7 0.02 DyD,W
Since xi 1 and yi 1, Dy D,i D, and Bx B,i B
Thus, D DyD,M DyD,W 29.7 1.4 31.1 kmol h
B Bx B,M Bx B,W 0.3 68.6 68.9 kmol h
Check: B D 100 F OK

59
DyD,M
yd,M 29.7 31.1 0.955
D
x B,M Bx B,M B 0.3 68.9 0.00435
a) D 31.1 & L0 D 2. Thus L0 62.2
Reflux liquid is in equilibrium with vapor y D,M 0.955
From equilibrium data (Table 2-7) x M,0 ~ 0.893 (linear interpolation)
b) E.B. Partial condenser: V1H1 Qc DH0 L0 h 0
V1 D L0 93.3
V1y1 Dy0 L0 x 0 y1 Dy0 L0 x 0 V1
y1,M 29.7 62.2 0.893 93.3 0.914; y1,W 1 y1,M 0.086
M 35, 270 J mol@64.5 C 35, 270 kJ / kmol choose MeOH reference 64.5ºC.
40, 656 J mol@100 C 40, 656 kJ/kmol choose water reference 100ºC.
The condenser is at 66.1ºC (linear interpolation Table 2-7).
H1 y
M 1,M y
W 1,W y1,M CP,V,M T1 64.5 y1,W CP,V,W T1 100
V1 is at T1 in equilibrium with y1,M 0.914. From Table 2-7 T1 ~ 67.6 C
Assuming only constant & linear T term are important in CP,V eqs., C P,V CP Tavg . For
67.6 64.5 67.6 100
methanol Tavg 66.65 C . For water, Tavg 83.8 C .
2 2
J 1000 mol kJ kJ
CPV,M 42.93 0.08301 66.05 48.41 48.41
mol kmol 1000J kmol C
J kJ
CP,V,W 33.46 0.00688 83.8 34.04 o
34.04
mol C kmol C
TM 67.6 64.5 3.1 ; TW 67.6 100 32.4
Then
H1 35270 0.914 40656 0.086 48.41 3.1 0.914 34.04 32.4 0.086
H1 35775.5 kJ kmol Note λ terms dominate.

H D is at yD 0.955@66.1 C
HD M yD,M W yD,W yD,M CP,V,M TD 64.5 yD,W CP,V,W TD 100
66.1 64.5
CP,V,M CP,V,M TM,avg . Tavg,M 65.3
2
CP,V,M 42.93 0.08301 65.3 48.35
100 66.1
CP,V,W CP,V,W TW,avg . TW,avg 83.05
2
CP,V,W 33.46 0.00688 83.05 34.03
TW 66.1 100 33.9

60
HD 35270 .955 40656 0.045 48.35 0.955 1.6 34.03 0.045 33.9
HD 33682.85 1829.5 73.88 51.91 35534.3
Reflux liquid at 66.1ºC and x M,0 0.893, x W,0 0.107
Reference MeOH 64.5ºC, water reference 100 ºC
h 0 CPL,M x M,0 TM CPL,W x W,0 TW
kJ
CPL,M CPL,M Tavg 75.86 0.1683 65.3 86.85
kmol C
h0 86.85 0.893 1.6 75.4 0.107 33.9 149 kJ kmol
QC DH D L0 h 0 V1H1 31.1 35534.3 62.2 149 93.3 35775.5 2, 242, 030 kJ h
Overall EB Fh F QC QR DH D Bh B
or QR DH D Bh B QC Fh F
h B is saturated liquid with x B,M 0.00435 and x B,W 0.99565
Interpolating in Table 2.7 TBot 99.2 C
hB CPL,M 0.00435 99.2 64.5 CPL,W 0.99565 99.2 100
99.2 64.5
Tavg,M 81.86 and CPL,M 75.86 0.1683 81.86 89.64
2
hB 13.53 60.06 46.5 kJ kmo l
Feed is saturated liquid at z M 0.3, z W 0.7.
From Table 2-7, TF 78 C
hF CP,LM 0.03 78 64.5 CPL,W 0.7 78 100
Tavg,M 78 64.5 2 71.25 and CP,L,M 75.86 0.1683 71.25 87.85
hF 805.4 kJ kmol Then
QR 31.1 35534.3 68.9 46.5 2, 242, 030 100 805.4
1,105,116 3204 2, 242, 030 80,536 3, 424, 479 kJ h

3.D5. Mass Balances: F = D + S + B, Fz Dx D Sx S Bx B


Solving simultaneously, B = 76.4 kg/min, D = 13.6 kg/min.
Condenser: QC V1 h 0 H1
V1 L0 D L D 1 D 4 13.6 54.4 kg/min
From Figure 2-4, h 0 7.7 kcal/kg (x = .9, T = 20°C),
H1 290 kcal/kg (y = .9, sat’d vapor).
Thus, Q C = 54.4 (7.7 – 290) = -15,357 kcal/min
Overall Energy Balance: Fh F QR QC Dh D ShS Bh B
QR Dh D ShS Bh B Fh F QC
From Figure 2-4,
hS 61 x S .7, sat'd Liq'd ; h F 200 z .2, 93 C ,

61
hB 99 x B .01, sat'd Liq'd , h D ho 7.7
Thus,
QR 13.6 7.7 10 61 76.4 99 100 200 15357 3635.3 kcal/min

z xB .4 .002
3.D6. From Eq. (3-3), D = F 2500 998 lbmol/h.
xD xB .999 .002
Then B = F = 1502.
Condenser: V Lo D Lo D D D
QC h D H V D Lo D 1
With 99.9% nC5 have essentially pure nC5 . Thus, it is at its boiling point.
h D HV C5 11,369 Btu/lbmol.
QC 11,369 998 4 45,385, 048 Btu/h
Overall: QR Dh D Bh B Fh F QC
Distillate is at boiling point of pure nC5 K C5 1.0 on DePriester Chart) = 35°C. Bottoms
is at boiling point of nC6 K C6 1.0 67°C.
Converting to °F: 35°C = 95°F, 67°C = 152.6°F, 30°C = 86°F.
Note feed is obviously a subcooled liquid. Arbitrarily, pick a liquid at 0°F as reference.
(This will not affect the result and other values can be used.)
CPF x C5CPLC5 z C6 CPLC6
CPF .4 39.7 .6 51.7 46.9 Btu/lbmol °F
hF CPF TF 0 46.9 86 4033.4 Btu/lbmol
Distillate is almost pure nC5 . Liquid at 95°F
hD CPLC5 TDist 0 39.7 95 3771.5 Btu/lbmol
Bottoms is almost pure liquid nC6 at 152.6°F.
hC pLC6 Tbot 0 51.7 152.6 7889.4 Btu/lbmol
QR 998 3771.5 1502 7889.4 2500 4033.4 45,385, 048 50,861, 491 Btu/h
z xB 0.7 0.001
3.D7. Eq. (3-3), D F 1000 700.4 kmol/h
xD xB 0.999 0.001
B F D 299.6 kmol/h
Condenser: Lo Lo D D 2.8 700.4 1961.1 kmol/h
Only this reflux is condensed since product is a vapor.
QC Lo where λ is for essentially pure n-pentane.
kmol Btu 2.20462 lbmol
QC 1966.1 11,369
h lbmol 1 kmol
Btu 1J J
QC 49,154, 204.85 -4
5.18176 1010
h 9.486 10 Btu h

62
From overall balance QR DHD Bh B FhF QC
Distillate is vapor at b.p. of pure n-pentane (35°C from DePriester chart, K C5 1.0 )
Bottoms is boiling n-hexane (67°C)
Conversions: 35°C = 95°F - distillate & Feed and 67°C = 152.6°F - bottoms

As reference, arbitrarily choose liquid at 0°F.


Feed is subcooled liquid.
CPF z C5CPLC5 z C6CPLC6 0.7 39.7 0.3 51.7 43.3Btu lbmolo F
hF CPF TF 0 43.3 95 0 4113.5Btu lbmol
Distillate H D C5 CPLC5 Tdist 0
H D 11,369 39.7 95 0 15,140.5 Btu lbmol
Bottoms is pure C6 @152.6 F
hB CPLC6 Tbot 0 51.7 152.6 0 7889.4 Btu lbmol
kmol Btu kmol Btu
QR 700.4 15,140.5 299.6 7889.4
h lbmol h lbmol
kmol Btu 2.20462 lbmol Btu
1000 4113.5 49,154, 204.85
h lbmol kmol h
Btu 1J
QR 68, 675,167.9 7.240 1010 J h
h 9.486 10-4 Btu
3.D8.
New Problem in 3rd Edition.
F 300, zE .3, zw .7
98% rec. E in distillate, 81% rec water in bot.
D Dist. .98 90 1 .81 300 .7 128.1 kmol/h
.98 90
y DE 0.6885
128.1
B Bottoms .02 90 .81 210 171.9 kmol h

yD , D, H D
b. Partial Condenser.

Qc
Vapor
H1 y1 V1
L0
2, L0 2D 2 128.1 256.2 kmol h. x 0 , L0 , h 0
D
x 0 in equilibrium with y 0 , thus from equation data x 0 0.575.
Entering vapor y1 (from graph) 0.61

63
V1 L0 D 256.2 128.1 384.3 kmol h.
c. E.B. on PC. V1H1 Qc DHdist L0 h 0 . Can use Figure 2-4 by converting mole fracs to mass
fracs. Basis 1 kmole.
Distillate .6885 mol E MW 46 31.671 kgE
5.607 kgW
.3115 mole W MW 18
37.28 kg total
Mass frac. E = 31.671 37.28 0.8496
Vapor V1 0.61 mole E 46 28.06 kgE
7.02 kgW
.39 mole W 18
35.08 kg total
Mass frac E = 28.06 35.08 0.7999
Liquid reflux L 0 0.575 mole E 46 26.45 kgE
7.65 kgE
0.425 mole W 18
34.1 total
Mass frac E = 26.45 34.1 0.7757
From Figure 2-4, Hdist ~ 310 kcal kg, H1 ~ 330 kcal kg, h 0 ~ 65 kcal kg
kmol 37.28 kg kcal
Qc DH dist L 0 h 0 V1H1 128.1 310 256.2 34.1 65
hr kg kg
35.08 kg
364.3 330 2, 400,517 kcal hr
kmol
Overall EB. Fh F QR Qc DHdist Bh B
Know Qc 2, 400,517 kcal h
and DHdist 1, 480, 426 kcal h.
To find Fh F and Bh B , need to convert mole frac to wt frac.
Basis 1 kmol
Feed 30 mole % E: .3 mole 46 13.8
12.6
70% W : .7 18
total 26.4 kg kmol
Mass frac E 13.8 26.4 0.5227
Bottoms 0.01047 mole 46 .48162
17.811
0.98953 mole 18
total 18.293 kg kmol
Mass frac E 0.48162 18.28 0.0263
From Figure 2-4 h F satd liqd 70 kcal kg
h B satd liqd 97

64
Then QR DHdist Bh B Fh F Qc
kmol 18.29316 kg 97 kcal 26.4 kg kcal
QR 1, 480, 426 171.9 300 70
h kmol kg kmol kg
2, 400,517 1, 480, 426 305,525 554, 408 2, 400,517 3,632,069kcal h

3D9. New Problem 3rd Edition. B = (xD – z)/(xD – xB)F = [(0.9999 - 0.76)/(0.9999 – 0.00002)](500) = 120

QR Lh VH Bh B and L V B
Assume h hB. QR L B h Vh V H h V
V V B B 1.5 120 180 kmol h.
Bottoms is almost pure water. w 9.72 kcal mol 9720 kcal kmol
QR 180 kmol/h 9720 1.750 106 kcal h

3.D10. 2 atm × 101.3 kPa/atm = 202.6 kPa. Pentane Recovery: 0.995 Fz P Dx D


0.995 1000 0.55
D 547.6333 kmol/h
0.9993
B = 1000 – 547.6333 = 452.3667
Since Bx B Pentane Recovery Bot F z p ,
1 .995 1000 0.55
xB 0.006079 mol frac pentane
452.3667
Distillate is essentially pure Pentane. Bottoms Pure in Hexane. From DePriester Chart
K P 1@ p 202.6 kPa when Tdist 59.5 C
Kn H 1@ p 202.6 kPa when Tbot 94 C
Lo
For Total Condenser, Eq. (3-14) QC 1 D h D H1
D
h D pure pentane CPLC5 Tdist Tref choose Tref 25 C
kcal kcal
hD 39.7 59.5 25 1369.65
kmol C kmol
H1 hD assuming λ is independent of temperature
Btu 1 lbmol 0.252 kcal kcal
H1 1369.65 11369 7680.196
lbmol 0.454 kmol Btu kmol
kmol kcal
Eq. (3-14) is QC 1 2.8 547.6333 6310.5 13,132, 288
h h
kcal kcal
h B pure hexane CPLC6 Tbot Tref 51.7 94 25 3567.3
kmol C kmol
Feed is a liquid at 65°C
hF CPC5 zC5 CPC6 zC6 TF Tref
hF 39.7 0.55 51.7 0.45 65.25 1804kcal / kmol

65
QR Dh D Bh B Fh F QC
QR 547.6333 1369.65 452.3667 3567.3 1000 1804 13,132, 288
QR 13, 692, 081 kcal/h.
Note that QC and Q R are relatively close.

3.E1. Was 3.D8 in 2nd Edition.


QC

D, x D 0.990
M.B. F + S = D + B

F Fz M SyS,M Dx D Bx B
p = 1.0 atm
100 kmol h
Since steam is pure, yS,M 0
z M 0.6

S Know F, z M , x D , x B
B
xB 0.02 Unknowns S, D, B, Need E.B.

Fh F QC SHS Dh D Bh B

Pick as basis liquid at 0°C, h W 0 & hM 0 (essentially steam table choice)


Assume ideal mixtures.
hF CPavg TP Tref where CPavg 0.4 CPW ,L 0.6 C PM ,L
Felder & Rouseau p. 637 CPW 0.0754 kJ/mol
CPM 0.07586 16.83 10 5 T kJ/mol
CPM CPM Tavg CPM 0 40 2 CPM 20 C
5
CPM 0.07586 16.83 10 20 0.079226
CPavg 0.4 0.0754 0.6 0.079226 0.077696
hF 0.077696 40 0 3.1078 kJ/mol feed = 3107.8 kJ/kmol
Can also use steam table for water
kJ 18.0 kg
H S is sat’d vapor steam 1 atm, HS 2676.0
kg kmol
Steam Table F&R, p. 645 H S =48,168 kJ/kmol
h D is sat’d liquid at x D 0.99 . From Table 2-7, T = 64.6°C
hD CPavg 64.6 0 where CPavg 0.01 CPW 0.99 CPM
5
CPM CPM Tavg CPM 32.3 C 0.07586 16.83 10 32.3 0.081296
CPavg 0.01 0.0754 0.99 0.081296 0.08124

66
HD 0.08124 64.6 5.2479 kJ mol 5247.9 kJ kmol
h B : Since leaving an equilibrium stage, sat’d liqd. 2% MeOH
Table 2-7, T = 96.4°C
hB CPavg 96.4 0 where CP avg 0.98 CPW 0.02 CPM
CPM CPM 96.4 0 2 C PM 48.2 C
5
CPM 0.07586 16.83 10 48.2 0.08397
CPavg 0.98 0.0754 0.02 0.08397 0.07557 kJ mol
hB 0.07557 96.4 7.28509 kJ mol 7285.09 kJ mol

QC do EB around condenser
Lo
QC dist V1 dist Lo D dist 1 D
D
dist 0.99 MeOH 0.01 W

Felder & Rousseau: M 35.27 kJ mol &


40.656 kJ mol W

kJ
dist 0.99 35.27 0.01 40.656 35.324 35,323,86 kJ kmol
mol
QC 35,323.86 2.3 1 D 116,568.7D kJ h

Plug Q C & numbers into E.B.


100 3107.8 116,568.7D 48,168S 5247.9D 7285.09B
or 310,780 + 48,168S = 121,816.6D + 7285.09B

Solve simultaneously with 2 MB. 100 + S = D + B


60 + 0 = 0.99D + 0.02B
One can use algebra or various computer packages.

Obtain: D = 56.33 kmol/h, B = 211.71 kmol/h


S 168.04 kmol/h, QC 6,566, 000 kJ/h.

kmol mol
E2. Was 3.D9 in 2nd Edition. F 500 500, 000
h h
F+S=D+B
Fz SyS Dx D Bx B
2 eq. 3 unknowns
Condenser: QC 1 L0 / D D h o H1
Note Eq (3-14) not valid.

For enthalpy pick reference pure liquid water 0°C and pure liquid methanol 0°C. Felder &
64.5 0
Rouseau: CPMeOH 75.86 0.01683T at Tavg 3225, CPMeOH 76.4 J mol C.
2

67
Assuming distillate pure methanol, boils at 64.5°C
hD CPMeoh ,liq T Tref 76.4 J/mol 64.5 0 4928.0 J mol
H1 hD 1 at 64.5 F 4928 35270 J/mol 40,198 J/mol
QC 4 4928 40198 D 141, 080D J/h where D is mol/h
Overall Energy balance: F h F SHS QC DhD BhB
Bottoms is essentially pure H 2 O at 100°C
J J
hB CPW ,liq T Tref 75.4 100 0 7540
mol C mol
HS hB W at 100 C 7540 40656 J/mol 48196
For feed. 60 mole % Methanol boils at 71.2°C (Table 2-7).
hF CPi zi T Tref 0.6 76.459 71.2 0.4 75.4 71.2 5413.7 J/mol
Now, Eqs are
(1) F + S = D + B or 500,000 + S = D + B
(2) Fz Dx D Bx B or (500,000) (.6) = 0.998D + 0.0013B
Lo
(3) QC 1 D h D H1 or QC 141, 080D
D
(4) Fh F Sh S QC Dh D Bh B or (500,000) (54137) + S (48196)
+ Q C = D(4928) + B (7540)
Solve simultaneously: D = 298.98, B = 1245.5, S = 1044.2 kmol/h
Q C = - 4.218 × 10+7 kJ/h

3.F1. An enthalpy composition diagram is available on p. 272 of Perry’s Chemical Engineer’s


Handbook, 3rd ed., 1950.
z xB 0.79 0.004
Eq. (3-3) D F 25, 000 19, 788.5 kmol/h
xD xB 0.997 0.004
Note that N 2 mole fractions were used since N 2 is more volatile. B = F – D = 5211.5
From enthalpy comp. diag. h D 0, H1 1350 kcal/kmol, h B 160, h F 1575 . Then,
QC 1 Lo D D h D H1 5 19788.5 0 1350 133,572, 000 kcal/h
QR Dh D Bh B Fh F QC
QR 0 5211.5 160 25, 000 1575 QC 95, 030, 000 kcal/h

3.F2. We will use the enthalpy composition diagram on p. 3-171 of Perry’s 6th edition or p. 3-158 of
Perry’s 5th ed.Do for 1 kmol of feed:
Conversion of feed from kg to moles. Basis 100 kg
30 kg NH 3 = 1.765 kmol
70 kg H 2O 3.888
Total 5.653 kmol

Thus 1 kmol is 100/5.653 = 17.69 kg

68
Will work problem in weight fractions since data is presented that way.
95% recovery: (0.95) Fz = Dx D or, D = (.95) Fz / x D = (.95) (17.69) (.3)/(.98) = 5.15 kg.
B = F – D = 12.54 kg
xB Fz Dx D B 17.69 .3 5.15 98 12.54 0.021
From diagram: h D 55, H1 415, h B 150, h F 5 kcal/kg
Eq. (3-14), QC 1 Lo D D h D H1 3 5.15 55 415 5562 kcal/kmol feed
and QR Dh D Bh B Fh F QC
QR 5.15 55 12.54 150 17.69 5 QC 7815 kcal/kmol of feed

G1. a.) Using NRTL. QC 778,863 kcal/h, QR 709,520 kcal/h


b.) QC 1, 064,820 kcal/h, QR 995, 478 kcal/h

G2. New Problem in 3rd Edition.


ASPENPlus. D = 988, L/D = 3, Peng Robinson, N 40, NF 20 (arbitrary values in Radfrac)
7
xD 1.000 x DC6 1.211 10
xB 0.0013316
QC 4.4426 107 Btu h,
QR 4.9852 107 Btu h

69
SPE 3rd Ed. Solution Manual Chapter 4

New Problems and new solutions are listed as new immediately after the solution number. These new
problems are: 4A6, 4A13, 4C10, 4C16, 4D6, 4D9, 4D13, 4D15, 4D18, 4E4, 4E5, 4H1 to 4H3.

4A1. Point A: streams leaving stage 2 (L2, V2)


Point B: vapor stream leaving stage 5 (V5)
liquid stream leaving stage 4 (L4)
Temp. of stage 2: know K y 2 / x 2 , can get T from temperature-composition graph or
DePriester chart of K = f(T,p).
Temp. in reboiler: same as above (reboiler is an equilibrium stage.)

4A2. a. Feed tray = .6, z = 0.51 (draw y = x line), yF =0.52, xF = 0.29.


b. Two-phase feed.
c. Higher

4A6. New Problem in 3rd Edition. Answer is a.

4A7. See Table 11-3 and 11-4 for a partial list.

4A13. New Problem in 3rd Edition.


A. Answer is b
B. Answer is a
C. Answer is a
D. Answer is a
E. Answer is b
F. Answer is a
G. Answer is b

4A14. If feed stage is non-optimum, the feed conditions can be changed to have an optimum
feed location.

4B2. a. Use columns in parallel. Lower F to each column allows for higher L/D and may be sufficient
for product specifications.
b. Add a reboiler instead of steam injection. Slightly less stages required and adds 1 stage.
c. Make the condenser a partial instead of a total condenser. Adds a stage.
d. Stop removing side stream. Fewer stages are now required for the same separation.
e. Remove the intermediate reboiler or condenser and use it at bottom (or top) of column. Fewer
stages, but all energy at highest T (reboilers) or lowest T (condenser) for same separation.
Many other ideas will be useful in certain cases.
4C7. Easiest proof is for a saturated liquid feed. Show point z, y D satisfies operating equation.
Solution: Op. Eq. y L V x L V 1 xB
Substitute in y yD , x z
yD V Lz L V xB
But q 1.0, V D, L F, L V B
y D D Fz Bx B
Which is external mass balance. QED.

70
Can do similar for enriching column for a saturated vapor feed.

4.C10. New Problem in 3rd Edition. If we consider λ, the latent heat per mole to be a positive quantity,
then QR V . With CMO and a saturated liquid feed V V (1 L / D) D , and then
QR / D (1 L / D) .

4.C16. New Problem in 3rd Edition. Define a fictitious total feed FT , z T , h T


F1z1 F2 z 2 F1h F1 F2 h F2
FT F1 F2 , z T , hT
FT FT
Intersection of top & bottom operating lines must occur at feed line for fictitious feed F T.
(Draw a column with a single mixed feed to prove this.)
This feed line goes through y x z T

with slope qT qT 1

H mix hT
where qT and H mix , h mix are saturated
FT H mix h mix

vapor and liquid enthalpies at feed stage of column with

mixed feed.

Given p, L/D, saturated liquid reflux, x D , x B


A
B z0 opt feed locations, z1 , z 2 , F1 , F2 , h F1 , h F2

z2
y
zT

z1
Plot top op line. Plot all 3 feed lines. Draw
xB
x line from point A to y = x = to obtain

q1F1 q 2 Fbot.
2
op. line. Connect pts B & C to get
b.) Does qT
FT
middle op. line.

71
F1h F1 F2 h F2
H mix
H mix hT FT F1 F2 H mix F1H F1 F2 h F2
check q T
H mix h mix H mix h mix H mix h mix FT
where H mix & h mix are vapor and liquid enthalpies on feed stage of mixed column

F1 H mix h F1 H mix h F2
F2
H mix h mix H mix h mix
qT
FT
Usual CMO assumption is λ >> latent heat effects in either vapor or liquid.
H mix h F1 H mix h F2
Then q1 and q2
H mix h mix H mix h mix
F1q1 F2 q 2
Thus q T if CMO is valid.
FT
L L L L D 1.25
4D1. a. Top op line: y x 1 x D and 0.5555
V V V 1 L D 2.25
Intersects y x xD 0.9
L
When x 0, y 1 xD 0.4 Plot – See diagram
V
L L L V B V B 1 3
b. Bottom op line: y x 1 x B , and
V V V V V B 2
Intersects y = x = xB = 0.05
1 0.5 / 2
@y 1 x 0.683 this is convenient point to plot
32
c. See diagram for stages. Optimum feed stage is #2 above partial reboiler.
5 equilibrium stages + PR is more than sufficient.

72
d. Feed line goes from y = x = z = 0.55 to intersection of two operating lines.
q
Slope 1.0 or q 0.5 .
q 1
This is a 2 phase feed which is ½ liquid & ½ vapor.

4D2. New Problem in 3rd Edition. Part a.

L F V 1 V/F .63
y x zE .6 Slope 1.703
V V V F .37
b. From Table 2-1, at 84.1° C y .5089
H hF
c. liquid at 20°C q and 40 mole % ethanol.
H h
The pressure in Figure 2-4 is very close to 1.0 atm, thus it can be used, but must convert to wt
frac.

73
Basis 1kmol feed.
.4 kmole E .4 MW 46 18.4 kg 0.63 wt frac.
10.8 kg
.6 kmol Water .6 MW 18
total 29.2 kg
From Figure 2-4 H 398 kcal kg , h 75, h F 20 C 10
398 10
q 1.20
398 75
q 1.2
Slope 6
q 1 .2
Alternate Solution: 40 mole % ethanol boils at 84.1°C (Table 2-1).
Then if pick reference as saturated liquid at 40 mole %
h F Cp,40%liq 20 84.1
h 0, H 40%E

kcal
d. 40 mole %E 63 wt%, H 398 kcal kg , h 65, h F 398 C p vapor 120 84.1
kg
CPvapor y E CPEvapor y w CPw ,vapor
Assume only 1st and 2nd terms in C P equations are significant.
From Problem 2.D9
CPvapor .4 14.66 0.03758T .6 7.88 .0032T
kcal/kmol T is C

which simplifies to CPvapor 10.592 0.16952T


120

For linear C p dT is equal to CPvapor @ Tavg


84.1

kcal
Tavg 84.1 120 2 102.05 . Then C Pv ,avg 10.592 0.16952 102.05 12.32
kmol
kcal kcal 1 kmol
hF 398 12.32 120 84.1
kg kmol 27.2 kg
hF 398 15.149 413.15 kcal kg
398 413.15 15.147
q 0.045.
398 65 333
F 13
e. q L L f, L L F L F
12 12
13 12
q 13 12 , slope q q 1 13
1 12

V L 1 V F .3 3
f. Flash .7,
F V V F .7 7
See graph for feed lines.

74
Graph for 4.D2

75
4.D3*. a. Basis 1 mole feed.
0.4 moles EtOH × 46 = 18.4 kg EtOH
0.6 moles H2O × 18 = 10.8 kg H2O
Total = 29.2
wt frac 18.4 / 29.2 0.63 wt frac EtOH
Calculate all enthalpies at 0.63 wt frac. Hv = 395, HL = 65 (from Figure 2-4). hF is liquid at
200°C. Assume Cp,liq is not a function of T. Estimate,
h h L 60 C h 20 46.1 23 kcal
C P ,liq .63 wt frac ~ 0.864
T 60 20 80 kg C
Then h F h L 200 CPL 200 60 h L 60 C .864 200 60 46.1 167.1
Hv hF 395 167.1 q 0.691
q 0.691, 2.24
Hv hL 395 65 q 1 0.309
b. From Figure 2-4 at 50 wt% ethanol Hv = 446 and hL = 70. Since CMO is valid obtaining both
enthalpies at 50% wt is OK. The feed is a liquid
h F C P,liq TF Tref CP,liq 250 0
CP,liq CP,EtOH z EtOH CPw z w in Mole fractions
Basis 100 kg solution
50 kg EtOH 46.07=1.085 kg/kgmole
50 kg W 18.016 2.775 kg moles
Total 3.860 kg moles
Avg M.W. 100 3.86 25.91 kg/kgmole
Thus, zW = 0.719 and zE = 0.281
CP,liq 37.96 .281 18.0 .719 23.61
CP 23.61
C P,liq in kcal kg C 0.911 . Then,
MWAVG 25.91
kcal
hF 0.911 250 C 228
kg C
Hv h F 446 228
q 0.58
Hv hL 446 70 4.D4a y=x
4.D4*. a. hF H CPv 350 50 H 25 300
25 300 z
H hF
q 1.5
H h feed
line
Slope q q 1 0.6. y x z 0.6 is intersection.

.5 .6 .7
b. q L L F where L L 0.6F. Then q L 0.6F L / F 0.6, and
slope q q 1 1.5
c. q L L F where L L F 5. q L F5 L F 1 5 , slope q q 1 16

76
h liq h reflux 3100 1500
4.D5*. fL 0.1111
H vap h liq 17500 3100
L0 L0 D 1.1
0.524
V1 L0 D 1 2.1
L1 1 fc L 0 V1 1.1111 .524
0.55
V2 1 f c L 0 V1 1 .111 .524
Alternate Solution
L1 H h0 17500 1500
For subcooled reflux, q 1.111
L0 H h1 17500 3100
Then, L1 qL0 1.1111 L0
L1 L1 L1 D L1 1.111 L0
, 1.111 1.1 1.2222
V2 L1 D L1 D 1 D D
L1 1.222
0.55
V2 2.222
4D6. New Problem in 3rd Edition. a) 175 F1 F2 B D
85 75 .6 100 0.4 0.1 B 0.9D
Solve simultaneously.
D 84.375 and B 90.625 kmol hr
b) Feed 1. q1 1, vertical at y x z1 0.6
Feed 2. 60% vapor = 40% liquid q 2 0.4
q2 0.4
Slope feed line 2 3 through y x z2 0.4
q2 1 .06
Bottom Op. Line y L V x L V 1 x B . Through y x xB
V B 1
Slope L V 32
V B
Middle L F2 B V
L F2 z 2 Bx B
Lx F2 z 2 Bx B Vy y x
V V
F2 z 2 Bx B
When x 0, y , Slope L V
V
Also intersects bot. op. line and Feed line 2.
Do External Balances and Find D & B. Then V V/B B 2B 181.25
L V B 271.875
At feed 2, L .4F L or L L 0.4F 271.875 40 231.875
V V 0.6F 181.25 60 241.25
L V 0.961
40 9.625
x 0, y 0.126 Plot Middle Op Line.
241.25

77
L L
y x 1 xD
V V
Know that y x x D and gives through interaction Middle and Feed line 1.
Also, L L F1 231.875 75 156.875 and V V 241.25 ; thus,
L V 156.875 241.25 0.65
c) See graph.

Graph for 4D6.

78
L
4.D7*. a. Plot top op. line: slope .8 , x y xD .9. Step off stages as shown on Figure.
V
L V
b. Plot bottom op. line: slope 1 12 , x y x B 0.13. Step off stages
V B
(reboiler is an equil stage). Find y2 = 0.515.
c. Total # stages = 8 + reboiler
Optimum feed plate = 7 or 8 from top. Plot feed line. Goes through x = y = z = .3, and
intersection of two operating lines.
9 q
slope gives q = 0.692.
4 q 1

4.D8*. The equilibrium data is plotted and shown in the figure. From the Solution to 4.D7c,
q 0.692 and q q 1 9 4
a. total reflux. Need 5 2/3 stages (from large graph) – 5.9 from small diagram shown.
.9 .462
b. L V min 0.660 (see figure)
.9 .236
L V min
L D min 1.941
1 L V min
L V .8
c. In 4.D7, L D act 4
1 L V .2
L Dact Multiplier L D min
Multiplier = 4/1.941 = 2.06

79
d. Operating lines are same as in Problem 4.D7. Start at bottom of column. Reboiler is an
equilibrium contact. Then use E MV AB AC 0.75 (illustrated for the first real stage)
Stage 1 is the optimum feed stage. 11 real stages plus a partial reboiler are sufficient.

4D9. New Problem in 3rd Edition. a)


F1 F2 D B 100 F1 80 B F1 B 20
F1z1 F2 z 2 Dx B Bx B F1 .42 18 .66 80 0.04 B
Solve simultaneously, B 113.68, F1 93.68
1 L L D 12 1
b) L D ,
2 V 1 L D 32 3
L
L D 40, V L D 120
D
Saturated Liquid Feed V V 120
L L F1 40 93.68 133.68, L V 1.114
c) Top Op. Line – Normal: y L V x 1 L V xD
2
Through y x x D , Slope 1 3, y intercept .66 .44
3
Bottom – Normal: y L V x L V 1 x B , through y x xB
L F2 .7
Also through intersection, F2 feed line and middle op. line. Feed line F2 slope
VF2 .3

80
D F1z1
Middle y L V x xD (or do around bottom)
V V
Slope L V . Through intersection feed line F1 and top op. line.
Dx D F1z1 80 .66 93.68 .42
Also, x 0, y 0.11212
V 120
d)Opt. Feed F2 stage 1 from bottom, Opt feed F1 , Stage 2. 4 stages + PR more than sufficient.

Graph for 4D9.

81
L L L D 4
4.D10*. Operating Line y L V x 1 x D , where .8
V V 1 L D 5
Thus, operating line is y = .8x + .192
y y1
a. Equilibrium is x or x1
1 y 1.79 .76y1
Start with y1 = .96 = x D
y1 .96
Equilibrium: x1 0.9317
1.76 .76y1 1.76 .76 .96
Operating: y2 .8x .192 .8 .9317 .192 0.93736
y2 .93736
Equilibrium: x2 0.89476
1.76 .76y 2 1.76 .76 .93736
Operating: y3 .8x 2 .192 .8 .89476 .192 0.9078
1.76x
b. Generate equilibrium data from: y
1 .76x
x 1.0 .9 .8 .7 .6 .5 .4
y 1.0 .9406 .8756 .8042 .7253 .6377 .5399

Plot equilibrium curve and operating line. (See Figure). Slope = L/V = .8, y intercept (x = 0)
= 0.192, y = x = x D = 0.96. Find x 6 = 0.660.

4.D11. a) Same as 4.D2 part g. q = 1.0668, slope feed line = 15.97.


b) Top y L V x 1 L V x D goes through y = x = x D = 0.99
L D
L V 0.6969 @ x = 0 y = (1-L/V) x D = (1-0.6969) 0.99 = 0.300
1 L D
Feed line: Slope q q 1 , y x z 0.6

82
Bottom Op line:

L V yV Lx Sy M,S Bx B
But y M,S 0 (Pure steam)
With CMO B L
B S
L L
y x xB
V V

y= 0, x = x B . Also goes through intersection of feed line and top op.line.


Stages: Accuracy at top is not real high. (Expand diagram for more occupancy).
As drawn opt. Feed = #6. Total = 9 is sufficient,

0.99 0.57
c. L V min
Slope 0.4242
0.99 0
L L V 0.4242
0.73684
D min 1 L V min
1 0.4242
Actual L/D is 3.12 × this value.

83
L D
4.D12. L V 34 slope. Top op line goes throug y x xD 0.998
1 L D
L L
y x 1 xD @ x 0, y .25 .998 0.2495
V V
B 1245167 From Soln to 3.D9 or
Bottom slope L V 1.19
S 1044168 from graph. 1.169
Feed line is vertical at z = 0.6. Can also plot top and feed lines, and then find bottom from 2
points y 0, x x B & intersect top & feed .
For accuracy – Use expanded portions near distillate & near bottoms.
From Table 2-7 from (x = .95, y = .979)
Draw straight line to (x = 1.0, y = 1.0)
From (x = 0, y = 0) draw straight line to (x = 0.02, y = 0.134)
or (x = 0.01, y = 0.067)

Opt feed = # 9 from top. Need 13 equilibrium stages.

84
85
4.D13. New Problem in 3rd Edition. a.) See Figure

L 0.665 0.95
b.) See figure. 0.4385
V MIN 0.30 0.95
L L L V 0.4385
0.7808
D MIN V L 1 L V 0.5615
L L L D 1.5616
c.) 2.0 L D MIN 1.5616 , 0.6096
D V 1 L D 2.5616
L L L
y intersect 1 yD 0.3709 . Top operating line y x 1 yD
V V V
Goes through y x yD 0.95
L L
Bottom y x 1 xB
V V
Goes through y x x B & intersection top operating line & feed line.
Feed Line: Vertical (saturated liquid, q = 1). Through y x z 0.3
Plot & Step off stages. Optimal feed = 5 below PC. 6 + PC + PR more than sufficient.
0.85 0.025
d.) Slope bottom: See figure for parts c & d. L V 1.941
0.45 0.025
1 1
V B V L V 1.0625 .
L V 1 0.941

86
Graph for problem 4.D13.

87
4.D14a.

88
New S.S.

External M.B. S = B

Sys = BxB . Since yS = 0 (pure water)

xB = 0

S B

4.D14b. Two approaches to answer. Common sense is all methanol leaks out and x MA 0.
McCabe-Thiele diagram: This is enriching column with z ys 0 . Intersection top op. line and
horizontal feed line is at x x M,b 0 , which is also a pinch point. Thus x M,d 0 also.
q
4.D15. New Problem in 3rd Edition. Saturated liquid. q 1, , feed line vertical @ z .3 .
q 1
L L L L D 2
Top operating line y x 1 y D , Slope
V V V 1 L D 3
y intercept 1 L V yD 1 3 0.6885 0.2295 and y x yD
Bottom operating line y L V x L V 1 x B goes through y x xB
And goes through interaction feed line and top operating line. See graph.
Optimum feed is stage 2 below partial condenser. Partial condenser + Partial reboiler + 3
equilibrium stages are more than enough to obtain separation.

89
Graph for problem 4.D15.

90
L-L Hv hF
4.D16*. q
F Hv hL
Using 32°F = 0°C as reference T, h F 4033.4 Btu/lbmole.
Hv hL . Approx. at feed conditions.
.4 11369 .6 13572 12691 Btu/lbmole
For approx. temperature of feed stage, do bubble pt. calc.
y1 1 K1x1 K1z1
Pick T = 48°C (~ 40% of way between boiling pts.)
K C5 1.5, K C6 .54, K1x1 1.5 .4 .54 .6 .92
.54
K C5 Tnew = =.594, Tnew 50 C
.92
K C5 50 C 1.6, K1 x 1 1.6 .4 .584 .6 .99 Close enough.
Hv h L 50 C feed
, 50 C 122 F
Hv CPfeed ,Liq 122 46.9 122 5721.8
Note : CP feed,liq 46.9 is from Prob. 3-D6.
Hv 5721.8 12691 18412.8 Btu/lbmole
HV hF 18412.8 4033.4
q 1.133
HV hL 12691
Note: h F is from Prob. 3.D6.
L L
4.D17. Top Op. Line: y x 1 x D , goes through y x xD 0.9
V V
L L D 7 2
7 9
V 1 L D 9 2
L 2
x 0, y 1 xD
.9 =0.2
V 9
Plot Top. Step off 2 stages. Find x S ~ 0.81
The vertical line at x xS 0.81 is the withdrawal line.
Bot. Op. Line intersects Top at x xS .
Also know it intersects feed line at x x B (unknown)
External Balances F D B S Don’t know D, B, or x B .
Fz Dx D Bx B Sx S
Feed enters as saturated vapor. Thus q 0&V F
Bottoms leaves an equilibrium contact, it is saturated liquid L B
Do flow balances
V F 100
V V 100 since S is removed as saturated liquid.

91
L
L V 7 9 100 77.777. D V L 100 77.777 22.222
V
L L S 77.7777 15 62.7777. L V 62.7777 100 0.6278
B L 62.777
Fz Dx D Sx S 60 22.222 0.9 15 0.81
xB 0.444
B 62.7777
Plot. Op. line
Step off stages. 9 is more than sufficient.

4.D18. New Problem in 3rd Edition. Feed F1 : z1 0.6, saturated liquid, q 1, q / (q 1)

92
Feed F2 : z 2 0.4, 80% vapor hence 20% liquid q LF / F 0.2F / F .2
q .2
14
q 1 .8
Part a.) Bottom operating line goes through point, y x xB 0.04
Max L V to point intersection feed F2 line and equilibrium curve.
L 1.0 .04
Slope 2.2326
V max
.47 .04
V V 1 1
0.8113
B min
L V L V 1 1.2326

Part b. L F1 100
L L L F2 100 .2 80 116
L V B and V B 1.5
116
116 L 1.5B B B 46.4
2.5
116
V L B 116 46.4 69.6 L V 1.66667
69.6
D V V VF2 69.6 .8 80 133.6
L 100
0.7485
V 133.6
Check overall balance F1 F2 D B 180 133.6 46.4 180.0 OK
To find y D use MVC mass balance
F1z1 F2 z 2 Dy D Bx B
F1z1 F2 z 2 Bx B 100 .6 80 .4 46.4 0.4
or yD 0.675
D 133.6
L L
Actual bottom op. line: y x 1 xB
V V
L V B V B 1 2.5 5
V V V B 1.5 3
Goes through y x x B 0.04 , Slope 5 3
2nd point y = 1, x = 0.616 (this was arbitrarily found by setting y = 1.)
Plot bottom op. line
L Dy D F1 z1
Top Op. line: yV F1z1 Lx Dy D . y x
V V
Goes through intersection feed line for F 2 and bot. op. line. Does NOT go through
y x yD .
Since D & F, passing streams, Point z1 , y D is on op. line.

93
Figure for 4D18

4.D19*. B = 0. Then from external balance F = D + B must have D = F = 1000. Acetone balance
becomes Fz Dx D or x D z 0.75 .
L L
To predict x B need operating lines. Top: y x 1 xD
V V
L L V 2
and y x xD .75
V 1 L D 3

94
Bottom: L V 1.0 . Thus y = x is operating line. From Figure x B 0.01 to 0.02
Feed line can be calculated but is not needed.

4.D20*. To use enthalpy composition diagram change to wt. fractions. Basis = 1 kg mole

Distillate: 0.8 ETOH = (.8)(46.07) = 36,856


0.2 Water = (.2)(18.016) = 3.6032
Total = 40.459
Weight Fractions: EtOH = .911, Water = .089

Feed: 0.32 (EtOH) = (.32)(46.07) = 14.7424


0.68 (W) = (.68)(18.016) = 12.25088
Total = 26.993
Weight Fractions: EtOH = .546, W = .454

Bottoms: 0.04 EtOH = (.04)(46.07) = 1.8428


0.96W = (.96)(18.016) = 17.295
Total = 19.1378
Weight Fractions: EtOH = .0963, W = .9037

Condenser Energy Balance is V1H1 Qc Lo h o Dh D which can be solved for L o D .


Lo Qc
1
D D ho H1
From chart: h D 54 Kcal/kg and H1 285 Kcal/kg
Need D in weight units. Convert feed to weight units.
100 kgmoles
Ethanol: .32 46.07 1474.24 kg/hr
hr
Water: (100)(.68)(18.016) = 1225 kg/hr
Total: F = 2699.328 kg/hr

95
F z xB 2699.328 .0963
Then, D 1489.93 kg/hr
xD xB .911 .0963
2, 065,113
Then, Lo D Q D ho H1 1 1 5.0
1489.98 54 285
Now do usual McCabe-Thiele analysis using molar units. Note L o D is the same in mass
and molar units.
L L L L D
Top Operating Line: y x 1 x D and
V V V 1 L D
L 5 L 5
; x y xD .8, y int ercept x 0 1 xD 1 .8 .133
V 6 V 6
Feed Line: Goes through y = x = z = .32
Weight fraction of feed = .546. Then, h f 15 kcal/kg, H v 430.7, and h1k1 69 .
HV hF 430.7 15 q 1.149
q 1.149 Slope 7.711
HV hL 430.7 69 q 1 1.149 1
L L
Bottom Operating Line: y x 1 x B . Goes through x y x B and
V V
intersection top operating line and feed line.

From Figure need about 8 equilibrium contacts including a reboiler. Stage 1 above reboiler is the
optimum feed stage location.

96
4.D21. Feed 1: q1 0 , slope feed line = 0
Feed 2: q 2 0.9 , slope q2 q2 1 0.9 0.1 9
L L D 1.375 L L
Top: 0.579 , y x 1 xD .
V 1 L D 2.375 V V
L
Goes through y x x D , When x = 0, y 1 xD 0.40
V
Bottom: Since F1 is saturated vapor, V F1 100 kmoles/hr

L
.1 F2 V V 0.1F2 100 8 108
At feed F2

.9 F2 V
L L 0.9 F2

L L V V 0.579 108 62.532


D V L 108 45.46
B F1 F2 D 100 80 45.46 134.532

But B is saturated liquid. L B 134.532


Check L L .9F2 62.532 0.9 80 134.532, OK
Draw top op line. Intersects with F2 feed line. Then draw bottom op line with slope
L V 1.3453 .
Intersection bottom op & q. line gives x B 0.09 .
F1z1 F2 z 2 Dx D 20 36 43.187
Check x B or x B 0.095 , OK
B 134.532
Check External MB
180 F1 F2 D B 45.46 134.53 179.99 , OK
F1z1 F2 z 2 Dx D Bx B
56 20.0 36.0 45.46 0.95 134.53 0.095 55.97 OK
See McCabe-Thiele diagram: Optimum feed = 5, 7 equilibrium stages (6.65) more than
sufficient. fraction ab/ac 0.65

97
4.D22*. Around top of column mass balances are: L D V C and Lx Dy D Vy Cx W
L D C
Solving, y x yD
xw
V V V
For pure entering water, x W 1.0 . With saturated liquid entering, L = C. Then from overall
balance, V = D. Thus L/V = C/D = ¾ and D/V = 1.0. Operating equation becomes
y 0.75x .92 .75 0.75x .17
Slope 0.75, y x 0 0.17, y x 1 0.92 yD , y x 0.68 Not y D

98
4.D23*. Note L/V ≠ C/D since C is subcooled. Let c = amount condensed. The energy required to
heat stream W to the boiling point must come from this condensation. That is,
H h c h hW C
h hW Cp T 18 212 100
c C C 0.1154C
H h 17465.4
L C c 1.1154C
V D c D 0.1154C
In addition, C/D = ¾ or D/C = 4/3.
L 1.1154C 1.1154 1.1154
0.77
V D .1154C 4 3 .1154 4 3 .1154
This compares to L/V = 0.75 if entering water is a saturated liquid. Very little effect since λ
is very large.
L L D 3.25
0.7647
V 1 L D 4.25
Goes through y x yD 0.85
When x 0, y 1 L V yD 0.20
L L
Bottom Op Line: y x 1 xB
V V
Through y x x D 0.05 and intersects top op line @ feed line
Opt. Feed is #4 below partial condenser – see diagram. Need 6 equil stages + P.C. (an equil.
contact)

Note – Commercial columns usually operate much closer to minimum reflux ratio and have
many more stages.

0.85 0.376 L L V Min


b.) L V min
0.558 , 1.26
0.85 0 D min 1 L V Min

c.) Total reflux 5 stages + PC sufficient or


4 3 4 equil contacts + PC = 5 3 4 eq. contacts
ab 7.6mm
where fraction 0.74 or .75
ac 10.3mm

99
100
4.D25. a. 99.9% methanol is essentially pure. Pure MeOH boils 64.5°C.
L 1 fc Lo D
Eq. (4-66) 1 where f c CPL TBP Treflux
V2 1 1 f c L o D
For pure MeOH, CPL 0.07586 16.83 10 5 T , average (40 + 64.5)/2 = 52.25°C
CPL 0.084654 kJ gmole , MeOH 35.27 kJ mole , TBP 64.5
L1 1.058804 1.2
fc 0.084654 24.5 35.27 0.058804 , 0.5596
V2 1 1.058804 1.2

101
L L D 1.2
b. 0.5454 or 2.59% more reflux with 24.5°C cooling! Usually
V 1 L D 2.2
subcooling not important.
4.D26. a) 50% feed: q L L F, L L amt vaporized L F 20
F 1 q 1 20 0.05
q L L F , Slope = 0.0476
20 20 q 1 1 20 1 1.05
35% feed: Saturated liquid – vertical feed line. Plot both feed lines. The one with lowest
intersection point with equilibrium curve will normally control V B
min

V 1
Find L V . Then V B
max min
L V L V 1
max

1 0.1 V 1
L V slope 2.497 , 0.6681
max
0.46 0.1 B min
2.497 1.0
b) L V 3 V B 2.0043
min

V B V B 1 3.0043
L V 1.49892 slope
V V B 2.0043
L L
Bot. y x 1 x B . Goes through y x x B with total reboiler
V V
L
y 1, x 1 L V x B or x 1 L V 1 xB / L V
V
1
x y 1 1 0.49892 0.1 0.6338
1.49892
Intersects feed line with 50% feed first.
Middle operating line: Do mass balance around bottom of column. Mass balance intersects
streams L & V (in column), F50% and B.
yV L x F50% z50% Bx B
L F50 z 50 Bx B
y x
V V
Intersects bottom operating line & 50% feed line.
@ x 0, y (F50 z 50 Bx B ) / V , Slope L V
For 50% feed, q 50 0.05 (L L ) / F50 .
External balances: 250 F1 F2 D B and F1z50 F2 z35 Dy D Bx B
Find D = 103.333 and B = 146.666 moles/min
Since V B 2.0034, V 293.96 and L V B 440.63
Then from q 50% : L q 50 F50 L 0.05 100 440.63 445.63
V L F50 B 445.63 100 146.666 398.964 , Slope L V 1.11698
y intercept (x = 0), y
[ 100 .5 146.666 0.1 ] / 398.964 0.0885
Top Intersects feed line for 35% feed and middle op. line and goes through
y x y D 0.85

102
EQ
Stages actual op
actual change liquid
0.75 = E ML =
change at equilibrium
eq. change

Figure for 4D26

103
L L
4.D27*. Top Op. Eqn: y x 1 xD
V V
L L D 1 L
, y intercept 1 xD .46, y x xD .92
V 1 L D 2 V
1
L F F L
L L 4 q 54
Feed: q 5 4 , slope 5 , y x z .48
F F q 1
14
L Bx B
Middle: V B L S , V y Bx B L x Sy s y s 0 or y x
V V
Does not intersect at y x x B or at y 0, x x B . Does intersect top op. line at feed line.
Need another point.
L L L V B V B 1 1.5
Bottom: y x 1 xB , 3, y x x B 0.08
V V V B V B .5
The steam is another feed to the column: Sat’d vapor q = 0, q/(q-1) = 0, y = x = z = ys = 0.
Middle Op. Line intersects this steam at bottom op. line (see figure).

This problem is a two-feed column with the lower feed (steam) input at a non-optimum feed
stage. Otimum feed is 3rd above partial reboiler. Need 5.6 equilibrium stages plus PR.

4.D28. This problem was 4.D35 in 2nd edition.


L L
Stripping Section: y x 1 xB, y x xB 0.02 . Feed line is vertical
V V

104
1.0 0.02 V V 1 1
L V 1.24, 4.167
max
0.81 0.02 B min
L V L V 1 0.24
max

V V V B V B 1 6.25 1
1.5 6.25, L V 1.16
B B min
V V B 6.25
L
1 1 xB
V 1 .16 .02
Op line y 1, x= 0.865 . Intersection Op & feed lines is y D
L V 1.16
Overall Balance: 10,000 = F = D + B
6000 Fz Dy D Bx B D .695 B .02 6000 .695D .02D 200
D 5800 / 0.675 8592.6 kgmoles/day, B 1407.4 . Need a use for impure distillate.

Figure for 4.D28

105
L L 1
4.D29. L D 3, 3 4, 1 xD 0.9 0.225
V V 4
Step off 3 stages on top op line. Find x S 0.76 . Point on top op line at x S 0.76 is also
on middle op. line

xD

3 xs, S = 15

V´ L´
F=D+B+S
F = 100 6
z = .6 Fz Dx D Bx B Sx S

Solve simultaneously D = 50.125, B = 34.875

10

x6 = .1

In top: L = 3D = 150.375 and V = L + D = 200.500

Since saturated liquid withdrawn, V´ = V = 200.500


and L´ = L – S = 150.375 – 15 = 135.375

Middle op. line slope L V 135.375 200.5 0.6752

Middle Op line: yV L x Sx S Dx D
L Sx S Dx D 11.4 45.1125
y x , when x 0, y 0.2819
V V 200.5

Feed z = 0.6, 20% vapor = 80% liquid q = 0.8.


Feed line slope q q 1 0.8 0.4 4

106
Lo Lo D 3
4.D30*. a. Subcooled Reflux: , L1 Lo c and V2 V1 c
V1 1 Lo D 4
4
L o V1
1 L1 4 Lo 3 3
c Lo 500 ,
3 V2 1 1 Lo
V1 Lo 1
3 3 V1
L1 1.0 1 4
Substituting in values, we have , Top op. line y x xD 0.92
L2 1 14 54 5
Step off two stages. x2 0.62 xS yS

107
Mixed feed to column: F S FM 1500 , Fz SyS FM z M
Solving for mixed feed, z M 0.52667
Energy balance for mixed feed, Fh F SHS FM H FM
Since H s h F (sat’d vapor), h FM HS h F (sat’d vapor), and q FM 0 (horizontal).
External Balances: F = D + B, Fz Dx D Bx B , Solving simultaneously D = 500 & B = 500
4
Lo 3D 1500, and L Lo 2000 (subcooled reflux), V L D 2500
3
L 1500 3
Middle: L L S 2000 500 1500 , V V 2500 , Slope .6
V 2500 5
Intersects Top Op. line at x S
Plot Bot. from y x x B to intersection feed line and middle
Step off stages (see figure). Need 4 8/9 equilibrium stages.

b. Mass balances for mixed feed injection: V FM V


V V FM 2500 1500 1000 , V B 1000 500 2

4.D31*. Was problem 4.D36 in 2nd edition. Solution is trial & error. Need to pick L/V. Final answer
shown in figure.
L .63 .385 0.245
.389
V .63 0.63
L V L 0.389
.636
1 L V D 0.611
Note feed stage is not optimum.

108
Figure for problem 4D31

4.D32*. External Balance: F = B = 50, and Fz Bx B . Thus x B z 0.4 .


.75 .452
4.D33*. a. L V min
0.397 (tangent pinch)
.75
L L V
0.659
D min 1 L V min

L D
b. L D act
1.318. L V 0.569
1 L D
Top operating Line through y x yD 0.75
Bottom through y 0, x yB 0.1 and intersection feed and top operating lines.
Feed: L L F .25F, q 5 4, slope q q 1 5
Optimum feed is 3rd from bottom. Need 9 real stages plus partial condenser (see figure).
c. From figure slope of bottom operating line L V 2.025
Since saturated steam and CMO valid, B S L V
Also have mass balances, S + F = B + D
SyS Fz Bx B Dy D ys 0
Solve 3 eqs. simultaneously. S = 760 lbmoles/hr = 13,680 lb steam/hr.

109
4.D34*. Trial and Error, Feed: L L F F 2, q 3 2, Slope 3.

Figure for 4.D34


4-E1. Find (L/V)min (see diagram)
0.95 0.613 L L V min
L V min 0.3547 , 0.5497
0.95 0 D min 1 L V min

110
L L L L D
2 1.0994 , 0.5237
D act D min V act 1 L D
External M.B. F = D + B, Fz Dx D Bx B
z xB 0.6 0.025 kgmol
or Eq. (4.3) D F 100 62.162 , B = F – D = 37.838
xD xB 0.95 0.025 h

D L L D D 68.030

V L
L F V = L + D = 130.192
V´ L´
At feed V V (sat’d liquid)

V″ L″
P= L L F 168.030
xP
V L

L
Top op line y x 1 L F x D normal . At x = 0, y = 0.4525
V
L L
1st middle y x 1 x B looks like usual bottom!
V V
Goes through y x x B , and intersection top & feed line.
L 168.030
Slope1.2906
130.192 V
At pump-around return, V V 130.192
L L P 208.030, L V 208.030 130.192 1.5979
At pump-around removal, V V 130.192 , L L P L 168.030
Check at bottom L V B or 130.192 168.030 37.838 , OK
L L
Bottom Op line y x 1 x B , Same as first middle!!!
V V
Step off P.R. stage 1 & 2 above. x P is liquid from stage 2, x P 0.335 . Vertical line at x P is
withdrawal line for pump-around and it is feed-line for return of pump-around. 2nd middle op line
slope L V intersects x P withdrawal & feed line where bottom & 1st middle intersect.

111
L P Bx B
Using MB: yV Px P Bx B Lx, y x xP
V V V
P Bx B 40 32838 0.025
When y 0, x xP .335 0.0690
L L 208.030 208.030
Draw, 2nd middle – Step off stage 2 & start 3. 3 is op loc. for feed and where pump-around is
returned. Need PR + 6 equil stages. (Actually 5 and a large fraction)

L L q
4-E2*. Feed: q , L L 1.5F, q 3 2 , slope 3
F q 1
Bottom op. line: Since steam is saturated vapor S V and B L
Thus, 1 S B L V 1.2 . Operating line goes through y 0, x xB 0.015
Middle op. line: V B Side L S or V L S B Side
V y Bx B Side x side L x SyS
L Bx B Side x side
Since yS 0 this is, y x
V V
Side stream is removed as a saturated liquid so q = 1.
Step off two stages (see figure) and find x side 0.0975

112
Find slope: V S, B L Side
L Side B Side B 1 0.4 1
1.68 slope
V S SB 0.833
Draw in the middle operating line. Step off 4 stages. Trial and error to find x D .85 (see figure
for final result).

4-E3*. This column has 4 sections. The exact shape is not known ahead of time. Plot top operating line
L L D 1.86 L L
0.650, y x 1 xD
V 1 L D 2.86 V V
L
1 xD .35 .8 .28, y x x D 0.8
V
Step off 8 stages and find x S 0.495 yS . Feed line for this vapor is horizontal. Feed line for
feed to column is vertical at z = 0.32. From figure the feed is injected below the liquid
withdrawal and above vapor stream from intermediate reboiler. Can now calculate flows in each
section of columns.
Overall Balance: Fz Dx D F D x B

113
F z xB .3 1000
D 385
xD xB .78
L
Flows: L D 716.1, V L D 1101.1 V V ,V V S 643.8
D
L L S 258.8, L V 0.235
L L L F 1258.8, L V 1.143
L V 1.955 (this is a check)
To plot: From stage 8 draw line of slope L´/V´. From intersection of first intermediate operating
line and feed line draw line of slope L″/V″. Draw line from intersection of second intermediate
operating line with line y y s to y x x B 0.02 . Check if slope L V 1.955 . Optimum
feed is 10th below condenser while vapor from intermediate reboiler is returned on 11 th stage.
Need 12 ½ stages.
Note: Small differences in stepping off stages may change column geometry.

4.E.4. New Problem in 3rd Edition. External Balance.


a) F D B, Fz Dx D Bx B
z xB .25 .025
D F 100 25.7 kmol h , B 74.3 kmol h
xD xB .9 .025
b) V B 1.0, V B 74.3
L V B 148.6, L V 2.0
At feed, amount condensed = C = F/10 = 10
1
L L F C L F F 148.6 110 38.6
10
V V C 74.3 10 64.3

114
At stage 2 L L L R and V V 64.3
L L0 21.4
L 21.4
0.333 1 3
V 64.3
LR L L 38.6 21.43 17.17 kgmoles hour
L LR D L L
c) Top Op. line: y x xD x 1 xD
V V V V

D LR D
y x, y xD x D sin ce V L LR D
L0 V L
Envelope for top

V L Slope L V 1 3, y x x D 0.9
LR L
x 0,y 1 xD 0.6
V

Plot
L 38.6
Middle: y x 1 L V x D . Slope 0.600 y x xD 0.9
V 64.3
Bottom: y L V x L V 1 xB
y x xB
V B 1
Slope L V 2
V B
Now have somewhat redundant information.
Can plot bottom.
Intersection bottom and feed line should also be on middle. – Or use this pt to find middle or
bottom op. line.

From graph: Opt. Feed = #4.


Need 6 stages + P.R.

115
Graph for 4.E.4.

L V B V 1 2.25 9 4 9
4.E.5. New Problem in 3rd Edition.
V V V B 1.25 54 5
L L
Bot. Op. Line: y x 1 xB, y x x B , Slope L V
V V
D z xB .3 0.10 .2 1
External M.B.: F D B and Fz Dx D Bx B ,
F xD xB 0.90 0.10 .8 4
D 250 kmol day , B 750 kmol day , V V B B 1.25 750 937.5 kmol day
L V B 937.50 750 1687.5 kmol day
At feed stage: V V 937.5, L L F or L L F 1687.5 1000 687.5
L 687.5
0.733333
V 937.5

116
L L
MB: L x Dy D V y, D V L , y x 1 yD
V V
Goes through y x y D , and intersects feed and bot. op. lines.
At side withdrawal: 687.5 L L , V S V or V 937.5 200 737.5
Op line by intermediate condenser: L D V S
L Dy D Sx S
L x Dy D V y Sx S or y x
V V
687.5
Find from intersection L V op line @ y yS plus slope 0.932 or intersection
737.5
top op line and x x S
L 487.5
At side stream feed point: L L S 487.5, V V 737.5. Thus, 0.661
V 737.5
L L
Top op. line: y x 1 yD Can draw, yint ercept 1 .661 .9 0.305
V V

117
Graph for 4.E5.
4-F1.
970.33 Since bottoms are very pure h B h water @ 212°F
V L
HS 1381.4
v H Equil 1192 (in column)
Extra heat 189.4 Btu/lb
S B
Since y ≈ x, MW are same

118
Must vaporize material in column.
extra heat MW 189.4
v S S 0.1952S
MW 970.33
B L v, V S v
L B v BS vS 2 0.1952
1.837
V S v 1 vS 1 0.1952
If super heat not included L V BS 2 , which is incorrect.

4-F3*. An approximate check is to compare molar latent heats of vaporization. Data is available in
Perry’s and in Himmelblau.
a. See Example 4-4.
b. isopropanol λ = 159.35 cal/g. MW = 60.09, λ = 9.575 kcal/mole. Water λ = 9.72 kcal/mole.
CMO is OK.
c. CMO is not valid. AA 5.83 kcal / gmole, W 9.72
d. nC4. λ = 5.331 kcal/mole, MW = 58.12. λ = 0.0917 kcal/g
nC5. λ = 6.16 kcal/mole, MW = 72.15. λ = 0.0854 kcal/g
Constant mass overflow is closer than constant molar overflow.
e. benzene. λ = 7.353 kcal/mole, MW = 78.11, λ = 0.0941 kcal/g
toluene. λ = 8.00 kcal/mole, MW = 92.13, λ = 0.0868 kcal/g
CMO is within about 6%.

4G1. a*. Answer should be close, but not identical, to result obtained in Example 4-4.

4G2. Was 4.G4 in 2nd edition. Used Peng-Robinson. QC 44, 437,300 Btu/hr,
QR 49,859, 400 Btu/hr.
Optimum N F 17 & N 27 (Total condenser is #1)
x D 0.9992 and x B 0.00187 .
4G3*. See answers to selected problems in back of book.

4.H.1. New Problem in 3rd Edition. Use VBA program in Appendix B of Chapter 4.
xd 0.995 xb 0.011 F 250 z 0.4
L/D 3 q 0 feed stg 4
partial reboiler total condenser Ethanol-water Prob. 4H1.
VLE 6th 5th 4th 3rd 2nd 1st constant
-24.75 85.897 -118.03 82.079 -30.803 6.6048 0
yeqatxint 0.584177 yint 0.4 xint 0.2016667
stage x y
1 0.011 0.069033
2 0.039445 0.217358
3 0.112146 0.451872
4 0.227091 0.607193
5 0.477924 0.769849
6 0.694798 0.867005
7 0.824341 0.919132
8 0.893842 0.954863
9 0.941484 0.979257

119
10 0.974009 0.992216
11 0.991288 0.996557

4.H.2. New Problem in 3rd Edition. The VBA program is in Appendix B of Chapter 4. With L/D = 6.94
the result is:
xd 0.7 xb 0.0001 F 1000 z 0.1
L/D 6.94 q 0 feed stg 28
partial reboiler total condenser Ethanol-water Prob. 4H3.
VLE 6th 5th 4th 3rd 2nd 1st constant
-47.949 161.42 -212.43 138.68 -46.65 7.9322 0
yeqatxint 0.099219 yint 0.1 xint 0.0135447
stage x y
Reflux rate too low Reflux rate too low

With L/D = 6.95 the result is given below. With feed stages below 85 the feed stage was too low.
xd 0.7 xb 0.0001 F 1000 z 0.1
L/D 6.95 q 0 feed stg 85
partial reboiler total condenser Ethanol-water Prob. 4H3.
VLE 6th 5th 4th 3rd 2nd 1st constant
-47.949 161.42 -212.43 138.68 -46.65 7.9322 0
yeqatxint 0.100057 yint 0.1 xint 0.0136691
stage x y
1 0.0001 0.000793
2 0.000194 0.001538
3 0.000295 0.002338
4 0.000404 0.003197
5 0.000521 0.004117
6 0.000646 0.005102
7 0.000779 0.006154
8 0.000922 0.007277
9 0.001075 0.008472
10 0.001237 0.009742
11 0.00141 0.011089
12 0.001593 0.012515
13 0.001786 0.014021
14 0.001991 0.015608
15 0.002206 0.017276
16 0.002433 0.019025
17 0.00267 0.020853
18 0.002919 0.022758
19 0.003178 0.024739
20 0.003447 0.026791
21 0.003725 0.02891
22 0.004013 0.03109
23 0.004309 0.033327
24 0.004613 0.035613
25 0.004924 0.037941
26 0.00524 0.040302
27 0.005561 0.042689
28 0.005885 0.045091

120
29 0.006211 0.0475
30 0.006538 0.049907
31 0.006865 0.052301
32 0.00719 0.054674
33 0.007513 0.057017
34 0.007831 0.059321
35 0.008144 0.061579
36 0.00845 0.063782
37 0.00875 0.065925
38 0.009041 0.068002
39 0.009323 0.070008
40 0.009595 0.071938
41 0.009858 0.07379
42 0.010109 0.075561
43 0.01035 0.077251
44 0.010579 0.078857
45 0.010797 0.08038
46 0.011004 0.08182
47 0.0112 0.083179
48 0.011384 0.084459
49 0.011558 0.085661
50 0.011721 0.086787
51 0.011874 0.087841
52 0.012018 0.088826
53 0.012151 0.089743
54 0.012276 0.090598
55 0.012392 0.091392
56 0.0125 0.092129
57 0.0126 0.092812
58 0.012693 0.093445
59 0.012779 0.09403
60 0.012858 0.094571
61 0.012932 0.09507
62 0.012999 0.095531
63 0.013062 0.095955
64 0.01312 0.096346
65 0.013173 0.096705
66 0.013222 0.097036
67 0.013267 0.09734
68 0.013308 0.09762
69 0.013346 0.097876
70 0.013381 0.098112
71 0.013413 0.098328
72 0.013442 0.098526
73 0.013469 0.098708
74 0.013494 0.098874
75 0.013516 0.099027
76 0.013537 0.099167
77 0.013556 0.099295
78 0.013573 0.099412

121
79 0.013589 0.099519
80 0.013604 0.099618
81 0.013617 0.099707
82 0.013629 0.09979
83 0.013641 0.099865
84 0.013651 0.099934
85 0.01366 0.099997
86 0.013665 0.100032
87 0.013706 0.100305
88 0.014018 0.102401
89 0.016416 0.11824
90 0.034534 0.223718
91 0.155188 0.516574
92 0.490181 0.652848
93 0.646064 0.724878

4.H.3. New Problem in 3rd Edition. The Spreadsheet is:


xd 0.7 xb 0.0001 F 1000 z 0.17
Multiplier 1.05 q 0.5 feed stg 17
partial reboiler total condenser Ethanol-water Problem 4.H4.
VLE 6th 5th 4th 3rd 2nd 1st constant
-47.949 161.42 -212.43 138.68 -46.65 7.9322 0
L/Dmin 1.687377 L/Vmin 0.62789 L/D 1.771746 L/V 0.639217
stage x y
1 0.0001 0.000793
2 0.000229 0.001812
3 0.000418 0.003309
4 0.000696 0.0055
5 0.001104 0.008697
6 0.001698 0.013331
7 0.002559 0.019994
8 0.003797 0.029452
9 0.005555 0.042644
10 0.008006 0.060585
11 0.01134 0.08415
12 0.015719 0.113686
13 0.021208 0.148522
14 0.027681 0.186647
15 0.034766 0.224911
16 0.041877 0.259918
17 0.048382 0.28916
18 0.057276 0.325158
19 0.113592 0.469947
20 0.340102 0.575494
21 0.505222 0.659654
22 0.636883 0.719124
Because the multiplier is close to 1.0, this answer is very sensitive to the data fit used.
One solution to the coding is the following VBA program:

Option Explicit

122
Sub McCabeThiele()
' Find minimum reflux ratio assuming it occurs at feed plate. Then
' L/D actual = L/D min times Multiplier. Steps off stages from the bottom up.
' Assumes that the feed stage is specified.
Sheets("Sheet2").Select
Range("A8", "G108").Clear
Dim i, feedstage As Integer
Dim D, B, xd, xb, F, z, q, LoverD, LoverV, x, y, xint, yint, yeq As Single
Dim a6, a5, a4, a3, a2, a1, a0, L, V, LbaroverVbar, LoverDmin As Single
Dim LoverVmin, LoverVdelta, Multiplier As Single
' Input values from spread sheet
xd = Cells(1, 2).Value
xb = Cells(1, 4).Value
F = Cells(1, 6).Value
z = Cells(1, 8).Value
Multiplier = Cells(2, 2).Value
q = Cells(2, 4).Value
feedstage = Cells(2, 8).Value
' Fit of equilibrium data to 6th order polynomial to find y. a6 is multiplied
' by x to the 6th power.
a6 = Cells(5, 1).Value
a5 = Cells(5, 2).Value
a4 = Cells(5, 3).Value
a3 = Cells(5, 4).Value
a2 = Cells(5, 5).Value
a1 = Cells(5, 6).Value
a0 = Cells(5, 7).Value
' Calculate intersection point of two operating lines and use this to find
' minimum L/D and L/V. Initialize
LoverV = 1
LoverVdelta = 0.00001
Do
LoverV = LoverV - LoverVdelta
xint = ((-(q - 1) * (1 - LoverV) * xd) - z) / (((q - 1) * LoverV) - q)
x = xint
yint = LoverV * xint + (1 - LoverV) * xd
' Equilibrium y at value of x intersection. When yint=yeq, have minimum L/V and L/D.
yeq = a6 * x ^ 6 + a5 * x ^ 5 + a4 * x ^ 4 + a3 * x ^ 3 + a2 * x ^ 2 + a1 * x + a0
Loop While yint < yeq
'Print intersection and equilibrium values.
LoverVmin = LoverV + LoverVdelta
LoverDmin = LoverVmin / (1 - LoverVmin)
LoverD = Multiplier * LoverDmin
LoverV = LoverD / (1 + LoverD)
Cells(6, 2).Value = LoverDmin
Cells(6, 4).Value = LoverVmin
Cells(6, 6).Value = LoverD
Cells(6, 8).Value = LoverV
' Calculate flow rates and ratios.
D = ((z - xb) / (xd - xb)) * F
L = LoverD * D

123
V=L+D
LbaroverVbar = (LoverV + (q * F / V)) / (1 - ((1 - q) * F / V))
' Step off stages from bottom up. First stage is partial reboiler. Initialize
x = xb
i=1
' Loop in stipping section stepping off stages with equilibrium and operating eqs.
Do While i < feedstage
y = a6 * x ^ 6 + a5 * x ^ 5 + a4 * x ^ 4 + a3 * x ^ 3 + a2 * x ^ 2 + a1 * x + a0
Cells(i + 7, 1).Value = i
Cells(i + 7, 2).Value = x
Cells(i + 7, 3).Value = y
i=i+1
x = (y / LbaroverVbar) + (LbaroverVbar - 1) * xb / LbaroverVbar
Loop
' Calculations in enriching section continues to Loop While y < xd.
Do While y < xd
y = a6 * x ^ 6 + a5 * x ^ 5 + a4 * x ^ 4 + a3 * x ^ 3 + a2 * x ^ 2 + a1 * x + a0
Cells(i + 7, 1).Value = i
Cells(i + 7, 2).Value = x
Cells(i + 7, 3).Value = y
i=i+1
x = (y / LoverV) - (1 - LoverV) * xd / LoverV
If x < 0 Then
Cells(i + 7, 4).Value = "Feed stage too low"
Exit Do
End If
If i > 100 Then
Cells(i + 7, 6).Value = "Too many stages"
Exit Do
End If
Loop
End Sub

124
SPE 3rd Ed. Solution Manual Chapter 5

New Problems and new solutions are listed as new immediately after the solution number. These new
problems are: 5A15, 5C1, 5D1, 5D2, 5D9, 5D10, 5H1 to 5H5. Problems and solutions from the first
edition that were not in the second edition are: 5D6, 5D8, 5D11-5D13, 5E1.

5.A6.

Ethane is less volatile than methane so it decreases toward distillate. At bottoms it is more
volatile than propane and butane, so must decrease towards bottoms. Thus ethane
concentrates within column.

5.A7. 1. c; 2. c; 3. a (saturated liquid feed); 4. c; 5. B

5.A9. Pure LK cannot be withdrawn because LNK is present. Pure LNK can be removed at
distillate if all LK is removed in side stream. However, recovery of LNK will be < 100%.

5.A13. If z HNK F (frac. rec. HK in bot) z HK F, then there is more HNK in bottoms and curves
cross.

5A15. New Problem in 3rd Edition. a. a, b. d, c. b, d. c, e. b.

5.C1. New Problem in 3 rd Edition. Start with equilibrium equation, yi,j = Ki,jxi,j and multiply right hand
side by Kref,j /Kref,j. One obtains yi,j = αi-ref,j Kref,jxi,j. Then 1 = Σ y = Σ (α i-ref,j Kref,jxi,j) . Since Kref,j is
constant, bring it outside the summation and solve for Kref,j = 1/ Σ (αi-ref,j xi,j). This is Eq. (5-29).

125
5.D1. New Problem in 3rd Edition.

a) Dx M ,d 1.00 2, 000 0.19 2280


Dx B,d 0 0
Dx E,d 0.978 2, 000 0.31 3638.16
Dx P ,d 1 0.994 12, 000 0.27 19.44

D x i,d D 5937.6 kg/h

b) assume NK (Methanol and n-butanol) do not distribute (all


methanol in top and all butanol in bottom).

wt frac. in distillate: M = 0.38399, E = 0.61273, P = 0.00327, B = 0.0.

wt. frac
x i,b
Bx M ,B 0 0
0
Bx B,B 1.0 12, 000 0.23 2760.0
0.4553
Bx E ,B 1 0.978 12, 000 0.31 81.84
0.0135
Bx P ,B 0.994 12, 000 0.27 3220.56 0.5312
B x i,b B 6062.4 kg hr
Check B+D=F, OK.

5.D2. New Problem in 3rd Edition. y4 0.30, y 5 0.5, y 6 0.20, P 760 mmHg
Raoult’s Law yi P VPi x i
B
Antoine’s Eqn. log10 VP A
T C
Dew Point condition is xi 1

yi P
xi (from Raoult’s Law)
VPi

126
B
VPi 10 A (from Antoine’s Eqn)
T C
Combine with Dew Point condition
y4P y5P y6P
B B B
1
4 5 6
A4 A5 A6
T C4 T C5 T C6
10 10 10

0.30 760 0.50 760 0.20 760


935.86 1064.8 1171.17
1
6.809 6.853 6.876
T 238.73 T 233.01 T 224.41
10 10 10

Using Goal Seek in Excel, T = 41.3ºC

5.D3. Assume that ethanol is HK. Then assume that HNK’s are totally in the bottoms.
x M,dist .99, x E,dist .01
2195.6 = (.998) (0.22) (10,000) = Dx MD
2195.6 2195.6
D 2217.78 and B F D 7782.22
x MD .99
Bottoms: MeOH: .0021 (.22) (10,000) = 4.4
4.4
x Mbot 0.0006
7782.22
1.0 .18 10, 000
x n propanol,bot 0.2313
7782.22
1.0 .13 10, 000
x n butanol,bot .1670
7782.22
x EtOH,bot 1 x MeOH,bot x n p,bot x nbut,bot 0.6011

5.D4. a. .99 F z C5 D x C5,dist (1), and .01 Fz C5 =B x C5,bot (2)


.98 F z C6,botBx C6,bot (3), and .02 F z C6 D z C6,dist (4)
Assume all heptane in bottoms
Fz C7 Bx bot,C6 (5), x dist,C7 0 (6)
Take Eqs. 1, 4 & 6: .99 (1000) (.4) = Dx C5d
.02 (1000) (.3) = D x C6d
0 = D x C7,dist
Dx l,dist D 402 kg moles/hr
B = F – D = 1000 – 402 = 598
b. x C7,dist 0
.99 1000 .4
x C5,d 0.9851
402
x C6,d 1 0.9851 0.0149

127
1000 .3 1.0
x C7,bot 0.5017
598
.98 1000 .3
x C6,bot 0.4916
598
x C5,bot 1 .5017 .4916 0.0067
c. L = (L/D)D = (2.5) (402) = 1005
V = L + D = 1005 + 402 = 1407
At feed stage: L = L + .6F = 1005 + 600 = 1605
V = V - .4F = 1407 – 400 = 1007

5.D5. Assume all methanol and ethanol in distillate.


Dx MeOH,dist 0.55 100 0.01 150 56.5
Dx EtOH,dist 0.21 100 0.03 150 25.5

Dx prop,dist 0.993 0.23 100 0.26 150 61.57


Dx bu tan ol,dis 1 0.995 0.01 100 0.70 150 0.53

D 144.1
B F1 F2 D 105.9
Check: Bx Pr op,bot 1 0.993 .23 100 .26 150 0.434
Bx but,dist 0.995 .01 100 0.7 150 105.47

Check = 105.90
Mole fractions:
0.434
x M ,bot 0 , x E ,bot 0, x prop,bot 0.0041
105.90
x but,bot 1 x prop,bot 0.9959

Dx MeOH ,dist 56.5


x M ,dist 0.392
D 144.1
25.5
x E,dist 0.1767
144.1
61.566
x p,dist 0.427
144.1
x But,dist 0.53 144.1 0.0037

Check = 1.000 OK

5.D6. This is 8.D1. in 1st ed.

128
129
5.D7. Assume 100% recovery C 2 , & propylene in distillate.
Assume 100% recovery pentane & hexane in bottoms.

Comp. Distillate kg/h xD


C2 0.3 (1000) + 0.02 (1500) = 330 0.2824
Propylene: 0.006 (1000) + 0.001 (1500) = 7.5 0.0064
n-C3 (0.991) [1000 (0.45) + 1500 (0.249)] = 816.0885 0.06983
n-C4 (0.02) [1000 (0.154) + 1500 (0.40)] = 15.08 0.0129
C5 & C6 0 = 15.08 0.0
D= = 1168.7

Bottoms flow rate = F1 F2 D 2500 1168.7 1331.3 kg/h


.009 1000 0.45 1500 0.249
x B,C2 0, x B,propylene 0, x B,C3 0.0056
1331.3
.98 1000 .154 1500 .40
x B,C4 0.5550
1331.3
1000 0.09 1500 0.18 1500 .15
x B,C5 0.2704, x B,C6 0.1690
1331.33 1331.31

5.D8. 8.D.6. in 1st edition. Assume all benzene is in the distillate.

130
131
5.D.9. New Problem in 3rd Edition.

132
5.D10. New Problem in 3rd Edition. At the bubble point yi 1.0

x C5 .40 , x C6 .60 , p 760 mmHg , y C5 y C6 K 5 x C5 K 6 x C6 1.0

133
VPC5 VPC6
or x C5 x C6 1.0 , VPC5 x C5 VPC6 x C6 Ptot
Ptot Ptot
1064.8 1171.17
6.853 6.876
T 233.01 T 224.41
0.40 10 0.60 10 760

TºC (C5 term) x (.4) C6 term x (.6) SUM


20 441.03 × .4 = 176.4 + 121.387 × .6 = 249.23
Final result is:
51 1270.095 × .4 = 508.04 + 420.28 × .6 = 252.17 760.21

5.D11. Was problem 6.D2 in 2nd edition SPE. Since x i known, want yi 1 K i p BP x i
K ref p old
Find new pressure from, K ref p new
Kixi
Use ethane, or n-pentane as reference.
First guess: Try K C5 1.0, p 115 kPa
K NC7 0.13 K C2 29
Ki xi 29 0.1 1.0 .35 0.13 .55 3.32 p too low.
1.0
K rep p new ~ 0.3 p new 440 kPa , K NC7 0.042 K C2 8.0
3.32
Ki xi 8.0 0.1 0.3 .35 0.042 0.55 0.927
0.3
K C5 p new .032 p new 400 kPa, K nC7 0.045 K C2 8.7
0.927
K i x i 8.7 0.1 0.32 0.35 0.045 0.55 1.004
Answer (within accuracy DePriester Chart) = 400 kPa

5.D12. Was problem 6.D3 in 2nd edition SPE. a. Highest B.P. Temp. is pure n-octane. K C8 1.0, T 174 C
b. Lowest B.P. Temp. is pure n-hexane. K C6 1.0, T 110 C

5.D13. Was problem 6.D6 in 2nd edition SPE. Let 1 = n-butane, 2 = n-pentane and 3 = n-hexane. p = 101.3 kPa.
Bubble Point: First guess. K1 1 at T 1 , K 2 1 at T 36 , K 3 1 at T 68 .
Tfirst z1T1 .2 1 .5 36 .3 68 38
K1 3.6, K 2 1.08, K 3 0.36. K1x1 .2 3.6 .5 1.08 .3 .36 1.368
Choose 2 as ref. Eq. (6-14) is: K 2 Tnew 1.8 1.368 0.789
Tnew 29 C. K1 2.7, K 5 0.26, and Kixi 1.013
Eq. (6-14) is: K 2 Tnew 0.789 1.013 0.779
Tnew 28.8 K1 2.7, K8 0.255, and Kx 1.006. OK. TBP 28.8 C.
i i

5. E1. This is 8.E4. in 1st edition.

134
135
136
5.H1. New Problem in 3rd Edition. Same program as 5.H5 except do not list y values as distillate.
Different input in spreadsheet – see below.
Ternary Distillation: Constant relative volatility. Step off stages from bottom up. Use whole stages.
System has A = LK, B = HK and C = HNK C5H9
alpha Alpha C-
A-B 3.58 alpha B-B 1.86 B 1 feedstage 8
zA 0.35 z B 0.4 z C 0.25 epsilon (values for 0.00000001
N loop(
F 200 q 1 L/D 6 convergence 100
df(HNK frac
frac rec B in dist 0.996 guess frac rec C bot 1 recovery) 0.9
frac rec A in dist 0.961
D 67.59011 B 132.4099 L/V 0.857143 Lbar/Vbar 1.279858988
Mass balance
xAdist 0.995264 xBdist 0.004734 xCdist 1.56E-06 values
Mass balance
xAbot 0.020618 xBbot 0.601768 xCbot 0.377614 values

137
Stage by stage calculations
i xA yA xB yB xC yC Note x1 =
1 0.020618 0.046992 0.601768 0.7125981 0.377614 0.240409437 xbot
2 0.041225 0.0869 0.688364 0.7538807 0.270411 0.159219617
3 0.072406 0.143487 0.720619 0.7419438 0.206975 0.114569647
4 0.11662 0.218289 0.711292 0.6917342 0.172088 0.089976459
5 0.175066 0.308791 0.672062 0.6158891 0.152873 0.075319919
6 0.245778 0.407144 0.612801 0.5274175 0.141421 0.0654388
7 0.322624 0.502187 0.543675 0.4396808 0.1337 0.058132319
8 0.396885 0.584094 0.475123 0.3632905 0.127992 0.05261578
9 0.515565 0.685129 0.42305 0.2920855 0.061385 0.02278591
10 0.633439 0.774848 0.339977 0.2160685 0.026583 0.009083172
11 0.738112 0.846817 0.251291 0.1497869 0.010597 0.003395923
12 0.822076 0.899855 0.173962 0.098934 0.003962 0.001211305
13 0.883953 0.936484 0.114634 0.0630978 0.001413 0.000418127
14 0.926687 0.960636 0.072825 0.0392226 0.000488 0.000141178
15 0.954865 0.97607 0.044971 0.0238835 0.000164 4.69552E-05
16 0.97287 0.985732 0.027075 0.0142529 5.45E-05 1.54307E-05
17 0.984143 0.991702 0.015839 0.0082925 1.77E-05 4.9941E-06
18 0.991109 0.995362 0.008886 0.0046363 5.57E-06 1.56158E-06

Calc frac recovery C in bottoms 0.9999979 j 3


5.H2. New Problem in 3rd Edition. The spreadsheet is,
Ternary Distillation with Constant relative volatility. Step off stages from top down.
System has A = LNK, B = LK and C = HK
alpha A-B 2.25 alpha B-B 1 Alpha C-B 0.21 feedstage 6
zA 0.25 zB 0.35 zC 0.4 epsilon (values for 0.0001
F 100 q 1 L/D 0.3 N loop( convergence 10
frac rec B in dist 0.9 frac rec C in bot 0.97 df(LNK frac recovery) 0.9
Guess: frac rec A in dist 1
D 57.66689 B 42.35298 L/V 0.230769 Lbar/Vbar 1.565105
xAdist 0.43295 xBdist 0.546241 xCdist 0.020809 Mass balance values
xAbot 0.001251 xBbot 0.082639 xCbot 0.91611 Mass balance values
Stage by stage calculations
i xA yA xB yB xC yC Note y1 =
1 0.229688 0.43295 0.65203 0.546241 0.118282 0.020809168 xdist
2 0.180904 0.386044 0.601681 0.570654 0.217416 0.043302907
3 0.16005 0.374786 0.537147 0.559034 0.302804 0.066179941
4 0.147137 0.369973 0.486906 0.544142 0.365957 0.085884852
5 0.13893 0.366993 0.453607 0.532548 0.407464 0.100458716
6 0.133981 0.365099 0.433372 0.524864 0.432647 0.1100371
7 0.062603 0.208987 0.425676 0.631573 0.511721 0.159440004
8 0.021494 0.097273 0.308019 0.619528 0.670486 0.283199293
9 0.004909 0.032934 0.146011 0.435383 0.84908 0.531682943
10 0.000766 0.006976 0.044919 0.181823 0.954315 0.811200627

Calc frac recovery A in distillate 0.998702 j 2

138
5.H.3. New Problem in 3rd Edition. (L/D)min = 0.26761 by trial and error using spreadsheet in Table 5.A-
1..

5.H.4. New Problem in 3rd Edition.


Ternary Distillation with Constant relative volatility. Step off stages from top down.
System has A = LK, B = sandwich and C = HK 5.G.e.
alpha A-B 1.4 alpha B-B 1 Alpha C-B 0.7 feedstage 19
zA 0.25 zB 0.35 zC 0.4 epsilon (values for 1E-10
F 100 q 1 L/D 5 N loop( convergence 100
frac rec B in dist 0.583 frac rec C in bot 0.995 df(LNK frac recovery) 0.8
Guess: frac rec A in dist 0.95
D 44.10818 B 55.89182 L/V 0.833333 Lbar/Vbar 1.21119
xAdist 0.532853 xBdist 0.462613 xCdist 0.004534 Mass balance values
xAbot 0.026781 xBbot 0.261129 xCbot 0.71209 Mass balance values
Stage by stage calculations
i xA yA xB yB xC yC Note y1 =
1 0.447934 0.532853 0.544443 0.462613 0.007623 0.004534307 xdist
2 0.378937 0.462087 0.609404 0.530804 0.011659 0.007108533
3 0.325116 0.40459 0.658055 0.584939 0.016829 0.010471366
4 0.284385 0.359739 0.692247 0.625481 0.023368 0.01477984
5 0.254167 0.325796 0.71427 0.653975 0.031563 0.020228923
6 0.231958 0.300615 0.726286 0.672327 0.041757 0.027058086
7 0.215598 0.282107 0.730061 0.68234 0.054342 0.035552935
8 0.203353 0.268473 0.726901 0.685486 0.069746 0.046040694
9 0.193893 0.258269 0.717703 0.682853 0.088403 0.058877558
10 0.18623 0.250387 0.703059 0.675188 0.110711 0.074425299
11 0.179649 0.244001 0.683384 0.662985 0.136967 0.093014635
12 0.173652 0.238516 0.65905 0.646589 0.167298 0.114894592
13 0.167915 0.233519 0.630501 0.62631 0.201584 0.140170911
14 0.162254 0.228738 0.598352 0.602519 0.239393 0.168742359
15 0.156599 0.224021 0.563437 0.575729 0.279964 0.200250121
16 0.150964 0.219308 0.526798 0.546633 0.322237 0.234059083
17 0.145428 0.214612 0.489617 0.516101 0.364955 0.269286831
18 0.140099 0.209999 0.453098 0.485116 0.406803 0.304884695
19 0.135091 0.205558 0.418339 0.454684 0.446571 0.339758041
20 0.100543 0.157965 0.402362 0.45154 0.497094 0.39049513
21 0.071479 0.116121 0.372448 0.43219 0.556074 0.45168881
22 0.048123 0.080918 0.329669 0.395957 0.622209 0.523124245
23 0.030232 0.05263 0.276757 0.344144 0.693012 0.603226557
24 0.017191 0.03096 0.2177 0.280057 0.765109 0.688982619
Mass balance: fraction stage, A, B, C calculated at bottom, % error B
0.264616 0.026781 0.261129 0.71209 6.65242E-09
Calc frac recovery A in distillate 0.940127 j 4

Part d. Fractional recovery of B in distillate. is 0.725. Note that B goes through a maximum of close to
1% on stages 7 and 8.
Ternary Distillation with Constant relative volatility. Step off stages from top down.
System has A = LK, B = trace sandwich and C = HK 5.G.e. Part d.
alpha A-B 1.4 alpha B-B 1 Alpha C-B 0.7 feedstage 19

139
zA 0.38 zB 0.02 zC 0.6 epsilon (values for 1E-10
F 100 q 1 L/D 4 N loop( convergence 100
frac rec B in dist 0.725 frac rec C in bot 0.995 df(LNK frac recovery) 0.8
Guess: frac rec A in dist 0.99
D 39.37044 B 60.62956 L/V 0.8 Lbar/Vbar 1.308
xAdist 0.95555 xBdist 0.03683 xCdist 0.00762 Mass balance values
xAbot 0.00626 xBbot 0.009071 xCbot 0.984668 Mass balance values
Stage by stage calculations
i xA yA xB yB xC yC Note y1 =
1 0.934659 0.95555 0.050434 0.03683 0.014907 0.00761993 xdist
2 0.909255 0.938837 0.064694 0.047713 0.026051 0.013449322
3 0.878109 0.918514 0.079128 0.059121 0.042762 0.02236485
4 0.839847 0.893598 0.092985 0.070669 0.067168 0.035733649
5 0.793216 0.862987 0.105202 0.081754 0.101582 0.055258739
6 0.737611 0.825683 0.114471 0.091527 0.147918 0.082789801
7 0.673775 0.781199 0.119471 0.098942 0.206753 0.119858439
8 0.604361 0.73013 0.119295 0.102943 0.276345 0.166926476
9 0.53382 0.674599 0.113888 0.102802 0.352293 0.222599678
10 0.467334 0.618166 0.104227 0.098476 0.428438 0.283358088
11 0.409234 0.564978 0.092025 0.090748 0.498741 0.344274648
12 0.361849 0.518497 0.079126 0.080986 0.559026 0.400517103
13 0.325379 0.480589 0.066982 0.070666 0.607639 0.448744495
14 0.298551 0.451414 0.056436 0.060951 0.645013 0.487634893
15 0.279454 0.429951 0.047786 0.052515 0.67276 0.517534464
16 0.266162 0.414673 0.040972 0.045595 0.692866 0.539732259
17 0.257043 0.40404 0.035754 0.040143 0.707203 0.555816779
18 0.250839 0.396745 0.031838 0.035969 0.717324 0.567286133
19 0.246633 0.391781 0.028939 0.032836 0.724428 0.575383054
20 0.193371 0.320667 0.029598 0.035058 0.777031 0.644274899
21 0.145301 0.251 0.029111 0.03592 0.825588 0.713080242
22 0.105056 0.188125 0.027585 0.035283 0.867359 0.776592117
23 0.073452 0.135485 0.025264 0.033287 0.901284 0.831228269
24 0.049874 0.094147 0.022436 0.030252 0.92769 0.875601733
25 0.032959 0.063306 0.019353 0.026552 0.947688 0.910141552
26 0.02117 0.041182 0.016207 0.02252 0.962623 0.93629802
27 0.013123 0.025762 0.013125 0.018405 0.973752 0.955833007
28 0.00771 0.015236 0.010183 0.014373 0.982107 0.970390508
29 0.004108 0.008156 0.007421 0.010525 0.988471 0.981318291
Mass balance: fraction stage, A, B, C calculated at bottom, % error B
0.402462 0.00626 0.009071 0.984668 5.53216E-07
Calc frac recovery A in distillate 0.990012 j 16

5.H.5. New Problem in 3rd Edition.


Ternary Distillation with Bubble point Calc. Step off stages from bottom
up.
System has A = LK, B = HK and C = HNK
feedstage 5
zA 0.3 zB 0.3 zC 0.4 epsilon 1E-08
F 100 q 1 L/D 8 N loop 100
frac rec B in bot 0.997 guess frac rec C bot 1 df 0.9

140
frac rec A in dist 0.995 Trebguess 500 p, psia 14.7
K const. aT1 aT2 aT6 ap1 ap2 ap3
nB=A -1280557 0 7.94986 -0.96455 0 0
nPen=B -1524891 0 7.33129 -0.89143 0 0
nHex=C -1778901 0 6.96783 -0.84634 0 0

D 29.940006 B 70.05999 L/V 0.88889 Lbar/Vbar 1.26


xAdist 0.9969938 xBdist 0.003006 xCdist 2.2E-07
xAbot 0.002141 xBbot 0.42692 xCbot 0.57094

stage xA yA xB yB xC yC T KB
1 0.002141 0.0103991 0.42692 0.6614042 0.57094 0.3282 582.283 1.549247
2 0.0086951 0.0361347 0.613018 0.7887656 0.37829 0.1751 570.622 1.286692
3 0.0291201 0.1069354 0.714099 0.7929732 0.25678 0.10009 561.854 1.110453
4 0.0853111 0.270011 0.717438 0.6674458 0.19725 0.06254 551.835 0.930318
5 0.214736 0.5324156 0.617813 0.4297616 0.16745 0.03782 536.481 0.695617
6 0.4743433 0.786406 0.483106 0.2080991 0.04255 0.00549 513.748 0.430752
7 0.7600825 0.9294091 0.233736 0.0700679 0.00618 0.00052 498.351 0.299774
8 0.920961 0.980028 0.078451 0.0199309 0.00059 4.1E-05 491.769 0.254057
9 0.9779073 0.9946891 0.022047 0.0053079 4.6E-05 3E-06 489.686 0.240759
10 0.994401 0.9986729 0.005596 0.0013269 3.4E-06 2.2E-07 489.103 0.237137

Distillate mole fracs = y values 0.9986729 0.00133 2.2E-07


Calc frac recovery C in bottoms 0.9999998 j 3

Option Explicit
Sub Ternary_bottom_up_BP()
' Ternary distillation with constant alpha. Frac recoveries of LK and HK given.
' There is a HNK present and its frac rec in bottoms is guessed.
Sheets("Sheet1").Select
Range("A18", "I150").Clear
Dim i, j, k, feedstage, N As Integer
Dim AaT1, AaT6, Aap1, BaT1, BaT6, Bap1, CaT1, CaT6, Cap1 As Double
Dim F, fracBbot, fracCbot, q, LoverD, LoverV As Double
Dim LbaroverVbar, D, B, L, V, Lbar, Vbar, Eqsum, fracAdist As Double
Dim xA, xB, xC, yA, yB, yC, zA, zB, zC, xAbot, xBbot, xCbot As Double
Dim DxA, DxB, DxC, BxA, BxB, BxC, xAdist, xBdist, xCdist As Double
Dim fracCbotcalc, difference, epsilon, df As Double
Dim T, p, Tinit, KA, KB, KC, sum As Double
'Input data
AaT1 = Cells(9, 2).Value
AaT6 = Cells(9, 4).Value
Aap1 = Cells(9, 5).Value
BaT1 = Cells(10, 2).Value
BaT6 = Cells(10, 4).Value
Bap1 = Cells(10, 5).Value
CaT1 = Cells(11, 2).Value
CaT6 = Cells(11, 4).Value
Cap1 = Cells(11, 5).Value
feedstage = Cells(3, 8).Value
F = Cells(5, 2).Value

141
q = Cells(5, 4).Value
LoverD = Cells(5, 6).Value
zA = Cells(4, 2).Value
zB = Cells(4, 4).Value
zC = Cells(4, 6).Value
fracBbot = Cells(6, 3).Value
fracCbot = Cells(6, 6).Value
fracAdist = Cells(7, 4).Value
epsilon = Cells(4, 8).Value
N = Cells(5, 8).Value
df = Cells(6, 8).Value
p = Cells(7, 8).Value
Tinit = Cells(7, 6).Value
' The For loop (remainder of program) is to obtain convergence of guess of
' fractional recovery of A in distillate.
For j = 1 To N
' Calculate compositions and flow rates.
DxA = F * zA * fracAdist
DxB = F * zB * (1 - fracBbot)
DxC = F * zC * (1 - fracCbot)
BxA = F * zA * (1 - fracAdist)
BxB = F * zB * fracBbot
BxC = F * zC * fracCbot
D = DxA + DxB + DxC
B = BxA + BxB + BxC
xAdist = DxA / D
xBdist = DxB / D
xCdist = DxC / D
xAbot = BxA / B
xBbot = BxB / B
xCbot = BxC / B
L = LoverD * D
V=L+D
LoverV = L / V
Lbar = L + q * F
Vbar = Lbar - B
LbaroverVbar = Lbar / Vbar
' Print values of flowrates and mole fractions
Cells(13, 2) = D
Cells(13, 4) = B
Cells(13, 6) = LoverV
Cells(13, 8) = LbaroverVbar
Cells(14, 2) = xAdist
Cells(14, 4) = xBdist
Cells(14, 6) = xCdist
Cells(15, 2) = xAbot
Cells(15, 4) = xBbot
Cells(15, 6) = xCbot
' initialize (reboiler =1) and start loops
i=1
xA = xAbot

142
xB = xBbot
xC = xCbot
T = Tinit
' Calculations in stripping section: equilibrium then operating.
Do While i < feedstage
' Bubble point calculaton
For k = 1 To 10
KB = 1
KA = Exp((AaT1 / (T * T)) + AaT6 + (Aap1 * Log(p)))
KB = Exp((BaT1 / (T * T)) + BaT6 + (Bap1 * Log(p)))
KC = Exp((CaT1 / (T * T)) + CaT6 + (Cap1 * Log(p)))
yA = KA * xA
yB = KB * xB
yC = KC * xC
sum = yA + yB + yC
KB = KB / sum
T = Sqr(BaT1 / (Log(KB) - BaT6 - (Bap1 * Log(p))))
Next k
' Print values
Cells(i + 17, 1).Value = i
Cells(i + 17, 2).Value = xA
Cells(i + 17, 3).Value = yA
Cells(i + 17, 4).Value = xB
Cells(i + 17, 5).Value = yB
Cells(i + 17, 6).Value = xC
Cells(i + 17, 7).Value = yC
Cells(i + 17, 8).Value = T
Cells(i + 17, 9).Value = KB
' Bottom operating line
i=i+1
xA = yA / LbaroverVbar + (1 - (1 / LbaroverVbar)) * xAbot
xB = yB / LbaroverVbar + (1 - (1 / LbaroverVbar)) * xBbot
xC = yC / LbaroverVbar + (1 - (1 / LbaroverVbar)) * xCbot
Loop
' Calculations in enriching section
Do
For k = 1 To 10
' Bubble point calculation
KA = Exp((AaT1 / (T * T)) + AaT6 + (Aap1 * Log(p)))
KB = Exp((BaT1 / (T * T)) + BaT6 + (Bap1 * Log(p)))
KC = Exp((CaT1 / (T * T)) + CaT6 + (Cap1 * Log(p)))
yA = KA * xA
yB = KB * xB
yC = KC * xC
sum = yA + yB + yC
KB = KB / sum
T = Sqr(BaT1 / (Log(KB) - BaT6 - (Bap1 * Log(p))))
Next k
' Print values
Cells(i + 17, 1).Value = i
Cells(i + 17, 2).Value = xA

143
Cells(i + 17, 3).Value = yA
Cells(i + 17, 4).Value = xB
Cells(i + 17, 5).Value = yB
Cells(i + 17, 6).Value = xC
Cells(i + 17, 7).Value = yC
Cells(i + 17, 8).Value = T
Cells(i + 17, 9).Value = KB
' Test for feed stage too low
If xA < 0 Or xB < 0 Or xC < 0 Then
Cells(i + 18, 3) = "Feed stage too low"
Exit For
End If
i=i+1
' Test for too many stages, which may mean reflux rate is too low.
If i > 100 Then
Cells(i + 18, 2).Value = "Too many stages"
Exit For
End If
' Top operating line
xA = yA / LoverV - ((1 / LoverV) - 1) * xAdist
xB = yB / LoverV - ((1 / LoverV) - 1) * xBdist
xC = yC / LoverV - ((1 / LoverV) - 1) * xCdist
' Test for calculations being done.
Loop While yA < xAdist
' Fractional recovery of C based on stage-by-stage calculation.
fracCbotcalc = 1 - (yC * D) / (F * zC)
difference = fracCbot - fracCbotcalc
If Abs(difference) < epsilon Then Exit For
fracCbot = fracCbot + df * (fracCbotcalc - fracCbot)
' Test if have convergence of fractional recovery of C.
Next j
Cells(i + 19, 1).Value = "Calc frac recovery C in bottoms"
Cells(i + 19, 5).Value = fracCbot
Cells(i + 19, 6).Value = "j"
Cells(i + 19, 7).Value = j
Cells(i + 18, 1).Value = "Distillate mole fracs = y values"
Cells(i + 18, 5).Value = yA
Cells(i + 18, 6).Value = yB
Cells(i + 18, 7).Value = yC
End Sub

144
Chapter 6

New Problems and new solutions are listed as new immediately after the solution number. These new
problems are: 6A1, 6A5, 6D3, 6D4, 6G4-6G7.

6.A1. New Problem in 3rd Edition. Trial and error. Try a feed stage and determine the distillate and
bottoms mole fractions of the key components. Repeat for additional feed stages. The feed stage
that produces the best separation is the optimum feed stage for this value of N.

6.A5. New Problem in 3rd Edition. Trial and error. Pick an N that you think is close (a systematic
method to do this is described in Chapter 7). Find the optimum feed stage. If you need more
separation to meet specifications, increase N, and if you exceed specifications, try decreasing N.
For an initial estimate of the optimum feed location for the new N, assume that the ratio
(Optimum feed stage)/(total number of stages) is constant. Continue process until specifications
are met or slightly exceeded.

6.C1. a. With a side stream, mass balance is,


Vj y j L j x j Sx S Vj 1 , y j 1 L j 1x j 1 Fjz j
where L j is the flow into the stage below, L j L j S, x S x j . Since L j x j Sx S L j x j , Eqs.
(6-4) to (6-6) are unchanged. Note that the L j 1
input into the matrix will be changed.
b. Now L jh j L jh j S jh j in the E.B. and
j j j 1 j 1
Lj Vj 1 D Fk Sk , L j 1 Vj D Fk Sk
k 1 k 1 k 1 k 1
Substituting into EB we find
j j 1
DEj Fj h Fj Qj D hj 1 hj FK h j FK h j 1
k 1 k 1
j 1 j
SK h j 1 SK h j
k 1 k 1

6.C2. Partial condenser mass balances is:


Dy1 L1x1 V2 y 2 F1z1 (6-7)
This becomes,
DK1 V2
1 1 K 2 2 F1z1
L1 L2
DK1 K 2 V2
Thus, B1 1 , C1 , and D1 F1z1 (6-8)
L1 L2
Note that only B1 differs.

6.D1. For n-pentane from Example 6-1, T = 60°C, K C5 1.05, L1 L2 825 kmole/hr,
L3 1825, L 4 450 B, V2 V3 V4 1375.
Matrix: j = 1 (total condenser),

145
K 2 V2 1.05 1375 D 550
C1 1.75 , B1 1 1 1.67, D1 0
L2 825 L1 825
K 3 V3 1.05 1375
j 2, C 2 0.791
L3 1825
V2 K 2
B2 1 2.75, A 2 1, D 2 0
L2
K 4 V4 1.05 1375
j 3, C 3 3.208
L4 450
V3 K 3
B3 1 1.791, A 3 1, D 3 Fz C51 1000 0.35 350
L3
1 V4 K 4
j 4 Reboiler , B 4 4.208, A 4 1, D 4 0
L4
1.67 1.75 0 0 1,C5 0
1 2.75 0.791 0 2,C5 0
0 1 1.791 3.208 3,C5 350
0 0 1 4.208 4,C5 0

Values for 1,C5 to 4,C5


are in Example 6-1.

6.D2. p = 5 atm: z C2 0.08, z C3 0.33, z C4 0.49, z C5 0.10


As sat’d liquid & for bp calculate z i x i . Want yi 1.0.
Pick C 3 as ref. 5 atm 101.3 kPa atm 506.5 kPa
st
1 Guess: Want K C3 1, K C4 1. DePriester Chart.
Try T = 20°C. K C2 5.4, K C3 1.7, K C4 0.47, K C5 0.14
yi 5.4 0.08 1.7 0.33 0.47 0.49 0.14 0.10 1.237
K C3 20 1.7
Need lower T. K C3 Tnew 1.37
Kixi 1.237
Tnew 12 , K C2 4.6, K C4 0.35, K C5 0.10
yi 0.368 0.4521 0.1715 0.0 1.0016 OK
Propane Matrix Analysis: K C3 1.37, B F D 1000 410 590 L6
D3 Fz c3 330, D1 D2 D4 D5 D6 0
V2 V3 V4 V5 V6 L1 D 1435
L1
L1 D 1025 L2 L3
D
L3 L 2 F 2025 L4 L5
Total Condenser (1):

146
B1 1 D L1 1.40, C1 K 2 V2 L 2 1.37 1435 1025 1.918
V2 K 2 V3 K 3 1435 1.37
Stage 2. A2 1, B2 1 =2.918, C 2 0.9708
L2 L3 2025
V3 K 3 V4 K 4 1435 1.37
Stage 3. A 3 1, B3 1 =1.9708, C 3 0.9708
L3 L4 2025
V4 K 4 V5 K 5
Stage 4: A4 1, B 4 1 =1.9708, C 4 0.9708
L4 L5
V5 K 5 V6 K 6 1435 1.37
Stage 5: A5 1, B5 1 =1.9708, C 5 3.32
L5 L6 590
V6 K 6
Reboiler (Stage 6). A 6 1, B6 1 =4.32
L6
Mass balance matrix.
1.40 -1.918 0 0 0 0
-1 2.918 -0.9708 0 0 0
0 1 1.9708 0.9708 0 0
0 0 1 1.9708 0.9708 0
0 0 0 -1 1.9708 -3.32
0 0 0 0 -1 4.32

6.D3. New Problem in 3rd Edition. p = 5 atm = 506.5 kPa


z C2 0.08, z C3 0.33, z C4 0.49, z C5 0.10
As sat’d liquid & for bp calculation at z i x i . Calculation is same as in 5.D11 to obtain T.
Result is: Tbp 12 , K C2 4.6, K C3 =1.37, K C4 0.35, K C5 0.10
n-butane Matrix Analysis: K C4 0.35, B F D 1000 410 590 L6
D3 Fz c3 490, D1 D2 D4 D5 D6 0
V2 V3 V4 V5 V6 L1 D 1435
L1
L1 D 1025 L 2 , L3 L 2 F 2025 L4 L5
D
Total Condenser (1):
B1 1 D L1 1.40, C1 K 2 V2 L 2 0.35 1435 1025 0.49
V2 K 2 V3 K 3 1435 0.35
Stage 2. A2 1, B 2 1 =1.49, C 2 0.2480
L2 L3 2025
V3 K 3 V4 K 4 1435 0.35
Stage3. A 3 1, B3 1 =1.2480, C 3 0.2480
L3 L4 2025
V4 K 4 V5 K 5
Stage 4. A4 1, B 4 1 =1.2480, C 4 0.2480
L4 L5

147
V5 K 5 V6 K 6 1435 0.35
Stage5: A 5 1, B5 1 =1.2480, C 5 0.8513
L5 L6 590
V6 K 6
Reboiler (Stage 6). A 6 1, B 6 1 =1.8513
L6
Mass balance matrix.
1.40 -0.49 0 0 0 0
-1 1.49 -0.248 0 0 0
0 1 1.248 0.248 0 0
0 0 1 1.248 0.248 0
0 0 0 -1 1.248 -0.851
0 0 0 0 -1 1.851

6.D4. New Problem in 3rd Edition. L L D D 5 60 300


V L D 360
Saturated liquid feed: V V 360; L L F 400

L1 L, L 2 L, L3 L, L 4 B F D 40
V1 0, V2 V, V3 V, V4 V
V1
2

L2 Bubble Pt. Set z i xi yi Ki xi


V3

3 F
yi 1.0 or Ki xi 1.0
V4 L3 M 3.58, E 2.17, NP 1, NB 0.412

xi i
yi
b. Eq. (5-30), xi i

zi i .3 3.58 .25 2.17 .35 1.0 0.1 .412 2.0077


z NP NP
0.35 1.0
Then y nP 0.1743
zi i 2.0077
0.1743
and K nP y x nP
0.4981
0.35

148
KM M NP K NP 1.7832; K E 1.0809, K n-B nB NP K NP 0.2052

c. Matrix for n-butanol K nB 0.2052, z nB 0.1


Stage 1. A1 , B1 1 D L1 1 0.2 1.2,

V2 K 2 360 0.2052
C1 0.2463, D1 0
L2 300

V2 K 2
Stage 2. A 2 1, B 2 1 1.2463
L2

V3 K 3 360 .2052
C2 0.1847, D 2 0
L3 400

V3 K 3
Stage 3. A 3 1, B3 1 1.1847
L3

V4 K 4 360 0.2052
C3 1.8468
L4 40

D3 F3z nBut 100 .1 10

V4 K 4
Stage 4. A 4 1 B4 1 2.8468, C 4 , D 4 0
B
1.2 0.2463 0 0 1 0
1 1.2463 0.1847 0 2 0
0 1 1.1847 1.8468 3 10
0 0 1 2.8468 4 0
.2463
d. y 1 V11 B1 1.2, V21 0, V31 .20525
1.2
j 2 V12 B2 A2 V3 1
1.2463 1 .20525 1.04105
V22 D2 A 2 V2 1
V12 0 1 0 0
0.1847
V32 C 2 V11 0.1539
1.2
j=3 V13 B3 A3 V3 2 1.1847 1 0.1539 1.0308
V23 D3 A3V22 V13 10 1 0 1.0308 9.7014
V33 C3 V12 1.8468 1.04105 1.7740

149
j=4 V14 B4 A4 V3 3
2.8468 1 1.7740 1.0728
V24 D4 A 4 V23 V14 0 1 9.7014 1.0728 9.0428
V34 C4 V13 not needed.
4,NB N V24 9.0428 (bottoms flow rate)

3NB V23 V33 4 9.7014 1.7740 9.0428 25.743

2 NB V22 V33 3 0 0.1539 25.743 3.9619

1NB V22 V31 2 0 .20525 3.9619 0.8132


VP VP in mmHg
nP
e. Raoult’s Law K nP
Ptot 760
VP K nP 760 0.4981 760 378.5 mmHg
Need to interpolate VP data. We know n VP 1T
n 200 5.2983, n 378.5 5.9362, n 400 5.99146
1 66.8 273.16 2.9415E 3 1 82.0+273.16 2.81563E 3
Linear Interpolation:
2.9415E 3 2.8163E 3
5.9362 5.2983 2.9415E 3 0.00284
5.2983 5.99146
TbP 273.16 351.74 or TbP 78.6 C.

6.F1. Plots of vapor pressure are available in Maxwell (see Table 2-2 for reference) while tabulated
values are in Perry’s K VPi p tot . Dew point calculation on feed gives 245.7°F.
Overall Mass Balances: D = 30, L = 5D = 150
V = L + D = 180, V V F 180 100 80 , L L 150 , B = F – D = 70
First Trial Values
Stage T L V KB KT Kx
4 245.7 70 = B 80 2.307 1.042 0.534
3 245.7 150 180 2.307 1.042 0.534
2 245.7 150 180 2.307 1.042 0.534
1 245.7 150 30 = D 2.307 1.042 0.534

Benzene
Stage C B A D ℓ
4 - 3.705 -1 0 7.9886
3 -2.705 3.8404 -1 35 29.5978
2 -2.8404 3.8404 -1 0 57.058
1 -2.8404 1.2 - 0 135.6569

Toluene
Stage C B A D ℓ
4 - 2.191 -1 0 27.1651
3 -1.191 2.2504 -1 40 59.5188
2 -1.2504 2.2504 -1 0 61.5875
1 -1.2504 1.2 - 0 64.1742
150
Xylenes
Stage C B A D ℓ
4 - 1.6103 -1 0 22.7361
3 -.6103 1.6408 -1 25 36.6120
2 -.6408 1.6408 - 0 21.1971
1 -.6408 1.2 - 0 11.3193

6.G1. Using Peng-Robinson. Aspen-Plus solution:

Stage T1°C L kmol/h V C4 C5 C8


1 38.31 825 0 x1 0.360 0.6013 0.0386
y1 0.6568 0.3424 0.00083
2 69.16 557.3 1375 x2 0.0993 0.4499 0.4508
y2 0.3601 0.6013 0.0386
3 107.02 1533.9 1107.3 x3 0.03355 0.1731 0.7934
y3 0.2288 0.5251 0.2461
4 140.92 450 1083.9 x4 0.00436 0.0429 0.9528
y4 0.04568 0.2271 0.7272

6.G2. 1. What VLE package did you use? Peng- Robinson.

2. Report the following values:


Temperature of condenser = - 2.77 oC
Temperature of reboiler = 79.97 oC
Distillate product mole fractions C2 0.3636, C3 0.6360, C4 0.0004
Bottoms product mole fractions C2 1.2 E 13, C3 0.000492, C4 0.9995
3. Was the specified feed stage the optimum feed stage? Yes No
If no, the feed stage should be: a. closer to the condenser, b. closer to the reboiler.

4. Which tray gives the largest column diameter (in meters) with sieve trays when one uses the
originally specified feed stage? Tray # 28 Diameter = 0.792 m.

5. Which components in the original problem are the key components? LK = Propane, HK =
butane

6. Change one specification in the operating conditions (keep original number of stages, feed
location, feed flow, feed composition, feed pressure, feed temperature/fraction vaporized
constant) to make ethane the light key and propane the heavy key.
What operating parameter did you change, and what is its new value? D = 20
Temperature of condenser = - 31.54 oC
Temperature of reboiler = 50.87 oC
Distillate product mole fractions C2 0.9955, C3 0.00448, C4 1.32 E 07
Bottoms product mole fractions C2 0.00112, C3 0.4364, C4 0.5625

151
6.G3. For column 1 report the following:
a. Final value of L/D 1.8
b. Split fractions of ethanol (distillate) 0.9999 and n-propanol (bottoms) 0.9913
c. Mole fractions in bottoms 1.70 E-5, 0.00871, 0.9913
d. Mole fractions in distillate 0.4545, 0.5383, 0.00714
For column 2:
a. Optimum feed location in the column. 18
b. Mole fractions in bottoms 0.00689, 0.9800, 0.0131
c. Mole fractions in distillate 0.9917, 0.0083, 0.0

6.G4. New Problem in 3rd Edition.


1. Temperature of condenser = 389.9_ K. Temperature of reboiler = __547.4 K
Qcondenser = _-772260____cal/sec, Qreboiler = _____912459__cal/sec
Distillate product mole fractions: B= 0.23529, T= 0.76471, BiP = 0.12E-08_________
Bottoms product mole fractions:_B = 0.5 E-10, T = 0.67 E-08, BiP= 1.0000_________

2. Was the specified feed stage the optimum feed stage? Yes No x
If no, the feed stage should be: a. closer to the condenser, b. closer to the reboiler. a__
(Note: Do minimum number of simulations to answer these questions. Do not optimize.)

3. Which tray gives the largest column diameter with sieve trays when one uses the originally
specified feed stage? Aspen Tray #__16______Column Diameter =______2.28____meters
[Use the default values for number of passes (1), tray spacing (0.6096 m), minimum downcomer
area (0.10), foaming factor (1), and over-design factor (1). Set the fractional approach to flooding
at 0.65. Use the “Fair” design method for flooding.]

4. Which components in the original problem are the key components (label light and heavy
keys) _____LK = toluene, HK = biphenyl_____________________________________________

5. Change one specification in the operating conditions (keep N, feed location, feed flow, feed
composition, feed pressure, feed temperature or fraction vaporized constant at original
conditions) to make benzene the light key and toluene the heavy key. Also increase the reflux
ratio to 4.0.
What operating parameter did you change (not including the reflux ratio), and what is its new
value? D = 40________
Temperature of condenser = _368.9____ K, Temperature of reboiler = 407.7____ K
Distillate product mole fractions: _B = 0.9283, T = 0.07173, BiP = 0.8 E-19________
Bottoms product mole fractions: _B = 0.01793, T = 0.79457, BiP = 0.1875_________

6.G.5. New Problem in 3rd Edition.


1. Temperature of condenser = _121.07___ K. Temperature of reboiler = _166.23___ K
Qcondenser = ____-757506.6____cal/sec, Qreboiler = ______1058466.75____cal/sec
Distillate product mole fractions:__B = 0.9779, T = 0.22070, pxy = 0.6004 E-05__
Bottoms product mole fractions:___B = 0.0055189, T = 0.55698, pxy = 0.43750___

2. Was the specified feed stage the optimum feed stage? Yes No x
If no, the feed stage should be: a. closer to the condenser, b. closer to the reboiler. a
(Note: Do minimum number of simulations to answer these questions. Do not optimize.)

3. Which tray gives the largest column diameter with sieve trays when one uses the originally
specified feed stage? Aspen Tray #____24______Column Diameter =______2.28___meters

152
[Use the default values for number of passes (1), tray spacing (0.6096 m), minimum downcomer
area (0.10), foaming factor (1), and over-design factor (1). Set the fractional approach to flooding
at 0.7. Use the “Fair” design method for flooding.]

4. Which components in the original problem are the key components (label light and heavy
keys) ________benzene = LK, toluene = HK______________________

5. Change one specification in the operating conditions (keep N, feed location, feed flow, feed
composition, feed pressure, feed temperature or fraction vaporized constant) to make toluene the
light key and p-xylene the heavy key.
What operating parameter did you change, and what is its new value?__D=260______
Temperature of condenser = _142.2____ K, Temperature of reboiler = _183.98__ K
Distillate product mole fractions: _B = 0.30769, T = 0.68850, Pxy = 0.003805________
Bottoms product mole fractions: __B= 0.3177 E-06, T = 0.007066, Pxy = 0.99293_____

6.G.6. New Problem in 3rd Edition. Part a. L D 27 . b. L D 60 . c. D = 147, S = 453 (liquid)


distillate mole fracs: E = 0.99007, B = 0.00993, P = 0.5 E-9
side stream mole fracs: E = 0.0009845, B = 0.98930, P = 0.000854
bottoms mole fracs: E = 0.7 E-14, B = 0.00043, P = 0.99957
d. distillate : E = 0.89146, B = 0.10854, P = 0.556 E-8
side: E = 0.041845, B = 0.95794, P = 0.000218
bottoms: E = 0.1 E-14, B = 0.0001095, P = 0.99989
Since vapor mole fraction ethane > liquid mole fraction (ethane is LK), have more ethane in
vapor side stream.
e. The separation of n-pentane and n-butane is much more difficult than between ethane and n-
butane. Thus side stream purity is less. Also feed has lot more pentane than ethane, which makes
side stream below feed less pure.

6.G.7. New Problem in 3rd edition.


1. Report the following values:
Temperature of condenser = _373.28____ K. Temperature of reboiler = ___411.75___ K
Qcondenser = _-829828_____cal/sec, Qreboiler = ____1012650_____cal/sec
Distillate product mole fractions: M = 0.59998, E = 0.36184, NP = 0.038177, NB = 0.3087E -05
Bottoms product mole fractions: M= 0.2042E-04, E = 0.03816, NP = 0.46182, NB = 0.50000_

2. Was the specified feed stage the optimum feed stage? Yes No X
If no, the feed stage should be: a. closer to the condenser, b. closer to the reboiler. Answer a
(Note: Do minimum number of simulations to answer these questions. Do not optimize.)

3. Which tray gives the largest column diameter with sieve trays when one uses the originally specified
feed stage? Aspen Tray #_____18_____Column Diameter =___1.77____meters
[Use the default values for number of passes (1), tray spacing (0.6096 m), minimum downcomer area
(0.10), foaming factor (1), and over-design factor (1). Set the fractional approach to flooding at 0.7. Use
the “Fair” design method for flooding.]

4. Which components in the original problem are the key components (label light and heavy keys)
______________LK = ethanol, HK = n-propanol_______________________

153
5. Change one specification in the operating conditions (keep N, feed location, feed flow, feed
composition, feed pressure, feed temperature or fraction vaporized constant) to make methanol the light
key and ethanol the heavy key.
What operating parameter did you change, and what is its new value?_____D = 60____
Temperature of condenser = __368.66__ K, Temperature of reboiler = _404.23___ K
Distillate product mole fractions: M = 0.97858, E = 0.021417, NP = 0.155 E-07, NB = 0.1 E-10_
Bottoms product mole fractions: M = 0.0091787, E = 0.27654, NP = 0.35714, NB = 0.35714__

154
Chapter 7

New Problems and new solutions are listed as new immediately after the solution number. These new
problems are: 7.A1, 7.A4, 7.D2, 7.D10, 7.D11, 7.D14, 7.D21, 7.G1.

7.A1. New problem in 3rd edition. f. none of the above.

7.A.4. New problem in 3rd edition. a. estimate fractional recoveries nonkeys at total reflux.

7.C4. Use yi, j 1 K i x i, j 1 . Then substituting into Eq. (7-20), we have


Vmin K i x i, j 1 L min x ij Dx i,dist
which is Vmin K i x i, j 1 L min x ij 1 Dx i,dist where K i i K HK . Rearranging,
Vmin K i K HK 1 Dx i,dist i
L min x i, j 1 Kx i,dist , or L min x i, j 1
L min Vmin K HK 1
L min i

c c
Dx i,dist i
Total flow rate L min is L min L min x ij 1
(A)
V K 1
i 1 i 1 min HK
L min i

Bx i ,bot i
By a similar analysis obtain, L min (B)
Vmin K HK 1
L min i

Vmin K1 Vmin K
Let L L
(C)
L min L min
Add Eqs. (A) and (B), and use external mass balance,
c Fz i
qF L feed L min L min i
(7-33 analogue)
i 1 1/ i L

c Dx i,dist i
From Eqs. (A) and (C) we have L min (7-29 analogue)
i 1 1/ i L

7.C5. For saturated vapor feed have VF F . For binary system Eq. (7-33) is,
z
1 1 2 z2
1
1 2
Clearing fractions we obtain
1 2 2 1 1 z 2 2 z 1

After some algebra this


2 z
1 1 2 2 z 1 2 0
Solutions are, 0 or 1 2 z
1 1 2 z2
For sat’d liq’d VF 0 . Clear fractions and equation is linear.

155
x 1 x dist .992 .008
n n
x 1 x bot
.014 .986
7.D1. a. Eq. (7-16), N min 10.36
n AB n 2.4
This includes the partial reboiler. Eq. (7-40a) gives,
x 1 x dist .992 .008
n n
z 1 z feed
.4 .6
N f ,min 5.97
n AB n 2.4
zB zT
b. Saturated liquid: Vf 0 . Eq. (7-33) becomes B T
0
B T

After clearing fractions and solving for ,

T zB 1.0 2.4
1.53846
B zB T zT 2.4 .4 1.0 .6
Which does lie between the α’s of the keys. To use Eq. (7-29) we need D. From mass
balances (Eq. (3-3)).
z x bot .4 .014
D F 10 3.9468 kg moles/hr
x dist x bot .992 .014
B Dx Bdist T Dx Tdist
Eq (7-29) is: Vmin .
B T

2.4 3.9468 .992 1.0 3.9468 .008


Vmin 10.848
2.4 1.53846 1 1.53846
L min Vmin D 6.9013 . L D min 1.75

L D L D
c. L D 1.1 L D min
1.9234 , x min
0.0598
L D 1
N N min
Using Eq. 7-42b, 0.5563
N 1
.5563 N min
Solving for N, N 24.6 (includes reboiler)
1 .5563
N F,min 5.97
From Eq. (7-40b), N F N 24.6 14.2
N min 10.36
Try stage 14 from top for feed stage.

7.D.2. New problem in 3rd edition. p 5 atm, z C2 0.08, z C3 0.33, z C4 0.49, z C5 0.10

Saturated liquid and for bp. Calc., z i x i . Want yi 1.0 .

kPa
Pick C 3 as reference (this is arbitrary). 5 atm 101.3 506.5 kPa
atm
1st Guess: Want K C3 1 (light key), K C4 1 (heavy key). Use DePriester Chart.

156
Try T 20 C , K C2 5.4 , K C3 1.7 , K C4 0.47 , K C5 0.14

yi 5.4 0.8 1.7 0.33 0.47 0.49 0.14 0.10

0.432 0.561 0.230 0.014 1.237


K C3 20 1.7
Need lower T. K C3 Tnew 1.37
Kixi 1.237

Tnew 12 , K C2 4.6, K C4 0.35, K C5 0.10


yi 0.368 0.4521 0.175 0.0 1.0016

For remainder, let C4 HK be reference. C3C4 1.37 0.35 3.914


0.997 0.998
n
0.003 0.002 12.01874
a) N MIN 8.808 (includes PR).
n 3.914 1.36456
b) C2 C4 4.6 0.35 13.143, C4 C4 1.0, C5 C4 0.1 0.35 0.286
Sat’d liquid feed VMIN VMIN . In Eq (7-33) divide through by F.

i zi 13.14 0.08 3.914 0.33 1.0 0.49 0.10 0.286


0
i 13.14 3.914 1.0 0.286
Solve for φ. Find φ = 1.74 (Note LK HK 3.914 HK HK )

i Dx i,dist
Eq (7-29) VMIN . Assume all C 2 in distillate & all C 5 in bottoms
i

Dx i,C2 80, Dx i,C5 0


Dx i C3
0.997 1000 0.33 329.01
Dx i C4
1 0.998 1000 0.49 0.98
D Dx i,d 409.99
13.14 80 329.01 3.914 0.98 1.0
VMIN 0 683.23 kmole h
13.14 1.74 3.914 1.74 1.0 1.74
L MIN VMIN D 273.24; L D MIN 0.6664
c) L MIN 1.15 L D MIN
0.7664
0.7664 0.6664
Ordinate Gilliland 0.05662
1.7664
N N MIN 0.002743
Eq (7-42b) 0.545827 0.591422 0.05662 0.56079
N 1 0.05662
0.5608 8.808
N 21.33 (include PR)
1 0.5608

157
Dx LK Dx HK dist 329.01 0.98
n n
z LK z HK 0.33 0.49
N F,MIN 4.552
n LK HK n3.914
N F,min NF .552
Eq. (7-40b) NF 21.33 11 (approximate)
N min N 8.808
xA xA
log
xB d
xB bot
7.D3. At total reflux use Fenske Eq. (7-11). N min
log AB

1 xA xA
Rearrange, log AB log
N min xB d
xB bot
1 N min
13
xA xA .545 .36
AB 1.287
xB d
xB bot
.455 .64
x 1 x dist .9915 .0085
n n
x 1 x bot
.01773 0.98227
7.D4. a.) N min 10.02
n AB n 2.4
b.) Feed is saturated liquid, feed line is vertical.

y* 0.9915 y*
(L/V)min =
0.9915 0.6
y
* z = .6 x
y* 0.7826
1 1 x x z 0.6

L L V
(L/V) min = 0.534, 1.144
D min 1 L V min

L D L D 2.2286 1.144
c.) Abscissa = min
0.336
L D 1 3.2286
From Eq. (7-42b)
N N min 0.002743
0.545827 0.591422 0.336 0.3553 ordinate
N 1 0.336
N min ordinate 10.022 0.3553
N 16.1
1 ordinate 1 0.3553
10.022 0.3474
From fitted curve ordinate = 0.3474, N 16.2
1 .3474
Error = 25-16 25 36% low. Aspen Plus equilibrium data is not α = 2.4. Note that α =
2.24 may be a better fit.

158
xD xB
log
1 xD 1 xB N min xD xB
7.D5. Fenske Eq. is: N min or
log 1 xD 1 xB
xD 1 xD
Solving for x B , we obtain: x B N min
xD 1 xD
Since N min 30, 1.30 and x D 0.984, this is x B 0.229

xD xB .993 .01
log log
1 xD 1 xB .007 .99
7.D6. Fenske Eq.: N min 10.82
log log 2.4
Determine y in equilibrium with feed z x 0.4 .
x 2.4 .4
y* .616
1 1 x 1 1.4 .4
L xD y* .993 .616 L L V
0.636 , 1.746 .
V min xD z .993 .4 D min 1 L V min

L
Then, 1.15 1.746 2.01
D act

L D L D 2.01 1.746 .264


Gilliland Correlation: Abscissa min
.0878
L D 1 3.01 3.01
N N min N 10.82
Ordinate 0.557
N 1 N 1
Clear fractions, and find N = 25.3 (including partial reboiler).

7.D7. p 5 atm. From the solution to problem 6.D9: Tbp 12 C


K C2 4.6, K C3 1.37, K C4 0.35, K C5 0.10
1.37
Let C 4 HK be reference. C3 C 4 3.914
0.35
0.98 0.992
n
0.02 0.008 8.7121
a) Eq. (7-15), Including PR N min 6.38
n 3.914 1.36456
b) C4 HK reference. C2-C4 4.6 0.35 13.143, C4 C4 1.0
C5 C4 0.10 0.35 0.286
Sat’d/liquid feed, Vmin Vmin
z
i i
13.14 0.08 3.914 0.33 1.0 .49 0.10 0.286
Eq. 7-33, 0
i 13.14 3.914 1.0 0.286
Want LK HK 3.914 1.0 HK HK . Converge to = 1.74.

159
Dx i C2
Fzi 80, Dx i C3
0.98 1000 0.33 323.4
Dx i C4
.008 1000 0.49 3.92, Dx i C5
0
13.14 80 323.4 3.914 3.92 1.0
Eq. (7-29), Vmin 0 671.05
13.14 1.74 3.914 1.74 1.0 1.74
D Dx i,d 407.32, Lmin Vmin D 263.73, L D min 0.647
c) L D 1.2 L D min
0.777 . Ordinate Gilliland 0.777 0.647 1.777 0.073
N N min 0.002743
Eq. (7-42b), 0.545827 0.591422 0.073 0.5402
N 1 0.073
0.5402 6.38
N 15.05, incl. PR. Nfeed ~ 9
1 0.5402
7.D8. a. Can do this graphically, or can calculate slope of a line from y x x D .992 to
intersection of feed line and equilibrium, or use Underwood. Easiest to calculate slope. Feed
line y z F .4 .
y .4
Equilibrium: x .3755
y 1 y .4 1.11 .6
L .992 .4 L L V
.958 , 22.83
V min .992 .3755 D min 1 L V
xD xB .992 .005
n
1 xD 1 xB .008 .995
b. N min 96.9
n n 1.11
This is 95.9 stages plus partial reboiler.
L D L D 27.4 22.83
c. L D 1.2 22.83 27.4 , Abscissa min
.161
L D 1 27.4 1
N N min
Ordinate .47 or N 181.9 which includes partial reboiler.
N 1
This separation would probably not be done by distillation.
LF Fz 8
7.D9. Feed 80% liquid, L F .8F, VF .2F. Feed line: y x , Slope 4
VF VF 2
L .8 0.386
a. See Graph. Min top op line is tangent. slope 0.5175
V min 0.8 0
L L V min 0.5175
1.0725
D min 1 L V min
1 .5175
3
b. N min 6 eq. contacts. See graph.
4
L
c. 1.05 1.0725 1.1262 . Abscissa, Gilliland Correlation is
D actual

160
L D L D min 1.1262 1.0725 0.053666
0.02524
L D 1 2.1262 2.1262
Ordinate ~ 0.63 from graph.
0.002743
From eq. (7-42b), Ordinate 0.545827 0.591422 0.02524 0.6396
0.02524
(agrees with graph).
N min ordinate 6.75 0.6396
Then N 20.5
1 ordinate 1 0.6396
Need 20 eqs. contacts + P.R.
N F,min 6
N F,min from graph = 6. N F N 20.5 18 (7-40b)
N min 6.75

7.D10. a) New problem in 3rd edition. Eq. (7-15)


FR E ,dist FR B,bot
n
1 FR E ,dist 1 FR B,bot
N MIN
n EB

161
0.989 0.998
n
0.011 0.002 10.7114
EB 13.14 N MIN 4.159
n 13.14 2.5756

b) N MIN 4.159 is known.. PB 3.91.


N MIN 4.159
PB 3.91
Eq. (7-17) FR P ,dist 0.3677
FR B,bot N MIN 0.998 4.159
PB
3.91
1 FR B,bot 0.002
c) Dx D i
FR i,dist Fzi
Ethane Dx DE 0.989 100 0.3 29.67
Propane Dx DP 0.3677 100 0.33 12.134
n-butane Dx DB 0.002 100 0.37 0.074
3 kmol
D Dx i,d 41.878
i 1 h
Also accept D = 0 since total reflux.
7.D.11. New problem in 3rd edition.

200

zn 0.35
2
z iP 0.4
z NP 0.25 V1
1

B
Use Underwood Eqns. – Case A Assume LNK (propane) is all in distillate. Dx p,dist Fz p 20
b) Vfeed F 1 q F since q 0

i F zi
Eq. (7-33). F 1 where φ is between α’s of two keys (B and H)
i
1.0 > φ > 0.2. Equation is,
2.04 0.2 1.0 .35 0.20 0.45
1.0
2.04 1.0 0.20
Solving for φ obtain φ = 0.62185.

162
Dx i,dist
Then Vmin . Find D from fractional recoveries.

Dx B,dist 0.99 100 .35 34.65


Dx p,dist 20
Dx H ,dist 1 0.98 100 .45 0.9
D 55.55

2.04 20 1.0 34.65 0.2 0.9


VMIN 119.98
2.04 0.62188 1.0 0.62188 0.2 0.62188
L min VMIN D 64.4314 and L D min 1.1599

7.D12. A = benzene (LK), AB 2.25, FR A,dist 0.98


B = toluene (HK), BB 1.0, FR B,bot 0.99
C = cumene (HNK), CB 0.210
FR A ,dist 1 FR B,bot
n
1 FR A ,dist FR B,bot
a. Use Fenske eqn. at total reflux. N min
n AB

0.98 0.01
n
0.02 0.99
N min 10.47
n 2.25
N min

AC AB 2.25
FR C,bot where AC 10.71
FR A ,dist N min
CB 0.21
AC
1 FR A ,dist
10.47
10.71
FR C,bot 1 . All cumene goes to bottoms.
0.98 10.47
10.71
0.02
(We can also substitute into Eq. (7-17)).
N min 10.47
AC
0.21 12
FR C,dist 8.1 10 0
FR B,bot N min .99 10.47
CB
0.21
1 FR B,bot .01
b. Underwood equations – Case B analysis
C Fz1
Feed is sat. vapor. q 0, Vfeed F 1 q F 100 , Vfeed i

i 1
1

2.25 40 1.0 30 0.21 30


100
2.25 1.0 0.21
Find 1.6516.

163
C i Dx i,dist
Vmin where Dx i,dist Fz1 FR i,dist
i 1
1

Dx A,dist 100 0.4 0.98 39.2, Dx B,dist 100 0.3 0.01 0.4, Dx C,dist 0, D 39.6
2.25 39.2 1.0 39.2
Vmin 0 146.78
2.25 1.6516 1 1.6516
L 107.2
From mass balance, L min Vmin D 146.78 39.6 107.2 , 2.71
D min 39.6
c. L D 1.25 L D min
1.25 2.71 3.39
L D L D
abscissa for Gilliland correlation min
0.155 . From Figure 7-3 the
L D 1
ordinate = 0.46. With N min 10.47, we find N = 20.24.
To find N feed , we need N feed
min

xA zA
n
xB dist
zB feed
N feed
min n AB

39.2
x A ,dist 0.9899, xB 1 .9899 0.0101
39.6
0.9899 0.4
n
0.0101 0.3
N feed 5.30.
min n 2.25
N feed N feed
min
gives N feed 10.25. Use stage 10 or 11.
N min N

.99 .02
7.D13. Use Fenske eq. FR T 1 FR T 99 , FR C 1 FR C 0.0204
.01 .98
log 99 0.0204 log 4851
N min 5.438
log TC log 1 0.21
2.5 .25 1.0 .30 0.21 45
Underwood: Vf V V 0, 0
2.5 1.0 21
3
1 Dx i,d
1.526 or 0.3374. Use 0.3374 as it is between keys. Vmin
i 1
1

Dx B,d Fz B 25 (assume all benzene in dist.)


Dx T,d .99 Fz T 29.7, Dx c,d 0.02 Fz c 0.9, D 55.6
2.5 25 1.0 29.7 .21 9
Vmin 72.24
2.5 0.3374 1.0 0.3374 0.21 0.3374
L min V D 16.64 and L D min 0.2993

164
N N min 9 5.438
Gilliland: Ordinate 0.3562
N 1 10
Abscissa ~ .29 (original Gilliland) or .36 (Liddle)
L L L
If use 0.29 have, 0.29 0.29
D D min D
L .29 0.2993
.83
D 1 .29
L .36 .2993
If use Abscissa = 0.36, 1.03 which are quite different. Safer to use
D 1 .36
higher value.
If BT 2.25, N min is same. Underwood Eq. gives 1.4666 or 0.3367. Use 0.3367.
Vmin 29.40 44.78 1.492 72.68 , L min V D 17.084 and L D min 0.3073
Which is 2.7% different than for BT 2.5.

7.D14. New problem in 3rd edition. Use Gilliland correlation to find the value of the minimum reflux
ratio, (L/D)min = 1.4
FR B,dist FR C,bot .9992 .9999
n n
1 FR B,dist 1 FR C,bot .0008 .0001
7-D15. Fenske: Eq. (7-15), N min 6.89
n 2.25
BC n
.21
N min
TC 1
Eq. (7-17), FR T ,dist where TC
FR C ,bot N min .21
TC
1 FR C ,bot
6.89
1
.21
FR tol,dist 6.89
0.8238, and FR tol,bot 0.1762
.9999 1
.0001 .21
x dist
Dx d ,tol 0.8238 167 137.57
0.2568
Dx d ,benz 0.9992 397 396.68 0.7418
.0436
Dx d ,cum 0.0001 436 0.0008
D 534.294
x LK z LK .7418 .397
n n
x HK dist
z HK .000815 .0436
N F,min 1.94 .
n LK-HK n 2.25 .21
Underwood: Use a Case C analysis since toluene is a sandwich key.
3
1Fz1
2.25 397 1.0 167 .21 436
Eq. (7-33): 0 VF is, 0
i 1
1 2.25 1.00 .21
1.216 and 0.3373 which lie between α’s.

165
Eq. (7-29):
3
i Dx i,dist 2.25 396.88 1.0 Dx tol .21 0.436
Vmin becomes Vmin
i 1
i 2.25 1.0 .21
Write for 1.216 and for 0.3373. Obtain 2 eqns and 2 unknowns: Vmin and Dx tol,dist .
For 1.216 , Vmin 863.525 4.629 Dx tol . For
.3373, Vmin 466.15 1.509 Dx tol .
Solving simultaneously, Vmin 563.84, Dx tol 64.740
D Dx i,d 396.88 64.74 .436 462.056
L min Vmin D 101.79 and L D min
0.2203
L D L D min 1.2 .2203
Gilliland Abscissa 0.445
L D 1 2.2
N N min
Ord. .245 (Original Gilliland)
N 1
Obtain N = 9.45 (includes reboiler)
N F,min 2.91
Estimate N F N 9.45 2.66 (Use stage 3)
N min 6.89

x x .99 .992
n n
1 x dist 1 x bot .01 .008
7.D16. a. Fenske: N min 12.69
n n 2.1
Fz E 1.0 F z p
b. Underwood: Vfeed EP
, zE .6, z P .4, V feed V V F
EP 1.0
2.1 .6 1.0 4
1 , 0 or 1.44 . Use 1.44 which is between 1.0 and 2.1
2.1 1.0
z xB .6 .008
To find D: D F 1000 602.85 . Then
xD xB .99 .008
Vmin 2.1 .99 1.0 .008
3.132
D 2.1 1.44 1.00 1.44
L
Vmin 1888.12, L min Vmin D 1285.3, 2.13
D min

N N min .30 12.69


c. Use Gilliland: Ordinate 0.558
N 1 31
L D L D min
Abscissa ~ 0.8 (Original Gilliland), L D 2.4
L D 1

VF i zi
7.D17. Sounds harder than it is: 0 f , tol-xy 3.03, -
xy xy 1
F i

166
z tol xy z xy
Expand & Solve for , 0 tol

tol xy

xy tol tol
Result is linear, sin ce xy 1
tol z tol xy z xy tol z tol 1 z tol

i Dx i,dist z xB z .008
Then Vmin , D F 100
i
xD xB 0.988
3.03 .996 1.0 .004
Vmin D , L min Vmin D
3.03 1

z tol D Vmin L min L


D min
0.1 3.03 9.3117 3.03 .996 .004 45.625 4.8997
2.51870 54.93685
3.03 .1 .9 3.03 2.51870 1 2.51870
0.3 3.03 29.5547 3.03 .996 .004 48.0836 1.6269
1.88316 D 77.6383
3.03 .3 .7 3.03 1.88316 1 1.88316
0.5 3.03 49.7976 3.03 .996 .004 48.2707 0.96934
1.503722 D 98.0683
3.03 .5 .5 3.03 1.503722 1 1.50372
.7 3.03 70.0405 3.03 .996 .004 47.6985 0.68101
1.25155 D 117.739
3.03 .7 .3 3.03 1.25155 1 1.25155
.9 3.03 90.2834 3.03 .996 .004 38.7990 0.429
1.071806 D 129.08
3.03 .9 .1 3.03 1.071808 1 1.071808

Check for z = 0.5.

xD Slope L V min

y* L xD y*
where
V min x D .5
y xF
y* 0.75186
.5 = z 1 1 xF

0.996 0.75186
x 0.4922
0.996 0.5

L L L V .4922
0.96934 Perfect
D min V L 1 L V 1 4922
As z , although L D min , Vmin , thus Qc min
Vmin as expected.
Q R,min also.

167
FR A ,dist FR B,bot
n
1 FR A ,dist 1 FR B,bot
7.D18.a. Fenske: Eq. (7-15): N min
n AB

Where A = propane, B = butane, AB 1/ .49 2.04 . (Note value α.)


.9854 .8791
n
.0146 .1209
N min 8.7
n 2.04
For N F,min assume no LNK is bot and no HNK in distillate
D = .229 + (.9854) (.368) + (.1209) (.322) = .631
.9854 .368 .1209 .322
x prop 0.575, x C4 0.0617
.631 .631
x C3 z C3 .575 .368
n n
x C4 dist
zC4 F .0617 .322
N F,min 2.94
n C3-C4 0.713
1 Fz1
Underwood Eqns. (Case A.) 0 Vfeed for 1.0 .49
1

9.92 .229 1.0 .368 .49 322 .081 0.10


0 f
9.92 1.0 .49 .10
9.92 .229 1.0 .363 .49 .0389
Find 0.6213 , Vmin 1.057
9.92 .6213 1.0 .6213 .49 .6213
L min Vmin D 1.057 .631 .426, L D min 0.676
L / D (L / D) min
Gilliland Correlation (Fig.7-3): abscissa 0.33
L/D 1
N N min
Ordinate .32 (Original Gilliland, ~.36 Liddle). Find N = 13.24 (14.13 Liddle)
N 1
b. With N = 20, ordinate to Gilliland correlation is,
N N min 20 8.7
0.538
N 1 21
Abscissa = 0.1. Since L D min 0.676, solve for L/D = 0.862.
N min
C6 C3
c. FR C 6 dist
FR C 3 bot N min
C6C3
1 FR C 3 bot

where C6 C3 0.10, FR C3bot 1 FR C3dist 0.0156, and N min 8.7.


8.7
0.10
FR C 6 0.00000013 , FR C6 0.99999987
dist
0.0156 8.7
bot
0.10
1 0.0156

168
For all practical purposes all C6 in bottoms at total reflux.

d. FR C3dist 0.999, L / D 1.5, FR B,bot 0.8791


FR C3dist FR C 4 bot .999 .8791
n n
1 FR C3dist 1 FR C 4bot .001 .1209
1) N min 12.47
n C3C 4 0.713

2) For D assume all LNK in dist, No HNK in dist

D = 0.299 + (0.999) (0.368) + (0.1209) (0.322) = 0.6356

9.92 .229 1.0 .368 .49 .332 .081 .1


Now f 0
9.92 1 .49 .1

Which is same [φ depends only on feed & α’s]. Thus, same φ = 0.6213

9.92 0.229 1.0 0.368 0.49 4689 ??


Vmin 1.0709
9.92 .6213 1 .6213 .49 .6213

L min 1.0709 0.6356 0.4345 L D min 0.6848. Very little change.

N N min 25 11
7.D19. Use Figure 7-3. Ordinate 0.5385
N 1 26
L D L D
Then Abscissa 0.08 with L D 2.2286 . L D
min
min
1.97
L D 1
Abscissa approximated between original & fitted curves.

7.D20. a) Distillate Dx Bdist Fz B 5, Dx Tdist Fz T 15 ,


5 15
x dist 1.0 becomes 0.57895 0.07018 1.0
D D

Find D = 57.001 kmoles/hr, B = 100 – D = 42.999


xA xB dist
n
xA xB bot
A xylene
b) Can use Fenske eq. (7-11) or alternatives. N min
n AB
B cumene
K xy K xy K tol xy 0.330
AB xylene cumene 1.57143
K cum K cum K tol cum 0.210
Xylene balance, Fz 35 57.00 0.57895 42.999 x x,bot
x x,bot 0.0465, x cum,bot 1 .0465 0.9535

169
0.57895 0.07018
n
0.04650 0.9534
N min 11.35
n 1.57143
This is # equil contacts at total reflux.
Dx x ,dist 57.001 0.57895
c) Alternative: FR xy,dist 0.94288
Fz x 35
Dx C,dist 57.001 0.07018
1 FR C,bot 0.088896 , FR cum,bot 0.91110
Fz C 45
FR A dist FR B,bot
n
1 FR A dist 1 FR B,bot A xylene
Use Eq. 7-15. N min ,
n AB
B cumene
0.94288 0.91110
n
0.057122 0.088896
N min 11.35
n 1.57143

7.D21. New problem in 3rd edition. Assume all ethanol in distillate and all n-butanol in bottoms.

Dx E ,dist Fz E 100 .3 30
Dx i P,dist Fz ip Frac Rec iP dist 100 .25 .986 24.65
Dx n P,dist Fz nP 1 Frac Rec nP dist 100 .35 .008 0.28
Dx n B,dist 0 0

D Dx i,dist 54.93

xE,dist = xE,dist/D = 0.5461, xi-P,dist = xi-P,dist/D = 0.4488, xn-P,dist = xn-P,dist/D = 0.0051

Bx E ,bot 0 0
Bx i P,bot Fz iP 1-Frac Rec iP dist 100 .25 .014 0.35
Bx n P,bot Fz nP Frac Rec nP bot 100 .35 0.992 34.72
Bx n B,bot Fz n B 100 0.10 10.0

B = 45.07
x i,bot Bx i,bot B
xi-P,bot = Bxi-P,bot /B = 0.0108, xn-P,bot = Bxn-P,bot /B = 0.7704, xn-B,bot = Bxn-B,bot /B = 0.2188

170
FR ip,dist FR nP ,bot
n
1 FR iP ,dist 1 FR nP ,bot
b. Fenske eq. (7-15) N Min
n iP nP

.986 .992
n
.014 0.008 n 8733 9.0748
N min 14.62
n 1.86 0.62058 0.62058
This includes PR
x iP z iP 0.4488 0.25
n n
x nP dist
z nP 0.0051 0.35
N F,MIN Eq. 7 40a , N F,MIN 7.76
n iP-NP 0.62058

c. Underwood Equation: Assume NKs do NOT distribute: Case A.


i Fz i
Eq. (7-33) Vfeed
i

z
i i
For saturated vapor Vfeed F divide (7-33) by F. 1 , which becomes
i

E nP zE iP nP z iP nP NP z NP NB NP z NB
1
ENP iP NP NP NP NB NP

3.58 0.3 1.86 0.25 1 0.35 0.412 0.10


1
3.58 1.86 1 0.412
Solve for φ between α values of keys. LK = i-propanol, HK=n-propanol. Thus, want
1.0 1.86 . From Goal Seek on spreadsheet 1.48648
i Dx i,dist
Then from Eq. (7-29) Vmin where Dx i,dist values from part a.
i

3.58 30 1.86 24.65 1.0 0.28


VMIN 0 173.47
3.58 1.48648 1.86 1.48648 1.0 1.48648
L MIN VMIN D 173.47 54.93 118.54 , L D MIN 118.54 54.93 2.16
L
d. L D 1.1
D Min

L D MIN 1
1 1
L D L D MIN L D 1.1
Gilliland abscissa, x
L D 1 1 1
1 1
L D 1.1 L D MIN
L D
1
L D MIN 1.1 1
or x 0.0683
L D 1 1
1.1
L D MIN L D MIN 2.16

171
N N MIN
From Eq. 7-42b, 0.5456
N 1
N 1 0.5456 0.5456 N MIN 33.4 N includes PR

NF N F,MIN 7.76
Assuming NF 33.4 17.7 or Stage 18 below total condenser.
N N MIN 14.62

7.F2 Equilibrium data is available in a variety of sources such as Perry’s Handbook. Data used
here is from Perry’s (3rd ed.), p. 574.
a) Need to obtain avg. α from equilibrium data.
yN2 1 x N2 0.1397 0.9615
x 0.0385, y 0.1397, 4.055
1 y N2 x N2 0.8603 0.0385
x N2 0.4783, y 0.7893 (needed for part b)
.9770 .0810
x N2 .9190, y N2 .9770, = 3.744
.0230 .9191
1/ 2
avg top&bot 4.01 3.744 3.875

x 1 x dist .998 .002


n n
x 1 x bot
.001 .999
Fenske N min 9.685
n AB n 3.875
xD z
b) L V min
where x* is in equilibrium with feed y z 0.79
xD x *
From equilibrium data x* ~ 0.48.
.998 .79 L L V min 0.40
L V min 0.40 , 0.66667
.998 .48 D min 1 L V min 1 0.40
c) L / D 1.1 L/D min
0.7333
L D L D 0.06666
Gilliland Correlation: abscissa min
0.0385
1 L D 1.7333

N N min
Original correlation, ordinate 0.6 N = 25.7 including PR. Need 25 equil. Stages
N 1
7.G.1. New problem in 3rd edition. a. At total reflux N MIN 9
b. L D MIN
0.92

172
Chapter 8

New Problems and new solutions are listed as new immediately after the solution number. These new
problems are: 8.A1, 8.A2, 8A7, 8A12, 8.D1, 8D6, 8D12, 8.D13, 8D15, 8D17, 8D20, 8.D22, 8D23 to
8.D25, 8.E1, 8G1-8G5, 8.H3.

8.A1. New Problem in 3rd edition. a. 2-pressure distillation

8.A2. New Problem in 3rd edition. b. extractive distillation

8.A7. New Problem in 3rd edition. If there are volatile and non-volatile organics, a single equilibrium
contact gives an organic layer that contains no non-volatiles. Extra stages do not increase the
separation. If there is entrainment, a second stage may be useful.

8A.12. New Problem in 3rd edition. Steam distillation is normally operated with 2 liquid phases in the
still pot and in the settler after the condenser. There is usually no reflux. Azeotropic
distillation is normally operated with one liquid phase in the column and in the reboiler, but
with 2 liquid phases in the condenser and settler. One of the liquid phases is refluxed to the
azeotropic column.

y org x org p org x org H org x org x org H org VP org x org in org
8.C2. org w in water in w in w
yw x w pw x w VP w
xw xw VP w
VP w
x org in w

At solubility pt. x w in w .975 and x org in w .025, x org in org .573


Vapor pressures (Perry & Green, 1984). N-butanol:
T = 70.1°C 84.3°C l00.8°C 117.5°C
VP = 100 mm Hg 200 400 760 mm Hg

Water: T = 100.8°C, VP = 782 mm Hg, T = 84.3, VP ~ 421.8 mm Hg


VPorg 200 VPorg 400
at 84.3°C: 0.474 , at 100.8: 0.5115
VPw 421.8 VPw 782
For org w
use average between 92 and 100°C. Can linearly interpolate at T = 96°C,
VPorg / VPw 0.501 , org w in w 0.501 .573 .025 11.483
w org in w 1 org w in w 1 11.483 0.0871
w o xw 0.0871x w
From y w generate equilibrium curve,
1 w o 1 xw 1 0.9129x w
xw 1.0 .995 .990 .985 .980 .975
yw 1.0 .9495 0.8961 0.8512 0.8102 0.7726
At constant x w , the calculated y w y w ,exp eri min tal . Difference at x B 0.975 is
approximately .7726 .752 .752 100 2.74% .

These equations work better for mixtures which are more completely immiscible.

173
8.D1. New Problem in 3rd edition. Top Op. Eq., y L V x 1 L V xD, xD .975
L L D 4 L
.8 , y intercept x 0 1 xD .2 .975 .195
V 1 L D 5 V
xB 0.11 (from diagram). Need 2 equil. Stages.

Graph for problem 8.D1.

8.D2. The columns are sketched in the Solution to Problem 8-C2. B1 is butanol phase and B 2 is water
product. Two equilibrium diagrams are shown.
a. F B1 B2 , Fz B1x B 1 B2 x B2

174
z x B2 .28 .995
B1 3743.45 , B2 F B1 1256.55
x B1 x B2 .04 .995
L L
b. Col. 1. Bottom Op: y x 1 x B1 , Intersects y x x B1 0.04 .
V 1
V
q .7 7
Feed: 70% Liquid, q = .7, . Intersects y = x = z = .28.
q 1 .3 3
L L
Top y x 1 x B2
V V
L 0.995 .436 L
From Figure 8-D2a: 0.562 , 1.23 .562 0.69
V min 0.995 0 V
L
y intercept x 0 1 x B2 0.307
V

Note that reflux is x 0 0.573 . Optimum feed stage = 3. Need about 3 stages + partial reboiler.
L L
Stripper (Column 2): y x2 1 x B2
V 2
V

L V B V B 1 1.132
2
8.57
V 2
V V B .132
2
Construction is shown in Figure 8-D2b. Need 1 2/3 eq. contacts or P.R. + 2/3 equil. stage.

175
8.D3.

a) y = 0.4, x = 0.09 from graph. B) V/F = 0.3

L F V 1 V/F .7 7
20 = Fz = Vy + Lx
V V V/F .3 3
L F 7 .99
100 = F = V + L y x z x
V V 3 0.3
20 = 0.4 V + 0.09 (100 – V) See Enlarged figure [Be careful with scales]
V 11 .31 35.48 kg moles / h y w 0.969, x w 0.999
L = 64.52 kg moles/h
F zw x w 100 0.99 0.999 kg moles/hr
V = 30
yw xw 0.969 0.999 hr
Use Table 8-2 to find V, Tdrum @ y w 0.40, xw 0.09 , L = 70 and Tdrum ~ 108 C

176
8.D4. Compositions x 0.573 , x 0.975 [Shown on Figure 8-D3a]
100 F L L and 88 = Fz = L x L x
F z x 100 .88 0.573
L = 76.37 kmoles/hr, L 23.63 kmoles/hr
x x .975 0.573
8.D5. a) Water conc. W is 0.975.
200 = F = W + B W = 200 – B
Water balance: (200) (.8) = Fz = W(0.975) + B (0.04)
Solve: B = 37.433 kmol/hr, W = 162.567 kmol/hr
V B V B 1 5 L L
b) L V , Bot. Op. Eq. y x 1 xB .
V V B 4 V V
Goes through y x xB

177
1 0.25 .04
Plot operating line: If y = 1, x 0.808
1.25
PR + 2 stages mores than sufficient. (see graph)
0.75 0.04
c) L V Slope 1.332
min
0.573 0.04
V 1 1
L V 3.012
min
L V L V 1 0.332

178
8.D6. New Problem in 3rd Edition.

F
D, x dist

Reflux

V L

B, x bot

8.D.6 Part a) F D B
F .65 .975D .02B
z xB .65 .02
D F 100 65.969
xD xB .975 .02
b) V V B B 136.124, Reflux L L V V 170.155.
Or V F D L 136.124 100 65.969 170.155
L V B V B 1 4 1
c) 1.25
V V V B 4
L L
y x 1 xB
V V
Goes through y x xB

Calculate arbitrary point at x .6


y 1.25 .6 .25 .02 .745
See Figure: Need 2 stages + PR

V V 1
d) What is V B ? 3.0894 On graph.
MIN B MIN
L V L V 1
MAX

179
180
y W x W in organic yA 0.0364, y W 1 0.0364 0.9636
8.D7. W A
in organic y A x A in organic x A in organic 0.628, x W in organic =1-0.628=0.372

0.9636 0.372
W A 44.69
in organic 0.0364 0.628

8.D8. Convert wt frac to mole frac.


MW C8 H14 O 72 14 16 102 and MW water 18

Basis 1000 kg
0.994 wt frac. ether: 6 kg W = .333 kg moles
994 kg E = 9.745
Total = 10.078
x W in organic 0.033

0.012 wt frac. ether: 988 kg W = 54.889 kg moles


12 kg E = 0.118 kg moles
x W in organic .998

y = 0.959 ether: 41 kg W = 2.278 kg moles


959 kg E = 9.402 kg moles
y W 0.155
y W x W in org .195 .805
W - E in org 7.026
yE x E .033 .967

z(wt) = 0.004 water: 4 kg W = 0.222 kg moles


996 kg E = 9.765 kg moles
z W 0.022

x 7.026 0.022
y in equil w feed 0.138
1 1 x 1 6.026 0.022

L xD y equil w feed .998 .138


0.882
V min xD z .998 .022

L L V min L L
7.467; 11.20; 0.918
D min 1 L V min
D act V act

Generate following equilibrium data using w E in org 7.026 :

xW 0 0.01 .022 .033


yW 0 0.066 0.137 0.195

181
L L
Top Op. line: y x 1 xD
V V
Where L/V = 0.918, x D .998 , and y = intercept = (1 - .918) (.998) = 0.082

Bottom: From y x x B to intersection of feed line and top operating line.


xB 0.0004 wt frac.: 0.4 kg W = 0.022 kg moles
999.6 kg E = 9.800 kg mole
Obtain x B 0.0023
3
See plot in Figure: Optimum feed is top stage. Need 4 equil. Contacts.
5

8.D9. Convert to Mole fractions:


MW C6 H14 O 72 14 16 102; MWwater 18

Basis for all conversions is1000 kg soln.


Top Layer Separator = 0.994 wt frac. ether
6 kg W = 0.333 kmole
994 kg ether = 9.745 kmol
Total = 10.078 kmol
0.333
Mole frac. x w in org 0.033
10.078

182
xD 0.998

0.033

Bottom Layer separator is 988 kg W = 54.889 kmol

= 0.012 wt frac. ether 12 kg E = 0.118 kmol

Mole frac. x W in org 0.998 xD


z = 0.02
41 kg W = 2.278 kmol
Vapor into Condenser is
959 kg E = 9.402 kmol
yazeotrope = 0.959 wt frac. ether
Mole frac. y W 0.195

y W x W in org 0.195 0.033


W E 7.026
in organ phase y E x E in org 1 .195 1 0.033
Feed is y W 0.02 mole frac water = z. In equil. With feed:
in org

y 0.02
x *f 0.002896
1 y 7.026 6.026 0.02
xD z 0.998 0.02 L L V
L V min *
0.9828 , 57.18 ,
xD x f 0.988 0.002896 D min 1 L V
L D
L D act
2 L D min 114.36 , L V act
0.9913
1 L D
Top Op. y L V x 1 L / V x D,W through y x x D,W 0.998 .
When x = 0, y = 0.00868
7.026x
Eq. Data. y . Generate curve,
1 6.026x

x 0 0.01 0.022 0.033


y 0 0.066 0.137 0.195
Plot on graph, and plot top op. line:
x 0, y 0.00868. x 0.04, y 0.9913 .04 0.00868 0.04833
x 0.03, y .9913 0.03 0.00868
x W ,bot is at intersection y = z = 0.0208 top op. line, x W,bot 0.0123
Step off stages from top down. 1 equil stage is sufficient.
But with this very high reflux rate consider alternatives.

183
8.D10. a. VP C10
x C10 VP W
xW 760
Assume the water layer is pure, x W 1.0. Try 95.5°C, VPC10 60, VPW 645.7.
(.99) (60) = 645.7 = 705.1. Too low. Try higher temperature. The attached plot of VP
vs. T allows estimation of vapor pressure. (Note: a plot of log (VP) vs 1/T will be easier
to interpolate and extrapolate.)
97.0°C: VPC10 63, VPW 682.07 , (.99) (63) + 682.07 = 744.4
97.5°C: VPC10 65, VPW 694.57 , (.99) (65) + 694.57 = 758.9
T = 97.6 gives VPW 697.1 which will be too high. Thus T = 97.5°C is close enough.

184
nw p tot VPorg x org 760 65 .99
b. 10.81
n org VPorg x org 65 .99
This is significantly less than in Example 8-2 where 296.8/4.12 = 72.04 mol decane are
used. Difference is due to higher n-decane concentration in liquid.

8-D11. x F,org 0.9, 95% recovery → 5% left. Octanol left = 0.05 (.9) (1.0) = 0.045 kmol/h
Nonvolatiles in bottoms = 0.10 kmol/h

octanol

water

0.045
x oc tan ol 0.3103
in org 1.10 0.045
xF

steam

185
B
a) Water VP can be fit to log10 VP A over short ranges T. T in C, VP is mmHg.
273.16 T
B
T = 95.5 log 0 645.67 A 2.8100 A B 368.66 (1)
273.16 95.5
B
T = 100 log10 760 A 2.8808 A B 373.16 (2)
273.16 100

To solve for A and B, subtract 1 from 2


B B
0.07080 0.00003271B B = 2164.42
368.66 373.16
B
A 2.8808 8.68105
373.16
Now find T for which p tot VPW x W VPO x O 760 mm Hg where x W 1.0, x O 0.3103
On Spread Sheet find T = 99.782°C
VP O x O 19.075
b) y O 0.025098 0.3103 0.007788
p tot 760
754.072
yW x 1.0 0.99220
760
a) Moles octanol = F z O .95 1.0 0.90 0.95 0.855 kmol/h
yW 0.9922
b) Moles water nW n org 0.855 108.93
y org 0.007788
0.855
Check Eq. (8-18): n W 760 19.075 0.3103 108.93
19.075 0.3103

x P,B2 x P,dist1
8.D12. New Problem in 3rd edition. All cases B1 60, B2 40 , D 2 B2
x P,dist1 x P,dist 2

1.0 .65
a) D2 40 140
.65 .55
D1 140 40 180 . Total feed Col 1 = 246
.35
b) D1 280 1400
.01
D1 1440. Total feed Col 1 = 1500

186
8.D13. New Problem in 3rd edition.

F1
D, x dist

V L
F2

V L

B, x bot

Part a. F1 F2 D B 100 80 D B
Water:
F1z1 F2 z 2Dx dist Bx bot 100 .84 80 .20 D 0.975 B 0.04
Solve simultaneously, D = 99.25 and B = 80.75
V
b) V B 121.125, L V B 201.875
B
Since feed 2 is saturated liquid V V 121.125
L L F2 121.875
c) Doing Mass balance around top Doing Mass balance around bottom
V y F1z1 Lx Dx dist V y Bx bot Lx F2 z 2
L Dx dist F1z1 L F2 z 2 Bx bot
y y x
V V V V

These two equations are equivalent.


L L
d) Bot. op. line: y x 1 x B . Goes through y x xB .
V V
L V B V B 1
Slope 5 3 . Plot Bot Op. line.
V V V B
L 121.875
Op. line above feed 2: Slope 1.0062
V 121.125
At intersection F2 feed line and bot op. line (at x .2, y 0.306667 ) with
slope 1.0062
2 stages + PR is more than sufficient (See graph).

187
Graph for 8.D13.

8.D14. Figure is on next page.

Part b. F B1 B2 , Fz B1x EB1 B2 x E


B2

z x B2 0.85 0.006
B1 F 100 85.60 , B2 F B1 14.40 kmol/h
x B1 x B2 0.992 0.006
z x b1 x b2 x d1 0.85 0.992 0.006 0.449
c) D2 F 100 21.196
x b2 x b1 x d1 xd2 0.006 0.992 0.449 0.75
D1 D2 B2 35.596 kmol/h

188
D2
D1
xE 0.75

101.3 kPa
1333
F
0.449 kPa
zE 0.85
ethanol

Water
Ethanol
99.4 mole % water
99.2 mole %
B1

8.D.15. Part a) New Problem in 3rd Edition.


p org VPoc tan ol x oc tan ol =Ptotal where x oc tan ol is mole fraction octanol in organic phase.
At 0.05 atm and boiling T, porg 0.05 atm. 38 mmHg
From Antoine equation,
1310.62
log10 VP oc tan ol
6.8379
T 136.05
At T 129.8C, VPoctanol 80.905 mmHg
p org 38
Since p org 38 mmHg, x octanol,mole 0.470
VPbenzene 80.905
Average mole wt solids and non volatile organics can be calculated. Basis 100 kg
mol octanol 15 130.23
0.470
mol octanol mol non-volatiles 15 85
130.23 MW
15 85 15
0.470 MW 654.04
130.23 MW 130.23
b) 95% recovery is true on both mass and mole basis.
Distillate octanol flow rate 0.95 100 0.15 14.25 kg h.
Since MWoc tan ol 130.23, this is 14.25 130.23 0.109 kmol h.
In waste there are 0.05 15 0.75 kg hr octanol and 85 kg h (organics + solids), or
85.75 kg h total. Wt frac octanol 0.75 85.75 0.00875.
0.75 130.23
Mole frac. octanol in waste 0.0424
0.75 130.23 85 654.04

189
c) For equilibrium in still pot VP oct
x oct in org VPw 1.0 p tot 760 mmHg .
The still pot is perfectly mixed; thus x oct in org x oct in waste 0.0424 mole frac.
Since water boils at 100°C when P 760 mmHg, T < 100°C.
Eq. (8-15) becomes VPoct 0.0424 VPw 1.0 760
Substituting in the Antoine equations for octanol and water and solving with a spread sheet, T =
99.97°C.
VPoct 19.27 mmHg and VPw 759.18 mmHg.
n oct VPoct x oct
d) From Eq. (8-18),
nw p tot VPoct x oct
From spread sheet n oct n w 0.001076
Since n oct 0.109 kmol h, n w n oct 0.001076 101.27 kmol h
kmol 18.016 kg
water 101.27 1824.5 kg h water in distillate.
kmol kmol
This is a lot of steam!

8.D16. Distillate 1: 0.997 EtOH, 0.0002 solvent. Calculate x d1,W 1 .9972 0.0028
Distillate 2: 0.999 water, 0.00035 solvent. Calculate x D2 E 1 0.99935 0.00065
F = 100, x F,E 0.81, sat'd liq'd, x F,solv 0
Find D1 , D 2 , M where Makeup is pure solvent.
0
Water: x W M Fz W D1x D1W D 2 x D2W
0
Ethanol: Mx E,M Fz E D1x d1,E D 2 x d 2,E

Ethylene Glycol: Mx Esolv 1.0 Fz solv D1x d1,solv Dx d 2,solv

Solving water & ethanol balances obtain: D 2 18.7913 and D1 81.2316 kmol/h.
From Ethylene Glycol balance,
M 81.2316 0.0002 18.7913 0.00035 0.02282 kmol/h
Can also use overall balance instead of EG bal. Then
M D1 D 2 F 18.7913 81.2316 100 0.2290 , OK

8.D17. New Problem in 3rd edition. Since everything now exits the bottoms, B = S + F, and x A =
FzA/(S+F), xB = FzB/(S+F), xsolvent = S/(S+F).

8.D18. Ethanol Product: 0.997E, 0.0002 solvent, 0.0028 water

Water Product: 0.9990W, 0.00035 solvent, 0.00065 ethanol

F 100, z E 0.20 z W 0.80

190
Water bal: x WM M Fz W PE x EP,W PW x WP,W (A)

E bal: x EM M Fz E PE x EP,E PW x WP,E (B)


0
EG x M,solvent M Fz solv PE x EP,solv PW x WP,solv where x M,solvent 1.0

Solve A & B for PE & PW : PW 80.0240 kmol/h, PE 20.0074 kmol/h

M 80.0240 0.00035 20.0074 0.0002 0.03201 kmoles/h

8.D19. M.B. around System. Since everything in wt. units do M.B. in wts.

Overall: F H B1 B2 where H = makeup hexane.

Ethanol: Fz E 0 B1x E,bot 2 B2 x E,bot 2


0
Hexane: Fz H H 1.0 B1x H,bot1 B2 0

Solving simultaneously, B1 8000.04, B2 2000.04 and H 0.08 kg/h.

8.D20. New Problem in 3rd edition. a. 1000 F E W


1000 0.8094 Ex E,Ethanol prod Wx E,wprod
809.4 0.998E 0.0001W
808.3
E 811.0 kmol h
0.9979
W F E 1000 811 189.0

Fx WF Ex W ,Ethanol prod 1000 0.1906 811.0 0.002


b) V V 629.93
y w ,1 0.300
629.93
boilup ratio V E Pr od 0.777
811.0

L V E Pr od. 629.93 811.0 1440.93


L L F sat 'd liquid feed 1440.93 1000 440.93
If CMO strictly valid then, L reflux 440.93

Can also estimate L from _ settler Pentane flow rate in V1 Ethanol flow rate in V1
Ethanol lost in Water Product.

Pentane flow rate y P,1V1 629.93 0.6455 406.62

191
Ethanol flow rate in V1 y E,1V1 629.93 0.0555 34.96
E in V1 E lost
x E,reflux
L from settler to Col1

Ethanol lost in water product W prod x E in Wprod 189.0 .0001 0.0189

Lfrom _ settler _ calculation 441.6

Match not perfect because: 1. CMO not totally valid


2. There is some water in reflux
3. K dE value may be incorrect.

c. Ethanol returned to distillation column


V1 y E1 WPr od x E,W Pr od 34.96 0.0189 34.19 kmol h.
440.93 441.6
Using average estimate for L 0 441.3
2
Then 34.91 441.3 x E in pentane
x E,pentane 0.0792 x E,Re flux,pantane _ layer

Then since assume K d 1, x E,Water layer 0.0792

192
d. x E,water 0.0792 V
V1 0.5 V 0.5W 94.5 kmol h.
W
L W V 283.5 kmol h.
1
L 283.5
3
V 94.5
L 0.0792 W .0001
y E1
V

283.5 .0792 189.0 .0001


y E1 0.237
94.5
W 189.0
x E,W Pr od 0.0001

8.D21.

193
L V B 1 1.5 L L
Bottom: 3, y 1 xB .Goes through y
x x x B with slope = 3
V V B 0.5 V V
Feed line = Horizontal (q = 0). Through y = x = z = 0.4
Top. MB: yV Lx Dx D and V L D
L L
y x 1
x D goes through y x x D 0.975
V V
Intersects Feed line where bottom op line does.
Opt. Feed #1 above reboiler. 3 equilibrium stages + PR is sufficient.

8.D22. New Problem in 3rd edition.

D, x dist

Reflux

V L

F B

Part a. F D B & Fz Dx dist Bx bot


z x bot 0.20 0.08
D F 100 13.41 kmol/h and B 86.59
x dist x bot 0.975 0.08

Part b. yV Lx Dx dist
L D L L
y x x dist Substitute in D V L to obtain y x 1 x dist
V V V V
L
Points on operating line: y x x dist 0.975 and x 0, y intercept 1 x dist
V
Alternative point is at feed line (y = z = 0.2) & x x bot 0.08
V F, L B, thus slope L V B F 86.59 100 0.8659
y intercept 1 0.8659 0.975 0.1307
Part c. Need 2 Stages. See graph.
Part d. Pinch at feed line intersection with equilibrium is at x 0.02 .

194
Figure for problem 8D22.

195
8.D23. New Problem in 3rd edition.

Water phase XNM = 0.086


N.M. Phase

xw = 0.312 W F
NM

Water
Nitro Methane Product
Product
Part a. External balances F NM Pr od WPr od

NM : F .25 NM Pr od .98 WPr od .01


.25 .01
NM Pr od 100 24.74, WProd 75.26
.98 .01

a. W Column: z = .25, horizontal feed line


L L
Top y NM x NM 1 x NM ,bot ,col NM Mass balance through top of W column
V V
and around col. NM. Can easily show that

196
y NM x NM x NM,bot,col NM
But do not know L/V so cannot plot yet.
L L
Bottom operating line looks familiar: y NM x 1 x NM ,bot col w
V V
y NM x NM x NM,bot,col w 0.01
L V B 1 54
5
V col w
V B 14
Can plot bottom operating line. Arbitrary point: x 0.2, y 5 0.2 .01 .96
Now can plot top operating line from intersection of bottom operating line and feed line to point
y NM x NM x NM,bot,col w 0.01
See graph. Need PR + ~ 1 2 stage. Build PR + 1 stage.
b. NM Column is a stripping column: yw L V xw L V 1 x w ,bot ,col NM

V
1
B 4
L V
col NM V 3
B
yw xw x w,bot,NM _ col 0.02
4 1
To plot, xw .3, y w .3 .02 0.3933 xw 0.3 is arbitrary point
3 3
1
Need PR + ~ 1 stages.
3
V 1 1
c. W col. Want V. V 100 V, V= B WProd75.26 18.81
B 4 4
V to cond. from Wcol 118.81 kmol hr
V
NM col want V. V= B 3 NM Pr od 3 24.74 74.22 kmol hr . To condenser.
B

197
Graph for 8.D23.

198
8.D24. New Problem in 3rd edition. From Equilibrium, y bu tan ol 0.092 at x bu tan ol 0.004
Overall Mass Balance: 100 F V B
Butanol MB: 100 0.025 V .092 B .004
2.5 .092 V F V .004
2.1 .088V V 23.864, B 76.136 kmol hr
This problem can also be solved graphically, but using basic mass balances is easier.

8.D25. New Problem in 3rd edition. Part a) porg VPbenzene x benzene where x benzene is
mole fraction benzene in organic. At boiling T, p org 1.0 atm.
1211.033
From Antoine equation, log10 VP benzene
6.90565
T 220.790
At T 93 C, VPbenzene 1112.44 mmHg . Since
p org 760
p org 760, x ben,mole 0.683
VPbenzene 1112.44
Average mole wt solids and non volatile organics can be calculated. Basis 100 kg

Moles benzene 20 78.11


0.683
Moles benzene + Moles non-volatiles 20 80
78.11 MW
20 80 20 80 .683 20
0.683 1 .683 MW 673.2
78.11 MW 78.11 MW 78.11

b) 90% recovery is true on both mass and mole basis.


Distillate benzene flow rate 0.9 100 0.2 18.0 kg h .
Since MWbenzene 78.11, this is 18 78.11 0.230 kmol h
In waste there are 2.0 kg/h benzene and 80 kg/h (organics + solids), or 82 kg/h total.
Wt frac benzene 2 82 0.0244
2 78.11
Mole frac. benzene 0.1773
2 78.11 80 673.2
c) For equilibrium in still pot VP b
x b in org VPw 1.0 p tot 760 mmHg
The still pot is perfectly mixed; thus, x b in org x b in waste 0.1773 mole frac.
Since water boils at 100ºC when P 760 mmHg, T 100 C . Benzene is more volatile and
boils at 80.1ºC, but mole fracs low.
2164.42
Antoine equation for water: log10 VPw 8.68105
273.16 T
Eq (8-15) becomes VPb 0.1773 VPw 1.0 760
Substituting in the Antoine equations for benzene and water and solving with a spread sheet,
T 92.0411 C . VPben 1082.5 mmHg and VPw 568.1 mmHg.

199
n ben VPben x ben
d) From Eq. (8-18),
nw p tot VPben x ben
From spread sheet n ben n w 0.337876
Since n ben 0.230 kmol hr, n w n ben 0.337876 0.6807 kmol hr

kmol 18.016 kg
water 0.6807 12.264 kg h water in distillate.
kmol kmol
n ben ben
e. To vaporize benzene condense moles water .
w

This occurs at 92.0411 C 365.1911K; From Perry’s table 2-237,

H hg h hf H h kJ kg

T = 360 886.7 498.7 388


T = 370 898.6 518.1 380.5

5.1911 kJ
Linear interpolate x 380.5 388 388 384.1
30, 002 kJ kmol.
10 kg
Note: 8th edition, Table 2-193 is very slightly different after unit conversion.
T = 360 2663 384.7 2278.3
Water Table 2-352. T = 370 2671 405.88 2265.67

0.0911
w 2265.67 2278.3 2278.3 2277.8 kJ kg 41,037 kJ kmol
5
0.230 30, 002
Moles water condensed 0.1682 kmol h water (in waste)
41037
kg h water in waste 0.1682 18.016 3.029 kg h

200
8.E.1. New Problem in 3rd edition.

Water phase XNM = 0.086


N.M. Phase

xw = 0.312 W F1
F2 NM

Water
Nitro Methane Product
Product

Part a. 250 F1 F2 PNM PW


NM balance F1z1NM F2 z 2 NM PMN x NM Prod Pw x w Pr od
NM mol frac NM mol frac

135.5 8 127.5 PNM .98 Pw 0.01


135.5 PNM 250 PNM 0.01 .97PNM 2.5
PNM 133 .97 137.11 kmol hr
Pw 250 137.11 112.89
b. Column W – Use y NM vs. x NM (water phase) plot.
F2 z NM 2
Top operating line y NM L V water x NM 1 L V wcol
x NM
col NM Pr od Vwcol

Bottom Operating Line y NM L V water x NM L V water 1 x NM


col col Water Pr od

V
V B V B 1 43 V B col w 37.63 V
L V 4. B .
V V B 13
L V B col w 150.52 L F1

201
Top IS NOT from y x x NM 0.98 to intersection feed and bottom operating line.
NM Pr od

Instead from intersection of feed and bottom operating line with slope
L V L F1 V 50.52 37.63 1.32 25 .
Optimum feed is top stage. Need PR + 1 stage.

F1z1,NM
c. Column NM. Top y L V NM xw 1 L V NM x w
col col W,W Prod. V
Bottom yw L V xw L V 1 xw
NMcol NMcol
in WM Prod.

L V B 1
yw xw xw 0.02. 2.00
NM Pr od V V B
Draw bottom operating line. Top is through intersection bottom operating line and feed line F 2 .
Slope L V 0.906 (see item d). Need 2 stages + PR. Optimum feed is stage above PR.

d. Column W: PW Bcol w 112.89


V V B Bcol w 1 3 112.89 37.63
L V Bcol w 150.52. L L F 50.52.
kmol
Saturated liquid feed: V V 37.63
hr.
Column NM: V B 1.0, PNM Bcol NM 137.11
V B 1
L V 2.0
V B
V V B B 137.11 kmol hr, L 2.0 137.11 274.22
Saturated liquid feed V V 137.11, L L 150 124.22 , L V 0.906.
e. Minimum boilup rate NM column gives combination bottom & top operating lines to go through
reflux point: y W 0.5, x w 0.312 .
From bottom operating line intersection with feed line x z2 0.15 is
L L
y INTER z2
1 x W ,NM Prod
V V
Slope of top operating line to reflux point is
L 0.5 y INTER L F2
V 0.312 0.15 V

V 1
V B . V V B B 137.11 V B . L V B
L V L V 1
L F2
Guess V B Calc V & L & L V Calc y int er Calc L V
V
Check is two calculated values L V are same.

202
Spreadsheet. V B 0.6105, L V 0.846
MIN

Graph for Problem 8.E1.

203
8.E2. Balances at mixing point for F & R.

To Butanol Column:
Overall: FT F R 100 R
Water: FT z T,W Fz W Rx W,reflux FT z T,W 30 0.573R
R .573 30
z T,W
100 R
External balances: 100 = W + B, water: 30 = 0.995W + 0.02 B
Solve simultaneously: W = 28.72 & B = 71.28 kmol/h

Butanol Col: V B 1.90, V 1.90B 135.432


FT L V B 206.712, L V 1.5263
y x x W,butonal 0.02
R FT F 206.712 100 106.712
106.712 0.573 30
zT 0.4409
206.712
Vertical feed line at z T intersects bot. operating Line at y = 0.67 (see graph)

V B V B 1
Water Col. V B 0.1143, L V 9.748
V V B

y L V x L V 1 x B,water,watercol, y x x B,W 0.995


watercol

See graph, y leaving column = 0.8

204
205
8.E3. Basis: 1000 kg sea water (1 h):
965 kg water kmol 18.016 kg 53.5635 kmol, x F,W 0.98894
0.5988
35 kg NaCl kmol 58.45 kg kmol, x F,salt 0.011056
Total 54.1623
Water Condensate = (0.60) (53.5635) = 32.1381 kmol/h = n W
Water Remaining 53.5635 – 32.1381 = 21.4254
21.4254 kg moles W
Waste Water is 0.9728 mole frac. water
21.4254+0.5988 salt

a. In still, organic phase is pure decane.


VPC10 VPW x W p tot 760 mmHg where x W 0.9728 .
Try T = 99°C. VPC10 ~ 68, VPW 733.2 mm Hg
68 + (0.9728) (433.2) = 781.27 mm Hg, which is too high.
Converge to T ~ 98.2°C.
nW yW pW VPW x W 707.27 0.9728
b. Distillate: 10.4225
n org y org p org VPorg x org 66 1.0
(This calculation is at 98°C, not 98.2, but will be close.)

206
32.1381 kmol water/h kmol
n C10 3.0819 C10 in distillate
10.425 mol water/mol organic h

8.F1. V.P. Data n-nonane (p. 3-59 Perry & Green, 1984)

VP = 20 40 60 100 200 400 760 mm Hg


T = 51.2 66 75.5 88.1 107.5 128.2 150.8°C

See Solution problem 8.D10 for plot. MWnC9 128.25 . From p. 3-268 (Perry & Green,
1984), nonane enthalpies are,
T hliquid Hgas
360 K 671.3 KJ/kg 998.2
380 K 722.5 1036.5
Water VP is given in Problem 8-D10 and on p. 3-45 of Perry and Green (1984).

a. Try 95.0°C. VPC9 127 mm Hg, VPW 633.9 . Assume water is pure.
Pressure: (.99) (127) + (1.0) 633.9 = 759.6. Close enough and lucky!

nW p tot VPorg x org 760 127 .99


b. 5.045 mol water/mol nonane
n org VPorg x org 127 .99

c. Need to calculate the energy required to vaporize the nonane. T = 273 + 95 = 368 K.
By linear interpolation for pure nonane: h1 ~ 691.78, Hgas ~ 1013.52. nonane 321.74 KJ/kg

Table 3-302 of Perry and Green (1984): h liq,W 397.36, H vap W 2667.8, W 2270.44 KJ/kg
mol water condensed C9
321.74 KJ/kg 128.25 kg/kmol
1.009
mol C9 vaporized W 2270.44 KJ/kg 18.016 kg/kmol
b. Now, VPC9 .020 VPW 760. Temperature will be higher.
Try 99°C: VPC9 149 mm Hg, VPW 733.24
(149) (.020) + 733.24 = 736.22 which is too low.

Try 99.9ºC: VPC9 154, VPW 757.29


154 (.020) + 7570.29 = 760.37. Close enough.

The low nonane conc. reduces nonane partial pressure and operation is much closer to 100°C.
nW p tot VPorg x org 760 154 .02
245.75
n org VPorg x org 154 .02

Need lot more steam!

207
8.F2.

n-nonane

water
F = 1000
95% n-nonane
organic

waste

steam
water

Basis: 1 hour
All junk in feed (0.50 kmol) is in bottoms
a)
Organic Bottoms is 0.95 n-C9 0.5 junk 1.45 (see part C)
.95
x C9,bot ,org 0.65517
1.45
b) Still T. p W p org p tot 102.633 kPa 770 mm Hg
pW VPW (T)x W where x W 1
porg K C9 T x c9,org,bot K C9 T 0.65517 770 mm Hg 504.481 K C9 T
Procedure: Guess T, determine VPW & K C8 check if pressure eq. is valid
VP W
504.481K C9 770 mm Hg?
Try T = 96°C VPW 657.62, K C9
0.16 (DePriester Chart)
652.62 80.717 738.34 Need higher T
T = 97°C, VPW 682.07 and K C9 0.17
682.07 + 85.762 = 767.83 slightly low, but close enough

c) 9.50 kmol n-nonane × .90 = 8.55 kmol n-nonane in distillate. 9.5 – 8.55 = 0.95 kmol n-C9 in
bottoms
n org p org 85.762 n org 8.55
d) Eq. (8-18) 0.12567, n W 68.031 kmol water
nW p W 682.07 0.12567 0.12567

e) EB simplies to W n W,condensed nonane n org,dist. where λ’s are at 97°C = 370 K.


C9 C9
n W ,condensed n org,dist. 8.55 kmol
W W

208
From Perry’s 6th ed. Table 3-268 or 7th ed. Table 2-292, nonane λ’s are:
h g h f , @ 360 K, 998.2 671.3 326.9 kJ/kg
380 K, 1036.5 722.5 314.0 kJ/kg
326.9 314
@ 370 K, λ ≈ 320.45 kJ/kg
2
MW C9 128.258 . Then at 370 K,
kJ 128.258 kg
C9 320.45 41,100.3 kJ/kmol
kg kmol
Water 370 K: (Perry’s 6th Ed., Table 3-302).
kJ 18.016 kg kJ
W 2671 405.8 2265.2 40,809.84
kg kmol kg
41,100.3
n W ,cond 8.55 8.611 kmol water
40,809.84

8.G.1. New Problem in 3rd Edition.


1. Final makeup solvent flow rate _________0.02__________ kmol/h.
2. Final value solvent recycle rate (B2) __1400___kmol/h and L/D in col 1 _0.100_.
3. Final values of flow rates D1 _140.0_, B1 _1460.02_, and D2 __60.02___ kmol/h.
4. Mole fractions in stream D1 _Pyr=0.0084259, W=0.99157, Bisphenol=.49E-10_
5. Mole fractions in stream D2 _Pyr = 0.98001, W = 0.019654, Bisphen = .000333___
6. Mole fractions in stream B1 _Pyr=0.040287, W= 0.0008079, Bisphen = 0.95890_
7. Mole fractions in stream B2 (solvent recycle stream) Pyr = .526E-8, W = .2E-12, Bisphenol =
1.0000__
8. Heat load in cooler on solvent recycle line__-0.15216E8___ cal/s.

8.G2. New Problem in 3rd edition. Aspen Plus Residue Plot 4.0 atm using NRTL

Pressure can have major effect on VLE for non-ideal systems. Compare T-xy diagrams for
acetone MEK at 1.0 and 4.0 atm.

Also compare residue curves for acetone-MEK-MIBK at 1.0 & 4.0 atm.

209
210
211
8.G3. New Problem in 3rd edition.
a. Final reflux ratio column 1___0.01_ and final reflux ratio column 2 _0.01_______. If these values are
not 0.01 you are not finished with Part B.
b. Flow rates furfural product ___166.0___ kmol/h and water product ___34.0__ kmol/h.
c. Boilup rate in column 2 _____8.0________ kmol/h.
d. Mole fraction furfural in furfural product ____0.99816___& mole fraction water in water product
___0.99102____.
e. Flow rate of distillate from column 1 _____42.10_____ kmol/h.
f. Column 1 condenser temperature __370.3___K, & column 1 reboiler temp. __433.59__ K.
g. Outlet temperature of decanter ____375.2______ K.
h. Molar ratio of water phase/total liquid in decanter _____0.8393_____

8.G.4. New Problem in 3rd edition.


Column 1: a. Bottoms product mole fraction acetonitrile______0.99915______
b. Distillate flow rate ___240____ kmol/h and bottoms flow rate ___170____kmol/h.
c. Distillate mole fraction acetonitrile ____0.67910___________ .

Column 2: a. Distillate flow rate ___210_______ kmol/h, and reflux ratio ___1.2____.
b. Bottoms product mole fraction water______0.99517______________ .
c. Distillate mole fraction acetonitrile _____.77542__________ .

8G5. New Problem in 3rd edition.


Results are residue curves and profiles of mole fraction vs plate location. For an equal molar feed, N = 10
does not give the desired purity even if L/D = 10. N = 50 does work with L/D = 2, but not for L/D = 1.0.

8.H1. Part b. Was 8.D12 in 2nd edition of SPE. Use Eq. (8-25b) with
AB 2.4, BB 1.0, BC 0.21.
A = benzene, B = toluene, C = cumene. Results from Spreadsheet:
Stage: Reboiler: x A 0.0003 x B 0.0097 x C 0.990
1 0.003298 0.04443 0.95227
2 0.03137 0.176085 0.79255
3 0.18019 0.42145 0.39836
4 0.46126 0.44952 0.08923
5 0.70274 0.28536 0.01189
6 0.85421 0.14453 0.001265
7 0.93403 0.06585 0.000121
8 0.97145 0.028535 1.102 E-5
9 0.98791 0.012091 9.802 E-7
10 0.99493 0.005074 8.637 E-8
11 0.99788 0.00212 7.580 E-9
12 0.99912 0.000885 6.641 E-10

8.H2. Was 8.D13 in 2nd edition of SPE. Use a spreadsheet with Eq. (8-30) as recursion equation.
Result is shown in Figure. The VBA program was given in Example 8-3. The results obtained
for the starting conditions given are:
k xA xB xC
1 0.990 0.001 0.009
100 0.9763 0.0017 0.0220
200 0.9431 0.0029 0.0534
300 0.8630 0.0049 0.1331

212
400 0.6740 0.0077 0.3183
450 0.5044 0.0089 0.4867
475 0.3946 0.0092 0.5962
500 0.2696 0.0089 0.7214
600 0.00042 0.00095 0.9986

Results for other starting conditions are shown in the figure.

213
Figure for problem 8H2.

8H.3. New Problem in 3rd edition. The spread sheet including the first 10 time steps and time steps 600
to 610, and the VBA program are listed.
Part a

214
Simple distillation calc (residue curves) with BP calcs.
aT1 aT6 ap1
iB -1166846 7.72668 -0.92213
nB -1280557 7.94986 -0.96455
iP -1481583 7.58071 -0.93159
nP -1524891 7.33129 -0.89143
nhex -1778901 6.96783 -0.84634

Residue curve calc. h 0.01 N 1000 epsilon 1E-09


x1iB 0.98 x1nB 0.01 x1iP 0 x1nP 0.01
sumx 1 T1guess,R 500 p,psia 14.7 x1nHex 0
time step xiB xnB xiP xnP x nHex TR
1 0.98 0.01 0 0.01 0 472.0604
2 0.979883 0.010032 0 0.010085 0 472.0642
3 0.979764 0.010065 0 0.010171 0 472.068
4 0.979645 0.010097 0 0.010258 0 472.0718
5 0.979525 0.01013 0 0.010345 0 472.0757
6 0.979405 0.010162 0 0.010433 0 472.0797
7 0.979283 0.010195 0 0.010522 0 472.0836
8 0.979161 0.010228 0 0.010612 0 472.0876
9 0.979037 0.010261 0 0.010702 0 472.0916
10 0.978913 0.010294 0 0.010793 0 472.0956

600 0.071054 0.009754 0 0.919192 0 542.3988


601 0.069204 0.009586 0 0.92121 0 542.71
602 0.067391 0.009418 0 0.923191 0 543.0165
603 0.065615 0.009253 0 0.925133 0 543.3183
604 0.063874 0.009089 0 0.927037 0 543.6155
605 0.06217 0.008927 0 0.928903 0 543.9078
606 0.060502 0.008766 0 0.930732 0 544.1955
607 0.058869 0.008608 0 0.932524 0 544.4783
608 0.057271 0.008451 0 0.934279 0 544.7563
609 0.055708 0.008296 0 0.935997 0 545.0295
610 0.054179 0.008142 0 0.937678 0 545.2978

Option Explicit
Sub Residue_Curve_BPcalc()
' K value data for nbutane,ibutane, ipentane,npentane and nhexane included.
' Only want 3 for residue curve. Thus, set x values = 0 for 2 components.
' The reference component is nbutane.
Dim i, N, j As Integer
Dim h, epsilon, xiB, xnB, xiP, xnP, xnHex As Double
Dim T, p, aT1iB, aT6iB, ap1iB, aT1nB, aT6nB, Ap1nB As Double
Dim aT1iP, aT6iP, ap1iP, aT1nP, aT6nP, ap1nP, aT1nHex, aT6nHex, ap1nHex As Double
Dim KiB, KnB, KiP, KnP, KnHex, Ksum, chksum, inside As Double
Dim yiB, ynB, yiP, ynP, ynHex As Double

Sheets("Sheet1").Select

215
Range("A15", "G1045").Clear
aT1iB = Cells(5, 2).Value
aT6iB = Cells(5, 3).Value
ap1iB = Cells(5, 4).Value
aT1nB = Cells(6, 2).Value
aT6nB = Cells(6, 3).Value
Ap1nB = Cells(6, 4).Value
aT1iP = Cells(7, 2).Value
aT6iP = Cells(7, 3).Value
ap1iP = Cells(7, 4).Value
aT1nP = Cells(8, 2).Value
aT6nP = Cells(8, 3).Value
ap1nP = Cells(8, 4).Value
aT1nHex = Cells(9, 2).Value
aT6nHex = Cells(9, 3).Value
ap1nHex = Cells(9, 4).Value
h = Cells(11, 4).Value
N = Cells(11, 6).Value
epsilon = Cells(11, 8).Value
xiB = Cells(12, 2).Value
xnB = Cells(12, 4).Value
xiP = Cells(12, 6).Value
xnP = Cells(12, 8).Value
xnHex = Cells(13, 8).Value
T = Cells(13, 4).Value
p = Cells(13, 6).Value
For i = 1 To N
j=i+1
Do
KiB = Exp((aT1iB / (T * T)) + aT6iB + (ap1iB * Log(p)))
KnB = Exp((aT1nB / (T * T)) + aT6nB + (Ap1nB * Log(p)))
KiP = Exp((aT1iP / (T * T)) + aT6iP + (ap1iP * Log(p)))
KnP = Exp((aT1nP / (T * T)) + aT6nP + (ap1nP * Log(p)))
KnHex = Exp((aT1nHex / (T * T)) + aT6nHex + (ap1nHex * Log(p)))
Ksum = KiB * xiB + KnB * xnB + KiP * xiP + KnP * xnP + KnHex * xnHex
KnB = KnB / Ksum
inside = aT1nB / (Log(KnB) - aT6nB - (Ap1nB * Log(p)))
T = Sqr(inside)
chksum = Ksum - 1
Loop While Abs(chksum) > epsilon
Cells(13 + i + 1, 1).Value = i
Cells(13 + i + 1, 2).Value = xiB
Cells(13 + i + 1, 3).Value = xnB
Cells(13 + i + 1, 4).Value = xiP
Cells(13 + i + 1, 5).Value = xnP
Cells(13 + i + 1, 6).Value = xnHex
Cells(13 + i + 1, 7).Value = T
yiB = xiB * KiB

216
ynB = xnB * KnB
yiP = xiP * KiP
ynP = xnP * KnP
ynHex = xnHex * KnHex
xiB = xiB + (h * (xiB - yiB))
xnB = xnB + (h * (xnB - ynB))
xiP = xiP + (h * (xiP - yiP))
xnP = xnP + (h * (xnP - ynP))
xnHex = xnHex + (h * (xnHex - ynHex))
Next i
End Sub

217
Chapter 9

New Problems and new solutions are listed as new immediately after the solution number. These new
problems are: 9.A4, 9.A5, 9C1, 9D1, 9.D5, 9.D8, 9D11, 9D13, 9D18, 9D19, 9D21, 9D22, 9D25, 9E2, 9.H1.

9.A4. New Problem in 3rd edition. Answer is g.

9.A5. New Problem in 3rd edition. Answer is c.

9.A6. b. The same.

9.B1.
Multi Stage Single Stage
F, x F , x Davg , N, L / D, Treflux , P F, x F , x Davg , P
F, x F , x Wf , N, L / D, Tr , P F, x F , x D tot , P
F, x F , Wfinal , N, L / D, Tr , P F, x F , x wf , P
F, x F , D tot , N, L / D, Tr , P F, x Wf , Wfinal , P
F, x F , x Davg , x Wf , N, Tr , P x F , x Davg , D tot , P
F, x F , x Davg , x Wf , L / D, Tr , P x F , x Wf , Wfinal , P
x F , x Davg , D tot , N, L / D, Tr , P x F , x Wf , D tot , P
x F , x Davg , D tot , x Wf , N, Tr , P F, x F , x Davg , x W
x F , x Davg , D tot , x Wf , L / D, Tr , P etc.
x F , x Davg , Wfinal , N, L / D, Tr , P
x F , x Wfinal , Wfinal , N, L D, Tr , P
etc.

9.B2. a. Replace the column with one containing more trays or more packing.
b. Retray or repack existing column.
c. Run a batch in several steps. For example, take the feed and operate so that the desired bottoms
concentration is met. Collect all the distillate and use this as the feed for a second batch. Operate
so that the distillate for this run meets specifications. The bottoms from this run can be used as
feed for a 3rd run, or it can be mixed with the next feed batch.
An alternate is to first collect distillate of desired purity. Then collect distillate which does not
meet purity requirements while bottoms is reduced to the desired purity. The material not
meeting requirements is then mixed with fresh feed for next batch.
Other operating variations are possible.
d. Hook up two batch stills in series – Either to run 1 batch or to run separate batches (second still
takes distillate from first as the charge).
e. See if product specifications can be relaxed.
f. Reducing the pressure increases the relative volatility and may help. However, one must watch
for earlier flooding.

9.C.1. Rayleigh eqn


xF
d xW
Wfinal F exp -
x W,fin
xD xW

218
Because x D constant, can integrate analytically.
xF
xF
d xW xD xF
n xD xW n
x w ,final
xD xW xD x W ,final
x w ,final

xD xF xD xF
Wfinal F exp n F
xD x W ,final xD x W ,final
Mass balances are F Wfinal D
Fx F Wfinal x W,final D xD
xD xF
Solve for Wfinal F
xD x W ,final
Thus results are identical.

9.C2. External balances over entire cycle


Fz Dx Dfinal Wx wAvg and F = D + W
a. Ignoring holdup on stages and in reboiler
- Out = Accum in accumulator is - x w dD d Dx D
which becomes, x w dD Ddx D x D dD
dD
Rearrange, dx D xW xD and integrate,
D
D final
dD x Dfinal
dx D D final x D final
dx D
which is, n
D F D xD xF xw xD F xF xD xw

b. Assume CMO and draw mass balance envelop around bottom of the column.

L V B Lx Vy Bx w
L B
y x xw
V V
L L
y x 1 xw
V V

219
xF
rd dx
9.D.1. New Problem in 3 Edition. Eq. (9-9) Wfinal F exp -
x W,final
y-x

0.1 0.00346 1 4 1
From Simpson’s rule Area
6 y x 0.00346
y x x 0.005173
y x 0.1

From equilibrium curve (Table 2-1).


x y y-x 1
y-x
0.00346 0.03096 0.027498 36.366
0.05173 .34 .28827 3.469
0.1 0.4416 .3416 2.9273

The y values are found by linear interpolation of data in Table 2-1. For example, at x = 0.00346,
linearly interpolated first 2 data pts Table 2-1.
0.170
y x for x 0.19.
0.019
0.170
For x = 0.00346, y 0.00346 0.03096
0.019
For y at x = 0.1,
y = 0.4375 +[ (.4704 - .4375)/(.1238 - .0966)](.10 - .0966) = 0.4416

[Alternatively, could fit equilibrium data to constant α.]


0.1 0.00346
Area 36.366 4 3.469 2.9273 0.8555
6
Wfinal 0.5 exp 0.8555 0.2125 kmol
D total 0.5 .2125 0.2875 kmol
F xF Wfinal x W ,final .5 .1 .2125 .00346
x DAvg 0.1714
D avg 0.2875
.75 dx
9.D2. Rayleigh equation is Wfinal F exp -
y-x .55

Most of values of 1/(y – x) are listed in Example 9-1. From Table 2-7 can easily generate
values for x = .55: y = 0.805, y – x = .255. (y – x)-1 = 3.92. The mid-point for Simpson’s
rule is at x = .65. Then from Eq. (9-12) and values in Example 9-1,
.75 dx .2
3.92 4 5.13 6.89 1.044
.55 y x 6
1.044
Wfinal 100e 35.20, D total F Wfinal 64.8
Fx F Wfinal x final 75 35.2 0.55
x Davg 0.859
D total 64.8
A graphical integration counting squares gives x Davg 0.861.
L0 D
9.D3. Operating equation is y L V x 1 L V x D where L V 0.65
1 L0 D

220
This is y = 0.65 x + 0.35 x D
We have two equil. stages (stillpot and one in the column). From McCabe-Thiele diagram
we can get the values of x D which are related to x W . Pick x D and get x W from figure.
From this we can generate the following table (only two values are shown in Figure).

xD xW 1/(xD-xW)
0.90 0.588 3.205
0.895 0.570 3.077
0.85 0.394 2.193
0.837 0.360 2.096
0.720 0.150 1.754
0.70 0.128 1.748

Simpson’s rule (Eq. (9-12)). [Midpoint x w 0.36 ]


xf
dx w .57 .15
3.077 4 2.096 1.754 0.925
x w ,final xD xW 6
Wfinal xF
dx 0.925
Raleigh Equation: , Wfinal Fe 39.7 kmol
F x wfinal xD xw
D total F Wfinal 100 39.7 60.3 kmol
Fx F Wfinal x wf 100 0.57 39.7 0.15
x Davg 0.847
D total 60.3

221
9.D4. Wfinal 2.0 kg moles, x F 0.8, x wf 0.4 . Find F, x DAvg , D total

x y(equil) 1
y x
1 0.80 0.86 16.666
16.66
y x
4.76 7.143 0.70 0.80 10

0.60 0.74 7.143


.4 .6 .8
0.50 0.67 5.882
x
0.40 0.61 4.76

Can use Simpson’s Rule (Eq. 9-12) or evaluate numerically.


xF
dx 0.4
4.76 4 7.143 16.666 3.3333
x Wfin y x 6
Wfinal 2.0
Rayleigh eqn F 56.063 kmol
xF
dx exp 3.3333
exp
x Wfin y x
D total F Wfin 54.063 kmol

Fx F Wf x Wfin 56.068 0.8 2.0 .4


x D AVG 0.815
D total 54.063

xF
dx
9.D.5. New Problem for 3rd Edition a) Wfinal F exp
x w ,fin y x
Can use Simpson’s rule, eq. (9-12) with equilibrium values from plot.
1 1
f x w ,final 2.7397
y x 0.028 0.645 0.28
x w ,fin xF 1 1
f 3.2787
2 y x 0.40
0.705 0.52
1 1
f xF 4.65116
y x 0.52
0.735 0.52

222
xF
dx xF x w fin
f x w ,fin 4f x w avg f xF
x w ,fin y x 6
0.52 0.28 0.24
2.7397 4 3.2787 4.65116 20.5056 0.8202
6 6
Wfin 3exp 0.8202 3 0.44033 1.321 , DV,tot F Wfin 1.6790
Fz Wfin x w fin
y D,avg 0.7088
D V,tot
b. Settler: D V,tot D1 D2 1.6790 D1 D2
DV,tot y D,avt D1x 2 D2 x B 1.6790 0.7088 0.573 D1 0.975 D 2
Solving simultaneously, D 2 0.567 and D1 1.112
WF 1 x W ,F 1 x F 1 xf
9.D6. Use eq. (9-13) n n n
F 1 x F 1 x W ,f 1 x W ,f

223
a) F = 1.3, x F .6, x Wf .3, 2.4
WF 1 .3 4 .4
n n n 0.8948 0.5596 1.4544
1 3 1.4 .6 7 .7
WF 1.3 0.2335 WF 0.3036 kmol
Fx F Wfin x Wf 1.3 0.6 0.3036 0.3
x DAVG 0.6914
F Wfin 1.3 0.3036
b) Now Wf 3.5 0.2335 Wf 0.81725
3.5 0.6 0.81725 0.3
x D AVG 0.6914
3.5 0.8125
c) F 2.0, x F 0.6, x DAVG 0.75

Wfin x D AVG xF
Since F Wfin x D AVG Fx F Wfin x Wf , .
F x D AVG x W ,f
Then Eq. (9-13) becomes
0.75 .6 x D AVG xf 1 x Wf .4 4
n n n n
0.75 x Wf x D AVG x Wf 1.4 .6 1 x Wf 1 x Wf

F x D AVG xf 2.0 .75 0.60


Solution is x w fin 0.5025 . Wfinal 1.212 kmol.
x D AVG x Wf .75 0.5025
L D 1
9.D7. L D 1/ 2, L V slope . Pick series x D values. Plot enriching section op
1 L D 3
1 xF
dx
line. Step off two stages. Find x W . Calculate , determine
xD xW x Wf xD xW
xD xW 1
f
xD xW

0.56 0.11 1/.45 = 2.2222


0.10 Interpolate 2.361
0.49 0.09 1/.40 = 2.50

0.44 0.06 1 .38 2.6316


0.38 0.05
0.26 0.02 1 .33 3.0303
1/.24 = 4.1666

Simpson’s Rule
xF
dx xF x Wf
f x WF 0.02 4f x W 0.06 f xF 0.10
x Wfin xD xW 6
0.10 0.02
4.1666 4 2.6316 2.361 0.22739
6

224
WFinal 4.0 exp 0.22739 4 0.7966 3.1864

Fx F Wf x Wf 4 .1 3.1864 0.02
x DAvg 0.4133
F Wf 4 3.1864

L L
9.D8. New Problem 3rd Edition. a) Op. Eqn. y x 1 xd
v V
L L 4 L
L V 1 xd , x d .8@ y x , y Intercept 1 xD .2 .8 .16
D D 5 V
From McCabe-Thiele plot x w final ~ 0.075

b) 10 F Wfinal D tot Wfinal 10 D tot


10 .4 F xF Wfinal x w final D tot x D 0.075 Wfinal .8 D tot
Substitute in for Wfinal , 4.0 0.075 10 D tot .8 D tot
4.0 10 .075
D tot 4.483
.8 .075
Wfinal 5.517

225
9.D8. Figure

9.D8. Part C.
Trial & Error to 3 stages ending at x F 0.4 (See figure)
L L
y INtercept 0.645 1 xD 1 .8
V V

226
L .645
1 .80625
V .8
L L L L V .15375
.19375, 0.24
V D Initial V L 1 L V 1 .19375

9.D9. a. Mass balances: F D total W or 20 D total W


F xF D total x D W x Wf or 8 .975 D total .28 W
Solving simultaneously: D total 3.453 and W 16.547
Can also use Rayleigh equation to obtain same result. (Use of the Rayleigh equation for this
type problem is illustrated in the Solution to Problem 9-D14.)

b. Vapor in equilibrium with x Wf must be within the two phase region. x y x


Minimum is when y x . This is x Wf ,min 0.21 . See graph.

227
L L
9.D10. Op. Eq., y x D , Slope, L V L D
x 1 1 LD 15
V V
Plot on McCabe-Thiele graph for series of x D values.
1
Simpson’s rule, at x W xF .52, x W x Wfin .20, and (x Wfin xf ) / 2 .36
xD xW

xF
dx W 0.52 0.20 1 4 1
x W ,fin xd xW 6 xD xW xF
xD xW x F x W ,fin xD xW x W ,fin
2

0.32
5.95 4 3.70 2.67 1.2491
6
Wfinal xF
dx W
Rayleigh eq. n Wfinal Fe
F x W ,fin xd xW
1.2491
or Wfinal 10 e 10 0.28677 2.8677 and D total F Wfinal 7.132
Fx F Wfinal x Wfinal 10 0.52 2.8677 0.2
x D,avg 0.648
D total 7.132

228
xD xW x D-x W 1/(x D-x W)
0.70 0.55 0.15 6.666
0.65 0.405 0.245 4.082
0.60 0.25 0.35 2.857
0.50 0.094 0.406 2.463
0.40 0.055 0.345 2.899
0.30 0.035 0.265 3.774
229
xW 1
xD xW
x F = 0.52 5.95
xF x Wfin 3.70
= 0.36
2
x W ,fin = 0.20 2.67

x butanol Inital
S dx pot
9D11. New Problem in 3rd edition. Eq. (9-17)
W x butanol final
y
Note W water
Equation is in terms of butanol.
1.0
S dx but pot
Simpson’s rule – need y at x pot 1.0, .8, .6 butanol.
W .6
y but

x but x water y water y but 1


y but
1.0 0 0 1.0 1.0
.8 .2 0.565 0.435 2.2999
.6 .4 0.70 0.30 3.333

.4
3.333 4 2.299 1.0 0.90191
6
S 0.9019 W 1.804 kmol.
More accurate if done in 2 steps. Thus add points below:

x but xw yw y but 1
y but
.9 .1 .42 .58 1.7241
.7 .3 0.66 .34 2.9412

1
.2
2.299 4 1.7241 1 0.33985
0.8
6
.8
.2
3.3333 4 2.9412 2.299 0.5799
.6
6

Total Area = .91975. S = 1.8395

230
xF
dx
9.D12. Wfinal F exp -
x Wfinal
y x

xF
dx xF x Wfinal 1 1 1
4
x Wfinal
y x 6 y x x W final
y x x Wfinal x F y x xF
2

xF x Wfinal
xF 0.48, x Wfinal 0.16, x avg 0.32
2
1
x y (from eq. data) (y-x)
y x
0.16 0.36 0.20 5.0
0.32 0.545 0.225 4.444
0.48 0.66 0.18 5.555
xF
dx 0.32
5.0 4 4.444 5.555 1.51 , Wfinal 3.0 exp 1.51 0.662 k/moles
x Wfin
y x 6
Fx F Wfin x Wfin 3.0 0.48 0.662 0.16
D tot F Wfinal 2.338 x DAVG 0.571
, D tot 2.338
xf
Wfinal dx
9.D.13. New Problem for 3rd Edition. a) Rayleigh equation: n
F x w final
y x
x y y-x 1
(from graph)
y x
0.4 0.7 0.3 3.333
0.24 0.61 0.37 2.7027
0.08 0.37 0.29 3.448

Simpson’s Rule
xF
0.4 0.08
3.333 4 2.7027 3.448 0.93826
x w ,fin
6

Wfinal F exp 0.93826 3.1305 D total F Wfinal 4.8695


F xF Wfinal x w ,final
x D,AVG 0.6057
D total
b) Two Liquids. x 0.573 and x 0.975
L L D total L D total L

L x L x D total x DAVG D total L .573 L .975 D total .6057

231
D total .6057 .573
L 0.3963 L D total .3963 4.4732
0.975 0.573

9.D14. a. p org pw 760 , p org VPC10 x C10 , and assuming water is pure, p W VPW .
VPC10 x C10 VPW 760
Vapor pressure data for C10 was shown in solution to Problem 8.D10. Guess 99.5ºC.
VPW 746.52, VPC10 ~ 70.5 mm Hg
7 + 746.5 = 753.5 < 760
At 100ºC VPC10 ~ 70.5 and VPW 760 .
7.05 + 760 = 767.05 > 760
6.5
By linear interpolation: T 99.5 .5 99.74 ºC
13.55
b. Use Mass balances. Initially 9 moles n-decane, 1 mole non-volatile
a .1
Final: a mol n-decane where 1 ; thus, a .111 mol
1 a .9
Wfinal 1.111 mol (Water free)
D total F Wfinal 10 1.111 8.889 kmol

Alternate Solution: Raleigh Eq. with


xF
dx xF
xD 1: Wfinal F exp - F exp n 1-x W
x W,final
1 xW x W ,final

Wfinal F exp n .1 n .9 .111 F 1.111


D total F Wfinal 8.889. Same result as mass balance.
p tot VPorg x org in org
c. nW D org
VPorg x org in org
Should really calculate numerically from integral for most accuracy
D
p tot VPorg x org in org
nW dn org
0 VPorg x org in org
However, estimate at final conditions with VPndecane ~ 70.25 (from part a)
nW 760 70.25 .1
107.185
n org 70.25 .1
This should be a good estimate.

9.D15. Column is similar to figure in Solution to 9.D13, but with 1 stage in column.

a) For finding Wfin & D tot don’t actually need to step off stages. Just want to make sure x Wfinal
is obtainable – I checked this at total reflux – It works. Use Mass balances:
F x F D tot x d Wfin x Wfin
Substitute in F D tot Wfinal

232
F xD xF 100 0.975 0.48
Then, Wfinal
xD x Wfin 0.975 0.08
Solution is Wfinal 55.307 kmol, D tot F Wfinal 44.673
b) Need to draw operating lines until: initial 2 stages gives x Feed .
final 2 stages gives x W,final . Then L/V = slope.
Initial – There will be a pinch at point reflux is returned.
y xd y int ercept 0.975 0.41
L V Initial Slope 0.579
xD 0 0.975 0
Final: A few trials resulted in final result.
y xd y int ercept 0.975 0.17
L V final Slope 0.826
xd 0 0.975 0

233
9.D16. a. Need L/V so that 3 stages go from x F .4 to x D 0.84 . This is line a in Figure (trial-and-
error) was used to find this).
L .84 .57 L L V a
.3214 and .4737
V a .84 0 D a 1 L V a

b. Need L/V so that 3 stages go from x wfinal 0.08 to x D 0.84 (see line b in Figure). Again,
trial-and-error was used to find line.
L .84 .13 L L V b
.8452 and 5.4615
V b .84 0 D b 1 L V b

c. F D total Wfinal 10 D total Wfinal


F x feed D total x D Wfinal x wfinal 4 .84 D total .08 Wfinal
Solving simultaneously, Wfinal = 5.789 kmol and D total 4.211 kmol.
The Rayleigh equation could be used, is not needed, but gives the same result.

9.D17. x init 1.0 . Start 1 kmol and keep 1 kmol. Add water as boil.
x W ,initial
S dx tan k
Eq. (9-17), Note balance is on original solvent, methanol.
W x W ,final
y
Use equilibrium data from Table 2-7. Generate table of methanol mole fractions:
x y 1/y
1.0 1.0 1.0
0.611 0.830 1.20489
0.222 .588 1.6722
0.11611 .450 2.222
0.01 0.07 14.9254

Use Simpson’s rule in two steps. Step 1 (x from 1.0 → 0.222)

234
0.778
1.0 4 1.20489 1.6722 0.97143.
6
Step 2 (x from 0.222 → 0.01)
0.216
1.6722 4 2.222 14.9254 0.9005.
6
Total = 1.87194 = S/W with W = 1.
y D ,final
rd F dy
9.D.18. New Problem 3 Edition Eq. (9-14) n
D final yF
y x

y D ,final
F dy
exp
D final yF
y x
F
D final y D ,final
dy
exp
yF
y x
Read x values from equilibrium diagram or interpolate from Table 2-1.
y x y-x 1
y x
0.1 0.008 0.092 10.87
0.3 0.045 .255 3.92
0.5 0.155 .345 2.90

0.5 0.1
Area 10.87 4 3.92 2.90 1.963
6
0.5
D final exp 0.0702 kmol
1.963
C total F Dfinal 0.4298 kmol
Ethanol MB:
Dfinal y D,final C total x C,avg F yF

F yF D final y D,final 0.5 0.1 0.0702 0.5


x C,avg 0.00346 mole frac ethanol
C total 0.4298

9.D19. New problem in 3rd edition.


VPw x w VPoct x oct Ptot , or in mm Hg, 526.123 1.0 10.964 .6 532.7

9.D20. Was 9.D18 in 2nd edition.


x F,C5 0.35 & x W,final,C5 0.05 : x C5,AVG 0.20, x C8,AVG 0.80, p 101.3 kPa.

235
B.P. yi Ki xi 1.0 . For average mole fractions, the BP calculation converges to T =
84º with K C5 3.7 and K C8 0.30 from the DePriester charts. The yi 0.98 which is
close enough to estimate α.
K C5 3.7
C5 C8 12.33
K C8 0.30
Wf 1 0.05 0.65 0.65
Eq. (9-13), n n n 0.5847
F 11.33 0.35 0.95 0.95
Wfin F 0.5573, Wfin 0.5573 1.5 0.8359 kmol
D total F Wfin 1.5 0.8359 0.6641 kmol
Fx F Wfin x W ,fin 1.5 .35 0.8359 0.05
x D,Avg 0.7276
D total 0.6641
L D
9.D.21. New Problem for 3rd Edition. L D 1.0 L V 12
1 L D
L L
y x 1 x D , but x D varies. Thus, plot series operating lines of arbitrary
V V
x D & slope 1 2. With a total of 21 equilibrium contacts there will be a pinch where the operating line
intersects the equilibrium curve. This intersection is x W for this x D value.
xD xW xD xW 1
xD xw

.7 .067 .633 1.5798


.6 .05 .55 1.818
.5 .038 .462 2.1645
.4 .027 .373 2.6801
.3 .018 .282 3.5461
.2 .01 .19 5.26316

Want to integrate from x W,final 0.02 to x F 0.06 and want middle point at x W 0.04 .
1
Plot vs x W and find values.
xD xW
1 1
x W ,final 0.02, 3.23; x W,Avg 0.04, 2.1
xD xW xD xW
1
xF 0.06, 1.65
xD xW
0.06 0.02
Area 3.23 4 2.1 1.65 0.08853
6

236
Wfinal F exp Area 2.5 exp 0.08853 2.288
F xF Wfinal x W ,final
D total F Wfinal 0.2118 x D,Avg 0.492
D total

237
D tot
9.D.22. New Problem for 3rd Edition. t OP toperating = __1.49 to 1.50 h
QR
9.D23. Prelim. Calc. Feed; Avg MW 0.1 MW E
0.9 MW water

MWavg 0.1 46 0.9 18 20.80 kg/mol


1000 kg
48.0769 kmol
20.8 kg mol
L L D 23
2 5 0.4
V 1 L D 53
All op. lines have slope 0.4.
Can draw op. line to x W . Ten stages will go from x D to x W because have large number
of stages. Thus, do not need to step off stages.

238
From Graph can create table of 1/(xd – 1/xw) versus xW.

xD xW xd xw 1
xd xw
0.665 0.10 0.565 1.770
0.630 0.08 0.550 1.8182
0.499 0.052 x Avg 0.447 2.237
0.440 0.040 0.400 2.500
0.278 0.020 0.258 3.876
0.140 0.010 0.130 7.692
0.057 0.004 0.053 18.868

239
Simpson’s Rule: x F 0.1, x W,final 0.004, x W,Avg 0.104 2 0.052
xF
dx W xF x Wfin 1 1 1
4
x Wfin
xd xW 6 xd xW x Wfin
xd xW x AVG
xd xW xF

0.096
18.868 4 2.237 1.770 0.4734
6
xF
WFinal dx W
exp 0.62289; Wfinal 48.0769 0.62289 29.9466
F x Wfin
xd xW

D total F WFinal 48.0769 29.9466 18.1303 kmole


Fx F WFinal x Wfin 4.80769 29.9466 0.004
x DAVG 0.2586
D total 18.1303

240
9.D24. Was 9D22 in 2nd edition.
a) F D total Wfin and Fx F =D total x D Wx Wfin
xF x Wfin
Fx F D total x D x Wfin +Fx Wfin D total F
xD x Wfin
0.62 0.45
D total 3.0 1.275 kmol, Wfin F Dtotal 1.725 kmol
0.85 0.45
L L
b) Want operating line where 2 equil. contacts gives x w fin 0.45 . y xD . x 1
V V
Surprisingly, with 2 contacts T & E not needed. – Start stepping off stages from top & from
bottom simultaneously. The intersection point must be on op. line as is y x x D .
L 0.85 0.44 L L L V
Slope 0.482, 0.932
V 0.85 0 D V L 1 L V

Figure for 9.D24.

9.D25. New Problem in 3rd edition.


Mix together F F1 F2 2.5

241
xF F1x F1 F2 x F2 2.5 0.8 2.5 0.32
xF
w FINAL dx xF x w FIN
n x AVG 0.21
F xw final
y x 2
xF
xF x FIN 1 4 1
x w Final
6 y x x w ,FIN
y x x w ,AVG
y x x w ,F

x y 1
y x Values are from methanol-water
0.1 0.4 3.333 equilibrium data.
0.21 .585 2.666
0.32 .675 2.817

.22
3.333 4 2.666 2.817 0.6166
6
area
Wfinal Fe 2.5 0.5398 1.349 kmol
Fx F WFIN x w FIN
D F Wfinal 1.151, x DAVG 0.578
D
Part b) First go from 0.4 to 0.2 with F1 D1

Then from 0.2 to 0.1 with F WFIN1 F2


.2
x y 1 2.666 4 2.703 3.030 0.55026
6
y x
0.2 0.575 2.666 WFINAL1 = F1e-area = 1.5 .5769 = .865
0.3 0.67 2.703
0.4 0.73 3.03 1.5 .4 .865 .2
D1 F1 WFinal,1 0.635, x D,AVG ,1
0.635

Now F WFIN1 F2 0.865 1.0 1.865, xF 0.2

0.1
x y 1 3.3333 4 2.7777 2.666 0.285185
6
y x area
Wfinal2 Fe 1.865 0.7518 1.4022
0.1 0.4 3.3333
0.15 0.51 2.7777
0.2 0.575 2.6666 D2 F Wfinal2 1.865 1.4022 0.46275
Total D total .635 .46275 1.09775
Ftot x tot Wfin 2 x Wfin 2.5 .32 1.4022 .1
Total x D,AVG 0.6010
D total 1.09775
Higher distillate mole fraction.

242
Part c) Go from 0.4 to 0.1 for F1
1
y x 0.3
x y 3.3333 4 2.7027 3.030 0.8587
0.1 0.4 3.3333 6
0.25 0.62 2.7027 Wfinal 1.5 exp .8587 0.6356
0.4 0.73 3.030 F1

For F2 go from 0.2 to 0.1. Same as 2nd part of Part b. 0.28518


area
Wfinal F2 e 1.0 .75187 0.75187
F2

Wfinal total 1.3874 Wfinal Wfin , D total F1 F2 Wfinal tot 1.11255


F1 F2

Differs from b – Numerical error!


Ftot .32 1.3874 .1
x DAVG 0.59436
1.11255
Should be same as part b. There are numerical errors in use of Simpson’s rule.

More accurate for .4 to .1 is .4 → .2 (Area = .55026) + .2 →.1 (Area = .285185)


Total Area = 0.835445, Wfinal1 F1 e area 1.5 .433681 0.65052
Then Wfin total 1.40239 , D total 2.5 1.40239 1.09761
2.5 .32 1.40239 .1
x D AVG 0.60109
1.09761
Same as for Part b.

L D 23
9.D.26. New Problem for 3rd Edition. a&b) L D 2 3, L V 0.4
1 L D 53
L D final
4, L V final
45 0.8
For Part b, draw op. lines with slope 0.4 for arbitrary x D 0.7 . Step off 2 stages.
For Part a, From x D 0.7 draw op. line slope = 0.8. Two stages gives
x w final 0.11 (See graph (labeled 9.D.c))
Also, draw a few lines with x D 0.7 and L V between 0.4 and 0.8 . Find x w values with
2 stages.
b) Generate Table x D , x w , 1 x D xw
1
Plot vs x w . Note there is a break in curve at x D 0.7 x w 0.185 to 0.110
xD xw
1. from x F 0.6 to x w 0.185
Find 2 areas
2. from x w 0.185to x w final 0.110
0.6 0.185
Area 1. Simpson’s rule. x w avg 0.3925 .
2

243
1
From graph 2.37 .
xD xw x w avg

0.6 0.185
Area 1 3.56 4 2.37 1.942 1.0363
6
1
Area 2. curve is straight line. Thus, Area = width x Avg height
xD xw
1.6949 1.942
0.185 0.110 0.13638
2
Total area = 1.1726.
Wfinal xF
dx w
Rayleigh eqn., n 1.1726
F x w final xD xw
1.1726
Wfinal 100 e 3.0955
D total F Wfinal 6.9045
Fx F Wfinal x Wfinal 6.0 3.0955 0.11
c) x D,AVG 0.8197
D total 6.9045

244
L/D xD xw 1
xD xw
2/3 0.9 0.65 4.0
2/3 .85 0.48 2.703
2/3 0.8 0.36 2.273
2/3 0.75 0.253 2.012
2/3 0.7 0.185 1.942
-- 0.7 0.145 1.8018
4.0 0.7 0.110 1.6949
x w ,final

245
9.D.25. New Problem for 3rd Edition. From the methanol-water equilibrium data, the following table can
be obtained.

246
xM yM 1 yM
1.0 1.0 1.0
.8 .92 1.08696
.6 .825 1.21212
x pot ,Initial
S dx MeOH
W x pot ,final
y MeOH
.4
Simpson’s rule: 1.0 4 1.08696 1.2121 0.4373
6
S 0.4373W 0.8747 kmol

9.E1.

octanol
water

xF 0.90, F=1.0 kmole

pot Final octanol in pot 1 0.95 .9 0.045 kmol

Nonvolatiles in pot 1.0 0.1 0.1

steam x oct,W,final 0.045 0.145 0.3103

2164.42
log10 VPW 8.68105 , T C, VPW mmHg
273.16 T
a) Final conditions x oct, in org 0.3103, x W, in W 1.0, T 99.782 C from solution to
problem 8.D11. Initial conditions: x oct,in org 0.90, x W in W 1.0
x i VPi 1.0 atm 760 mm Hg
From spread sheet find T = 99.377ºC
1 z 1.0 0.9
b) Wfinal,org F 1.0 0.1450
1 x Wfin 1 0.3103
c) Dorg F Wfinal 0.8550 n org
D org
p tot VP oct
x oct
d) Eq. (9-24) nW dn oct
0
VP oct
x oct
Estimate VPoct at average T (99.782 99.377) / 2 99.5795 , VPoc tan ol 18.87 mm Hg
D org D org D org
p tot dn oct p tot dn oct
nW dn org D org
VPoct 0
x oct 0
VPoct 0
x oct

247
F
F 1.0, n org F W, x org 1 1 x feed 1 0.1 F W
W
1 dn org
Step-by-Step integration, dn org
x org x org
n org dn org W x org Avg x org dn oct
x org,avg
0 1.0 .9
> 0.8944
0.1 .1 .9 .8888 0.11181
> 0.8819
0.2 .1 .8 0.875 0.11339
> 0.8661
0.3 .1 .7 0.8571 0.11546
> 0.8452
0.4 .1 .6 0.8333 0.1183
> 0.81666
0.5 .1 .5 0.8000 0.12245
> 0.775
0.6 .1 .4 0.750 0.12903
> 0.7083
0.7 .1 .3 0.6666 0.14118
> 0.58333
0.8 .1 .2 0.50 0.17143
0.855 0.055 0.145 0.3103 > 0.40517
0.1357
1.1588
Porg

p tot dn org 760


nW D org 1.15882 0.8550 45.826 kmol
VPorg O
x org 18.8666

e. Continuous had 108.93 kmoles water/kmole organic fed. The continuous always operates at
lowest octanol mole fraction in liquid & thus y oct is always at lowest value. Thus, requires
more water to carry over octanol then the batch operation.

9.E2. New Problem in 3rd edition. Parts a & b. See solution to problem 8.D25.
mol B 78.11
x benz
mol B 78.11 80 673.2
c. Find T from Eq. 8-15 With Spread Sheet:

SPE, Problem 9E5. Solution for temperature


T deg C 92.04234 Do step by step Antoine VP values
VPW A,B,C 8.68105 2164.42 273.16 2.754418 568.0906
VPben A,B,C 6.90565 1211.033 220.79 3.034461 1082.583
X ben 0.17727 xw 1 ptot 760
Eq8 -5E-05 Goal seek B6 to zero changing B2
massbeninit 20 massbenfin 2

248
dmorg-sum 18 massben-still 2 dmassorg 1
dnw/dnorg 9-23 2.960203 dnw 0.037898

d) Step-by-step integration. dn W dn org Eq. 9-23 Use Spread Sheet for each time
step. Did the addition of steps off-line in table below.
nW dn W for n org still init 20 78.11 n benz 2 78.11
still final

Set dm org 1 kg. Values from spreadsheet.


dn W divide all TºC
Mass benz still,
dm org kg values by 78.11 from spreadsheet
777 (kg)
kmol
Initial T 0→1 20 → 19 .664431 75.99
1→2 19 → 18 .677266 76.27
2→3 18 → 17 .691548 76.577
3 17 .707537 76.91
4 16 .725558 77.269
5 15 .746073 77.66
6 14 .769462 78.0996
7 13 .79657 78.58
8 12 .828276 79.12
9 11 .865852 79.715
10 10 .91108 80.39
11 9 .966543 81.156
12 8 1.036129 82.03398
13 7 1.12596 83.0487
14 6 1.246275 84.2358
15 5 1.415548 85.6435
16 4 1.670846 87.339
17 → 18 3→2 2.098857 89.422
final T 18 2 2.960203 92.04

d) For accuracy add time steps 1 to 17 + (time steps 0 & 18)/2. This is then identical to the use of
average values of dm for every step.
Steps_1 to 17 17.2794168
0.5 Steps _ 0 18 1.8122565
n W 19.0916733 78.11 18.016 4.4034 kg water
Compared to continuous, nw = 12.3, batch requires less because benzene mole fraction is higher
in batch operation for most of batch. If we take final cut and used that value for entire operation,
18.016
dn W 18 kg benzene overhead 12.3 same as continuous.
78.011
e) To vaporize (n ben ben ) / w varies since T is different.
For accuracy, do for each dm org and find λ values at these T.
x d ,fin
dx d 0.13 1 4 1
9.E3. Simpson’s Rule:
xF
xd xB 6 xd xB x d .63
xd xB x d 0.565
xd xB x D 0.50

249
Generate table from graph
xB xd 1 0.13
3.0303 4 2.15 2.004 0.2954
xd xB 6
0.3 0.36 3.0303 D final Fe 1 e 0.2954
7.442 gmole
0.2 0.597 2.5189
0.1 0.565 2.15 B F Dfinal 2.558 mole.
0.04 0.500 2.004 F xF Dx D,fin 5.0 4.6886
x B,avg 0.1218
B 2.558

x init
dx
9.E4. Wfinal F exp
x W ,fin
y x
T & E since dilution effects F & x init .
Dilute with 5 kg water → Start with 6, x init 16 0.1667
Dilute with 4 kg water → Start 5, x init 15 0.2
Dilute with 3.5 → Start 4.5, x init 1 4.5 0.2222
Dilute with 2.75 → Start with 3.75, x init 1 3.75 0.26667
Dilute with 1.75 → Start with 2.75, x init 1 2.75 0.364

250
For each dilution want to integrate using Simpson’s rule until find Wfinal 1.0. Thus,
need values of 1 y x at x init , x avg , and x final 0.01 for each dilution. These values
are determined in the following table.

Table of Values for Integrations

Dilute x y y-x 1/(y – x)


5 kg 0.16667 0.537671 0.3710 2.695412
Water 0.08833 0.38708 0.29875 3.34724
0.01 0.067 0.057 17.544
4 kg 0.200 0.579 0.379 2.63
Water 0.105 0.428 0.3230 3.095975
0.01 0.067 0.057 17.544
3.5 kg 0.2222 0.598 0.376 2.6596
Water 0.11611 0.450 0.334 2.994
0.01 0.067 0.057 17.544
1.75 kg 0.364 0.706 0.342 2.923977
Water 0.1870 0.563 0.376 2.6596
0.01 0.067 0.057 17.544
2.75 kg 0.2666 0.636 0.369 2.7100
Water 0.138 0.493 0.355 2.8169
0.01 0.067 0.057 17.544

Integrations: Use Simpson’s rule for each addition.


0.15667
Dilute 5 kg: 2.695412 4 3.34724 17.544 0.87809
6
Wfinal 6 exp -0.87809 2.4934 . Value is too high. Want 1.0 kg.
0.19
Dilute 4 kg: 32.5579 1.031
6
Wfin 5 exp -1.031 1.783 too high
0.2122
Dilute 3.5: 32.1796 1.138
6
Wfin 4.5 exp -1.138 1.442 too high
.354
Dilute 1.75: 31.106 1.833
6
Wfin 2.75 exp -1.835 0.4389 too low
.2566
Dilute 2.75: 31.52 1.348
6
Wfin 3.75 exp -1.348 0.9740
Close to desired 1.0 kg. Thus, 2.75 kg water. The final still pot is 99% water so have (.99)
(.974) = 0.964 moles water remaining. Moles of water distilled off is 2.75 – 0.964 = 1.786.
9.H.1. New Problem in 3rd edition. This problem is challenging for students because they must first
derive the forms of the equations they need to use.

251
A. Define. The system is the simple still pot shown in Figure 9-1. Find Wfinal, D, xA,Wfinal, and xA,dist,avg.
B. Explore. At first it may appear that the problem in Part a is under specified since there are now five
unknowns. However, in specifying the problem based on the fractional recovery of benzene in
the distillate we have added the equation for the definition of fractional recovery of A in the
distillate. This equation is most conveniently written as,

FzA (1 – Frac. Rec. A in distillate) = Wfinal xA,Wfinal (9-35a)

which becomes, Wfinal = FzA (1 – Frac. Rec. A in distillate)/ xA,Wfinal (9-35b)

C. Plan. If we write Eq. (9-13) for A and substitute in Eq. (9-35b) we obtain Eq. (9-36),
1 x A ,W ,final 1 x A ,F 1 x A ,F z A 1 Frac. Rec. A.dist
0 n n n
AB 1 x A ,F 1 x A ,W ,final 1 x A ,W ,final x A ,W ,final
Part a. In a spreadsheet Eq. (9-36) is easily solved for xA,Wfinal using Goal Seek. Then W final can
be determined from Eq. (9-35b). Then DTotal is determined from Eq. (9-11) and xA,dist,avg is
determined from Eq. (9-10) written for component A or from the fractional recovery.

Part b. Now solve Eq. (9-36) for frac. rec. of A in distillate using Goal Seek. For both parts a and b can
use fractional recovery values and DTotal to find xA,dist,avg = FzA(Frac Rec. A in distillate)/ DTotal

D. Do It. Because Eq. (9-36) for xA,Wfinal is nonlinear, it is easiest to solve this problem with a spreadsheet
and use Goal Seek to solve Eq. (9-36). The spreadsheets are shown below.
Part a
F 5 zA 0.37 The 0.37 is in cell D2
alpha AC 10.71
frac rec A in distillate 0.75
xA,Wfin 0.143185
9-36 term 1 -1.25687 term 2 -0.3075 term 3 0.436926

Eq 9-36 -1.7E-05 Use Goal seek


Wfinal from 9-35b 3.230095 D total 1.769905 xAdist,avg 0.78394
Part a. Use Goal Seek for cell B8, setting it equal to zero by varying cell B5 (xA,Wfin).
Part b. Use Goal Seek for cell B8, setting it equal to zero by varying cell C4.

Part b
F 5 zA 0.37 The 0.37 is in cell D2
alpha AC 10.71
frac rec A in distillate 0.930081
xA,Wfin 0.05
9-36 term 1 -2.41222 term 2 -0.41074 term 3 0.658942

Eq 9-36 -0.00023 Use Goal seek


Wfinal from 9-35b 2.586992 D total 2.413008 xAdist,avg 0.713073
Part a. Use Goal Seek for cell B8, setting it equal to zero by varying cell B5.
Part b. Use Goal Seek for cell B8, setting it equal to zero by varying cell C4 (frac rec A in distillate).

252
SPE 3rd Edition Solution Manual Chapter 10

New Problems and new solutions are listed as new immediately after the solution number. These new
problems are: 10A4, 10A5, 10A7, 10A12, 10A17, 10C4, 10C5, 10D13, 10D19, 10D21, 10G1-
10G4.

10.A9. A good packing will have: good contact between liquid and vapor, high surface area, low
pressure drop, inert, inexpensive, and self-wetting.
Marbles have low surface area, poor contact and relatively high ∆p.

10.A13. Figure 10-25 shows that if viscosity increases the ordinate increases and ∆p/foot increases.

10.A14. a.Want low F since this gives low ∆p.


b. F decreases as size increases.
c. Ceramics have much thicker walls than metal or plastic. Ceramics are used in corrosive
environments.

10.A16. 1. a. fewer; 2.b. larger; 3. a. lower

10.A.17. Answer is c.

10.B1. The trick is to have maximum and minimum positions of the valve with a larger area for
vapor flow at the maximum position.
a. Use a cage to prevent excess movement.
b. Use feet.
c. Have a flap that moves up and down.
d. Use a spring to provide force and maximum position.
e. Sliding valve controlled by an external feedback mechanism.
f. Two flaps to make a roof.
Many other ideas can be generated.

10.B3. Some possible candidates:


Bottle caps SOS pads
Bent bottle caps Scotch brite pads
Natural sponge Steel wool
Synthetic sponge Cooper cleaning pads
Miscellaneous junk Plastic coated wire
Broken crockery or glassware Tin foil - crushed
Cut up tubing (Tygon) String - balls
Glass tubing (broken) lines tied together
Crushed beer cans lines twisted
Cut up - crushed beer cans lines stretched taught
Coal Rope
Egg shells frayed rope
Styrofoam packing material Rope tied into bow ties
Old seat cushions Porous rock pieces

253
Nuts/bolts/screws/nails Pumice
Metal filings Ash from Mt. St. Helens
Wood shavings Ashes from coal stove
Kindling Pieces of cement block
Left-over redwood Pieces of brick
Staples Pop-tops
Window screens, rolled up Old watch bands - twisted
Chicken or barbed wire Bent wire coat hangers
Christmas Tree Ornaments Cookie cutters
Corn cobs Combination of the above

10.C1. L/V constant since L/D const. Thus L/G const. In F v only G
changes. From Perfect Gas
n P
Law G MWv MWv . Thus, F v increases as P . However, curve is almost
v RT
flat in this range and Cs,bflood ~ constant.
.5 .5
C sb flood L G 1
Then u flood 0.2
since L G .
20 / G P
L xD z
V L D 1 D and D F so V F
D xD xB
F F F
Thus, Dia
u flood 1 P
V P
P
2
Then, F Dia P , and exponent = ½

10.C2. See derivation in solution to Problem 10.C1.


L L xD z
V 1 D 1 F
D D xD xB
Then from Eq. (10-16)
1/ 2
L xD z
4F 1
D xD xB
Dia
3600 p fraction u flood

1/ 2
Since Dia F1/ 2 , Dia L D 1
1/ 2
L G
10.C3. Plot points based on ∆p and on Figure 10-25. From ordinate values calculate F
G L

for each point. Use an average value.

254
10.C4. New Problem in 3rd edition. Part of the operating lines become closer to the equilibrium curve.
Thus, for the same separation more stages are needed. Fortunately, this effect is often small.

10.C5. New Problem in 3rd edition. You can show this by proving that the minimum reflux ratio (Figure
10-18A) or the minimum boilup ratio (Figure 10-19A) must increase compared to the base
cases. These ratios increase because cooling the entire feed (Figure 10-18A) or heating the
entire feed (Figure 10-19A) automatically changes the feed line and moves the minimum
reflux (Figure 10-18A) or minimum boilup (Figure 10-19A) operating lines towards the y = x
line. This means larger minimum external reflux ratio or larger minimum boilup ratio.

10.D1. K C6 y/x and K C7 1 y 1 x


1 K C7
Solve simultaneously. x , y K C6 x
K C6 K C7
Pick T and generate equilibrium curve. Bounds are K C6 1.0 and K C7 1.0 .
T KC6 KC7 x y
o
149 C 1.0 - 1.0 1.0
169 1.3 .72 .483 .63
171 1.34 .75 .424 0.568

193 - 1.0 0 0
Average temperature is T = 171ºC = 444.1 K.
y x .568 .424
1.786
1 y 1 x .432 .576
Viscosity equation and terms are given in Example 10-1.
1 1
log10 C6 362.79 .935, C6 0.16 .
T 207.09
1 1
log10 C7 436.73 .895, C7 0.127 .
T 232.53
n MIX .5 n .116 .5 n .127 2.107, MIX 0.122.
From Eq. (10-6) for 0.217, E o 0.730 . The higher pressure results in higher
temperatures and lower viscosities. This increases the predicted column efficiency by 24%.

10.D2. T 98.4 273.1 371.5 K (almost pure n-heptane at bottom).


pMW 1 100.2
v 0.205 lb/ft 3 , L 0.684 62.4 42.68 lb/ft 3 , 20
RT 1.314 371.5
Need L V . First, find y at intersection of operating lines.
L .999 y
Top Operating Line Slope .8
V .999 .5
y .999 .8 .499 0.5998
L 0.5998 0.001
Then, 1.20
V 0.5 0.001

255
L L MWL L
1.2 . This is at bottom where MWL ~ MWv .
G V MWv V
0.5 0.5
L v .205
Fv 1.2 0.08317
G L 42.68
0.2 0.2
12.5
From Figure 10-16, Csb 0.32, C sb 0.32 0.29
20 20
0.5
42.68 0.205
u flood 0.29 4.19
0.205
Saturated liquid feed V V 2500. Use η = .90
1/ 2
4 2500 1.314 371.5
D 12.35 ft
.90 3600 1 .75 4.19
Somewhat larger. Would design at bottom of column. Use a 12 ft. diameter column.

10.D3. New 12’ dia col. First, redo entrainment calculation.


0.5
L 0.205
1.2, F g 0.0832 , L 1.2 V 3000
1.2
V 42.68
For Fig. 10-17 need % flood. In problem 12.D2 designed for 75% flood.
1/ 2
const 1/ 2
D 11.78 , const1/ 2 11.78 .75 10.20
75% flood
1/ 2
Use 12 feet: % flood 10.202 /12.0. % flood 72.3% . Then Fig. 10-17, ψ = 0.022.
L 0.022 3000
e 67.48 , L e 3067.48
1 1 .022
This is reasonable amount.
2
A total 12.0 4 113.1 ft 2
Ad 1 0.9 113.1 11.3 ft 2
Table 10.2, 0.9, weir Dia 0.726 , weir 0.726 12 ft 8.71 ft
A active 113.1 1 .2 90.48 ft 2 , A hole 0.1 90.48 9.05 ft2
14 gauge, t tray 0.078 in, 3 16 holes, d 0 t tray 2.4
VMW v
v0 37.51 ft s , C0 0.759 (unchanged from Ex. 10-3)
3600 v 9.05
2 59.87 1 0.01
h p ,d ry 0.003 37.51 0.205 2
2.086 in
42.68 0.759
100.2 7.48
Lg 3067.48 897.8 gpm
42.68 60

256
Lg 897.8
Abscissa 2.5 2.5
4.01
weir 8.71
Parameter w Dia 0.726, Fweir Fig.10 20 1.03
2/3
897.8
Eq. (10-26), h crest 0.092 1.03 2.083 in
8.71
With 1″ gap, A du 8.71 12 0.726 ft 2
2
897.8
Eq. (12-27), h du 0.56 4.248 in
449 0.726
h dc 2.086 2 2.083 0 4.248 10.417
h dc,aereated 10.417 0.5 20.83 OK, but close to distance between trays.
11.3 10.417 3600 42.68
t res =4.904 This is OK.
3067.48 100.2 12
0.040 12.5
Weeping, Eq. (10-32), h 0.0625 in
42.68 3 16
Eq. (10-31), LHS h p,dry h 2.086 0.0625 2.148
x 2 2.083 0 4.083
2
RHS 0.10392 0.25119 4.083 0.021675 4.083 0.7682, OK
LHS > RHS
Operation is slightly marginal with high downcomer backup.
1.5
Increase apron gap to 1.5 inches: A du 8.71 ft 1.089 ft 2
12
2
897.8
h du1.5 gap 0.56 1.888 in
449 1.089
h dc 2.086 2 2.083 1.888 8.05 in
h dc,areated 16.11 inch, OK

1/ 2
C v Wvalve 2g
10.D4. v o,bal (10-36)
K vA v v

Given Wvalve 0.08 lb, g 32.3 ft s 2 , K v,closed 33C v 1.25, K v,open 5.5
Pressure drop in terms of inches of liquid of density L :
Wvalve Kv
h p,valve Cv v
v 02
Av L 2g L

1
Av 0.02182 ft 2 , v 0.1917 lb ft 3 , L 41.12 lb ft 3
12

257
1/ 2
1.25 .08 2 32.2
v o,bal,closed 6.83 ft sec
33 0.2182 .1917
1/ 2
1.25 .08 2 32.2
v o,bal,open 16.73 ft sec
5.5 0.218 .1917
C v Wvalve 1.25 0.8 lb
At balance point, h p,valve
AL L 0.2182 ft 3 41.14 lb ft 3

0.1115 ft 1.338 in liquid


33 .1917
closed: h p,valve v 02 for v 0 6.83
2 32.2 41.12
3
2.39 10 v 02 ft 2.87 10 2
v 02 inches
5.5 .1917
open: h p,valve v 02 for v 0 16.73
2 32.2 41.12
4
3.98 10 v 02 ft 4.78 10 3
v 02 inches

10.D5. Do calculation at total reflux. From a McCabe-Thiele diagram (not shown). Total #
Contacts = 4.2 N = Total – 1 (P.R.) = 3.2
Length 1
HETP 0.31 m
N 3.2

10.D6. From Fenske eq. and definition of HETP


z
HETP
x x
n
1 x dist 1 x bot

n AB

x x .987 .008
n n 9.150
1 x dist 1 x bot .013 .992

z = 3.5 meters. Obtain:


a. α = 2.315, HETP = 0.321
b. α = 2.61, HETP = 0.367
1/ 2
c. AVG 2.315 2.61 2.4581 and HETPAVG 0.344
Can also use McCabe-Thiele diagrams although the solution shown is easier.

10.D7. Current:
F v 0.090, 12 spacing, Ordinate 0.2 U nf const., const. 0.2 U nf 0.2 6.0
New:
L Vnew 1.11 L V old , then L G new 1.11 L G old , F v,new 1.11F v,old 0.0999
Trays Spacing 24″, Ordinate ~ 0.32, Ordinate = U nf × Const.

258
Ordinate 0.32
U Nf 9.6 ft s
Const 0.2 6

10.D8. At Fv 0.5, Csb,f 1 0.12


0.2 0.2
L G 6.0 L G 6.0 1
Uf1 C sb 0.2
20 G 0.12 20 G G
0.12 L
20
n MWv p
Since pV nRT, G MWv
V RT
0.5 0.5
G ,new 1
New Condition: Fv 0.5 0.5 0.25; C sb ~ 0.18
G ,old 4
U f 1,new C sb,new G ,old C sb,new old 0.18 3
4 2 3
U f 1,old C sb,old G ,new C sb,old new
0.12 2
U fl,new 3 6 18 ft/s

D z xB 0.6 0.01
10.D9. Mass Balance: 0.59656
F x D x B 0.999 0.01
D 596.56 kmol/h, B 403.44
At top: L V 0.6, V L D V D 0.6V
D 596.56
V 1491.47 kmol h
0.4 0.4
L V D 491.41 kmol h
At top of col. WL WV L V since pure MeOH, same mol. wt. 0.6
kmol 32.04 kg MeOH 2.046 lbm lbm
V 1491.41 105, 346
h kmol kg h
Assume ideal gas. Top of column is essentially pure MeOH.
lb
1 atm 32.04
n MWM p lbmol lbm
v MWM 0.07219
V RT 3
ft atm ft 3
0.7302 607.79 R
lbmol R
1.8 R
where pure MeOH boils at 64.5 C 337.66K
607.79 R
K
1 kg 2.2046 lbm 28317 cm 3 lbm
L MeOH 0.7914 g cm 3 49.405
1000g kg ft 3 ft 3
At top 24 0.0773 64.5 19.0

259
1/ 2 1/ 2
WL G 0.07219
Fv .6 0.02294
WG L 49.405
Fig 10-16 with 18″ tray spacing: Csb 0.28
0.2 0.2
L V 19 49.405 0.07219
U flood C sb 0.28 7.25 ft s
20 V 20 0.07219
4V lbm h
Use Eq. 10-14 (Modified), D
V frac u flood 3600

4 105, 346 lbm h


D
ft
0.90 0.07219 lbm ft 3 3600 s h0.75 7.25
s
D = 10.27ft Use either 10 ft (slightly higher frac flood) or 11 ft – (lower frac. flood).

x
10.D10. Ref. Bonilla (1993). 3.0, y= , at z 0.4 x, y feed 23
1+ -1 x
.9 0.66667 L L V 0.46667
L V min
0.46667 , L D min
0.875
.9 .4 V L 1 L V 1 .46667
L D 1.75
L Dact 2 L D min 1.75 , L V act
0.636
1 L D 2.75
1 L V xD 0.3273
At Minimum (Pinch Point), Llow V L V min
or Llow D L D min
0.875
L low 0.875D , L avg 1.75D , Llow 0.5 Lavg
Generalize Llow Lavg M where L D actual
M L D min

10.D11. a. Since Liquid & Vapor have the same mole fractions L G L V
pV = nRT, G n MWv V p MWv RT , R 45.6 cm3 atm gmoles-1 o
R1

260
MWv .8 46 .2 18 40.4

G 1 ATM 40.4 g mol 45.6 cm3 atm mol 1 R 1 176 460 R .001393 g cm3

V L D 3D 6750 lb day , L 2D 2 2250 4500 lb day


L V 4500 6750 .6667 L G
1/ 2 1/2
3
L G G L .6667 1.393 10 .82 =0.0275

H2O L 1 g cm3 .82 g cm3 1 .82


F = 97 (Table 10-3)
Ordinate G F2 F .2
G L gc .197 , from Figure 10-25 (flooding line).
This is,
2
2 1 0.2 1 g cm 3
G F 97 .52
.82 62.4 lb ft 3
3
.197
1.393 10 g cm 3 .82 g cm 3 32.2
GF 0.5216 lb (s ft 2 ) 8.64 10 4 s day 45067 lb (day ft 2 ) , G .75 GF

FIND AREA AND DIAMETER FOR 75% OF FLOODING


AREA V .75 G 6750 lb Day 45067 lb Day ft 2 .75 .19970 ft 2
D 2 4 .19970 ft 2 , D .5042 ft 6.05 in
COLUMN DIAMETER
6.05 5 8 9.68 which is probably OK.
PACKING DIAMETER

1/2
b. From Fig. 10-25, G 2 F 0.2
( G L g c ) .036 at L G G L .0275
2 1 0.2 2
G 97 .52 1 62.4
.82
3
0.036
1.393 10 .82 32.2
2
G 0.2230 lb (s ft 2 ) 8.64 104 s day 1.927 10 4 lb (day ft )
AREA 6750 lb day 19267 lb (day ft 2 ) 0.3503 ft 2
AREA D 2 4 .3503, D .6679 ft 8.01 inches
1/ 2
L G
c. L G will be the same; thus will be the same, and G will be the same.
G L

Area V .75 6 , V 3D as before 67,500 lb day


area 67500 .75 .521 8.64 10 4 1.998 ft 2 10 earlier value
1/ 2
4 1.998
D 1.59 ft 19.14 inches

261
10.D12. a. y L V x 1 L V x D . When x 0, y 1 L V x D 0.1828 . See figure. Need
2 equilibrium stages. Stop where feed line and operating line intersect.
HETP 5 / 2 2.5 ft, x B ~ 0.065
b. M is at x in y1 .43
L L M 6.13
3.88
L L M 1.58
L L L L
Within accuracy of graph, 0.8
V L L L L 1
If try a shorter column with same feed won’t work.
L L
.8 and must adjust column.
L V

10.D.13. New Problem in 3rd edition. Saturated vapor feed in problem 10.D.9 has minimum L/V =
(0.999-0.6)/(0.999-0.22)=0.5122. This is (L/D)min = 1.05. The actual L/V = 0.6, which is an
L/D = 1.5. Thus, the multiplier M of the minimum was M = (L/D)/(L/D)min = 1.5/1.05 =
1.43.
For a saturated liquid feed (L/V)min = (0.999-0.825)/(0.999-0.6) = 0.4361, which corresponds to
(L/D)min = 0.7733. If we use the same multiplier, L/D =1.43(0.7733) =1.106 and L/V =
0.525.
D z xB 0.6 0.01
Mass Balance: 0.59656
F x D x B 0.999 0.01
D 596.56 kmol/h, B 403.44 . These are same as in 10.D9.
At top: L V 0.525, V L D V 0.525V D

262
D 596.56
V 1255.9 kmol h
0.475 0.475
L V D 659.4 kmol h
At top of col. WL WV L V since pure MeOH, same mol. wt. 0.525
kmol 32.04 kg MeOH 2.046 lbm lbm
V 1255.9 82, 330
h kmol kg h
The density and surface tension calculations are the same as in 10.D9.
Assume ideal gas. Top of column is essentially pure MeOH.
lb
1 atm 32.04
n MWM p lbmol lbm
v MWM 0.07219
V RT 3
ft atm ft 3
0.7302 607.79 R
lbmol R
1.8 R
where pure MeOH boils at 64.5 C 337.66K
607.79 R
K
1 kg 2.2046 lbm 28317 cm 3 lbm
L MeOH 0.7914 g cm 3 49.405
1000g kg ft 3 ft 3
At top 24 0.0773 64.5 19.0
1/ 2 1/ 2
WL G 0.07219
Fv .525 0.02007
WG L 49.405
Fig 10-16 with 18″ tray spacing: Csb 0.28
0.2 0.2
L V 19 49.405 0.07219
U flood C sb 0.28 7.25 ft s
20 V 20 0.07219
4V lbm h
Use Eq. 10-14 (Modified), D
V frac u flood 3600

4 82, 330 lbm h


D 9.08ft
ft 3
0.90 0.07219 lbm ft 0.75 7.25 3600 s h
s
Probably use 9 ft, which is a slightly higher fraction of flooding. This compares with 10.27 ft for the
saturated vapor feed. The smaller diameter column will be less expensive.
With a saturated liquid feed and CMO, the vapor flow rate in the bottom of the column is the same as in
the top, V = 1255.9 kmol/hr. For problem 10.D9 with a saturated vapor feed,
V V F 1491.47 1000 491.47 . Since QR V,
QR ,liquid _ feed (1255.9 / 491.7)QR ,vapor _ feed 2.55QR ,vapor _ feed .

Thus, in this case there is a significant energy price for reducing the column diameter by this method.

z xB 0.4 0.0001
10.D14. D F 1000 400.7415 , B 1000 D 599.258 kmol/day
xD xB 0.998 0.0001

263
V V L D 1 D 3 400.7415 1202.225 kmol day
kmol 1 day 1h
or V 1202.225 0.013915 kmol s
day 24 h 3600 s
At bottom, L V B 1202.225 599.258 1801.483 kmol day
L 1801.483
1.49846
V 1202.225
Bottom of column is essentially pure water. Also y boilup xB
Thus L G L V 1.49846 G is lb (s ft 2 )
lb kmol 18.016 kg 2.20462 lb
V in 0.013915 0.55268 lb s
s s kmol 1 kg
kg 2.20462 lb m3 lbm
L density water at 100 C 958.365 3 3
59.83
m kg 35.31454 ft ft 3
Data from Perry’s, 7th ed., p. 2-92. W L 1.0
lbm
1 atm 18.016
n p ft 3 lbm
pV nRT G MWw MWw 0.0367
V RT 3
ft atm ft 3
0.7302 671.688 R
lbmol R
1.8 R
where 100ºC 373.16K 671.688 R
K
1/ 2 1/ 2
L G 0.0367
Fv 1.49846 0.03713
G L 59.83
From Fig. 10.25, Ordinate at flooding = 0.18
1/ 2
0.18 G L gc
G flood 0.2
, gc 32.2, F 110 Table 10 3
F
1/ 2
0.18 0.0367 59.83 32.2 1/ 2 lbm
G flood 0.2
0.15147 0.3892
110 1.0 0.26 s ft 2

Where w 100 0.26 cp from Perry’s p. 2-323.


G actual 0.80 G flood 0.80 0.3892 0.31136
V in lb s 0.55268
Area 2
1.77505 ft 2
G actual lb s ft 0.8 0.3892
1/ 2 1/ 2
4 area 4 1.77505
Diameter 1.503 ft
1/ 4 1/ 4
Fint 33
b. F = 33, Diameter Intalox Dia Berl
1.503 1.112 ft
plastic FBerl 110

264
WL v
10.D15. Fv 0.03713 from prob. 10.D14
Wv L

From Fig. 10-16 with 12″ tray spacing , Csb,flood 0.21


0.2

K Csb where σ = surface tension water at 100ºC.


20
10 5 dynes 1m dynes
Perry’s, 7th ed., p. 2-306 @ 373.15 K, 0.0589 N m 58.9
1N 100 cm cm
0.2
K 0.21 58.9 20 0.21 1.2411 0.2606

L v 59.83 0.0367
u flood K 0.206 10.52 ft s , u act .8 10.52 8.416 ft s
v 0.0367

lb
4V
s
Dia Eq. (10-14) modified for units. V is from Problem 10.D14.
lb ft
v 3
u act
ft s

4 0.55268
Dia 1.637 ft . This can be compared to 1.5 ft for packed.
0.85 0.0367 8.416
Tray columns with this small a diameter are seldom used in industry.

10.D16. From Eq. (10-44). Diameter F1/4 . F1 98 and F3 22 Table 10-3


1/ 4 1/ 4
F3 22
Diameter (3 ) Diameter (1 ) 14.54 10.0 ft
F1 98
Can also repeat entire calculation which is a lot more work.

10.D17. At the bottom of the column have essentially pure n-heptane. Then, following Example 10-4,
we have.
p MW 1 100.2
v 0.205 lb ft 3
RT 1.314 371.4
1 y
Need L V . Since L V .8 where z .5, we have y 0.6 at intersection of
1 z
0.6 0
operating lines. Then L V 1.2 .
.5 0
L L MWL
1.2 1.0 1.2
G V MWv
1/ 2 1/ 2
L v 0.205
1.2 0.084
G L 1.684 62.4
Figure 10-25, Ordinate = 0.05 at ∆p = 0.5

265
0.05 .205 0.684 62.4 32.2
G .2
0.375
98 .9595 .684 0.205
The value 0.9595 is Water
at 98.4ºC, and Water L
. Since feed is a saturated liquid
V V 0.6944 lbmol/s.
VMWv 0.6944 100.2
Area 185.5 ft 2
G 0.375
1/ 2
D 4 Area 15.37 ft
This is somewhat larger than in Example 10-4. Therefore design at bottom.

G L gc ordinate
10.D18. G flood 0.2
F
Assume changing p changes only G
& ordinate. Then take ratios
G flood ,new G ,new ordinate, new
G flood ,old G ,old ordinate, old

G ,new
p MW RT new p new
4 . Assumes small change in
G ,old p MW RT p old
old

T (in Kelvin). T set by boiling conditions (Vapor Press) not by ideal gas law.
0.5 0.5 0.5
L G
F v ,new G ,new p new
Fv , 2.0 , F v,new 2F v,old 0.4 .
G L F v ,old L ,old p old
New Ordinate Value ~ 0.5, old value ~ 0.09
p new ordinate, new 0.05
G flood ,new G flood ,old 4 0.5 0.75
p old ordinate, old 0.09

10.D.19. New Problem in 3rd edition. Saturated vapor feed in problem 10.D.9 has minimum L/V =
(0.999-0.6)/(0.999-0.22)=0.5122.
This is (L/D)min = 1.05. The actual L/V = 0.6, which is an L/D = 1.5. Thus, the multiplier M of the
minimum was M = (L/D)/(L/D)min = 1.5/1.05 = 1.43. For a saturated liquid feed (L/V) min = (0.999-
0.825)/(0.999-0.6) = 0.4361, which corresponds to (L/D)min = 0.7733. If we use the same multiplier as in
10.D9, L/D =1.43(0.7733) =1.106 and L/V = 0.525. This is the slope we use in the top section for the 2-
enthalpy feed. In the middle section of the column at minimum reflux conditions the slope of the middle
operating line is L / V (0.825 0.6) / (0.6 0.22) 0.592 .

The external mass balances still gives D 596.56 kgmoles/hr, B 403.44 . At minimum reflux
Lmin ( L / D) min D 0.7733(596.56) 461.3 and Vmin Lmin D 461.3 596.56 1057.86 . At
the saturated liquid feed V V and L L Fliquid . Thus,
[( L Fliquid ) / V ]min ( L / V ) min 0.592 and Fliquid ,min 0.592V L 165.06 .
Since the total feed rate is 1000, the fraction liquefied is 0.16506. The same fraction can be liquefied at
the finite reflux ratios. Thus, Fliq 165.06 and Fvap 834.94 .

266
At top use saturated liquid reflux ratio L V 0.525, V L D V 0.525V D
D 596.56
V 1255.9 kmol h
0.475 0.475
This is the same as for problem 10.D13 and the remainder of the calculation of the diameter is identical to
that calculation. The result of the calculation at the top of the column is
4 82, 330 lbm h
D 9.08ft
ft 3
0.90 0.07219 lbm ft 0.75 7.25 3600 s h
s
We now need to calculate the vapor flow rate in the bottom. Assuming CMO, in the middle section
V V 1255.9 . In the bottom section,
V V Fvap 1255.9 834.94 420.96 .
Since QR V , QR ,2 enthalpy _ feed (420.96 / 491.7)QR , vapor _ feed 0.86QR , vapor _ feed .
Thus, in this case the two-enthalpy feed design results in the same reduction in diameter as liquefying the
entire feed, and it has energy savings compared to the vapor feed. However, the two enthalpy feed
system will require more stages than the other systems. A complete economic analysis is required to
determine the most economical system.

10.D20. Use Fig. 10-16 to find C sb . Gas is N 2 . Liquid is ammonia.


Since system very dilute, treat as pure ammonia liquid & pure N 2 gas.
L WL kg h L kmol h MWL 17.03
27.36 16.642
G WG kg h V kmol h MWV 28.08
3
g 1 kg 100 cm
L 0.61 3 3
610 kg m 3
cm 1000 g m

pMW v 175 atm 28.02 g mol 1000L kg kg


G 3
236.0
RT L atm m 1000 g m3
0.08206 253.2 K
mol K
1/ 2
236
Fv 16.642 10.35
610
Off chart. Extrapolate using Eq. (10-10e).
2
log10 Csb 0.94506 0.70234 log10 10.35 0.22618 log10 10.35
log10 Csb 1.891 Csb 0.01286
0.2
Assume 20 1.0. Then
610 236
u flood 0.01286 0.0162 ft s , u op 0.75 uflood 0.01215
236
4 V MW V
D ,V 100 kmol h , MWv 28.02 kg kmol
v u op 3600
0.85, v 236.0 kg m3 , u op 0.01215 ft s , Need to watch units

267
4 100 28.02
D =1.155 m 3.79 ft
0.85 236.0 0.01215 3600 1 3.2808 ft
Probably use 4 ft diameter column – (standard size)
Using larger diameter helps take into account the uncertainty in extrapolating to find C sb .

10.D.21. New Problem in 3rd edition. The mass balance and flow rate calculations are the same as for
problem 10.D14.

z xB 0.4 0.0001
D F 1000 400.7415 , B 1000 D 599.258 kmol/day
xD xB 0.998 0.0001
V L D 1 D 3 400.7415 1202.225 kmol day
kmol 1 day 1h
or V 1202.225 0.013915 kmol s
day 24 h 3600 s
L/V = 2/3. Top of column is close to pure methanol
Thus L G L V 0.66667 G is lb (s ft 2 )
lb kmol 32.04 kg 2.20462 lb
V in 0.013915 0.98290 lb s
s s kmol 1 kg
1.8 R
Pure MeOH boils at 64.5 C 337.66K 607.79 R
K
1 kg 2.2046 lbm 28317 cm 3 lbm
L MeOH 0.7914 g cm 3 3
49.405
1000g kg ft ft 3

W 59.83 / 49.405 1.211


L
Assume ideal gas. Top of column is essentially pure MeOH.
lb
1 atm 32.04
n MWM p lbmol lbm
v MWM 0.07219
V RT ft atm 3
ft 3
0.7302 607.79 R
lbmol R
1/ 2 1/ 2
WL G 0.07219
Fv .66667 0.02548
WG L 49.405
From Fig. 10-25, Ordinate at flooding = 0.20
1/ 2
0.20 G L gc
G flood 0.2
, gc 32.2, F 110 Table 10 3
F
1/ 2
0.20 0.07219 49.405 32.2 1/ 2 lbm
G flood 0.2
0.2146 0.4633
110 1.211 0.34 s ft 2

Where methanol 64.5 C 0.34 cp from Perry’s (8th ed.) p. 2-449.


G actual 0.80 G flood 0.80 0.4633 0.3706

268
V in lb s 0.98290
Area 2
2.6521 ft 2
G actual lb s ft 0.3706
1/ 2 1/ 2
4 area 4 2.6521
Diameter 1.838 ft

Note that this is larger than the calculation of 10.D14 at the bottom of the column. Thus, do calculations
at top of column.
1/ 4 1/ 4
Fint 33
b. F = 33, Diameter Intalox Dia Berl
1.838 1.360 ft
plastic FBerl 110

D kmol L 2
10.E1. op time where D 18.1303 kmol , , L V 0.4
D kmol hr D 3
D
Then D V L V 0.4V 0.6V and t op
0.6V
Use Fig. 10-25 or Eq. (10-39a) to find flooding at the end of the operation at bottom of column.
kg
2 MWliq
L lbm s ft L kmol h kmol
G lbm s ft 2
V kmol h kg
MWvapor
kmol
At end of operation at bottom of column x 0.004, y 0.036 (pinch)
L 18.128
0.4 0.381
G 19.023
MWavg,liq 0.004 46 0.996 18.016 18.128 & MWvap 0.036 46 0.964 18.016 19.023

L 62.4 lbm ft 3 (Essentially pure water). Boils at ~ 100ºC = 373 K


lbm
1.0 atm 19.023
p MW v lbmol lbm
G 0.038806
RT atm ft 3
ft 3
1.314 373 K
K lbmol
1/ 2 1/ 2
L G 0.038806
Then FV 0.381 0.009501
G L 62.4
From Eq. 10-39a.
2
log10 ordinate 1.6678 1.085 log10 0.009501 0.29655 log10 F v
log10 ordinate 0.6864 & ordinate 0.2059 agrees with Fig. 10 25
2 0.2059 G L gc
G 0.2
F
From Table 10-3, F 70, 1.0, g c 32.2, water 100 C ~ 0.26 c p (Perry’s 5th ed., 3-213)

269
0.5
0.2059 0.03698 62.4 32.2 lbm
G flood 0.2
0.53488
70 1.0 0.26 s ft 2

lbmol Area ft 2 lbm


From Eq. (10-41), V G
s lb s ft 2
MWvapor
l bmol
2
12
Area 0.19635 ft 2 and G act 0.7 G flood
4
lbmol 0.19635 0.53488 0.7
V 0.0038646
s 19.023
kmol lbmol 3600s 0.453593 kmol
V 0.0038646 6.310665 kmol h
hr s h 1.0 lbmol
D 18.1303 kmol
Then t op 4.7883 h 287.3 min.
0.6V 0.6 6.310665
yM 1 x M
10.F1. Need average . From Equil. data
x M 1 yM
0.134 0.98 0.979 0.05
Bot : 7.582 , Top 2.454
0.02 0.866 0.95 0.021
Geometric avg 7.582 2.454 4.31
Column temperature varies from 64.5º to ~ 98.2 ºC.
64.5+98.2
Avg T= 81.35 C
2
liquids
from Perry’s 7th ed., T = 81.35ºC, p. 2-323. M 0.28 cp, W 0.35 cp
(Note 40% MeOH is probably wt%)
Estimate: n mix x M n M x W n W
Feed is 60% M 40% W
n mix 0.60 n 0.28 0.40 n 0.35 1.1837 , mix 0.306
Then mix 4.31 0.306 1.3195
From O’Connell’s Correlation, Fig. 10-14, Eo 45%
2
Eq. (10-6): E o 0.52782 0.27511 log 10 1.3195 0.044923 log 10 1.3195 49.5%
If conservative use 45%

10.F2. To use O’Connell’s correlation (Fig. 10-14), need α and viscosity of feed.
KM yM x M yM 1 x M
MW
. Used Table 2-7 for values.
K W yW x W x M 1 yM
Can estimate a geometric average at bottom, feed & top
0.134 0.98 0.729 0.6
M W ,bot 7.582 , MW ,feed x .4 4.035
0.02 1.866 0.4 0.271

270
0.979 0.05 1/ 3
MW ,top 2.454 , avg bot 4.2184
F T
0.75 0.021
Averages can be calculated many other ways.
The feed is saturated liquid. From Table 2-7, T = 75.3ºC
Viscosities from Perry’s, p. 2-323, W 0.39cp & MeOH 0.30
Note: (MeOH, 40% probably refers to wt % - p. 2-322 Perry’s)
Estimate n mix x1 n 1 x 2 n 2
n mix .4 n 0.30 .6 n 0.39 1.0465 , mix 0.351
Then Feed 4.2184 0.351 1.481
Overall Plate effic. = 43.7%

z xB F 0.30 0.01 100


10.F3. D 36.71 B F D 100 36.71 63.29 lb mol h
xD xB 0.8 0.01
At top of column L = D(L/D) = 73.4 and V = L + D = 110.1
L F F3 L
Stripping section L L qF where q 4 3
F
L 206.7, V L B 143.3
Feed line has slope 4/3 and goes through y x z .3 . Top operating line has slope L/V =
0.667 and goes through y x xD 0.8 . Bottom operating line goes through
y x x b 0.01 and the intersection of top operating line and feed line. McCabe-Thiele
solution is shown in Figure. Optimum feed is 8th from top. Need 8 7/8 equilibrium stages
plus partial reboiler.

271
Overall Efficiency. For O’Connell Correlation, need AVG and Feed Tcol
yE 1 x E
. Using Table 2-1 we find α and following mole fraction.
1 yE x E
0.170 .981
x 0.019, y 0.170 : 10.575
.830 .019
.5826 .6727
x 0.3273, y .5826 : 2.87
.4174 .3273
.7815 .2528
x .7472, y .7815 : 1.210
.2185 .7472
1/ 3
AVG 1 2 3 3.324
Can estimate μ from p. 99 Ethyl Alcohol Handbook at z .3 .523 wt.
frac., 0.55 cp. Thus αμ = 1.83. From O’Connell Correlation E o .42 .
N 8.875 .42 21.1 . Thus, need 22 stages plus partial reboiler
Height 22 18 18 disengagement 48 (bottom sump) 38.5 ft

4V MW V
Diameter Calculation Dia
v flooding fraction u flood 3600
Use average values of parameters in stripping and enriching sections.
MW v MW eth
yeth MW W
y W for both MW V and MW L .
stripping section: 18.25 MW 21.5

WL L MW L 206.75 20 4135 lb h

WV V MW V 143.46 20 2869.2 lb h

0.96225 g ml 60.07 lb ft 3 Bottom


L
Bottoms T =100ºC = 672ºR
V
P MW V RT 1.0 20 0.7302 672 R 0.04076 lb ft 3

Fv WL WV V L 4135 2869.2 0.04076 60.07 0.0375, Csb 0.28 .


Enriching section: 26.4 MW 40.4

WL L MW L 73.42 35 2569.7

WV V MW V 110.13 35 3854.55 lb h

L0.766 g ml 47.92 lb ft 3 Distillate


Distillate T = 82ºC = T = 639.6ºR;
V P MW V RT 1 35 0.7302 639.6 R 0.05472 lb ft 3

272
Fv WL WV V L 2569.7 3854.55 0.05472 47.92 0.0225, Csb 0.28

0.2
K Csb 20 , ft/sec. σ, surface tension in dyne/cm. 57th ed. Hdbk of Physics + Chemistry, F-45.
Bottoms, σ ~ 46 dyn/cm, Middle, σ ~ 25 dyn/cm, Top σ ~ 18.6 dyn/cm
0.20
stripping: K 0.28 46 20 0.33075 ft s
0.20
enriching: K 0.285 18.6 20 0.2809 ft s
u flood K L V V , ft s

60.07 0.04076
stripping: u flood 0.33075 12.693 ft s
0.04076
47.90 0.05472
enriching: u flood 0.2809 8.31 ft s
0.05472
1/ 2
Dia. 4V MW 0.90 V 0.75 u flood 3600
V

stripping: V MW V V 143.46 lbmol h 20 0.04076 lb ft 3 70392.5

enriching: V MW V V 110.13 lbmol h 35 0.05472 lb ft 3 70441.37


Diameters: stripping section: Dia = 1.7 ft and enriching section: Dia = 2.1 ft
Probably use 2.5 ft diameter since there is little if any cost penalty.

10.F4. Numbers from solution of Problem 10.F3 are used.


V lbmol s MWV lb lbmol
Cross Sectional Area
G lb (s ft 2 )
Bottom: L G L V MWL MWV 1.44 ~ 1 1.44
Top: L G L V MWL MWV 0.667 ~ 1 0.667
1” metal Pall rings: F 48, 0.15, 0.15
ψ = Density of water/density of liquid At Top: 61 47.92 1.27
At 81ºC , w 0.35 cp, E 0.45 cp , n MIX x1 n 1 x2 n 2

At top y= x d 0.8, n MIX .8 n .45 .2 n .35 , and MIX 0.43


MWv y MWE 1 y MWw 0.8 46 .2 18 40.4

G 0.05472 lb ft 3 , L 47.92
1/ 2 1/ 2
L G G L 0.667 0.05472 47.92 0.02254
2 0.2
G F G L gc 0.078 from Figure 10-25
1/ 2
0.078 gc 0.078 0.05472 47.92 32.2
G G L
0.2 0.2
0.358 lbm (s ft 2 )
F 48 1.27 0.43
V 110.13 mol h 0.0306 lbmol s

273
V MW v
Area D2 4 , or
G
1/ 2
1/ 2
4V MW v 4 0.0306 40.4
D 2.096
G 0.358
Probably use 2 or 2.5 foot diameter columns.
The calculation at the bottom of the column gives a smaller diameter.
HETP N Height of Packing, or 1.2 ~ 10 12.0 ft

10.G.1. New Problem in 3rd edition. The result from Wankat (2007a) is listed in the following Table:
Results for distillation of vapor feed 5 mole % methanol, 95 mole % water. Distillate is 0.9543 mole
fraction methanol and bottoms is 0.9976 mole fraction water. Tray spacing = 0.4572 m. Base case
conditions are listed in Tables 1 and 2 in Wankat (2007a). When two trays are listed, they have the same
diameters. The decrease in volume and increase in QR are compared to the base case.

FL NF,V NF,L N dia A Vol tray QR Qc, total decr Vol incr QR
0(base) 10 -- 20 2.84 6.33 55.0 2 1065 -12,330 -- --

Two-enthalpy feed:
500 11 6 20 2.08 3.38 29.4 2 1070 -12,340 46.5 % 0.5 %
500 12 6 22 2.07 3.38 32.5 2 1065 -12,330 40.9 % 0
600 12 6 20 1.89 2.80 24.3 2 1070 -12340 55.8 % 0.5 %
750 13 6 20 1.56 1.92 16.7 2 1090 -12,360 69.6 % 2.3 %
750 16 7 26 1.56 1.91 21.8 2 1065 -12,330 60.3 % 0
1000 (all liquid) 9 20 1.20 1.13 9.8 2 2415 -13,680 82.1% 127 %

Intermediate condenser:
FWithdr NF,V NV,with NL,ret N dia A Vol tray QR Qc, total decr Vol incr QR
300 11 10 6 20 2.41 4.56 39.6 2 1067 -12,340 28.0 % 0.2 %
450 11 10 6 20 2.22 3.88 33.7 10/11 1067 -12,340 38.6 % 0.2 %

Two-enthalpy feed (FL = 600 kmol/hr) plus one intermediate condenser:


Fwithdr NF,V NF,L NV,with NL,ret N dia A Vol tray QR,total Qc decr Vol incr QR
100 12 6 5 5 20 1.69 2.23 19.4 2 1073 -12,340 64.7 % 0.8 %
80 12 6 13 6 20 1.72 2.33 20.2 2 1079 -12,345 63.2 % 1.3 %

Two-enthalpy feed (FL=600 kmol/hr, NF,V =12, NF,L=6, N=20) plus two intermediate condensers:
Fwthd1 NVwth1 NLret1 Fwthd2 NVwth2 NLret2 dia A Vol tray QR,total Qc decr Vol incr QR
100 5 4 80 13 8 1.50 1.77 15.4 2/6 1081 -12,350 72.1 % 1.6 %

Two-enthalpy feed (FL = 680 kmol/hr) plus one intermediate condenser:


Fwithdr NF,V NF,L NV,with NL,ret N dia A Vol tray QR,total Qc decr Vol incr QR
100 12 6 5 5 20 1.51 1.79 15.5 6 1081 -12,350 71.7 % 1.6 %

With constant Qc and QR, the two-enthalpy feed with FV = 750 and N = 26 appears to be best.

10.G.2. New Problem in 3rd edition. Results are from Wankat, P. C., "Balancing Diameters of
Distillation Column with Vapor Feeds," Ind. Engr Chem. Research, 46, 8813-8826 (2007).

274
Table 1. Simulation conditions and results for base cases. F 1000 kmol / h, D = distillate flow rate
kmol/h, N = number of trays + condenser + reboiler, tray spacing = 18 inches = 0.4572 m,
operation at 80% of flooding, dia = maximum calculated diameter in m, tray = tray at which max
diameter occurred. A = max calculated column area in m 2 , Vol = column volume in m 3 = A(N
– 2 +1) (tray spacing) where N – 2 is the number of trays and +1 is for disengagement space for
liquid and vapor, Q R and Q c = reboiler and condenser duties in kW, p cond condenser pressure
in atm, p pressure drop in psi/tray, N feed = optimum feed stage, and condenser is stage 1.
Note this solution has a p, each stage. Thus, solution slightly different than students’ solutions.

N Feed N Dia A Vol tray Q R Q c , L1 D D p cond p


Ethanol (10 mole %). Water (90 mole%) Vapor Feed Base Case:
23 26 2.61 5.35 85.6 2 902 -10,700 8.0 125 1.80 0.1

Table 2. Diameters calculated for standard distillation base cases listed in Table 1. Vapor flow
rate Vj and liquid flow rate L j are in kmol/hr, diameter is in m, area is m 2 .
Ethanol-water, vapor
Tray Vj Lj Dia Area
2 1125 997 2.61 5.35
23 1082 949 1.99 3.11
24 74.3 950 0.71 0.396
35 79.4 956 0.71 0.396

Table 3. Simulation conditions and results for a distillation column separating a vapor 10 mole %
ethanol, 90 % water feed (see Table 1 for base conditions). Partial condenser is stage 1. N F,V and N F,L
are optimum feed locations for vapor and liquid portions of the feed, respectively. Decrease in column
volume Vol (equal to change in area when the number of stages is unchanged) and increases in Q R are
compared to the ethanol-water base case (Table 1). For both runs y D,E 0.7901 and x B,W 0.9986.
FL N F,V N F,L N Dia A Vol Tray Q R Q C,col
0(base) 23 -- 36 2.61 2.24 35.8 2 902 -10,700

FL N F,V N F,L N Dia A Vol Tray QR Q C,tot Decr Vol Incr Q R


600 23 17 36 1.69 2.24 35.8 2 902 -10,700 58.2% 0
Qc,total Qc,col Qc,feed condenser

10.G3. New Problem in 3rd edition. Part a. S Dia = 2.032 m. Distillate mole fractions (vapor) = 0.22222
Eth, 0.77765 Propane, 0.12383 E-03 B, and 0.28503 E-9 pentane. Bottoms mole fractions = 0.14843 E-
10 Eth, 0.10132 E-03 Propane, 0.81808 Butane and 0.18182 pentane. Other values are in Table for 10.G4.
Part b. Worst backup is 0.232 m on stages 30 and 31. Maximum weir loading is 0.0204 m2/s on plate 31.
Part c. Same mole fractions, same Qc and QR. Max backup 0.1614 m in panel A on stages 29 to 32.
Maximum weir loading is 0.01183 m2/s on plate 31 of panel A which is acceptable.

10.G4. New Problem in 3rd edition.

275
V feed L feed Qc kW QR kW Max Dia Stage yD,C4 xB,C3
Kmol/s Kmol/s m max dia
0* 1 pass .1(NF=16) -1463 2827 2.032 31 .000124 .000101
0 part d .1(NF=15) -1463 2827 2.032 30 .000115 .0000942
0* 2 pass .1(NF = 16) -1463 2827 2.032 31 .000124 .000101
.01 .09 -1463 2600 1.956 31 .000145 .000119
.02 .08 -1463 2373 1.876 31 .000199 .000163
.03 .07 -1463 2147 1.792 31 .000300 .000246
.04 .06 -1464 1921 1.905 32 .000525 .000429
.05 .05 -1466 1696 1.650 18 .00112 .000915
.06 .04 -1472 1474 1.600 18 .003188 .002609
b. Change N=41 NF,liq=18 NFvap=21
N
.03 .07 -1463 2146 1.793 34 .0000817 .0000668
* Values from problem 10.G3.
Part c. Tray rating program with Dia = 1.793 m and defaults for tray spacing (0.6096m) & for DC
clearance (0.0373m) obtain 0.2207 m backup on tray 34, which is acceptable. Maximum weir loading is
0.01887 m2/s on tray 34 which is acceptable.
Part d. Shown above, plus maximum backup is 0.2320 on plate 31 (acceptable) and maximum
weir loading is 0.0204 m2/s on plate 31, which is marginal.

276
SPE 3rd Edition Solution Manual Chapter 11

New Problems and new solutions are listed as new immediately after the solution number. These new
problems are: 11.A19-11.A22,11.B6, 11.C3, 11.D5,11.D8, 11.G3-11.G4.

11.A3. As feed temperature increases L D MIN


increases, hence L/V increases and Q c increases.
L V increases and Q R decreases. The number of stages probably decreases.
(See Figure 7-3. The abscissa increases as L D min
and L/D increase. Thus ordinate drops.
Since N MIN is constant, N decreases).

11.A.6. New Problem for 3rd edition.


b. A liquid side-stream between the feed and the distillate. 1 point for b.
c. A vapor side-stream between the feed and the bottoms. 2 points for c.

11.A.9. If the feed rate is consistently one half the design capacity, the entire economy of scale will be
lost. In addition, distillation columns probably operate at lower than design efficiencies.

11.A11. Column 2 would have to be at a lower pressure than column 1.

11.A.12. New Problem for 3rd edition. The heuristics to not do would include items such as:
1. Remove dangerous, corrosive, and reactive components last.
2. Use distillation only for very difficult separations (α < 1.10). And so forth.
X,T 0.33
11.B4. Use heuristics. Hardest separation is xylene-cumene, x ,c 1.57
C,T 0.21
Toulene is most abundant. Use heuristics to:
Remove toluene early
Do hardest separation last
One-by-one in overhead

If use equil-molal splits:

B T X

F
C

Can also derive coupled systems.

277
B X

F
T
C

11.B5. Add in Heuristic 9 for sloppy separation.


T T
B B

X
F OR F

C C

11.C.1. New Problem for 3rd edition. Take the log of both sides of Eq. (11-2).

log (cost A/cost B) = (exponent) log (size A/size B)

log (cost A) – log (cost B) = (exponent) [log (size A) – log (size B)]

exponent = [log (cost A) – log (cost B)]/ [log (size A) – log (size B)]

11.D1. NOTE: This solution requires the solution to Problem 10-D1. Estimate α at feed composition
x = z = 0.5
1 K c7
a&b. For binary, x c6 , y c6 K c6 x c6 . Use Fig. 2-11.
K c6 K c7
T = 168ºC: K c6 1.29, K c7 0.71, x c6 0.50, y c6 0.645
(Guess was aided by solution to Problem 10-D1)
.645 .5
1.817
.355 .5
.999 .001
n
.001 .999
N MIN 23.13 stages
n 1.817
L x D y 99 .645
0.7094
V min x D z .999 .5

278
L L V
2.441
D min 1 L V min

L D L D 4 2.441
c. Gilliland: x MIN
0.3117
L D 1 5
Eq. (7-42b): N N MIN N 1 0.37027
N 0.3703 23.13 1 0.3703 37.3 contacts or 36.3 stages
d. N act N eq E o 36.3 0.73 49.7 or 50 stages
24” tray spacing × 50 trays = 100 ft
+ 4 disengagement
H = 104 ft = 31.7 m
m
D 9 ft 2.743 m
3.2808 ft
D2
Tray area 5.91 m 2 . Column Vol area H 187.3 m 3
4
cos t $700
Fig. 11-1. $700 m 3 , C P,tower 3
187.3 m 3 $131,110
vol m
Tray cos t
Fig. 11-2. $710 m 2 , C P, tray 710 5.91 m 2 $4196 tray
area
Eq. 11-8. CBM,tower 131,110 2.25 1.82 2.3 1.0 $844, 000
pD 7 2.743
700 kPa 7 bar, Eq (11-5), Fp .5 .5 2.30
10.71 0.00756p 10.71 0.00756 7
$ 209,800
Fig.11-9. C BM.tray 4196 50 1 1
total $1,054,000

11.D2. 69ºC (bp) = 156.2ºF. F 1000, D 500, B 500, L V 0.8, L D 4


L 2000 lbmol h, V 2500
Energy Bal: VH v Qc Vh D gives, Q c V ho Hv
h D is pure boiling hexane, H v is vapor.
Thus, h D Hv c6 13,572 Btu/lbmol
Qc 2500 13,572 3.39 107 Btu h
QR Dh D Bh B Fh F QC
Pick 25ºC as basis.
1.8 F
hD C PL ,c 6 69 25 51.7 44 1.8 4094.6
C
Btu
hB C PL,c7 98.4 25 1.8 50.8 73.4 1.8 6711.69
lbmol
Feed is a saturated liquid. From Example 10-1, T = 80ºC

279
hF CPL,c6 z c6 CPL,c7 z c7 80 25 1.8
hF 51.7 .5 50.8 .5 55 1.8 5073.75 Btu lbmol
QR 500 4094.64 500 6711.69 1000 5073.75 3.39 10 7
QR 3.423 107 Btu h
Q 3.423 10 7 Btu h
Reboiler: A 32, 800 ft 2
U TBtu 1.8 F
50
110 98.4 C
h ft 2 F C
where U is average from Table 11-2.
Qc 3.39 10 7
Condenser: A 2850 ft 2
U TAvg 110 70
180 156.2
2
Note these areas are very approximate. For detailed design need a much better estimate of U.
2
2 1m
Costs: Condenser A 2850 ft 2
264.8 m 2
3.2808 ft
Fixed Tube Sheet S&T Fig. 11-3, Cost = $125/m2
2
2 1m
Reboiler A 32,800 ft 2
3047 m 2 large because of low U.
3.2808 ft
Extrapolate $70/m2
1 atm, Fp 1.0, Fm 1.0 . Eq. (11-9) CBM Cp 1.63 1.66 Fm FQ 3.29 C p
$125
Condenser C BM 3.29 264.8 m 2 $109, 000
m2
$702, 000
Reboiler C BM 3.29 $70 m 2 304 m 2
$811, 000 Very sensitive to U.

11.D3. Note: This solution requires the solution to Problem 11-D2.


lb Q R 3.423 10 7 lb
Steam rate, 35, 704.6
h 958.7 h
where Q R is from Solution to problem 11-D2.
$ lb $20.00
Steam Cost 35, 704.6 $714 h.
h h 1000 lb
lb Qc 3.39 10 7 lb
Cooling water, 847, 500
h C p w Tw 1.0 40 h
where Q R is from Solution to problem 11-D2.
$ lb $3.00 1
Water Cost 847, 500 $306 h
h h 1000 gal 8.3 lb gal

280
11.D4. From Example 11-1 needed 21.09 eq stages + P.R.
h pack 21.09 1.1 ft stage 23.2 ft 7.07 m
3
D2 2 3 3 1m
Vol h pack 15 23.2 4099.6 ft 4099.6 ft 3
116.1 m 3
4 4 3.2808 ft
From Fig. 11-2 Cp ~ $250 m3 packing
Tower 23.2 ft + 2 ft between sections + 2 ft top + 2 ft top = 29.2 ft
1m
h = 29.2 ft 8.9 m , Vessel Vol. 146.1 m 3
3.2808 ft
Fig. 11.1 Cp $700 m3 for tower

Fp 1.0 1 atm , Tower FM 1 carbon steel , C BM C p 2.25 1.82 $700 146.1 4.07
$416, 312
Packing, Fm 4.1.
C BM C p 1.63 1.66 4.1 250 116.1 8.436 244, 855
Total $661,000
Does not include cost distributors, supports, hold down plates, etc.

11.D.5. New Problem for 3rd edition. n = [log (cost A) – log (cost B)]/[ log (size A) – log (size B)]
Let size A = 10 m2 and size B = 1.0 m2. The cost A = $400/m2 = ($400/m2)(10 m2) = 4000, and
cost B = $2100.
n = [log (4000) – log (2100)]/[log (10) – log (1)] = [3.602 – 3.322]/[1 - 0] = 0.28

11.D6. See residue curves in Figure. The recycle is pure MB. Mixing point is determined in same way
as in Fig. 11-11. Now mixing point splits into light (L) component methanol on B1 . Thus line
LM is extended to 0.0 mole fraction methanol to find location of B1 (0.73333 MB and 0.26667
toluene). We can use mass balance to find point B1 accurately. If D1 is pure methanol, D1 = 50
(all methanol in feed) and B1 = 150. Then from toluene balance 0.2 × 200 = 150 x tol,B1 , which
gives x tol,B1 0.266667 . B1 F2 which is then split into I (Some of which is recycled) and B 2
which is toluene product.
F Re cycle 100 100 M B1 D1
Since D1 contains no toluene B1 0.26667 F .4 Re cycle 0
.4
B1 100 150 kmo h. F2 , D1 M B1 50.0 kmol h
0.26667
For Column 2: B 2 essentially pure toluene, B2 0.266667 F2 0.26667 150 40
OK – Satisfies overall external M.B.
D 2 Re cycle F2 B2 150 40 110
D2 10.0, which also satisfies external M.B.

281
11-D7. a.) Proposed Split: Bottoms – Essentially pure toluene
Distillate ~ .83 methanol. (See figure for Solution to 11.D7)
F 100, z M 0.5, z MB .1, z T .4
Assume all toluene in bottoms & bottoms is pure.
B 100 .4 40, D 60
60 x M,dist 50 , x M,dist 0.8333 & xMB,dist 0.166667
b.) Proposed Split. Distillate pure M
Bottoms 0.2 MB, 0.8 T (See figure).
D 100 .5 50, B 50
Note – Doubtful this will work.

282
11.D.8. New Problem for 3rd edition. Part a.
Cost in June 2010 $947, 000 556 397 $1,326, 000
Part b.
F 2 x F of Example 11-2.
Since Dia F , Dia 2F11 2 12 ft 2 16.97 or 17 feet.
1m
Diameter 17 5.182m, Tray Area 21.08m 2 .
3.2808 ft
Tower Volume 488.2m3 Fig. 11-1, C0p Cost vol ~ $550 m3
CP,tower $550 m3 488.2 m3 $268,500
Fig. 11-2.
Tray cos t area ~ $750 m3 extrapolate , Cp,tray $750 m 2 21.08m2 $15,800 tray
CBM,tower 268,500 2.25 1.82 1.0 1.0 $1, 093, 000

283
CBM,trays $15,800 36 1.0 1.0 $569, 000
Total bare module cost September 2001 = $1,662,000
In June 2010, Cost $1, 662, 000 556 397 $2,328, 000
Part c. Original feed rate 1000 lbmol h .
At Foriginal , cost lbmol $1326 lbmol
At 2 x Foriginal , cost lbmol $1164 lbmol

11.G1. - Use Fig. 11-10b as flowsheet. Use NRTL.


Feed : 1 atm, sat'd Liq, 100 kmol/h, 0.5 MeOH, 0.4 T, 0.1 MB
Fed to Stage 30
Int. Recycle sat’d Liq, 100 kmol/h, 100% MB, fed to Stage 20

Block B1 : N = 46, total condenser, partial reboiler, D = 50, L/D = 3


Dist : 0.999759 MeOH, 0.000241 tol, 1.537 E 0.8 MB
Bot : 8.02 E 0.5 MeOH, 0.266586 tol, 0.73333 MB

Block B2: N = 85, L/D = 9, B = 40, feed = 41


Dist: 0.0001094 MeOH, 0.0008684 Tol, 0.999022 MB
Bot : 2.855 E 35 MeOH, 0.99731 tol, 0.0026888 MB
Thus, this is feasible.

11.G2. a) For Fig. 11-10A.


L Bottom 0.999186 tol
Col 1. N 90, N f 41 , 8 D 60
D 0.000814 MB

Dist. 0.99938 tol, 0.000605 MB,


Col 2. N 20, N f 10 , L D 2, B 10 Bot .996679 MB, 0.003088 MeOH,
0.000233 toluene
If increase L/D in column 1, Col 1. L D 9, N 90, N f 41, Bot. 0.99941 toluene.
Col 2. L D 2, N 24, N f 12, Dist. 0.99954 MeOH, Bot. 0.9976 MB

Which now meets specifications. Thus Figure 11-10a without recycle is feasible.

b.) For Fig. 11-10B – converged N = 30, L/D = 6


Dist. Col 1. 88.7% M & 11.3% T – No MB
(azeotrope)
Would not converge higher N.
Does not appear to work; thus, not feasible.

11.G.3. New Problem for 3rd edition. F 100, 10% Ethanol, 5.0 atm, Sat’d liquid feed
N = 10 includes partial reboiler, total condenser, D = 10, L D 2

284
P=1 NF = 5 QR = 52,948 xD,E = .72033 xB,E = .031085
NF = 6 QR = 53,029 .74564 .028263
NF = 7 .76332 .026298
NF = 8 53,148 .77539 .024957
NF = 9 53,175 QC = -78614.5 .78037 .024363←
NF = 10 52896.9 cal/s .69964 .033373

Pcol = 3 atm L D 2
NF = 8 QR = 67995.5 .75453 0.027275
NF = 9 QR = 68022.1 Qc = - 74274 0.76027 0.026637←
NF = 10 67794 .69695 .033672

Pcol = 5 atm L D 2
NF = 8 QR = 76435.3 .74276 .028582
NF = 9 7646 Qc 71692.8 .74840 .027955 ←
NF = 10 76264 .69411 0.033988

Size optimal feed columns. Sieve tray 1 uses 18 inch tray spacing at 85% approach flooding, Fair
calculation method for flooding.
Pcol 5 Max diameter tray 2 0.34867m
Pcol 3 Max diameter tray 2 0.38070m
Pcol 1 Max diameter tray 2 0.46429m

Part d. D1. 1.0 atm gives the best separation because the relative volatility is highest.
D2. The lowest Qreboiler is 1.0 atm. The effect of pressure on Qreboiler in this problem occurs
because the feed is always a saturated liquid at 5.0 atm. For the 5 atm column this feed remains a
saturated liquid and the feed line is vertical. At lower column pressures the feed flashes and is a two-
phase feed in the column. These feed lines have a negative slope. For the feed lines at lower pressures
the slopes of the bottom operating lines are steeper, which means lower boilup ratios, Vreboiler/B. This
means lower Qreboiler at the lower pressures. Another way to think about this is the flashing feed produces
vapor and thus less vapor is required from the reboiler.
D3. The lowest absolute value of Qcondenser is 5 atm. All columns have the same D and L/D.
Thus, V entering the condenser is the same. At higher pressures the latent heat of vaporization λ is lower.
Since Qcondenser = Vλ, the result is a lower absolute value of Qcondenser at the higher pressures.
D4. The smallest diameter column is at 5 atm. Vapor density is highest.

Part e. Increasing pressure above 1 atm for the same purity requires more stages, but smaller column
diameter. Thus capital cost initially goes down. Above 8 atm the column must be designed for high
pressure operation, which makes it more expensive. Operating cost may go up if a higher L/D is required
to achieve the desired purity.

11.G.4. New Problem for 3rd edition.


a. (L/D)min = 1.3962 → L/D = 1.5358. Obtain N (Aspen Notation) = 19 with Nfeed = 9 (on stage).
Distillate is 0.75056 ethanol and bottoms is 0.00005987 mole fraction ethanol. Q R = 569,172 cal/s,

285
Qc = - 443187 cal/s, Dia = 1.0755 m, A = 0.90843 m2, H = 11.5824 m, Vol = 10.52 m3

Part b. Note that balancing the flow rates to achieve the desired separation in each column is trial and
error. The easiest was to do this is to first find a flow rate that works for the high pressure column (note
that D changes every time the flow rate is changed). This gives a value for Q C,high pressure = - QR,low pressure.
Then design the low pressure column with this QR. Check that both columns work. The correct flow rate
into the high pressure column is between 570.6 and 577.275. The results are reported below with the
latter value:
High pressure column: F = 577.275 kmol/h, QR = 0.7555(569172) = 430,000 cal/s. N = 20, Nfeed = 9,
distillate is 0.75004 mole fraction ethanol and bottoms is 0.00009733 mole fraction ethanol. Q c = -
243,900 cal/s, L/D = 1.5618, T cond = 382.16 K, Dia = 0.79443 m, A = 0.49568, H = 12.192 m, Vol =
6.043 m3.

Low pressure column: F = 422.725 kmol/h, QR = 243,900 cal/s, N = 19, Nfeed = 9, distillate is 0.75087
mole fraction ethanol and bottoms is 0.00003829 mole fraction ethanol. Qc = -190,627 cal/s, L/D =
1.5803, Tcond = 351.56 K, Treb =373.16 K, Dia = 0.7014 m, A = 0.3864, H = 11.5824 m, Vol = 4.475 m3.

Part c. Energy requirement of multieffect distillation system was set at 75.55 % of the single column.
This is not optimized, but was set because it was known to work. Cooling is only in the low pressure
column of the multieffect system, and is significantly less than with only one column. The total volume
of columns is the same; however, this is misleading because volume in the single column would have
been less if it was designed at 3.0 atm. The capital cost will be higher for building two columns than one
larger one, but energy costs are less.

286
SPE 3rd Edition Solution Manual Chapter 12

New Problems and new solutions are listed as new immediately after the solution number. These new
problems are: 12.A5, 12.A6, 12.C2, 12.D1, 12.D3, 12D8, 12D13, 12D19, 12D21, 12D22, 12.G3.

12.A1. By raising T or dropping p can make gas desorb. The direction of transfer of solute
controls whether a column is a stripper or absorber. If operating line (on Y vs X) is
below equilibrium have a stripper.

12.A5. New Problem in 3rd edition. c. AspenPlus

12.A6. New Problem in 3rd edition. A. a. B. d. C. e. D. h. E. i

12.B1. Calculate: N or y out , or N for A and y out for B, or m, or N for A and m for B, or L/V,
or feed composition, or b, or m and b from 2 experiments, or Overall Efficiency.

12.B2. Two feeds, Sidestream, Reboiled absorber, Coupled absorber and stripper (see Figure
12-2), Interstage coolers (absorption) or heaters (strippers), Packed columns, Two
different solvents, Two different stripping gases, Add solid adsorbed to solvent (see
Chapt. 14). Cross-flow, Co-current flow, Combinations of flow patterns, etc.

12.B3. See Isom and Rogers (1994).


Eq 12 22
12.C4. Eo N equil N actual (10-1)
Eq 12 34
mV mV
n 1 E mV 1 n 1 E mV 1
L L
Eo
n ( L mV) n L mV
mV
n 1 E mV 1
L
Eo (10-4)
n mV L
QED

Ref. Lacks, D.J., Chem. Engr. Educ., 302 (Fall 1998).

x dy
12.C5. y , m 2
1 1 x dx 1 1 x
dy
When x 0, , b 0
dx
dy 1 1
When x 1, , b
dx
Apply Kremser as x 0 or x 1.0

yN 1 y1
12.C6. Graphically, L V MIN
where y N 1 & x N are in equilibrium
xN x0

287
yN 1 mx N b for linear.
L L
For absorbers m &1
V min mV min

L
Thus as N , 0
mV min

yN y1 L
Eq. (12-23) becomes 1
*
where y1* mx 0 b
yN 1 y 1 mV min

yN 1 y1 L yN 1 y1 L
or
yN 1 b V xN x0 V
x0 min min

m m
which agrees with graphical analysis.

H 1.186
12.D1. New Problem 3rd Edition. y x x 0.395 x
p tot 3
m 0.395 b 0

L y1 0.0002

x 0 .0001 unknown
unknown
1
L x0 Gy N 1 Gy1 L xN

y1* m x0 .395 0.0001 0.0000395


N

y*N unknown.
L xN G yN 1 0.004 Since x N unknown, 1

unknown
Eq. (12-30) or (12-31) easiest to use

mV y N 1 y1* mV
n 1
L y1 y1* L
Eq. 12-30 5
mV
n
L

39.5 0.004 0.0000395 39.5 39.5 39.5


n 1 n 1 24.67
L 0.0002 0.0000395 L L L
5
L L
n n
39.5 39.5

288
T & E → be sure L mV 1
L mV/L L/mV RHS
50 0.79 1.2658 7.58
75 0.52668 1.8987 3.90
65 0.60769 1.64557 4.67
63 0.62698 1.59494 4.895
62 0.637097 1.56962 5.015 OK
G 100
xN yN 1 y1 x0 0.004 0.0002 0.0001 0.006229
L 62
n1 0
There are alternative solution paths, but L = 39.5 is not valid, it becomes .
n1 0
Alternative: Trial-and-error McCabe-Thiele solution.

12.D2.
a) L & V constant. 97% rec. H2S 3% left in gas.
y out 0.03 0.0012 0.000036
p H 2S 423x
Equil.: y 169.2x
p tot 2.5
M.B. with Const. L & V: 0.0012 V 0.000036 V x out L
V 10
x out 0.0012 0.000036 0.001164 5.82E 6
L 2000
L L
b) See Figure. y x y in x out where y in , x out & y out , x in are on op. line .
V V
c)

Eq Note that L G min can


169.2 be calculated from a
yin 0.000012
crude sketch.
x* 7.09E 6
LG min
y
L 0.0012 0.000036
164.124
G min 7.09 E 6
L L
y out 0.0000036 200 M
G act G min M = 1.2186
0
0 x

289
d) L/G too high – L too high – Liquid too dilute.
Need much better solvent (e.g., MEA solution).

12.D3. New Problem 3rd Edition. Mass Balance:

H
Vy IN Lx IN y out V Lx , Equil., y x
p tot
Substitute equil. into M.B.
HVx HVx
Lx IN Lx L x IN x
p tot p tot
6
HVx 10.96 10 0.4 10
Solve p tot 6 6
0.274 atm
L x IN x 100 2 10 0.4 10

H 10.96 6 6
b. y out x out 0.4 10 16.0 10
p tot 0.274
Can also do graphically, with Kremser equation (trial and error) or by solving mass balance first,
L
y out x in x out 16 10 6 Then p tot H x out yout 0.274 atm.
V

12.D4. Mass bal. yin Vin Lin x in y out V Lx out

290
L L L
y out y in x in x out x in x out 12 0.0002 0.00001 0.00228
V V V
L L
Op. Eq., y x y in
x out
V V
point yin , x out 0.0, 0.00001 and point y out , x in 0.00228, 0.0002 are on op.
line. See graph.

Can also use Kremser equation for this problem.

291
12.D5. Since equilibrium is linear in weight ratio units can do either McCabe-Thiele or
Kremser solutions. For McCabe-Thiele solution know that points
Yout , Xin and Yin , X out are on operating line. McCabe-Thiele diagram is shown in
Figure. N = 5.9 stages. HETP = 10/.59 = 1.69 ft.

Kremser: Several different forms can be used. We will illustrate with Eq. (12-26)
written in ratio units.
*
YN 1 Yin 0.02, YN+1 mX out 1.5 0.06 0.09
Y1 Yout 0.50, Y1* mX in 1.5 .40 0.60
n .02 .09 .5 .6
N 5.88
n .02 .5 .09 .6
HETP h N 10 5.88 1.70 feet

12.D6. First, assume Nitrogen is an ideal gas: 1 lbmol = 359 cu ft at 0˚C and 1 atm
333.16
359 437 cu ft/lbmol at 60˚C
273.16
N 2 flow rate 2500 437 5.72 lbmol/h
Water flow rate: Ignore CO 2 in water. MWw 18
100, 000
5560 lbmol/h
18

292
H
Equilibrium at 60˚C: H = 3410, y = x 3410x
PTOT
L 5560
972.0 . External mass balance is:
V 5.72
L L L
y out x in y in x out x in x out 972. 9 10 6 0.00875
V V V
Can solve with either McCabe-Thiele diagram or with Kremser Eq. (12-34). The
McCabe-Thiele solution is shown in the figure. Note different scales on axes. Need 5
real stages.

Kremser: y N 1 0, y1 0.00875, y1* mx 0 3410 9.2 10 6


0.03137 , m V L =3.508

0 0.03137
n 1 3.508 3.508
0.00875 0.03137
N 5.07
n 1 .4 3.508 1
Probably use 6 real stages.

kg gas
12.D7. G Wt 0.828 wt frac air 1050 869.4 kg air/h
h
869.4
G mole 29.98 kmol air/h
29
y in 0.172 0.172 kg NH 3
Yin 0.2077
1 y in
1 0.172 .828 kg air
29
Yin ,molar 0.2077 0.3543
17
Inlet NH3 0.172 1050 180.6 kg NH 3
2% remains in gas 3.612 kg NH 3

293
3.612 17
Yout mole 0.007087
869.4 29
p NH 3 p, mmHg p, mmHg
Equilibrium data Table 12-2 - y mole
PTOT 1.30 760 988

kgNH 3 y
X wt p Ymole
kgW y mole frac 1 y
0.05 11.2 0.01134 0.011466
0.075 17.7 0.017915 0.01824
0.100 25.1 0.0254 0.02607
0.15 42.7 0.04322 0.04517
0.20 64 0.06478 0.06926
0.25 89.5 0.090587 0.09961
0.30 119 0.12044 0.13694
0.40 190 0.19231 0.2381
0.50 275 0.27834 0.3857
0.60 380 0.3846 0.6250

X in L LX wt
M.B. G mol Ymol GYout ,
17 kg NH 3
17
kmol NH 3
L wt
Ymole X wt Yout
17 G mole

Note: Units, although mixed, work in Mass Balance & in Operating equation. See graph.

Minimum Solvent
0.3543 0.007087 L wt ,min
Slope = 0.7279=
0.477 0 17 G mole
L wt,min .7279 17 29.98 370.99 kg W h
kg W
Actual Solvent, L wt 1.5 L min 556.48
h
L wt 556.48
Op Line Slope 1.092
17 G mole 17 29.98

294
295
12.D8. New Problem 3rd Edition.

X IN 0 y out 0.002 Yout 0.002004


L p tot 2 atm 760. 2 mmHg

L 1520 mmHg
G
F2 0.5 .475 air
L G
.05 mol HCl
0.05 mole frac HCl y 0.05263
.95 mol air
X out
F1 1.0, G=.8
.2 mol HCl
y1IN 0.20 mole frac HCl YIN =0.25
.8 mol air

Assume Water (not total liquid) flow rate is constant in both sections. Assume air flow rate (not total gas)
constant is each section. In bottom section G 0.8 mol air h . In top section
G 0.8 0.5 .95 1.275 mol air h. Keep X as kg HCl kg water (from equil. data).
Convert p to y (mole fraction) to Y (mole ratios).
kg HCl p kg HCl y
X p y Y
kg water p tot kmol air 1 y

0 0 0 0

0.0870 0.000583 3.8355 E-7 3.8355E-7

0.1905 0.016 0.00001053 0.00001053

0.316 0.43 0.0002829 0.0002830

0.47 11.8 0.007763 0.0078240.

0.563 56.4 0.037105 0.038535

0.667 233 .15329 .18104

0.786 840 .55263 1.2353

296
Y vs X equilibrium data is curved.
Using these units, we need L in kg water hr and G and G in kmol air hr , and we need to
convert the X terms from kg HCl/h to kmol HCl/h.
Top Operating Line GY (L / MWHCl )X IN (L / MWHCl )X GYout , X IN 0
L L
Y X Yout 0.002004 Goes through (0, 0.002004)
G(MWHCl ) 1.275(MWHCl )
Bottom Operating Line GY (L / MWHCl )X out GYin (L / MWHCl )X
L L
Y X YIN X out L 0.8MWHCl 0.25 L 0.8MWHCl X out
G(MWHCl ) G(MWHCl )
a) Feed line saturated gas at Y 0.05263. Two operating lines intersect at feed line.

Y1,IN 0.25 X *out

Sketch for Min L determination

Y
L (G (MWHCl ))

YF2 0.05263
X int er sec t
L /(G(MWHCl ))
0.002 Yout
X
*
From plot X OUT 0.69 (see figure)
L
Top operating line: Yout X int er sec t 0 YF2
(G(MWHCl ))
L
Bottom operating line: YF2 X *OUT X int er sec t YFIN
(G(MWHCl ))
Solve for X int er sec t G(MWHCl ) YF2 YOUT L. Then
L G(MWHCl )
YF2 X *OUT YF2 YOUT YFIN
(G(MWHCl )) L
L G G(MW) G(MW)
X *OUT YFIN YF2 YF2 YOUT L MIN *
YFIN YF2 YF2 YOUT
(G(MW)) G X OUT X *OUT
0.8(MW) 1.275(MW)
L MIN .25 0.05263 0.05263 0.00200
0.69 0.69
L MIN / MWHCl 0.22883 0.09355 .3224 kg water h

Since MWmin = 36.461, Lmin = 11.755 kg/h, L = 1.2407 Lmin = 14.584 kg/h. L/(MW)HCl = 0.40

297
298
b. M.B. F2 YF2 GYIN (L / MWHCl )X IN GYout (L / MWHCl )X out
F2 YF2 GYIN GYout
X out X IN
(L(MWHCl ))
0.475 0.05263 0.8 0.25 1.275 0.002
X out 0 0.5561 kg HCl kg water
14.584 / 36.461

(L / MW) 0.4
Top 0.3137
G 1.275
(L / MW) 0.4
Bottom 0.5
G 0.8
(L/MWHCl )
Top goes through X IN 0, Yout 0.002, Slope 0.3137
G
Arbitrary point for plotting: X .4, Y .3137 .4 0.1255
Y .1255 .002 0.1275
Bottom from X out 0.5561, YIN 0.25
To intersection Top Operating and Feed Line.
Need ~ 1.6 stages. Opt. Feed for F2 is Stage 1 (Feed 1 is at bottom.)
.25 0
Check Slope bottom 0.504 0K.
.5561 .06

299
Figure for 12.D8.

300
12.D9. Repeat 12.D2 with Kremser.
mV 169.2 2000
y mx m 169.2, b 0, 0.846 , L V 200
L 200 10
y1* m x0 b 0, x 0 x in 0 , y1 y out 0.000036, y N 1 0.0012

0.0012 0
n 1 0.846 0.846
0.000036 0
Eq. (12-22) N 10.69
n 1 0.846

Graphical solution was 10.4. Pretty close!

12.D10. Use Kremser equation such as


y A out y A* 1 L mV
out
N 1
y A in y A* 1 L mV
out

N 4, m 1.414, L V .65, y Ain 0, y A* m x Ain 1.414 .02 .02828


out

L mV .65 1.414 .459


Equation becomes: y Aout .02828 .02828 .552 .01267
Overall mass balance: yin V L x in y out V L x out

V V 1 4
x out y out y in x in .01267 0 .02 4.93 10
L L .65

12.D11. Any of the vapor forms of Kremser equation can be used but problem is trial and error.
For example, use Eq. (12-21) inverted for L m V 1
L 1.2
1 1
y1 y1* mV m
N 1
becomes, .27 5
y N 1 y1* L 1.2
1 1
mV m
Set up table and try values of m.

m 1.0 1.2 1.3 1.4 1.41 1.415


RHS .1344 0/0 .233 .2658 .2691 .2706

By linear interpolation m = 1.414. Note that m = 1.2 is a trap for the unsuspecting
student since L/(mV) = 1.0 and special form of Kremser is required.

12.D12. Note this requires information in Section 13.4.

301
L 796
L .796 1000 796 kg solvent/h , 0.0316
G All stages 25,190

.204 0.025 .0012


X in .256, X out 0.02564, Y1,in .001201
.796 0.975 1 .0012

Equilibrium, y = 0.04 x
x X y Y
.05 0.0526 .002 .002004
.10 .11111 .004 .00402
.15 .1765 .006 0.00604
.20 .25 .008 .00806
.25 .3333 .01 .0101

Plot weight ratios.


L L
Op. Line: Yj Xj Y0Xj 1
G G
Slope = - L/G = - 0.0316. Step off stages backwards (start w. stage N) since it is
different than other stages and we wouldn’t be sure when to switch if stepping off
forwards. Need 4 equilibrium stages.
Note: Can also plot y vs X, since y ~ Y and G ~ V

302
12.D13. New Problem 3rd Edition. Strip Vinyl chloride from water at 25ºC and 850 mmHg.
H 1243.84x
y x 1147.904x
p tot 850 760
Want 0.1 ppm water leaving. Entering air is pure, L 1 kmole hr.
x0 x IN
* y out y1
y out

N
1147.904
y IN 0 yN 1

x out
yN
5.0 x IN x0

xN x out 1 ppm

y1* y*out 1147.904 x IN 5739.52 ppm (mole)


L 5739.52 0 kmol
1171.33 G MIN 0.00085373
G mix 5.0 0.1 h
b and c. Want G 2 G MIN 0.00170746 kmol h air
L G 585.665 (See figure for part b – labeled HW5 Prob 1b). m = 1147.904.
c. Eq. 12-28 x *N 0
585.665 5.0 0 585.665
N n 1
1147.904 0.1 0 1147.904 n 25.0
4.78327
n 1147.904 585.665 0.672944

d. y out Lx IN Gy IN Lx out G
For L 1 kmol h , G 0.00170746 kmol h
L
y out x IN x out y IN 585.665 5.0 0.1 0
F
2869.76 ppm 0.00286976 mole frac.
Probably send waste gas to incinerator. Will require additional fuel to burn.
e. All concentrations are dilute enough that L G and equilibrium are straight and operation is very
close to isothermal.

303
304
12.D14. a) 95% removal CH4, 5% remains – Constant V: Yout 0.05 0.00129 0.0000645
CH 4

b.
CH 4 Eq.
yin 0.00129 p CH 4 3600 x CH 4
y CH 4 20.5714 x CH 4
p TOT 175
L V y CH 4 in
min 0.00129
y CH 4 x* 0.00006271
20.5714 20.5714

y out 0.000645
CH 4

x in x CH 4

0.00129 0.0000645
Slope L V min
19.5429
0.00006271 0
L V actual 1.4 L V min 27.360; L 27.360 V 2736.0
V V
c) Ext. bal. x out y in x CH 4 ,in y CH 4 ,out
CH 4 L CH 4 L
V 100
x CH 4 ,out
y CH 4 ,in y CH 4 ,out 0.00129 0.0000645 0.00004479
L 2736
d) Use methane values in Kremser eqn. (12-22) to find N
mV 20.57 100
m 3600 / 175 20.57, 0.75183; y1* mx 0 b 0
L 2736

0.00129 0
n 1 0.75183 0.75183
0.0000645 0
N 6.11 stages
n 1 0.75183
e) Now use Argon values with N = 6.11 to find y Ar,out & x Ar,out .
7700
m Ar 44.00, y Ar ,in yN 1,Ar 0.00024, x Ar ,in x Ar ,0 0.0
175
mV 44 100
1.6082, y1* mx 0 b 0
L 2736 Ar

7.11
0.00024 y Ar ,1 1 1.6082 1 1.6082
Eq. (12-23) 7.11
0.60846
0.00024 0 1
1
1.6082
y Ar,out y Ar,1 0.00024 0.00024 0.60846 0.00009397
V 100
x Ar ,out y Ar ,in y Ar ,out 0.00024 0.00009397 0.00000534
L 2736

305
0.00000534 2736
% Argon recovery in liquid 100 60.85%
100 0.00024

12D15. Need equilibrium data. From DePriester chart:


K C3 y C3 x C3 1.23 m C3 , K C4 y C4 x C4 0.34 m C4
L mV
Butane is a design problem: 5.882, 0.17
mV C 4 L C4
1.2% of the butane leaves as a gas. Thus,
*
y1,C4 0.006 0.012 0.0000072, y1,C4 m C4 x 0,C4 0

mV y N 1 y1* mV .0006 0
1 n .83 .17
L y1 y1* L .0000072 0
N 2.39
L n 5.882
n
mV
* L mV
Propane is simulation: y N 1,C3 0.0017, y1,C3 0, 1.626, 0.615
mV C3 L C3
N 1
L
1
yN 1 y1* yN 1 mV yN 1
*
, y1 0.000298
y1 y 1 y1 1 L mV 5.7034

12.D16. Was 12.D19 in 2nd edition.


H CO2 atm
. a.) Equil. y CO 2 x CO 2 , H CO 2 25 C 1640
PTOT mol frac
50 mmHg 50 760 0.06579 atm
PTOT 1.0 7
x CO 2 y CO 2 .00035 2.134 10
Feed
H CO 2 1640
H CO 2 1640
Equilibrium in column: y CO 2 x CO 2 x CO 2 24982 x CO 2
PTOT 0.06579

Basis: L = 1 kmol total/h. Assume L & G constant. Input = 2.1341 × 10-7 kmol CO2/h. 95%
removal = (.95) (2.1341 × 10-7) = 2.027395 × 10-7 kmol CO 2 h in outlet gas.
7
5% CO2 remains in liquid .05 2.1341 10 0.106705 10 7 kmol CO 2 h
7
0.106705 10 7
x in x out outlet liquid mole frac 0.106705 10
1 kmol h

b.)

306
7 3
y max out 24982 2.1341 10 5.3314 10
Slope 24982

y L y max y in 5.3314 10 3
0
7
V max x in x out 2.1341 0.10605 10
Slope
L Vmax
yin 0
x out x x in 2.1341 10 7 L 4
2.62968 10
V max

Since L = 1, 1
Vmin 4
3.803 10 5 kmol h
2.62968 10

c.) V 1.5 Vmin 1.5 3.803 105 5.704 10 5 kmol h


Conditions for Kremser eq. are satisfied.
CO2 Mass Bal: 2.1341 10 7 in 0.106705 10 5 out w. water 5.704 105 yCO2,out
7
2.027395 10 3
y1 y CO 2 ,out 5
3.5543 10
5.704 10
n xN x *N x0 x *0
Eq. 12-29 N
L
n
mV
3
yN y1 3.5543 10
x *N 1
0, x *0 1.4227 10 7

m m 24982
7
0.106705 10 0
n 7 7
2.1341 10 1.4227 10
N 5.3569
1.0
n 5
24982 5.704 10
L L
12D.17. y1* .0002 1.414 .0002828,
mV 14.14
Can use variety of forms of Kremser equation, but cannot easily use forms with y*N 1 since
y*N 1 mx N and x N is unknown and hard to calculate. Try Eq. (12-21).
N 1
L
1
yN 1 y1* .0083 .0002828 mV
*
36.91
y1 y .0005 .0002828 L
1
1
mV
Do by trial-and-error
L/mV 2 3 2.9 2.91
RHS 15 40 36.699 37.02

307
Linearly interpolate L/mV = 2.907. Then, L = (2.907) (14.14) = 41.10 kmol/h.

p 11.5 0.00757
12.D18. a. y 0.00757 , x 0.0127, K y x 0.596
p total 1520 0.0127

L, x 0 0 y1 0.0004, x1 .01, y .00596

1 L L L 100
y1 x yi x0 slope 23
VT V VT 150
VN 1 100 , VF 50, L 100
VF
y F 0.003 Ext. MB: Lx 0 VF y F VN 1 y N 1 Lx N VT y1

150 0.0004 50 .003 100 .0058


xN 0.0067
N 100

xN VN 1
yN 1 0.0058

L L L 100
Bot. of Column: y x yN 1 xN, Slope 1.0
V V VN 1 100

308
c. Minimum L.

xN
yN 1
Eq.
y
yF L min
L min 100 slope
VN 1
y1
L min L min
slope
VT 150
x0 x

Pinch is at y F . x F y F 0.596 0.003 0.596 0.00503


L min VT Slope min 0.003 0.0004 0.00503 0 0.51653
L min 0.51653 VT 0.51653 150 77.48 kmol h .

309
12.D19*. New Problem 3rd Edition. Found m and L/V in Example 12-1.
L/(mV) = (L/V)/m = 133/105.6 = 1.259, (mV)/L = 0.794, y1* = 105.6×0 = 0

If use Eq. (12-22), N = {ln[(1 - .794)((100-0)/(10-0)) + .794]}/ln[1.259] = 4.5

12.D20. a) 99% removal H 2 O , 1% left, .000024 1000 .01 0.00024 moles out in L
0.00024
x H 2Sout
0.00000024
1000
Moles H 2S out in gas = (.000024) (1000) (.99) = 0.02376
0.02376 0.02376
y H 2Sout 0.00691
V 3.44
H H 2S 26800
b) Equil. H 2S. y H 2S x H 2S x H 2S 1729.03 x H 2S , m H 2S 1729.03
p tot 15.5
H CO2 728
y CO2 x CO 2 x CO 2 46.9677 x CO 2 , m CO 2 46.9677
p tot 15.5
Can use Kremser eq. for H 2S design [dilute linear system]. For example, Eq. (12-28)

L x0 x *N L
n 1
mV xN x *N H 2S
mV
N
mV
n
L
L 1000 mV
0.16813, 5.9479
mV H 2S 1729.03 3.44 L H 2S

x*N yN 1 m 0, x 0 H 2S
0.000024, x N H 2S
0.00000024
.000024
n 1 0.16813 0.16813
.00000024
N 2.4807
n 5.9479
c) For CO 2 . x N unknown, x*N yN 1,CO2 mCO2 0, x 0 0.000038 Kremser (12-31)
mV
1
xN x *
N
L CO 2 mV 46.9677 3.44
* N 1
, with N 2.4807 and 0.16157
x0 x N CO mV L CO 2 1000
2
1
L CO 2

mV
1
L 1 0.16157
x N ,CO 2 x0 N 1
0.000038 3.4807
0.000031916 CO 2
mV 1 .16157
1
L
Little amount of CO 2

310
23.78
12.D21. New Problem 3rd Edition. Abs. y x 4.756x
5
23.78
Stripper y x 118.90x
0.2

y IN 0.00098 4.756 slope equilibrium


Abs
y
Slope L MIN G
x EQ 0.00098 4.756 0.00020605
0.00098 0.000079
Operating line Slope 4.5956
y out 0.000079 0.00020605 0.00001
L MIN 100 4.5956 459.56 kmol h
x
x IN 0.00001 L act 1.6 L abs,MIN 735.302 kmol h

735.302
L V Abs
7.353
100
L abs x abs,IN V y abs,IN y abs,out
External M.B. x abs,out
L abs
735.302 0.00001 100 0.00098 0.000079
x abs,out
735.302

x abs,out x strip,IN 0.00013253


7.353 mV
Kremser Eq. m 4.756, b 0, L mV 1.5460, 0.64681
4.756 L
yN 1 y IN 0.00098, y1 y out 0.000079

y1* mx IN 4.756 .00001 0.00004756


Eq. (12-22).
mV y N 1 y1* mV 0.00098 0.00004766
n 1 n .35319 0.64681
L y1 y1* L 0.000079 .00004756
N
L n 1.5460
n
mV

n 11.1216 2.40889
N abs 5.53
n 1.5460 0.4357

311
Stripper

118.90 = Slope Equilibrium

y EQ 118.90 0.00013253 0.015758


y

L
= Slope Operating line
VMIN
y IN 0
118.90x N x out 0.00001
x IN x out,abs 0.00013253 x0
118.90
L 0.015758 0
128.6037
V MIN 0.00013253 0.00001
Strip

735.302
VMIN ,Strip L Strip 128.6037 5.718 kmol h.
128.6037
VStrip 1.5 5.718 8.576 kmol h.

VStrip y Strip IN L Strip x Strip IN x Strip out 0 735.302 0.00013253 0.00001


y Strip out
VStrip 8.576
yStrip,IN yStrip,out 0.010506, x *N 0 yN 1 118.9

Kremser (12-28)
L x0 x *N L
n 1
mV xN x *N mV L 735.302
N , 0.721106
n mV L mV 118.90 8.576
0.00013253 0
n 0.27889 0.721106
0.00001 0
N 4.54
n 1.38676

12.D22. New Problem 3rd Edition. K 0.22. y Kx 0.22x. Plot e.g., at x 0.006, y 0.00132
L 75, V 150, L V 0.5
External M.B.:

Lx N Vy1 Lx 0 Vy N 1

V
xN yN 1 y1 2 0.003 0.0004 0.0052
L
Points x 0 , y1 0, 0.0004 , and x N , y N 1 0.0052, 0.003 are on op. line.
Plot Op. Line. See graph (labeled 12.D.b). 2 stages more than sufficient.

312
313
12D23. Apparatus similar to Figure 12-2, except part of treated gas is heated and used as stripping gas.

Absorber: Work in terms of mole ratios.


Yi n .15 .85 0.1765, Yout 0.005 .995 0.00502, X in 0.005 .995 0.00502
G 1400 .85 1190 mol carrier gas/day, L 800 .995 796 mol solvent/day
Equilibrium: y .5x . Convert to mole ratios
x X y Y
0 0 0 0
.05 .0526 .025 .0256
.1 .1111 .05 .0526
.15 .1765 .075 .0811
.2 .25 .1 .1111

Plot ratios on McCabe-Thiele diagram


L L L
Absorber: Op. Line: Y X Yout X in ,
0.669
G G G
External balance Absorber: 796 .00502 1190 .1765 1190 00502 796 X out
X out Abs 0.2614
Step off stages as shown in figure. Need 8 equilibrium stages.

Stripper: Yin .00502 same as Yout abs , Xin 0.2614 same as Xout abs
X out 0.00502 same as X in abs , L 796 mol solvent day, have 4 stages
Stripper equilibrium: y = 3 x. Convert to mole ratios.

x X y Y
0 0 0 0
.025 .0256 .075 .0811
.05 .0526 .15 .1765
.075 .0811 .225 .290
.1 .111 .3 .4286
.15 .1765 .45 .818

Problem is trial-and-error. Select Yout . Draw operating line from Yout , X in to Yin , X out .
See if need 4 stages. When need exactly 4 stages, L/G = slope.

From Figure, final result shown: G strip L slope 796 1.0998 723.7 mol carrier gas/day.
Yout strip ~ 0.287 mol ratio

314
315
12.E1. Convert ppm(wt) to mole fraction: ppm 10 6 wt frac
Basis 1000 g. of steam: Feed Liquid 1000 ppm = 0.001 wt frac
mole frac
lg Nitrobenzene 0.00812 gmole 0.0001465

999 g water = 55.4507 0.9998535


Total 55.4588 mol
1000g
Avg mol. wt 18.0314
55.4588
Liquid Water F 1726 g h 95.7219 mol h
L F C where C S Vout 99 61.8 37.2g water , C 2.0648 mol
L 95.7219 2.0648 97.7867 mol h
Outlet: Basis 1000 g
6
28.1 ppm 28.1 10 wt. frac. moles Mol frac.
3 4 6
28.1 10g nitrobenzene 2.2825 10 4.11 10
999.9997 18.016 55.5062
1000 28.1 10 3 g water ~ 1.0
55.5064 mol
V S 99 18.016 5.495 mol h .
Equil. y mx b, b 0, m H p tot 28.0
Kremser Eq. – Several forms can be used. → Use Eq. 12-28.
L 97.7867 mV yN 1 b
0.63555, 1.5734 , x *N 0
mV 28.0 5.495 L m
L x0 x *N L
n 1
mV xN x *N mV
N 5.763
mV
n
L
N Eq 5.763
Effic 0.524
N act 11
Ref. Hwang et al, IEC Research, 31 (7) 1992, 1753 & 1759.

H 59.3
12-F1. H = 59.3 (Perry’s 4th ed., p 14-4). y x 11.86x
PTOT 5
To be absolutely correct should convert this to mole ratios, although at these low
concentrations could use mole fractions with small error.
y x
Y , X
1 y 1 x

316
x y X Y
0 0 0 0
.001 .01186 .001001 .012
.0015 .01779 .001501 .01810
.002 .02372 .002003 .0243
Change specified conditions to mole ratios.
x in 0, X in 0; yin .02, Yin .02041; y out .002; Yout .002003; x out .001, X out .001001
See Figure for plot of operating line, equilibrium and stages. N = 3.3
Height 5 ft
HETP 1.515
N 3.3 equil stage
L Yin Yout .02041 .002003
18.4
G X out X in .001001 0
If use mole fractions find L/V = 18.0

12.F2. K E 26.0, K p 0.6, K 0.019


After one pass of mass balance obtain:

317
ethane pentane octane
xi,1 0.032 0.005 0.963
xi,2 0.035 0.031 0.934
xi,3 0.033 0.162 0.805
yi,1 0.975 0.003 0.021
yi,2 0.962 0.019 0.019
yi,3 0.885 0.099 0.016

For new temperature used multi-variant Newtonian convergence.


T1,New 73.90 F, T2,New 80.93 F, T3,New 99.15 F.

2.G1. a) N 4, L 570, y A,out 0.00317


b) N 8, L 500, y A,out 0.00315 while with L 490, y A 0.00358
c) N 16, L 490, y A,out 0.002978 while with L 480, y A 0.00337
d) Have a pinch point.

12.G2. Used Peng-Robinson.


a. Total number of stages required 8
b. Feed stage location for the solvent 1
c. Feed stage location for stream A 8
d. Feed stage location for stream B 6
e. Outlet mole fractions of gas stream leaving absorber 0.9991009, 0.00024691,
0.00019067, 0.0004617
f. Outlet mole fractions of liquid leaving absorber 0.012878, 0.0310992, 0.0198928,
0.93612992
g. Outlet gas flow rate 161.6478 kmol/h
h. Outlet liquid flow rate 213.352 kmol/h
i. Highest temperature in column 19.1287 ˚C and stage it occurs on 8

12.G.3. New Problem 3rd Edition. Used NRTL.


Column pressure = 1.0 atm. Feed gas flow rate = 752 kmol/h. Feed gas temperature = 100oC.
Liquid feed temperature = 75oC. Recovery of isopropyl alcohol = 0.98000.
T1 = 302.5K, T2 = 298.6K, T3 = 299.9K, T4 = 301.9K, T5 = 303.3K.
Leaving gas: G = 802.2 kmol/h, Mole fractions: IPA = 0.02443, W = 0.033836, N2 = 0.93721
Leaving liquid: L = 149.8 kmol/h, Mole fractions: IPA = 0.002671, W = 0.99638, N2 = 0.000950
Column diameter = 1.5455 m.

318
SPE 3rd Edition Solution Manual Chapter 13

New Problems and new solutions are listed as new immediately after the solution number. These new
problems are: 13.A12, 13.A13, 13.D3, 13.D5, 13D6, 13D10, 13D22, 13D30-13D34, 13D36-13D42,
13.E2, 13.E3, 13.G1, 13.G2 . Chapters 13 and 14 from the 2nd edition were rearranged to place all the
extraction material into chapter 13 and the material for other separations in Chapter 14. Thus, the
numbers of many problems have changed.

13.A3. The amount of solvent should be increased. This will decrease F/S and move the mixing
point M towards S. As a result the saturated extract product E N will be moved down
(less solute). The difference point ∆ will be moved towards the triangular diagram. The
combined effect will be that fewer stages are required. By adjusting F/S a condition
requiring exactly two stages can be found.

13.A5. The vertical axis will be the extract phase and the hypotenuse will be the raffinate phase.
These will be connected by tie lines. Usual procedure can be used.

13.A7. Situation where E = R and ∆ point is at infinity. All operating lines are parallel.
However, this does not correspond to minimum number of stages in extraction.

13.A.11 c.

13.A12. a. C will be spread out and go into both raffinate and extract streams.
b. C will concentrate around the feed edge. If C is very dilute in the feed, can concentrate C.
Then by stopping the feed but continuing to flow solvents, solutes A and B can be
removed. Solute C can now be collected by withdrawing a stream near the feed stage.

13.B1. Specify: T, p, z A , z B , F, x Ao , x B , y A N , y BN plus:


y B1 , R, E, N F
x B N , R, E, N F
x A N , R, E, N F
R, E, y B1 , x A N
N, N F , y A1 , E
N, N F , x B N , R
N, N F , x A N , R, etc.
Could also not be given one of standard variables (such as solvent concentration).

13.B2. a). One can build stages which are cross-flow (e.g. see Figure 12-12) within a counter-
current cascade. This effectively increases stage efficiency. Not that upward flowing less dense
liquid will be mixed.
b.) Build chambered stages within a counter-current cascade to prevent mixing of the
dense liquid and give better cross flow on each stage.
c.) Put in baffles to prevent MIXING of both less and more dense liquids. This will be
more effective if counter-current is arranged so that flow across stages is always in same
direction (see sketch)

319
13.C.7. Start by defining ∆ and the coordinates of ∆ as:
E o R1 , x A E o y Ao R1x A1 , x D E o y Do R1x D1
nd rd
Removing ∆ from 2 and 3 equations we obtain
xA E o y Ao R 1 x A1 Eo R1 (13-43a)

xD E o y Do R 1 x D1 Eo R1 (13-43b)
Assume that E o R 1 . Next write the three independent mass balances around stages 1 to j.
Rj 1 Ej , xA R j 1x A j 1 E j y Aj , xD R j 1x D j 1 E j y Dj
These equations are now in a form similar to the form of the mixing equations developed
previously. To develop the three point form of a straight line use the first equation to remove ∆
from the other two equations, solve for R j 1 E j in each of these equations, and finally set the
results equal to each other. The development proceeds as follows:

Use Ej R j 1 to remove ∆ from the other mass balances.


Ej Rj 1 xA R j 1x A j 1 E j yA j , E j Rj 1 xD R j 1x Dj 1 E j y Dj
Rj 1
yAj xA Rj 1
yDj xD
Solve for R j 1 Ej , ,
Ej xAj 1 xA Ej xDj 1 xD
Finally, set these equations equal to each other.
yA j x A yDj x D yA j x A yAj 1 xA
rearrange to:
xAj 1 xA xDj 1 xD xDj xD x Dj 1 xD

This last equation says that the slope of the line between the points y D j , y A j and

xD , xA is equal to the slope of the line between the points x D j 1 , x A j 1 and x D , x A and
thus the lines are colinear. Furthermore, the lever-arm rule is valid for this system.

13.D1. a. If we have a single column with only pure solvent then

320
R R
y x x N yN 1 and point x N , y N 1 0.001, 0.0
E E
is on op. line. E 44, R 100 , Slope R E 2.273
Op line intersect equilibrium at x 0.0037 . Thus, cannot get to x o 0.012 .
b. Now E 44 30 74 and R E 1.35 .
44 0 30 0.004
Still want x N 0.001, but y N 1 0.00162
74
At x 0.001, equilibrium value of y 1.613 x 0.001613 . Alternative works, but
have pinch point and need very large number of stages.

c. This alternative (Say use 25 kg/min of 0.004 butanol)


44 0 25 .004
yN 1 0.00145 which is below equilibrium point.
69
Now R E 100 69 1.449 m equil 1.613 Thus, this will work. Obtain
0.00145 69 0.012 100 0.001 100
y1 0.0174
69
Op line closer to equilibrium – require lot more stages.
20 0.004
If use 20 kg/min of 0.004 butanol: y N 1 0.00125
64
0.00125 64 0.011 100
R E 100 64 1.5625 , y1 0.01844
64
Will also work. Becoming close to pinch at top equil y1* 0.019356
15 .004
If 15 kg/min, y N 1 0.0010, R E 100 59 1.6949 m eq
59
1.0010 59 0.011 100
y1 0.1964 y1* 0.19356
59
Won’t work.
Thus, there is a small range where option c will work, but with many stages.

13.D2. R 20, E 20, x IN x F , y IN 0, m 8.3333, b 0, N 2


R 20
Kremser equation 0.12 , y1* mx 0 b 8.3333 x F , y N+1 0
mE 8.333 20
R
* 1
y1 y 1 mE
Eq (13-11) N 1
yN 1 y1* R
1
mE
1-0.12
y1 8.3333 x F 8.3333 x F 3
7.3460 x F , → y1 0.9873 x F
1- 0.12

321
20 x F 19.746 x F
Mass balance, Rx F Rx N Ey1 , x N 0.01269 x F
20
Recovery = 1 x N x F 0.9873 , which is higher than 0.963 obtained in cross-flow.

13.D3. New Problem in 3rd edition.


From M.B. R .013 E .001 R .007 E y out
Where R = 100 and the unknowns are E and yout.

and Equilibrium: y out 1.613 x out 1.613 .007 0.01129

R .013 .007 100 0.006


S E 58.309 kg h
y out 0.001 0.01129 0.001

Alternative Solution: 1 Equilibrium Stage

y 1.613x

1.613

y y out 0.01129
from graph
R
R 0.001 0.01129
E 1.715
E 0.013 0.007
R
E 58.309
1.715
.001
x .013
x out .007

Another alternative solution:

322
Op. Line:
E, y1 R, x 0 Eq.
y x x0
R R
y2 x1 y1 x0
E E
Or 1 y1
Points x1 , y 2 , x 0 , y1
R E Slope
On Op. Line
R, x1
E, y 2
x But x1 and y1 unknown

where y 2 y1,in y2 y1,IN x1 T & E in this configuration

If Eq. line is straight, can Use Kremser with N=1. Slope known, N = 1
Both representations are correct. Treating similar to a flash is easier.

13.D4. Since concentrations are low, use wt. fractions and total flow rates.
Equilibrium: y 0.828 x or m 0.828
R 550 lb h, E 700 lb h, x 0 0.0097, x N 0.00046, y N 1 0.0003,
mE R 1.0538 and R mE .94893
*
y 1 mx IN b 0.828 .0097 0 0.00803
mE y N 1 y1* mE
n 1
R y1 y1* R
Kremser Eq. (13-11b), N
R
n
mE
R R 550 550
y1 yN 1 x0 xN .0003 0.0097 .00046 0.0075
E E 700 700
.0077316
n .0538 1.5038
.0004716
N 33.6
n .94893

13.D5. New problem in 3rd edition.

Part a) Can do with Kremser eq or graphically.


y m x b, m 0.828, b 0
R 400
R 400, E 560, 0.862664
mE 0.828 560
mE 1
x0 0.005, x N 0.0003, y N+1 0.0001, 1.159
R .862664

323
R
Since 1.0 , can use equation such as 13-11b
mE
y1* m x 0 b 0.828 0.005 0.00414
mE y N 1 y1* mE
n 1
R y1 y1* R
N
R
n
mE
0.0001 0.00414
n 1 1.1592 1.1592
0.0034571 0.00414
N
n0.862664
R 400
where y1 yN 1 x0 xN 0.0001 0.005 0.0003 0.003457
E 560
1.52637
N 10.332
0.14773
Alternate solution: Eq. (12-28) becomes
R x0 x *N R
L R, V E , N n 1 n mE R
mE xN x *N mE
yN b 0.0001 1.52637
x *N 1
0.000120773 , N 10.332
m 0.828 .14773
b)

y1* 0.828 x 0 0.828 0.005 0.00414


Equil.
y
Slope = 0.828 R y1* yN 1 0.00414 0.0001
0.85957
E min x0 xN 0.005 0.0003
yN 0.0001 R E min
slope
1
400
E MIN R 0.85957 465.3 kg h
xN 0.0003 x0 0.005 0.85957

13.D6. New problem in 3rd edition.


Part a. Ey N 1 Rx 0 Ey1 Rx N Ext. M.B.
R R
y1 x0 yN 1 xN
E E
100 100
y1 0.005 0.0002 0.0005 0.003414
140 140
Part b. Kremser Eq.

324
yN 0.0002
Convert x x L R y y V E, x *N 1
0.00016556
m 1.208
L R 100
0.5913
mV mE 1.208 140
Lots of different forms can be used.
L x0 x *N L
n 1
mV x0 x *N mV
For example N
n mV L
R x0 x *N R
n 1
mE xN x *N mE
Becomes N
n mE R
0.005 0.00016556
n 0.4087 0.5913
0.0005 0.00016556 1.8717
N 3.6
1 0.5254
n
0.5913
Part c. Eq. y 1.208x

y EQ 1.208x 0.00604

R
slope
y N 1 0.0002 E MIN
y 1.208x
xN 0.0005 x
x0 0.005

R 0.00604 0.0002
1.29777
E MIN 0.005 0.0005
R 100
E MIN 77.05
1.29777 1.29777
Maximum extract out y EQ x 0 0.00604.
Part d. The roles of extract and diluents are switched in the two problems, which changes the definitions
of y and x.

13.D7. N 30, R 500, y N 1 0.0002, x 0 0.0111, x N 0.00037


Equilibrium: y 0.828x, m 0.828, x *N yN 1 .828 0.00024155

325
Since rather dilute and linear equilibrium use one of the Kremser equations.
R xo x *N R
n 1
mE xN x *N ME
N (12-28 (modified))
mE
n
R
x0 x *N
Where 83.756 . Solution is trial-and-error.
xN x *N

E R/mE Calculated N
500 1.21 R
Negative-Not possible Need 1
mE
700 0.8626 17.01 E too high
650 .929 26.175 E too high
640 .9435 29.84 E too high
639 .945015 30.30 E too low

By linear interpolation need E ~ 639.6 kg/h. Can use other forms of the Kremser equation.

13.D8. Was 13.D10 in 2nd edition.


x is raffinate R L
Convert Kremser y mx m K, b 0
y extract, V E
Use 12-31 xN x *N 1 mV L 1 KE R
a) * N 1 3
Other forms OK x 0 x N mV KE
1 1
L R
x *N yn 1 K 0

KE 30.488 25
1 1
xN x0 R 0.00092 100 0.00001376
3 3
KE 30.488 25
1 1
R 100
b) Can use External balance or Kremser to find y out y1
Ey1 Ey N 1 R x0 xN
R 100
y1 x0 xN 0.00092 0.00001379 0.003625
E 25

13.D9. Assume very dilute, R = 1500 kg/h, E = 750 kg/h


Equil. Y K d X becomes y K d x
From Table 13-3. K d,oleic m oleic 4.14, K d,linoleic md,linoleic 2.17
99% recovery oleic: .99 .0025 1500 y1,oleic 750 → y 1,oleic .00495

326
m oleic E 4.14 750
Use Kremser, Eq. (13-11b). 2.07
R 1500
y1* m oleic x 0,oleic 4.14 .0025 0.01035
mE y N 1 y1* mE
n 1
R y1 y1* R
N 5.44
R
n
mE
R 1500
For linoleic acid: .9216 ,
m lin E 750 2.17
yN 1 0, N 5.44, y1* m L x 0,L 2.17 003 .00651
R
1
y1 y1* mE
Can use Eq. (13-11a): N 1
yN 1 y1* R
1
mE
.07834
y1 .00651 .00651 .00124796 → y1linoleic 0.00526
.40866
Recovery of linoleic: Re c .003 1500 .00526 750 → Rec = 0.877
th
13.D10. New problem in 4 edition.

Analytical or graphical solution OK.


Stage 1 F1x F1 E i y1,in Ey1,out R 1 x1

Equilibrium y1,out 1.02 x1

Fx F1 1.02 E R1 x1

F xF 100 0.015
x1 0.0099338
1.02 E R1 1.02 50 100

Stage 2 R 1 x1 E2 y 2,in E 2 y 2,out R 2 x 2,out , R1 R2 F1 100

y 2,out 1.02 x 2,out

R 1 x1 0.0099338 100
x 2,out 0.006579, R 2 R1 F1 100
1.02 E 2 R2 151

Mix with Feed 2 R 3,in 100 70 170

327
x 2,out R 2 x F2 F2 0.006579 100 0.005 70
x 3,in
R 3,in 170

x 3,in 0.0059286

Stage 3 R 3 x 3,in E3 y3,in E 3 y3,out R 3 x 3,out

y3,out 1.02 x 3,out

R 3 x 3,in 170 0.0059286


x 3,out 0.00456
1.02 E 3 R3 1.02 50 170

Stage 4 R 4 x 4,out E4 y 4,in E 4 y 4,out R 4 x 4,out , R 4 R3 170


y 4,out 1.02 x 4,out

R 4 x 4,out 170 0.00456


x 4,out 0.003508
1.02 E 4 R4 51 170

y 4,out 1.02 x 4out 0.003578

328
329
13.D11. R R F 2501, E E 1000
Equilibrium: K D 1.57 . For dilute this becomes m K D in wt. frac. units.
Abietic Acid Recovery: xN R .95 F x F .95 1.0 0.5 .0475
.0475 .0475 .05 F x F .05 1.0 .05
xN 0.0000190 , y1 0.0000025
R 2501 E 1000
R R
Top op. Eq.: y x y1 x0
E E
R 2500
Goes through pt x 0 0, y1 . Slope 2.5
E 1000
R R
Bottom Op. Eq.: y x yN 1 x N through point x N , y N 1 0 , R E 2.501
E E
Need 8 ½ stages (see Figure).

13.D12. m 1.613, R mE 1.2399 , y N 1 y 4 0, y*1 mxin 0.00742


y1 0.00742 1 1.2399
Eq. (13-11a) 4
0.17594
0 0.00742 1 1.2399
y1 0.17594 0.00742 0.00742 0.006114
Rx in Ey in Ey out 10 0.0046 5 0.006114
Overall bal. x3 0.00154
R 10

13.D13. a. xy . 90% recovery, 10% left 0.1 1000 0.003 0.3 kg out
x ,out 0.0003

330
O xy 95% recovery, 5% left 0.05 1000 0.005 0.25 , x O,out 0.00025

For ortho, y max 0.15 0.005 0.00075


R 0.00075 0
0.1579
E max,ortho 0.005 0.00025
For para y max 0.08 0.003 0.00024
R 0.00024 0
0.08888
E max,para 0.003 0.0003
1000
b. The p-xylene recovery controls. E min 11, 250
0.08888
R
E 1.5 11250 16875 , 0.0592592
E
Can use Kremser eq. (13-11b) for ρ-xy to find N
mE y N 1 y1* mE
n 1
R y1 y1* R
N
R
n
mE
m 0.080, R E 0.0592592, y N 1 0 , y1* mx o,p 0.080 0.003 0.00024
Mass balance: 90% entering ρ-xy leaves w. solvent.
0.9 1000 0.003
y1 0.00016 wt frac
16,875
R 0.0592592 R mE .080
0.74074, n 0.300106 , 1.35
mE 0.080 mE R 0.0592592

331
0 0.00024
n .35 1.35
0.00016 0.00024 n 0.30
N 4.012
0.300106 0.30016
Note: Can use other forms of Kremser eq if desired.
c. For o-xy check if recovery > 95%
R
* 1 y1 unknown, y N 1 0
y1 y1 mE
Eq. (13-11a)
y N 1 y1* R
N 1
y1* mx 0 0.15 0.005 0.00075
1
mE
R 0.0592592
0.39506, N 4.012
mE 0.15
R
1
mE 1 0.39506
y1 yN 1 y1* N 1
y1* 0.00075 5.012
0.00075 0.0029194
R 1 0.39506
1
mE
External M.B. Ey1 R xN Ey N 1 R x0
Rx 0 Ey1 5 16875 .00029194
xN 7.3584 E 5
R 1000
Ey1
% Recovery 100 98.53%
Rx 0
S 10.0 2 MF
13.D14. (was 14.D4. in 2nd ed.) a)
F 15.0 3 SM

Once have M, use trial-and-error to find tie through M. (final result is shown). This
gives E and R. y A .115, yw 0.04, xA .23, xw .73.
b) Plot raffinate, R x A .1 . Find tie line through this point (not trial-and-error). This gives
E. Draw Line ER. Intersection with line SF gives M.
S S MF
. Find S 85.7 kg/h.
F 15.0 SM

332
13.D15. Since dilute, use Kremser equations. Assume units are weight fractions.
a) Column 1 at 40ºC. x N 0.0008, N 11,, x 0 0.01, E 1000,, R 100
Equilibrium: m 0.1022, thus y1* mx 0 0.001022. Kremser (Eq. 13-11a):
1
1
y1 0.001022 1.022
12
0.93664
y N 1 0.001022 1
1
1.022
This simplifies to: y1 .093664y N 1 .00092628
External MB: y N 1E Rx o y1E Rx N , y N 1 1000 1 1000 y1 .08
1
which simplifies to: yN 1 1000 y1 .92
1000
5
Solve 2 eqs and 2 unknowns: y1,coll 0.00092693, y N+1,coll 0.6929 10
b) Column 2 at 25ºC: y N 1,col2 y1,col1 0.00092693 ,
y1,col2 yN 1,col1 .6929 10 5 , x 0 0, N 9, m 0.0328, E 1000, y1* mx 0 0

Use Kremser to solve for R´. This is trial and error. For example, Using Eq. (13-11a),
R R
* 1 1
y1 y1 mE 0.0328 1000
0.007475 N 1 10
y N 1 y1* R R
1 1
mE 32.8
R 50 60 50.5 50.35
RHS 0.007855 0.001981 0.007307 0.007467
Within error R´ = 50.35
y N 1E R x 0 y1E .92693 0 .006929
xN 0.0183
R 50.35

333
c) Could be practical if m’s were larger, and have bigger shift in m. A similar scheme is used
commercially for citric acid. Not practical here since have to pump around too much
solvent. In addition, benzene is carcinogenic and would probably not be used as solvent.

13.D16. a.) R E 10 8 1.25, R mE 1.25 1.613 0.77495


*
y1 m x A0 1.613 0.01 0.01613. Use Eq. (13-11a),
y1 0.01613 1 0.77495
7
0.27044 → y1 0.01182
0.0002 0.01613 1 0.77495
E E
xA x A0 yN 1 y1 7.02498 E 4
R R
b.) Graphical check works fine (not shown)

R R R 10
13.D17. yj xj y IN x j 1, 5
Ej E0 Ej 2
x6 0.0018 (See graph)
Note: x6 x N,countercurrent 0.000702 even though use more total solvent.

334
13.D18. (was 14.D2. in 2nd ed.) Plot S, F, R and E. Draw lines SF and RE. Intersection is point M.

S MF 20.3
Lever arm rule: 4.511 → S 100 4.511 451.1 kg/h
F SM 4.5

Or Mass Bal. S + R = M and S y A F xA M x A ( S .15 .5 F .21 M )

Solve simultaneously S = 483.3

335
Difference is due to accuracy in reading numbers. Lever-Arm Rule more accurate!

13.D19. Equil. Kd yA x A 0.287 0.158 1.816


0.005
Acetone y0 0 Y0 0, x 1 0.005 X1 0.00503
0.995
0.10
xN 1 0.10 wt frac X N+1 0.1111
.9
FD 1000 .9 900 kg/h water, FS 1371 .995 1364.1 kg/h chloroform.
900
FD FS 0.6598
1364.1
Equil.

336
xA XA yA = 1.86 xA YA
0 0 0 0
0.01 0.0101 0.01816 0.01850
0.03 0.0309 0.05448 0.0576

0.05 0.0526 0.0908 0.09987

0.07 0.0753 0.1271 0.1456

0.09 0.9890 0.1634 0.1954

0.1 0.1111 0.1816 0.2219

External M.B.
FD FD
XN 1 Y6 X1 YN or YN 0.6598 0.1111 0.6598 .00503
FS FS
YN
0.06999, y N 0.0655
1 YN
1 2
Results pretty close to 13.D43. 2 vs 2 w i accuracy of graphs.
2 3

Note: The graph below should read acetone, not acetic acid as the solute.

337
13.D20. a) Batch Operation – Mix together & settle. Find fraction recovered:
R R
Operating Eq.: y x x 0 , R 5, S 4, x 0 x F
S S
Which is, y 1.25 x 1.25 x F
Equilibrium y 8.333 x, m 8.333
R̂ Sˆ x 0 y iN 1.25 x F 0
Eq. (13-21) written for batch x 0.1304 x F
m Rˆ Sˆ 9.583
Frac. Rec 1 0.1304 0.8696
b) Continuous solvent addition:
Sˆ 1 1 x t ,final
Eq. (13-28) n x t ,final x t ,feed 0.8 n
Rˆ t m 8.333 xF
x t,final x F exp 0.8 8.33 0.00127
Recovery = 99.87%.

338
13.D21 (Was 14.D1 in 2nd ed.) a. Let A = methylcyclohexane and D = n-heptane.
Mass Balances: F1 F2 S M or M 350
F1 x AF F2 x AF S y AS M x AM , F1 x DF F2 x DF S y DS M x DM
1 2 1 2

F1 x A F F2 x A F S y AS 100 .6 50 .2 0
Then x AM 1 2
0.2
M 350
F1 x D F F2 x D F S y DS 100 4 50 .8 0
x DM 1 2
0.229
M 350

Plot M. Find tie line through M. (See figure.) This gives location of points E and R.
Find x DR 0.48, x AR 0.42, y AE 0.06, y DE 0.05 .

b. Mass balances: M E R and Mx AM Ey AE Rx AR


Solving simultaneously: E = 214 and R = 136 kg/h

13.D.22. New problem in 3rd edition. With interface at center, heavy phase flow area is
1 1
Af D s2 4 0.411 and Pperf D s D 5 2.630
2 2

r Ds 2 0.5115

Center
θ .1
2
2 2 C
Chord .4115 r .1
Interface θ 2
α
(length = C)
r
.1 C 1.00326 m
arc α

C/2 339
Draw right triangle for interface below center to calculate new perimeter.
0.1 .1
sin .1955 11.274
r .5115
Then angle of arc, 180 2 157.452
r 3.14159 0.5115 157.452
Length of arc 1.4056
180 180
Perf C arc length 2.4089m
Mensuration formulas are from CRC Standard Mathematical Table.
3
Re settler 4Q c Perf c 4 0.006 998 2.4089 0.95 10 10, 466

Interference somewhat more likely than in Example 13-5.

13.D23 (was 14.D7. in 2nd ed.) a) F S 500 300 M


Pyrdine F x AF S y AO 500 .3 0 M x AM → x AM 150 800 0.1875
Plot M on line FS . By T & E find tie line through M (Use Conjugate line)
y p 0.223, x p 0.84 ; y w 0.02, x w 0.84 ;
Mass balances: R1 E1 M 800 , 0.84R 0.02E 0.43M
Solve simultaneously, E1 ~ 400, R1 ~ 400 (Note: More accurate than pyrdine values.)
b) R 1 S2 400 300 700 M2
R1x A1 S2 y A0 400 0.15 60 M 2 x AM 2
60
x pyr M 2 0.086
700

Find tie line by T & E: y pyr2 0.120; x pyr2 0.053 ; y w 2 0.005, x w 2 0.945

MB: R 2xw2 E2 yw 2 M x m2w → 0.945 R 2 0.005 E 2 700 0.48

R2 E2 M 700

Solve simultaneously: E 2 346 and R 2 354

340
341
13.D24 (was 14.D10. in 2nd ed.) a) Feed 40% MCH 55% n-heptane, F = 200. Solvent 95% aniline
& 5% n-heptane, Stotal 600 . S F M 800
S FM
Lever arm rule: 3 . Find M (Easy way is divide line FS into 4 parts)
F MS

Use tie line through M to find points E & R (T & E)

Extract: y MCH ~ 0.045, Raffinate:x MCH ~ 0.36 wt fracs

E MR
Mass balance E + R = 800 = M and lever arm rule . Measure distances on figure.
R ME

Solve simultaneously: R = 124.61 kg/h, E = 800 – R = 675.39

S 3 FM
b) 2 stage cross flow. Stage 1: F = 200, ρ = 300, . Find point M. Tie line through M
F 2 MS
gives points R1 and E1.
R1 E 1M 1
Mass balance 500 F S M R1 E1 and lever arm rule
E1 M 1R 1
Find: R1 207.04 kg h , E1 292.95

Note: Isotherms are very sensitive. Thus, calculation is not extremely accurate.
S2 M 2R1
Stage 2: Mass balance R 1 S2 M2 507.04 R2 E 2 and lever arm
E1 M 2S 2
Find M 2 and from tie line through M 2 find R 2 . Then can find R2 and E2 from mass balance
R2 E 2M
(given above) and new application of lever arm rule,
E2 R 2M
Solving simultaneously, R 2 196.16 kg h. E 2 310.88

342
13.D25 (was 14.D9. in 2nd ed.) a. Draw lines from S to F and from R 1 to E N . Intersection gives point
M (see Figure). Then from lever-arm,

S FM
1.25 → S 1.25 2000 2500
F SM

b. ∆ is at intersection of lines E N R N 1 and E 0 R1 . Then step off stages as shown. Need 2 stages.

343
13.D26. (was 14.D6. in 2nd ed.) Guess a value for M and step off stages. Repeat until need 3 stages.
After three trials found M shown in Figure. This required 3 1/10 stages which is close enough.
Extract Composition: Acetic Acid = 10.5%, Water = 3.5%.
Raffinate Composition: Acetic Acid = 5%, Water = 93%
Solvent Flow Rate: F S F SM SF 15 57 2000 S 2000 → S = 5600 kg/h
Raffinate Flow Rate: R1 E 0 E0 R1 R 1 5600, R 1 770 kg/h.
Extract Flow Rate: EN F S R1 2000 5600 772 6830

13.D27 (was 14.D12. in 2nd ed.) y AE y wE0 0 (Pure solvent)


0

E0 MF
Lever arm rule: 1.112 . Find M. Line RM intersects sat’d extract at E N , y A N 0.18
F E0M
Lines F E N & R1E 0 intersect at . Step off stages 3 more than enough. Need ~ 2 ¼
EN MR 1
M.B. E N R1 M 211.2 kg/h & lever arm: 2.287 (from graph).
R1 ENM
Solve simultaneously, R1 64.25, E N 146.95 kg/h

344
345
13.D28 (was 14.D14. in 2nd ed.) To find ∆: 1) Plot E N and R N 1 F
2) Ej Rj 1 EN RN 1 1500
E N x AN R N 1x A N 1
xA 0.06666

3) ∆ is on line through points E N and R N 1 .


Plot ∆. Or, use lever-rule.
R N 1E N
RN 1 1.5
EN
Step off three stages starting at point E N . This gives points
R 1 x A1 0.275, x D1 0.675 and E 0 y A1 .13, and y D 0 0.0 .

Mass Balance: E 0 RN 1 R1 E N → E0 R1 EN RN 1 R 1 1500


and E 0 0.13 1000 0.4 R 1 0.275 2500 0.2
Solving simultaneously, R 1 655 kg/h, E 0 2155 kg/h

13.D29 (was 14.D16. in 2nd ed.) a) Plot Points F, S, E N and R 1


Find ∆ point at intersection of lines FE N and R 1S
2 stages is more than enough. (see graph)

b) Draw lines FS and E N NOT calc. value E 2 R1 .


Intersection is mixing point M
F dist. S to M
Mass balance F + S = M and Lever arm 0.786
S dist F to M
Give S F 0.786 1000 0.786 1272 kg/h.

346
Alternate: Overall MB, F S M and Diluent mass balance,
650 F x F,D S yS,D M x M,D 0.28 M
Solve simultaneously: M 2321 and S 1321 kg/h. But this is less accurate.

13.D.30. New problem in 3rd edition. Equation (13-59) becomes Qc /Ai < ut /(1 + safety factor).
Using the equals sign and solving for the safety factor Sf we have,
Sf = ut Ai / Qc -1 = 0.00172 (1.0)(4.0)/.006 – 1 = 0.1467
where Ai = Ds Ls. Thus safety factor is 14.67% instead of 20%. This may still be acceptable.
50
13.D.31. New problem in 3rd edition. Soln. A. Kremser Soln. R mE 0.30998 1.0
161.3
R 50, E 100, m 1.613, b 0, y 2 0.0, x 0 0.01
For example, Use 13-11. y1* mx 0 b 0.01613
R
1
y1 y1* mE y1 0.01613 1 0.30998
* N 1
becomes 2
0.7633696
yN 1 y 1 R 0 0.01613 1 0.30998
1
mE
y1 0.01613 0.01613 .7633696 0.00381684
0.00381684
x1 y1 m 0.0023663
1.613
Soln. b.) Do mass balances and equilibrium for single stage.

347
Sy IN Fx F Sy Fx
0 0.5 100y 50x
also y x 1.613 . Solve simultaneously and obtain identical result.
Soln. c. Do graphically as single stage system.
Soln. d. Do graphically as counter-current system, N=1. Solution is valid, but awkward.

13.D.32. New problem in 3rd edition. Fixed Dispersed Phase.


Q tol Q sol Q feed Q feed Q sol Q feed
At feed conditions tol
Q tol Q feed Q sol 1 Q sol Q feed
Q feed Q feed
Q feed
0.6 .006
Q sol Q feed .6
a) tol .375
.006 .6 .006 Q sol 1.6
1
Q feed
Equation 13-48 operation in ambivalent range.
0.3
.3 3
0.375 865 0.95 10
L L H
3
1 L H L 0.625 998. 0.59 10
0.3
From Example 13.5 1.10235
0.375
The 1.10235 0.6614
0.625
Either phase can be dispersed.
1.0
b) d 0.5 , also ambivalent range
2.0
.5
1.10235 1.10235 Either phase dispersed
.5
2.0
c) d .6667. According to 13-48 at border.
3.0
.6667
1.10235 2.2 water probably dispersed
.3333
5.0
d) d .8333 Equation (13-48), water (heavy) dispersed.
6.0
0.8333
1.10235 5.5 water dispersed.
0.16667
13.D.33. New problem in 3rd edition. t re s Vliq Qd Qc 1.5 min 90s
Qd Qc 0.0072 m3 s , Vliq 90 s 0.0072 m3 s 0.648m3
Note that there is a 1 inch air gap at top
Vliq H t 0.0254 d 2tan k 4 0.648 , H t 2d tan k
Vliq 2d tan k 0.0254 d 2tan k 4 0.648
Using Goal Seek d tan k 0.7489 and H tan k 1.4978

348
13.D.34. New problem in 3rd edition. N = 500 rpm = 8.335 rps
d i 0.20 d tan k 0.2 0.8279 0.16558 m
Use water values for M w 998 kg m3 and M w 0.95 10 3 kg m s
2
d i2 N L
0.16558 8.335 998
Re L ,estimate 3
240, 064
L 0.95 10
Curve b in Figure 13-32 again predicts a constant N p0 40
2
Then from Equation (13-52), P N P0 M d 5i g c where g c 1.0 and N 8.335.
2 5
P 4.0 M 8.335 .16558 1.0 0.034587 M
(A)

M
will be fairly close to c W 998 since Q W 5QToluene (see Equation (13-53)).
The series of messy terms for Equation (13-56a) can be calculated. Since the tank dimensions and
physical properties are the same as in Example 13-5, the only term on the RHS of Equation (13-56a) that
is different is P. Thus the result in the same as Equation B in Example 13-5, d 0.0576 P 0.3 (B)
In addition to Equations A and B, we need to solve Equation (13-53)
M d d 1 d c 865 d 998 1 d (C)
Solving equations A, B and C with Goal Seek we obtain d 0.146 and d d,feed 0.874.
Then solving Equation C, M d d 1 d c 0.146 865 0.854 998 978.6
Equation (B) P 0.3 d 0.05076 2.876 P 33.84 W.
nd
13.D35 (was 14.D11. in 2 edition) From Eq. (12-46),
E1 K 1 E2K 2
B1 1 , C1 , D1 R 0 x 0
R1 R2
E jK j E j 1K j 1
(Eq. 6-6) For 1 < j < N A j 1, B j 1 , Cj , Dj Fjz j 0
Rj Rj 1

ENKN
(Eq. 12-48) For Stage N A N 1, B N 1 , DN FN z N E N 1y N 1
RN
Example 13-4: R 0 1000, x A0 0.35, x D,0 0.65, N 6 , EN 1 1475, yA,N 1 0, yD,N+1 0
For Acetic Acid, K A j y Aj x A j : Use Fig. 14-4 to estimate K A, j .
0.03 0.5 0.09
K A1 0.3, K A 2 0.33, K A 3 0.43
0.1 0.15 0.21
0.12 0.14 0.16
K A4 0.5, K A 5 0.5, K A 6 0.5,
0.24 0.28 0.32
For first guess assume constant E 1475 and R 1000.
E 1K A1 1475 0.3
Then B1 1 1 1.4425
R1 1000
E 2K 2 1475
C1 0.33 0.48675
R2 1000
D1 R D x A ,0 1000 0.35 350

349
and so forth with D6 DN 475 0 0 . Thus matrix for acetic acid is,

1 2 3 4 5 6

1 1.4425 -0.48675 0 0 0 0
1475 1475
2 -1 1 .33 .43 0 0 0
1000 1000
1475 1475 0 0
3 0 -1 1 .43 0.5
1000 1000
1475 1475 0
4 0 0 -1 1 0.5 0.5
1000 1000

0 1475 1475
5 0 0 -1 1 0.5 0.5
1000 1000
1475
6 0 0 0 0 -1 1 0.5
1000

13.D.36. Part a. New problem in 3rd edition. See figure


Forg C Aq ,0 C Aq ,N 0.10 0.008
* *
0.736
FAq C org ,1 C org ,N 1 0.133 0.008
Min

L
Forg,Min 0.736 FAq 0.736 200 L h 147.2
h
Forg 206.08
b. Forg 1.4 147.2 206.08 , 1.0304
FAq 200
Operating line goes through CAq,N 0.008 and Corg,N 1 0.008 with slope 1.0304.
Corg,1 0.097
3
See Figure. 3 stages more than enough. ~2 stages needed.
4

350
Part c. MW Zr NO3 4
91.22 4 14.0067 3 15.994
MW water 2 1.00797 15.994 18.00994

351
Basis 1 liter 0.10 mol Zr NO3 4

Have 33.917g Zr NO3 4


and 1000 g 33.917 966.083 g water
which is 966.083 18.00994 53.64 mol water
.1
Mole frac. Zr NO 3 4 0.00186
53.64 .1
33.9179
Mass frac. Zr NO 3 4 0.033917
1000 g
System is dilute if consider mole fraction, less so if use mass fractions. If densities are constant,
then constant flow rates is valid. Even with variable density, solving problem with mole fractions and
constant molar flow rates would be accurate. This would require converting equilibrium data to mole
fractions. Use of fractions with concentrations in mol/L is NOT correct.

13.D.37. New problem in 3rd edition.

Part a. HETPlarge-scale = HETPpilot (Dlarge/Dpilot)0.38 = (0.24 m) (1.1 m/.05 m).038 = 0.78 m

flarge-scale = fpilot (Dpilot/Dlarge)0.14 = (1.4 s-1)(.05 m/1.1 m)0.14 = 0.91 s-1

Part b. HETPlarge-scale = HETPpilot (Dlarge/Dpilot)0 = HETPpilot = 0.24 m

flarge-scale = fpilot (Dpilot/Dlarge)0 = fpilot = 1.4 s-1

c. Use of the more conservative design developed for difficult systems (n 1 = 0.38, n2 = 0.14) results in a
much higher HETP and thus a much taller column and more expensive column than use of the design
procedure for easy systems (n1 = 0, n2 = 0). Considerably more data is needed for a large variety of
systems to determine best design practice. If a variable speed motor is used in the large-scale system the
difference in predicted optimum frequency is not as serious because the system can tuned to find the
optimum frequency.

13.D.38. New problem in 3rd edition.


C extract C toluene
MWwater 18.02, MWtoluene 92.14 , m 20.8
C raffinate C water
F 1.0 kmol hr , S 0.06 kmol hr. x IN 0.00023 , y IN 0
F x IN Fx out Sy out and → x out Fx in / F Sm
y out m x out
Note m m. m is equilibrium in mole fraction units. Assume extract has properties
toluene and raffinate properties of water.
kmole benzoic 1 92.14 kg toluene
m 3 extract 865 kg tol m 3 kmol toluene
m 20.8 122.71
kmole benzoic 1 18.02 kgW
m 3 raffinate 998 kgW m 3 kmol W
kmol benzoic kmol extract
Units on m are
kmol benzoic kmol raffinate

352
1.0 0.00023
x out 0.0000275 , y out 122.71 0.0000275 0.00337
1.0 0.06 122.71
If use m 20.8 find x out 1 .00023 1 .06 20.8 0.000102, WRONG!

13.D.39. New problem in 3rd edition.


Feed is 0.1 CC 4 , 0.9 AA. F 10 kmol h . Solvent pure. S 10 kmol h.
1 equil. stage
S 10 FM x F,CC x M ,CC S
Lever arm: , Alternatively 1
4 4
1
F 10 SM x M ,CC 4
x S,CC 4
F
Then x M,CC 4 0.05 Find Mixing Point M.
[The figure is shown at the end of problem 13D39 as the single stage mixing line.]
Phases split along the line –TE to find the line through M
Rafinate: x CC 4 0.041, x AA 0.54 . Extract: yCC 4 0.095, y AA 0.07
Overall Balance: E+R+=F+S+=20
CCℓ4 Balance: .095E+0.041R = (0.0) S+0.1 for F=1.0
Solve simultaneously, R 16.6667, E 20 R 3.3333
NOTE: Since CCℓ4 mole fracs can be read more accurately, the CCℓ4 balance is
probably more accurate than the acetic acid balance equations.

13.D.40. S1 S
2
CCℓ4
1 2 R2 R1 = R2 single stage = 16.6667

Mix with S2 = 10 (pure)


E1 E2

16.6667 R1 SM 2 x M 2 ,acetic x Sacetic x M 2 ,AA 0

10 S R 1M 2 x R 1 ,acetic x M 2 acetic 0.54 x M 2 ,AA


x M2,AA .54 1.6667 2.6667 .3375
Find M2 and by trial and error find a tie line though M2. See figure on next page.
Extract 2, yCC 4 0.046 y AA 0.065
Raffinate 2, x CC 4
0.018 x AA 0.57
R2 E2 R 1 S 16.6667 10 26.6667
CC 4 balance 0.018R 2 0.0046E 2 0.041 16.6667 0.0 10
Substitution 0.018 R 2 0.046 26.6667 R 2 0.68333
R2 19.40 and E 2 7.16 kmol h .

353
354
13.D41. New problem in 3rd edition.
R N 1 F 10, x CC 4,N 1 0.1, x AA,N 1 0.9
E0 S 14.5, y TEA,0 1.0 , y CC 4,N 0.091

Mixing. Use lever arm rule.


14.5 S FM xN 1,CC x M ,CC
1.45 4 4

10 F SM x M ,CC 4
y S,CC 4

S
xN 1,CC y S,CC
4
F 4
.1 1.45 0
x M ,CC 0.041
4
SF 1 2.45

Find M. Draw E N MR 1 line. See figure on next page.


Raffinate: x1,CC 4
0.008 x1,AA .58
Passing Streams E N R N 1 & E O R 1 intersect at .
Very close to parallel. Use parallel lines to step off stages.
Estimate # Stages = 3.
Flow rates 24.5 F S E 3 R 1
CC 4 balance. F .1 S 0 1.0 E 3 .091 R1 0.008
1.0 24.5 0.008
E3 9.69 kmol h , R 1 24.5 9.69 14.81
.091 .008
Can compare to 13.G.2 Part c.
Extract 10.066 and Raffinate 14.433
Extract Mole fraction y TEA 0.841 x CC 4 0.0913 y AA 0.067
Raffinate Mole fraction x TEA .418 x CC 4
0.0056 x AA .577
Two results are reasonably close.

355
356
13.D42. a. First, plot points EN and R1 on the saturated extract and saturated raffinate curves,
respectively.
Second, Find point Δ at the intersection of lines FENΔ and R1SΔ.
Third, step off equilibrium stages. Need about 3. See graph.
Part b. Easiest: use the lever-arm rule. Find mixing point M at the intersection of lines FS and ENR1.Then
S FM
0.81 F 1235kg / h
F SM
Can also write 3 mass balances (overall, pyridine, and water) and solve for the unknown flow
rates F, EN and R1. Unfortunately, this will not be very accurate because it is difficult to read the water
values accurately.

13D.43 (was 14.D5. in 2nd ed.) Plot points for F, S E 0 , and R 1 (on saturated raffinate line)
E0M F 1000
Use lever-arm rule to find point M.
FM S 1371
Line R 1M intersects the saturated extract curve at E N . x acetone 0.067 .
Lines FE N and R 1E 0 intersect at ∆ (a second piece of paper was attached to find ∆
accurately). Step off stages. 3 is more than sufficient. Need about 2 & 2/3 stages. This
is close to the 2 + ½ estimated in problem 13.D19 with a McCabe-Thiele analysis.

357
358
13.E1. K Dm 0.05, K Do 0.15, R 20, E 200, F 1
E E
Since K Do 1 ortho goes up column and since K D M 1 meta goes down.
R R
y m,N 1 yo,N 1 0, x m,0 x o,0 0
Estimate: E E .52F and R R .48F
R 20 R 20.48
E 200.52 and R 20.48, 0.09974 and 0.1024
E 200.52 E 200
Recoveries: .92 .52 1 E y ortho,1 or y ortho,1 0.002386
x ortho,N 0.00203
.94 .48 1 Rx meta,N or x meta,N .02179
Plot equilibrium curves and operating lines (see Figure)
Feed cannot be 3rd stage since cannot get x m N desired. Cannot be 5 as will be past
intersection of R E and meta op lines.
Thus feed must be 4th stage. Do not get match of total number of stages.
Need 8 1/3 for ortho and ~ 5 2/3 for meta.
A very slight adjustment of recovery meta will change this. (Meta is approaching a pinch
point at feed stage). 93% recovery was not enough. Therefore, need ~ 93.5% recovery
with ~ 8 stages.

359
13.E.2. New problem in 3rd edition. Part a. 96% recovery. 4% p-xy left in diluent ,

x N,p xy 0.04 .004 0.00016 wt. frac.


y yN
Part b. Paraxylene: Ka 0.080 m, E 20, 000, R 1000 x *N 1
0. Thus
x m
Eq. (12-28) converted to extraction notation is convenient. L R V E

R x0 x *N R
n 1
mE xN x *N mE
N
mE
n
R
R 1000
x0 0.004, x N 0.00016, 0.625
mE 0.080 20, 000
0.004
n .375 .625
0.00016 2.3025
N 4.899
n 1 .625 0.470
mE 0.150 20000
Part c. ortho-xy m 0.150, x 0 0.006, x *N 0, 3
R 1000
xN x *N 1 mE R
Eq. (12-31) Converted: * N 1
x0 x N mE
1
R
1 3
xN 0.006 1.842 E - 5
1 35.899
Part d. Alternative Solutions are presented below for meta-xylene.
m-xy m 0.050, x 0 0.005, x*N 0, N 4.899 E 20,000, R 1000, b 0
0.05 20, 000
mE R 1
1000
Must use special form. But the L mV 1 form in terms of x is not available. Thus, need to
derive, or translate or find in another source. Looking at development of Eq. (12-12).
N x x0 x N
x0 xN
N
Solving for N, x
Where Δx is determined in same way Δy was determined for Eq. (12-12),
L
y1 x0 b
V y1
x x j x j 1 const x0
L V L V
L
yN xN b
1
V yN 1
Alternatively, x xN
L V L V

360
Translating to this extraction problem, L V R E , yN 1 0, x xN
x0 xN x0 xN
N
x xN
x0 0.005
And solving for xN, x N 0.0008476
N 1 5.899

Alternative Solution: Redefine terms to match Eq. 12-12 [Relating y to solvent and x to raffinate is
arbitrary. Switch these definitions.]
y N 1 meta xylene in hexane 0.005
y1 mxy out is unknown x 0 is now inlet solvent 0
1 1 L 20, 000
m 20, b 0, L E 20, 000; 1 , V 1000
Kd 0.05 mV 20 1000
L
yN 1 y1 N y1 x0 b 4.899 y1 20 0 0
V

361
Solve for y1,
yN 1 0.005
y1 0.0008476
5.899 5.899

This is actually x N in normal notation.

Part e. Shown for normal notation. pxy equil slope = 0.080


y EQ 0.080 .004 0.00032

yN 1 0
x x 0,pxy
x N ,pxy
0.004
0.00016
0.00032 0
Slope Operating line 0.08333
0.004 0.00016
R R 1000
Slope 0.08333 , E MIN 12, 000 kg h
E MIN 0.08333 0.08333

13.E.3. New problem in 3rd edition. Part a. Plot the equilibrium data and points F and S. Straight line
from power F to point S passes through mixing point M. Since amounts of F and S are equal,
M is at the half-way point of the line. Find tie line through M by trial-and-error. This is
difficult since tie line is very sensitive.

Approximately, raffinate x AR 0.326 x DR 0.575


and extract y AE 0.046 y DE 0.058

Mass Balances: E R S E 40 R 40 E
Ey AE Rx A,S Sy A,s Fx AF 20 0 20 .4 8
Solve simultaneously, E 18.0 kg , R 22.0 kg.

Part b. First add solvent until reach saturated raffinate curve at intersection with FS line.
Initial Raffinate x AR 0.36, x D 0.54
R INIT x AR Fx AF SINIT x AS 20 .4 SINIT 0 8
R init 8 x AR 8 0.36 22.22 kg
SINIT R INIT F 2.22 kg
Second, use Eq. (13-27) for the continuous solvent addition batch extraction.

362
x t ,final ,A
S dx t ,A
R x t ,feed ,A
yA
x t,feed,A is the raffinate after solvent addition to form two phases
x t,feed,A x A,initial raffinate 0.36 , x t ,final,A 0.292
From equilibrium find values y A (extract), Approximate values are:
x A,t yA 1y
0.36 0.048 20.8
0.326 0.046 21.7
0.292 0.045 22.22

0.292
dx A 0.36 0.292
20.8 4 21.7 22.22 1.47
0.36
yA 6
Sadded 1.47R t Eq. A
In the derivation R t is assumed constant, R t R t,INIT 22.22 kg
Sadded 32.66 kg
With this approximation E Sadded . 32.66 kg
Solute mass balance
R t x A,INIT Sadded y A,added R t x A,final Ey A,Avg
y A,added 0, x A,INIT 0.36, x A,final 0.292
22.22 0.36 0.292
y A ,Avg 0.046
32.66

If we do not assume R is constant, then Eq. (13-27) is

Sadded x t ,A
d R x t ,A
dS added
0 x t ,INITIAL ,raf
yA
We would need to do a numerical integration with a calculation of R x t,A versus y A . this
can be done, but is challenging.

13.G.1. New problem in 3rd edition.


Extract 1: flow 3.90769, xTRA 0.84986, xcarbontet 0.085102, xAcetic acid 0.065042
Raffinate 1: flow 16.03923, x TEA 0.41361, x carbontet 0.041332, x Acetic Acid 0.54506
Extract 1: flow 11.63396, x TEA 0.91426, x carbontet 0.036586, x Acetic Acid 0.049149
Raffinate 2: flow 14.40527, x TEA 0.41633, x carbontet =0.016472, x Acetic Acid =0.56719 .
Entering carbon tet 0.10 10 1.0 kmoles hr
Leaving in raffinate 0.016472 14.40527 0.23728
Extracted In Out in Raffinate 0.7627
% extracted = 76.27%

363
13.G.2. New problem in 3rd edition.
Part a, 3 stage cross-flow. All flow rates are kmol/h
Total Flow rate TEA flow CCl4 flow Acetic flow
Extract 1 3.961 3.366 .3371 .2576
Extract 2 11.634 10.637 .4256 .5718
Extract 3 11.052 10.419 .1554 .4777
Raffinate 3 13.353 5.580 .0819 7.693
Carbon tet remaining in raffinate 3 is 0.0819 kmol/h. Since carbon tet feed was 1.0 kmol/h, 0.9181
kmol/h was extracted. Fraction extracted = 0.9181/1.0 = 0.9181.

Part b. 3 stage counter-current with S = 10 kmol/h.


Extract 1 4.9142 3.723 .7242 .4672
Raffinate 3 15.086 6.277 .2758 8.533

Carbon tet remaining in raffinate 3 is 0.2758 kmol/h. Since carbon tet feed was 1.0 kmol/h, 0.7242
kmol/h was extracted. Fraction extracted = 0.7242/1.0 = 0.7242.

Part c. 3 stage counter-current with S set to give same fraction extracted as in part a (0.9181) and outlet
raffinate carbon tet flow rate of 0.0819 kmol/h. This is trial-and-error.
First trial: S = 20 and CCl4 raf 3 flow rate = 0.0289
Second trial: S = 18 and CCl4 raf 3 flow rate = 0.04045
Third trial: S = 16 and CCl4 raf 3 flow rate = 0.0590
Fourth trial: S = 14 and CCl4 raf 3 flow rate = 0.0908
Fifth trial: S = 14.5 and CCl4 raf 3 flow rate = 0.08104
This is close enough.
Final Results:
Extract 1 10.066 8.469 0.9189 .6786
Raffinate 3 14.433 6.031 0.0810 8.321

364
SPE 3rd Edition Solution Manual Chapter 14

New Problems and new solutions are listed as new immediately after the solution number. These new
problems are:14.A3, 14.A4, 14.C5, 14.D6, 14.D9, 14.D11, 14.D15-14D17, 14.E2, 14.E3. Chapters 13
and 14 from the 2nd edition were rearranged to place all the extraction material into chapter 13 and the
material for other separations in Chapter 14. Thus, the numbers of many problems have changed.

14.C.5. New problem in 3rd edition. Part a. y y, x x, m 1, F U, S O, R U, E O


where F, U, S, O, R, E are kg
Eq. (13-27b) becomes
U U
y x x F y IN
O O
U
and (13-21) x x IN y IN 1 U O and y = x
O
Part b. Eq. (13-29b) becomes
O 1
n x t ,final x t ,feed
U K
Where K y x at equilibrium = 1.0 in washing.

U x0 x *N U
n 1
mO xN x *N mO
14.D1. (was 13D29 in 2nd ed.) a) Translate eq. (12-28), N
n mO U
Note: x in wt frac. translates to x in kg m 3 if densities are constant. Densities cancel. For
washing equilibrium is equal overflow & underflow concentrations. Thus, m = 1, b = 0
yN 1 b
H 2SO 4 x *N y N 1 0, x 0 1.0, x N 0.09
m
U 40 mO 1
0.8 and 1.25
mO 1.0 50 U 0.8
1.0 0
n 1 0.8 0.8
0.09 0
N 4.96
n 1 0.8
xN x *N 1 mO U
b) HCℓ Use Eq. (12-31) or (14-8) * N 1
x0 x N 1 mO U
1 mO U 1 1.25
xN x0 N 1
0.75 0.0674 kg m 3
1 mO U 1 1.255.96

mO 1.0 50 yN 1HC b
N 4.96, 1.25, x *N HC 0
U 40 m
Alternative:

363
xN xN
Note: =0.09
x0 HC
x0 H 2 SO 4

Thus, if one is clever and realizes change will be same for HCℓ & H 2SO 4
mO
since & N are identical , don’t need to use Kremser eqn for part b.
U

14.D2. (was 13.D22 in 2nd ed.) a. 1000 cc sand = 400 cc underflow liquid. This is about 400 g = 0.4 kg
liquid. Equil: y = x. Use nomenclature of Table 13-4.
U U
Operating Eq. y j xj 1 y in x out
O O
U .4
Slope 0.8. Goes through point (y = 0, x = 0.002)
O .5
Overall bal. O yin U x in U x out O y out
.0140 .0008
O .4 0.035 .4 .002 .5 yout → y out 0.0264
.5
Need 6 2/3 Stages – See Graph (Can also use Kremser eq.)

b. Mass Balance: U xj 1 O j y jin U xj O jy j


U U
Op. Eq.: yj xj y jin xj 1
Oj Oj
U
U 0.4, O 0.2, 2 slope , x out 0.002 (see graph)
O
Obtain approximately same separation, but use much more wash water.
14.D3. (was 13D23 in 2nd ed.)

364
y y y y
4 3 2 1

U=3

4 3 2 1 U=3
x
4

O 2 O 2 O 2 O 2
4 3 2 1
y 0 y 0 y 0 y 0
in 4 in 3 in 2 in 1

Basis: 1 kg CaCO 3 solids


Feed Mole frac. can be arbitrary. Pick x 0 0.01 as basis
M.B. U x iN O yiN U x out O y out
U U
y out x outx iN y iN
O O
y out , x out at Equil (y = x) line
Point x in , yin x in , 0 is on op line
U 3
Slope Op line
O 2
See graph.
x4 0.00127
Find 0.127
x0 0.01
x4
Recovery 1 1 0.127 0.873
x0
Recovery is significantly better with counter-current process.

365
14.D4. (was 13D24 in 2nd ed.)
0.8 0.8 0.2 0.8, 4, O 4000 kg/h
1

366
kg liquid
1.0
kg L liquid L liquid kg liquid
U F1 1000 dry solids 4 1600
h L solid kg solid h
2.5
L solid
kg .8 1.0 kg liquid
U F2 2000 3200
h .22.5 h
kg liquid
UT U F1 U F2 4800
h
UT UT
In section 2: yj xj 1 y0 x1
O O
Slope UT O 4800 4000 1.2 Goes through point y0 , x1 0, 0.006
Intermediate feed at x xF 0.02
U F1 U F1
In Section 1: y j xj 1 yN xN 1
O O
Slope U F1 O 1600 4000 0.4 . Goes through point y N , x N+1
Also intersects Section 2 op. line at feed line. (Or calculate y N from mass balance). Equilibrium
is y = x. Step off stages (see Figure). Need 5.4 equilibrium stages. Opt. Feed is 4 th.

14.D5. (was 13D25 in 2nd ed.) 0.8, 1 0.2

kg dry 1 1 2000 L
F1: 1000 F1 5 wt %
h kg dry L solid h
2.25 0.2
L L under flow
kg dry 1 1 L
F2 : 2000 4000 F2 2 wt %
h 2.5 0.2 h

367
2000L L liq
Liquid Volumes: total 0.8 liq 1600 5 wt %
h h
3200 L liq
4000 0.8 liq 2 wt %
h
FT : Total liqd h 4800
underflow

kg liquid
U0 f 4000 4800 where f 1.0 kg L
h
U kg liq kg N a 0H
ON 1 4000 kg h , y N 1 0, 1.2, U 0x 0 1600 0.05 3200 0.02
O h kg liq
x0 144 4800 0.030 , x N specified 0.006
Ext. MB, ON 1 y N U0 x 0 UNx N O1 y1
1

U0x0 U U 4800
y1 xN x0 xN 0.030 0.006 0.0288
O O O 4000
Convert to Kremser
O V, U L, m 1, y1* mx 0 x0 0.030, mV L 4000 4800 0.8333
0 0.030
n 1 0.8333 0.83333
0.0288 0.030
Eq. (12-30) N 8.83 or 9 stages
n 1.2
Use 2 feeds!

14.D.6. New problem in 3rd Edition. 2.5 kg wet is 1 kg dry solids-insoluble, and 1.5 kg underflow liquid.
1 kg dry solids
Part a. 10 kg total 4 kg dry insoluble solids
2.5 kg total
1.5 kg liquid
U 4 kg dry solids 6 kg liquid. , Ov 10 kg liquid.
kg dry solids
Before 1st mixing: 0.05 frac BaS 6 kg liquid 0.3 kg BaS
0.3 kg BaS
1st Mix: 0.01875 mass frac in U & Ov.
16 kg liquid total
Settle – (6 kg liquid in U) 0.01875 0.1125 kg BaS
0.1125 kg BaS
2nd Mix Pure Water 0.00703 mass frac in U & Ov.
16 kg liquid
Settle – (6 kg liquid in U) 0.00703 0.0421875 kg BaS
0.0421875kg
3rd Mix Pure Water 0.00264 mass frac BaS in U and Ov.
16 kg liquid
Part b. Result is same. Can also be done graphically.
Part c. Countercurrent. Easiest solution approach is to use Kremser equation.
x N x *N 1 m Ov U
* N 1
x 0 x N 1 m Ov U

368
N 3, x 0 0.05, m 1, Ov 30, U 6, x *N yN 1 m 0
1 30 6
xN x0 4
0.05 0.00641 0.0003205
1 30 6
External M.B. x0U y N 1Ov xNU y1Ov
y1 U x0 xN Ov 6 0.05 0.0003205 30 0.009936

14.D7. (was 13D27 in 2nd ed.) Equilibrium: xj y j wt. fractions


U U
Operating Eq.: yj xj y j,in xj 1
O O
Basis 1000 cc wet sand.
vol water 1.0 g kg
U .4 1000 cm 3 wet sand 0.4 kg
vol sand wet cc 1000 g
.4
O = 0.2 kg. Thus, each operating line has slope 2.
.2
Each op line goes through pt. y j,in , x j 1

x0 0.035, y N,in yN 1,in yN 2,in 0


xN 0.002, yS,in y N , y 2,in y N 1 , y1,in yN 2

Start at stage N where x N = 0.002. Find y N then work backwards to stage N-2. This
gives inlets for first 3 stages so can then work forward (see Figure). Note: that stages 5 and
N-2 are not connected. 8 stages gives more than enough separation, but 7 is not enough.

369
14.D8. (was 13D28 in 2nd ed.) Use Kremser equation
Fsolv Fsolid .95, y mx is equilibrium with m = 1.18, and N = 11.
Recovery is 1 x N x0 .
Eq. (12-31) becomes x *N yN 1 m 0 .
m Fsolv
*
1
xN x N xN Fsolid
* N 1
x0 x N x0 m Fsolv
1
Fsolid
m Fsolv xN 1 1.121
1.18 .95 1.121. Then 12
0.041
Fsolid x0 1 1.121
Thus Recovery = 0.959

14.D.9. New problem in 3rd Edition. Assume FSolid and Fsolvent are constant despite removal of sugar from solid.
FSolid 1.0, x F 0.055, y solv,IN 0, m E 1.18,
FSolid
Eq. (13-21) becomes x xF y solv ,IN mE FSolid Fsolv
Fsolv
a. Fsolv 3.0, FSolid Fsolv 13
0.055 1
x 1.18 0.01211 , y mE x 1.18 0.01211 0.0143 wt frac.
3 3
b. x = 0.004. Solve for Fsolv .
FSolid xm E y solv,IN
Fsolv xF x
xF x 0.055 0.004
Fsolv FSolid 1.0 10.805 kg
mE x ySolv,IN 1.18 .004 0

14.D10. (was 13D30 in 2nd ed.) G H 2 stream 100 lb h of H 2 CONSTANT


L Slurry stream 120 lb h CONSTANT
L L
Operating Line. Must work in weight ratios. Y X X in Yout
G G
x y x in
x , Y , X in 0
1 x 1 y 1 x in
lb CH 4 in 30 lb CH 4 out .05
Yin 0.30, Yout .0527
lb H 2 in
100 lb H 2 out
.95
G Yin Yout L X out
100 X .206
X out 0.30 0.0527 0.206 , x out 0.171
120 1 X 1.206

370
Operating line becomes,
L L 120
Y X Yout where 1.20 and goes through X in , Yout .
G G 100
Y X 1.2 X
Equilibrium Curve: y = 1.2 x becomes 1.2 → Y
Y 1 X 1 1 .2 X
Plot Y vs X
x X Y
0 0 0
.05 .0526 .0038
.10 .1111 .1364
.15 .1765 .2195
.20 .2500 .3158
.25 .3333 .4286

See Figure for Plot. Need 5 1/8 stages.


14.D.11. New problem in 3rd Edition. 10,000 kg h wet solids, frac. vol. liquid ,
1 frac vol dry solids.
Basis 1 m 3 wet solids :
Weight liquid + weight solids 1.0 1000 kg m3 1.0 1 1500 kg m 3
0.4
400 kg 900 kg 1300 kg total m 3 wet solids.
400 400 kg
Thus of weight is underflow liquid, U 10, 000 3076.9 .
1300 1300 h

14.D12. (was 13D32 in 2nd ed.) Fsolv Fsolid 1.36

371
Fsolid Fsolid
Op. Eq.: y x y1 x0
Fsolv Fsolv
Where y and x are kg m 3 . y m E x is equilibrium.
xF x 0 , x N 1 .975 x F .025 x 0 , y N 1 0, x *N 0, N 5, Fsolv Fsolid 1.36
Can use any of Kremser equations such as Eq. (12-31).
m Fsolv
*
1
xN xN Fsolid 1 1.36 m
0.025 * N 1 6
x0 xN m Fsolv 1 1.36 m
1-
Fsolid
Which becomes: 0.1582 m 6 1.36 m 0.975 0 Find m = 1.313
1 1.313 1.36
Check: .025 6
0.025005 which is OK.
1 1.313 1.36
14.D13. (was 13D33 in 2nd ed.)
R̂ Sˆ x 0 y in
a) Use Eq. (13-21), x , equil. y m E x, y in 0
m Rˆ Sˆ
g L in liqd 0.8 x F 0 0.8
R̂ Sˆ 10 12.5 0.8, m E 1.18 , x xF 0.4040 x F
g L in solid 1.18 0.8 1.98
Frac. Rec. 1 0.404040 0.5959596
Sˆ 1 x t ,final
b) Eq. (13-29b) n
Rˆ mE x t ,feed
x t,final x F exp 1.25 1.18 0.228779 x F , Frac Rec = 1 – 0.228779 = 0.7712

14.D14. (was 13D34 in 2nd ed.) BaSO 4 coal BaS 2 CO 2


Equil: Soln conc in underflow = soln conc in overflow. Thus really washing
Equil : y x, m 1, b 0
kg kg solution
U 350 in sol. 1.5 525 kg soln., x in 0.20, x out 0.00001
h kg insoluble solid
kg U0 x0 xN 525
O 2075 , y in 0.0, x *N 0, y1 0.2 0.00001 0.0506, x *0 0.0506
h O 2075
n xN x *N x0 x *0 n 0.00001 .2 .0506
Eq. (12-29) N 6.99 or 7.0
n L mV n 525 1.0 2075
14.D.15. New problem in 3rd Edition. With 1000 kg/h dry solids U 1.5 1000 1500 kg h
a) Can use Kremser eq. with large N to find Ov Min or a sketch

372
y1* Equilibrium is y x

y U y1* 0 0.15
1.0101
U Ov Min x0 xN 0.15 0.0015
0 Min
yN 0 0 1500
x0 .15 Ov Min 1485
10101
xN 1 .99 .15 0.0015

b. Ov 1.2 Ov Min 1782


U 1500
0.84175
Ov 1782
y y V Ov
Kremser:
x x L U
Eq. (12-28) xN 0.0015, x *N 0, x 0 .15, m 1, U Ov 0.84175
U x0 x *N U
n 1
m Ov xN x *N m Ov
N
m
n
U Ov
0.15 0
n 1 .84175 .84175
.0015 0 2.81
N 16.33
1 .17227
n
.84175
In theory, can use McCabe-Thiele, but it is difficult to accurately step off this large number of
stages.
U 1500
c. Ov 2000, .75 m 1
Ov 2000
n 1 .75 100 .75
N 11.29
1
n
.75
N eq 11.29
N act 15 E overall 0.753
N sub actual 15
For m E use N = 15 and change mE with same equation
.75 .75
n 1 100
mE mE
N
mE
n
.75

373
Vary mE until N = 15. m E .911
On a McCabe-Thiele diagram this is trial and error. Kremser is much easier.

14.D.16. New problem in 3rd Edition. Part a. U 2 kg, O 2 kg, x IN 0.06, y IN 0


Solution (translation of Eq. (13-21)) is
U
x x IN y IN 1 U O 1 .06 0 2 .03
O
Part b. Want x 0.005 O is unknown, x IN 0.06, y N 0, U 2 Solve for O
U U O x y IN x IN x
x x x IN y IN , O U
O O U x IN x x y IN
0.06 0.005
O 2 22 kg water
0.005 0
O
14.D.17. New problem in 3rd Edition. K = 1 Eq (13–28) becomes n x t ,final x t ,feed
U
Part a. O 2, U 2, x t,feed 0.06
O 1
x t ,final x t ,feed exp 0.06 e 0.02207
U
Part b. U 2, x t,feed 0.06, x t,final 0.005
0.005
O U n x t ,final x t ,feed 2 n 4.97 kg
0.06
Part c. x t,final x in Part a.
O O normal batch in Part b.

14.D18. One equilibrium stage. F 1000, x A N+1 .2, S 662, y AS y DS 0


E 0 y A,0 F x A,N+1
x A,M 0.12 (same as Example 14-2)
E0 F
Plot M. By trial and error find tie line through M (Final result shown in Figure).
y A1 .238, y D1 0; x A1 .078, x D1 .656
Flow rates: Diluent balance: R1x D1 F x D,N+1
R1 F x D,N+1 x D1 1219.5
E1 M R1 1662 1219.5 442.5

374
14.D19. This problem is essentially a repeat of Example 14-2, except using exactly 3 stages.
Clearly, x A1 0.04 since now have more stages. F, E 0 and M are unchanged. Problem
is trial-and-error. Guess location of R 1 . Find E N and ∆. Step off 3 stages and see if
have correct location of E N . The third and final trial is shown in the figure.
x A1 0.026 and y A3 0.38.

14.D20. Although this is leaching, this cross-flow problem is very similar to cross-flow extraction.
We can derive
R j 1 x A j 1 E j,in y A j,in
x A Mj
R j 1 E j,in

M j x A Mj yA j
M Rj 1 E j,in where R j
xAj yA j
Stage 1: R0 1000, E1,in 421, x A0 .2 y a1,in 0 , x AM1 200 1421 .1407
Find M on line SR 0 at x AM1 (see Figure). By trial-and-error find tie line through M.

375
This gives E1 and R 1. Find y A1 .35, x A1 .113, M1 1421
1421 .1407 .35
R1 1254.9
.113 .35
1254.9 .113 0
Stage 2: x A M 2 0.085
1254.9 421
y A2 .18, x A2 .058, from tie line , M 2 1675.9
1675.9 0.085 .18
R2 1305.0
.058 .18

1305 .058 0
Stage 3: x A M 3 0.044 , y A,3 .09, xA3 .03, M3 1726
1726

1726 0.044 .09


R3 1323.3 kg/h
.03 .09

14.D21. a. Basis 1 kg mix in underflow: x NaC values 0.8 1.0 0.2 yNaC
crystals

Since crystals are pure NaCℓ, NaOH is in liquid only. Since 20% of the underflow is liquid,
x NaOH 0.2 y NaOH . Generate equilibrium table.

376
Soln (y)
x NaOH Mass frac NaOH y NaC x NaC
0 0 .270 .854
0.004 .02 .253 .8506
0.008 .04 .236 .8472
0.012 .06 .219 .8438
0.016 .08 .203 .8406
0.020 .10 .187 .8374
0.024 .12 .171 .8342
0.028 .14 .156 .8312
0.032 .16 .141 .8282
0.036 .18 .126 .8252

Feed is 45 wt% NaCℓ crystals. x values: NaCℓ (soln) = 0.5193, NaOH (soln) = 0.099, water 1-0.5193-
0.099 = 0.3817. Since feed is 55% liquid,
x F,NaOH 0.55 y NaOH 0.099 y NaOH 0.099 0.55 0.18
,
y NaC 0.126
From the equilibrium data
F = 100, S = 20, Plot F & S and find M.
FM 20
,
SM 100 Tie line through M gives E & R.
E RM
1.119
R EM (measured on figure)
E R 120 1.119 R R 120
R 56.63 kg/min, E 63.37 kg/min
R : Raffinate 0.833 x NaC , 0.026 x NaOH
E : Extract y NaC 0.16, y NaOH 0.135

The underflow is z wt frac crystals (Pure NaCℓ) + (1-z) wt frac solution


y NaC 0.16 is soln in equil
z 1.0 1 z 0.16 0.833
Thus,
0.333 0.16
z 80.1% OK
0.84
was 80% solids in problem statement.

R1 R1 EN
c. Same M. Plot draw line M to .

2 stages more than sufficient


EN R1 120 1.137 R 1 R1 120
EN R 1M 103.5
1.137
R1 ENM 91.0

377
R1 56.14 E 63.86
kg/min, N kg/min
R1 : x1,NaC 0.845, x1,NaOH 0.01
E N : y NaC 0.152 y NaOH 0.147

378
379
14.E1a. This is difficult part – converting data
Basis 1 lb oil-free solids

1.0 y oil z
z x solids x oil
y oil ysolvent 1 z 1 z
0 1.0 0.20 0.830 0
0.1 0.9 .242 0.80515 0.01948
0.2 0.8 .283 0.7794 0.044115
0.3 0.7 .339 0.74683 0.07595
0.4 0.6 0.405 0.71174 0.1153
0.5 0.5 0.489 0.67159 0.16420
0.6 0.4 0.600 0.625 0.2250
0.65 0.35 0.672 0.598086 0.26124
0.70 0.3 0.765 0.56657 0.303399
0.72 0.28 0.810 0.552486 0.3222

Note: ysolids 0 for all streams, Z = lb solution/lb oil free solids.

Plot data on triangular diagram. See Figure 14.E1a, b, c, d, e.

b&c. F + S = M1 = 1500

1000
F x oil,F S yoil,S M1x oil,M , x oil,M1 0.252 0.168
1500

See Figure 14.E1a, b, c, d, e.

M 1S F 2
Check: Lever Arm . Find tie line through M1.
M 1F S 1

Extract E1 , y oil,1 0.34; Raffinate: x oil,1 0.092 and x solids,1 0.730

Mass Balances: 1500 E1 R 1 , 252 0.34 E1 +0.922 R 1


R1 1040.3, E1 459.7 lb
Finish step c) Stage 2: R1 S2 M2 1540.3 , R1 0.092 M2 xoil,M
x oil,M 2 0.062 . Plot M 2 and find tie line through M 2 .
Extract: yoil 2
0.115; Raffinate: x oil,2 0.025 and x solids,2 0.80 .
MB: 1540.3 E2 R 2 , 95.7 0.115 E 2 0.025 R 2
R 2 904.8 lb, E 2 635.5 lb
d & e – Same answer as b & c but R & E are flowrates.
1
f. See Figure 14.E1f. 3 stages is more than enough. Need ~ 2 equil stages.
3

380
Lines E N R N 1 and E 0 R 1 intersect at .

381
382
14.E.2. New problem in 3rd Edition. Converting data is the difficult part, but is obviously identical to
Problem 14.E.1.
Basis 1 kg oil-free solids

1.0 y oil z
x solids x oil
1 z 1 z
y oil ysolvent z

0 1.0 0.20 0.830 0

0.1 0.9 .242 0.80515 0.01948

0.2 0.8 .283 0.7794 0.044115

0.3 0.7 .339 0.74683 0.07595

0.4 0.6 0.405 0.71174 0.1153

0.5 0.5 0.489 0.67159 0.16420

0.6 0.4 0.600 0.625 0.2250

0.65 0.35 0.672 0.598086 0.26124

0.70 0.3 0.765 0.56657 0.303399

0.72 0.28 0.810 0.552486 0.3222

x t ,final
S dx t
Approximate solution, use Eq. (13-29a) Oil balance:
Rt x c ,feed
y

S = Mass Solvent, R t Mass raffinate (solids + solute)


x = Mass frac. solute (oil) in raffinate
y = Mass frac. solute (oil) in raffinate in extract (solvent)

a) M is now at saturated raffinate curve. x oil,M 0.21, x solids,M 0.63


Mass balance F + S = M
Solids .748F + (0) (S) = 0.63M

0.748
M F 1187.3 kg R initial
0.63
S 187.3 kg
b) Now mixing is from S to a point on raffinate curve.
From equilibrium curve in solution to 14.E.1.

383
x oil y oil 1 y oil
0.21 .54 1.852
0.1625 .498 2.0080
0.115 .40 2.50
0.0675 .28 3.57
0.02 0.1 10.0

Insoluble Solids M.B.


Initial 0.748, F = 100, Final 0.81, R t final
.81 R t final 748 R tfinal 923.5 kg
Raffinate is 0.81 solids, 0.02 oil and 0.17 solvent
Solvent remaining in raffinate is 0.17 923.5 157.0 kg
Needs to be recovered by evaporation.
Do Simpson’s rule in 2 parts.
0.21 0.115
1.852 4 2.008 2.50 .1961
1
6
0.115 0.02 0.4241
2.50 4 3.57 10
2
6 0.6202
Sadded 0.6202 R t , but what is R t ?
Eq. (13-29a) assumes R t Const.
Use average value of R t .
1
R t ,avg R t init R t ,final 1187.3 923.5 1055.4
2
or Sadded 0.6202 R t,avg 0.6202 1055.4 654.6
Stotal Initial addition + Sadded 187.3 654.6 841.9
Extract amt S Stotal Sremain in raffinate 841.9 157.0 684.9 by solvent
Oil in extract x F,0.1 F x final,oil R t,final 0.252 1000 0.02 923.5 233.5
Total wt extract 684.9 233.5 918.4
yoil 233.5 918.4 0.254
ysolvent 0.746

14.E.3. New problem in 3rd Edition. Solid Matrix is insoluble. Solids = (.748) 1000 = 748 kg. R t not
Constant, but Solid is.
Solids
Rt
x Solids
xA
ydS d R xA Solids d
x Solids

384
x final ,A x Solids
S d x A x Solids
Solids x A ,raf ,init y
x Solids ,raf ,init

Changes limits integration.


x oil 0.21, x Solids 0.63, x oil x Solids 0.21 0.63 .3333
x oil 0.115, x Solids 0.705, x oil x Solids 0.115 .705 0.163
x oil 0.02, x Solids 0.81, x oil x Solids 0.02 0.81 0.0247
Numbers for use in Simpson’s rule are from Solution 14.E.2.

.3333 0.163
1.852 4 2.008 2.50 0.3515
1
6

0.163 0.0247
2.50 4 3.57 10 0.6173
2
6
Total 0.9688
Sadded Solids total integral 748 0.9688 724.6 kg
Stotal initial added 187.3 724.6 911.9 kg
Extract Amount Solvent Stotal Sraf ,final 911.9 157 754.9 kg
Oil in extract = 0.252 (1000) – 0.02 (923.5) = 233.5
Total weight extract 754.9 233.5 988.4

wt frac solvent = 0.764, wt frac oil = 0.236

385
Chapter 15 Solution Manual
Since this is a new chapter, all problems are new.

A. Discussion Problems.
15.A1. The mole fraction water is constant but since the temperature within the vessel varies the total
molar density Cm varies and the water concentration = Cw = ywCm also varies. Thus, Eq. (15-
10a) incorrectly predicts molecular diffusion. Equation (15-10b) predicts no molecular
diffusion because dyw/dz = 0.

B. Generation of Alternatives.
15.B1. For example, one could operate with both inflow and outflow at the bottom of the tube. If flow is
controlled with a constant head tank, the height of liquid in the tube will be very close to
constant.

C. Derivations.
15.C4. Substitute in q = (μ Re)/(4ρ) into Eq. (15-35d) and obtain δ = [(3μ2Re)/(4ρ2g)]1/3.

15.C5. Start with Eq. 15-52a), set vB=0 and solve for yAvA. Then NA = Cm yAvA. Substitute in the
expression for yAvA and Eq. (15-52e) for JA. This gives the desired result.

15.C6. This problem is included to show that one can derive the expressions in books. There is a lot of
algebra, but the derivation works. First, can expand the derivative,
1 AB 2 (1 2 x1 x12 )
x1 x [B (A B ) x1 ]2
Then take the derivative and expand all terms. The denominator becomes
[ A ( A B) x1 ]3 [ Bx2 Ax1 ]3 and the numerator simplifies to 2 A2 B 2 x2 . Multiply by x1. Q.E.D.

*
15.C7. With CMO and y as mole fraction, vmol y Av A yB vB y Av A (1 y A )vB 0 . Since NA = -NB,
CAvA = -CBvB and for an ideal gas Ci = yi Cm. The total molar concentration Cm is constant.
Then, vA = -(1-yA)vB/yA (Eq. A)
In terms of mass fractions yA =(yA,mass/MWA)/[yA,mass/MWA + (1 – yA,mass)/MWB]. (Eq. B)
Substitute Eq. B into Eq. A and simplify.
(1 y A, mass ) / MWB
vA vB (Eq. C)
y A, mass / MWA
*
Then in mass terms vmass y A,mass vA yB ,mass vB which after substituting in Eq. C and simplifying
*
vmass (1 y A,mass ) ( MWA / MWB )(1 y A,mass ) vB . (Eq. D)
*
If MWA = MWB, vmass = 0. We can write
vB = NBCB = NByBCm = NBCm(1 – yA) (Eq. E)
where the y are mole fractions. Substituting Eq. B into Eq. E and then substituting this into Eq. D,
we obtain
1 y A , mass MW A
Cm N B (1 y A , mass ) (1 y A , mass )
* MW B MW B
vmass (Eq. F)
y A , mass / MW A (1 y A , mass ) / MW B
*
Since yA,mass varies throughout the distillation, vmass is different for each stage.

386
D. Problems.
dC A
15.D1. Dprop,water = 0.87E-9 m2/s. Eq. (15-9), J A , z ( D AB / L )(C A , L C A ,0 ) . If C A,0 = 1.2
D AB
dz
kg/m3 is the known value, C A, L can be larger or smaller than C A,0 . For smaller C A, L we have
C A, L =1.2 – (0.2E-5)(0.0001)/0.87E-9 = 0.9701
If it is larger value, then C A, L =1.2 +(0.2E-5)(0.0001)/0.87E-9 = 1.430

15.D2. Taking the ratio of Eq. (15-23c) at the unknown T and at T =298.16,
exp[ Eo / (TR )]
D (T ) D (298.16) = 1.52E-09 for T = 335.18K. Flux
exp[ Eo / (298.16 R )]
dC A 9 5
J A, z D AB ( D AB / L )(C A , L C A ,0 ) (1.52 10 / 0.0001)(0.9701 1.2) 0.35 10
dz

The temperature can be found with Goal Seek from a spread sheet, but one has to trick Goal Seek
into working. Multiply the desired and the calculated fluxes by 1,000,000 and have Goal Seek
match these two values.

15.D3.a. 0.181cm2/s, b. 0.198 cm2/s, c. 0.0725 cm2/s, d. 0.198 cm2/s

15.D4. a. 0.0875 cm2/s, b. 0.096 cm2/s, c. 0.175 cm2/s, d. 0.096 cm2/s.

15.D5. Use Arrhenius form in Eq. (15-23c) but for mole fraction 0.0332 instead of infinite dilution. Write
the equation for both known temperatures and divide one of these equations by the other. The
constant Do divides out. Take the natural log of both sides and solve for E/R. The result is
D AB (T1 ) 1 1
E/R ln /
D AB (T2 ) T2 T1
The constant Do can be found from the known conditions at T 1
Do DAB (T1 ) / exp[ E / ( RT1 )]
Or from the known conditions at T 2. The results are: E/R = 1348.3, E = 2677.6 cal/mol, DAB
(x=0.0332, T=300) = 1.313×10-9m2/s.

15.D6. Same equations as in 15.D5. At 298.16 K for the infinite dilution value set C sucrose = 0. Final
results are Eo = 4953.8 cal/mol, DAB(infinite dilution, T = 320K) = 0.925×10-9m2/s.

15.D7. For an ideal solution the term in brackets in Eq. (15-22) is equal to 1.0. Write this equation for
two of the xA values with the corresponding diffusivities (e.g., x = 0.0332 with D = 1.007×10-9
m2/s and x = 0.7617 with D = 1.226×10-9m2/s). Then have two equations with the two unknowns:
o
DAB and DBAo
. Solve for the two unknowns. Results are DAB o
= 0.998×10-9 m2/s and DBA o
=
-9 2
1.308×10 m /s. Check results with the other two mole fractions and find that the fit is good.

15.D8. From http://www.engineeringtoolbox.com/ the density of methanol at bp is 750.5 kg/m3 (used a


linear interpolation), which means partial molar volume = 1/(density/MW)= 0.0426 kg/m3.
Viscosity of water is 1.0 cp = 0.001 Pa s = 0.001 kg/(m∙s).
a. With φB = 2.26, DAB = 1.43×10-9 m2/s.
b. With φB = 2.26, DAB = 1.56×10-9 m2/s.

387
1/3
15.D9. Combining Eqs. (15-35b) (15-35d), vvertical ,max,liq 0.5 9 gq 2 / Assume that the bulk is pure
water with infinite dilution of ethanol. From Perry’s Chemical Engineer’s Handbook, 8th edition,
(p. 2-305) at 1.0 bar (0.1 MPa) water has ρW,m,liq = 55.212 kmol/m3 →ρW,liq=994.64 kg/m3 and
ρW,m,vapor = 0.032769kmol/m3 → ρW,vapor = 0.5903 kg/m3. The water boils at 372.76K. At this
temperature, from p. 2-432, the viscosity of liquid water in Pa∙s is,
W ,liq exp[ 52,843 3703.6 / T 5.866 ln T (5.879 10 29 )T 10 ] 2.807 10 4 Pa s
The viscosity of the vapor at 372.76K is (p. 2-426)
W , vapor (1.7096 10 8 )T 1.1146 1.2561 10 5 Pa s or kg/(m∙s).
Now we can calculate the vertical velocity of the liquid water for q = 7.5×10-6m2/s (remember to use
liquid properties).
1/3
2
1/3 9(994.64)(9.81)(7.5 E 6) 2
vvertical ,max,liq 0.5 9 gq / 0.5 0.130 m / s
0.0002807
A check of the units show they work. The modified Reynolds number (using gas properties) is,
d tube ( v gas vliq , y ,max ) (0.10)(0.5903)(0.81 0.130)
Re 5
3195.6
1.256 10
/
The gas phase Schmidt number is Sc gas The viscosity and density were found earlier.
D EW gas

The diffusivity of ethanol and water in the vapor phase at 372.76K and 1.0 bar = 0.98717 atm can be
estimated from the Chapman-Enskog theory with the parameters in Table 15-2. This value of DEW =
1.658×10-5 m2/s. Then Scgas = 1.283. Since both Re and Scgas are in the range for Eq. (15-47a), the
modified Sherwood number is,
k p d tube ( pB )lm
0.0328(Re ) 0.77 Scgas 0.33 0.0328(3195.6).77 (1.283).33 17.79
DAB ptot

15.D10. From the Chapman-Enskog theory DNH3-air = 2.05×10-5m2/s at 318.16K and p = 1.2 atm.
Problem 15.D10a, 3rd ed.
MW A 28.9 MW B 17 const 1.86E-07
T 318.16 p 1.2 T^3/2 5675.033
sigma A 3.711 sigma B 2.9 sigma AB 3.3055
eos A/kB 78.6 eps B/kB 558.3 eps AB/kB 209.4812
kT/EpsAB 1.5188 Col integ 1.197 Linear interpolation table 15-2
D AB 2.05E-05
D, cm^2/s 2.05E-01

The concentration at z = L is CNH3 (L) = CNH3 (z = 0) + JNH3L/DNH3-air. Results are 0.0002483 kmol/m3
and 0.0002117 kmol/m3.

15D11. D = JL/ΔC = 4.114×10-5m2/s. Set up spreadsheet to obtain this value. Since the collision integral
was entered manually, had to do several iterations. After 6 iterations T = 396.2K (see
spreadsheet, and note that collision integral does not exactly match the value of kT/εAB.

388
Problem 15.D11, 3rd ed.
MW A 28.9 MW B 17 const 1.86E-07
T 396.1642 p 0.9 T^3/2 7885.202
sigma A 3.711 sigma B 2.9 sigma AB 3.3055
eos A/kB 78.6 eps B/kB 558.3 eps AB/kB 209.4812
kT/EpsAB 1.891168 Col integ 1.1069 Linear interpolation table 15-2
D AB 4.11E-05
D desired 4.11E-05 chkB7-B8 1.90E-10
chk x E5 1.90E-05 Goal seek to zero changing B3

Problem 15.D11, 3rd ed.


MW A 28.9 MW B 17 const 0.0000001858
T 396.164199034186 p 0.9 T^3/2 =B3*SQRT(B3)
sigma A 3.711 sigma B 2.9 sigma AB =0.5*(B4+D4)
eos A/kB 78.6 eps B/kB 558.3 eps AB/kB =SQRT(B5*D5)
kT/EpsAB =B3/F5 Col integ 1.1069 Linear interpolation
D AB =F2*F3*SQRT(1/B2+1/D2)/(D3*F4*F4*D6) in Table 15-2
D desired 0.00004114 chkB7-B8 =B7-B8
chk x E5 =100000*D8 Goal seek to zero
changing B3

15.D12*. From http://www.engineeringtoolbox.com/ viscosity is 1.0 cp = 0.001 Pa∙s = 0.001 kg/(m∙s).


At 298.16 K, DAB = 1.114 m2/s. Density water = 998.3 kg/m3, viscosity water = 0.001 kg/(ms).
Calculate δ = 0.000115282 m, vy,avg = 0.04338 m/s, Re = 19.966. This is a long residence time
with Re< 20 so there are no ripples. Shavg = 3.41 and kavg = 3.295E-05 m/s, and 0.000168 kg/s
carbon dioxide are absorbed.

15.D13. From http://www.engineeringtoolbox.com/ viscosity is 1.0 cp = 0.001 Pa∙s = 0.001 kg/(m∙s). At


298.16 K, DAB = 1.114 m2/s. Density water = 998.3 kg/m3, viscosity water = 0.001 kg/(m∙s).
Calculate δ = 0.0001663 m, vy,avg = 0.090241 m/s, Re = 59.898. This is a long residence time,
laminar flow, with no surfactant so there are ripples. Shavg = 5.8 and kavg = 3.89E-05 m/s, and
0.000198 kg/s carbon dioxide are absorbed

15.D14. From http://www.engineeringtoolbox.com/ viscosity is 1.0 cp = 0.001 Pa∙s = 0.001 kg/(m∙s). At


298.16 K, DAB = 1.114 m2/s. Density water = 998.3 kg/m3, viscosity water = 0.001 kg/(m∙s).
Calculate δ = 0.0007717 m, vy,avg = 1.9441 m/s, Re = 5989.8. This is turbulent flow with 1300 <
Re < 8300. Scliq = 899.2, Shavg = 255.5 and kavg = 0.0003689 m/s, and 0.00188 kg/s carbon
dioxide are absorbed

15.D15. From http://www.engineeringtoolbox.com/ viscosity is 1.0 cp = 0.001 Pa s = 0.001 kg/(m∙s). At


298.16 K, DAB = 1.114 m2/s. Density water = 998.3 kg/m3, viscosity water = 0.001 kg/(m∙s).
Calculate δ = 0.000115282 m, vy,avg = 0.04338 m/s, Re = 19.966. This is a short residence time
with Re< 20 so there are no ripples. Shavg = 9.942 and kavg = 9.61E-05 m/s, 7.851E-09 kg/s
carbon dioxide are absorbed.

15.D16. From http://www.engineeringtoolbox.com/ viscosity is 1.0 cp = 0.001 Pa∙s = 0.001 kg/(m∙s). At


298.16 K, DAB = 1.114 m2/s. Density water = 998.3 kg/m3, viscosity water = 0.001 kg/(m∙s).

389
Calculate δ = 0.0001663 m, vy,avg = 0.090241 m/s, Re = 59.898. This is a long residence time,
laminar flow, with surfactant so there are no ripples. Sh avg = 3.41 and kavg = 2.28E-05 m/s, and
0.0001165 kg/s carbon dioxide are absorbed.

15.D17. Used a spreadsheet set up to solve Example 15-6. For δ = 0.001 meter one obtains xNH3 =
0.04988, yNH3,surface = .21593, Nwater = 0.5393, NNH3 = 0.0228307. The concentrations are the same
as in Example 15-6, but the fluxes are 10× larger.

15.D18. Part a. For two part solution need values at xE = 0.25 and 0.35. The average molecular weights
are calculated as in Example 15-5, and are used to determine the average molar densities. The
Fickian diffusivities are estimated by interpolating between values given in the Table in Example
15-5. The activity coefficients are determined in the same way as in Example 15-5. Then the
Maxwell-Stefan diffusivities are found by the same method. The values are listed below

MWavg m
,kmol/m3 DEW, m2/s γE DEW , m2/s
XE = 0.25 25.0 36.28 0.633×10-9 1.9028 1.495×10-9
-9
XE = 0.35 27.8 31.62 0.625×10 1.5553 1.609×10-9
Write Eq. (15-61c) for both intervals. For Δz from xE = 0.2 to 0.3 we obtain (values at xE = 0.2 and 0.3
are in Example 15-5),
36.28(1.495 10 9 )[1.7083(0.3) 2.1582(0.2)] 9
zN E 9.3445 10
1.9028(0.25)
From xE = 0.3 to 0.4 (interval is over length δ- Δz) we obtain,
31.62(1.609 10 9 )[1.4338(0.4) 1.7083(0.3)] 9
(0.00068 z) N E 5.7027 10
1.5553(0.35)
Adding the two equations to remove the unknown Δz and then solving for NE and Δz,, we obtain
NE = -2.128×10-5kmol/s and Δz = 0.0004223m

Part b. Since the interval Δz is greater than the interval δ – Δz = 0.0002577m, we subdivide the interval
from xE = 0.2 to 0.3 into 2 parts. The values needed are given below.

MWavg m
,kmol/m3 DEW, m2/s γE DEW , m2/s
XE = 0.225 24.3 37.625 0.659×10-9 2.01976 1.482×10-9
XE = 0.275 25.7 34.99 0.624×10-9 1.79959 1.532×10-9
9
z1 N E 5.5371 10
9
Equation (15-61c) is now written 3 times: z2 N E 3.9846 10
( z1 z2 ) N E 5.7027 10 9
and solved for the 3 unknowns Δz1, Δz2, and NE. Obtain NE = -2.2389×10-5kmol/s, Δz1 = , 0.0002473, and
Δz2 = 0.0001780m.

5
15.D19 (Optional, Unsteady diffusion) At the average C = 0.001 mol/L Dsucrose 0.5228 10 cm 2 / s .
CA z
Equation becomes 1 erf
C A,0 4(0.5228 10 5 )t
Numerical values of C A / C A,0 are easily obtained with a spreadsheet or with the use of Table 17-7.

390
z, cm t = 1000 s t = 10000 s t = 100000 s
0 1 1 1
0.01 0.9221 0.9753 0.9922
0.05 0.6249 0.8771 0.9610
0.1 0.3281 0.7571 0.9221
0.2 0.0505 0.5362 0.8449
0.3 0.00335 0.3535 0.7692
0.4 9.16E-05 0.2161 0.6957
0.5 1.009E-06 0.1220 0.6249
0.6 0.6352 0.5574
0.7 7.61E-12 0.0304 0.4936
0.8 0.0134 0.4346
0.9 0.00538 0.3788
1.0 0.00198 0.3281
1.2 0.000206 0.2406
1.4 1.49E-05 0.1710
1.6 7.49E-07 0.1177
1.8 0.0784
2.0 6.2E-10 0.0505
3.0 0.00335
3.5 6.20E-04
3.56 4.99E-04
4.00 9.16E-05
5.0 1.01E-06

Part b. C 1.0 10 6 when C / C0 5.0 10 4 , which for t = 100000 s occurs for a thickness of <3.56
cm (Goal Seek gives 3.559 cm). Thus, at this time a layer 3.56 cm or thicker appears to be
infinitely thick.
Part c. C 1.0 10 6 when C / C0 5.0 10 4 , which for =0.10 cm occurs at t = 78.938 s (done with
Goal Seek on spreadsheet).

H. Spreadsheet Problems

15H1. Let A = air, B = hydrogen, and C = ammonia. Then NC = -NA – NB. Substitute this expression
into Eqs. (15-65a, b)

m yA yB yC yA yA yA
NA NB
z D AB D AC D AC D AB D AC

m yB yB yB yA yC yB
NA NB
z DBA DBC DBA DBC DBC

Determine NB from the 1st equation and NA from the second.

391
m yA yB yC yA
NA
z D AB D AC D AC
NB
yA yA
D AB D AC

m yB yA yC yB
NB
z D BA D BC D BC
NA
yB yB
D BA D BC

Put these equations and the values for mole fractions at the boundaries, diffusivities, ρm, and Δz = δ into a
spread sheet. Guess a value for NA,guess, calculate NB and NA,calc, and use Goal Seek to make
NA,guess - NA,calc = 0 by changing the value of N A,guess.
Results Nair = -6.209E-5, NH2 = 14.026E-5, and NNH3 = -7.817E-5 kmol/s. As expected hydrogen
diffuses in the positive direction and ammonia in the negative direction. The surprise is the
substantial negative diffusion rate of air. (Spreadsheet shown in 15.H4, but with different
numbers.)

15.H2. Used a spreadsheet set up to solve Example 15-6. For a bulk gas that is 40% air, 15 % NH3 and
45% water obtain xNH3 = 0.05596, yNH3,surface = .24225, Nwater = 0.032964, NNH3 = 0.001955 kmol/s

15.H3a. See solution to 15.H1 for procedure and 15.H4 for example spreadsheet.
Results: Nair = -1.87E-8, NH2 = 2.98E-7, NNH3 = -2.80E-7 kmol/s.
b. For ammonia Deff = 1.5656E-5 m2/s. Estimating dC/dz with the difference approximation for a very
dilute solution, N = J = -Deff ρm Δy/(Δz)= -(1.5656E-5)(0.08928 kmol/m3)(.002)/(.01m) = -2.80E-
7 kmol/s. Thus, this is accurate. For hydrogen and air Dair-H2 = 3.0550E-5 m2/s. Then
Nair = Jair = (3.0550E-5)(0.08928 kmol/m3)(-0.001)/(.01m) = 2.73E-7 kmol/s. The same value is
obtained for hydrogen. The hydrogen value is close, but the air value is not close. Conclusion:
Use the Maxwell-Stefan approach.

15.H4. See solution to 15.H1 for procedure. Results: Nair = -5.903E-5, NH2 = 14.069E-5, NNH3 = -
8.166E-5 kmol/s. Note the substantial negative diffusion of air despite the zero “driving force.”
The air is dragged along with the ammonia. The spreadsheet is given below (labeled as 15.D22),
first with the numbers, and then with the formulas.

392
HW 15-D22, 3rd Ed. SPE
A=air, B=H2, C = NH3
T 273.000000000 p 2.000000000

D AB 1 atm 0.000061100 D AC 1 atm 0.000019800 DBC 1 atm 0.000074800


D AB 0.000030550 D AC 0.000009900 DBC 0.000037400

NA guess -0.000059028

del z 0.010000000 ρ 0.089278949

yA z=0 0.520000000 y B z=0 0.480000000 y C z=0 0.000000000


yA z=δ 0.520000000 yB z=δ 0.000000000 yC z=δ 0.480000000
yA avg 0.520000000 yB avg 0.240000000 yC avg 0.240000000
ρΔyA/δz 0.000000000 ρΔ yB/δz -4.285389534

NB 0.000140693 kmol/s
NA calc -0.000059028 kmol/s
chk NA-Nacalc 0.000000000
100000 chk 0.000000000 Goal seek to zero
NC -0.000081665 change B10

HW 15-D22, 3rd Ed. SPE


A=air, B=H2, C = NH3
T 273 p 2

D AB 1 atm 0.0000611 D AC 1 atm 0.0000198 DBC 1 atm 0.0000748


D AB =B7/D3 D AC =D7/D3 DBC =F7/D3

NA guess -0.0000590279043468439

del z 0.01 ρ =D3/(0.0820575*B3)

yA z=0 0.52 y B z=0 0.48 y C z=0 =1-B14-D14


yA z=δ 0.52 yB z=δ =B9*B11/D3 yC z=δ =1-B15-D15
yA avg =(B14+B15)/2 yB avg =(D14+D15)/2 yC avg =(F14+F15)/2
ρΔyA/δz =-D12*(B14-B15)/B12 ρΔ yB/δz =-D12*(D14-D15)/B12

NB =(B17+(D16/B8+F16/D8+B16/D8)*B10)/(B16/B8-B16/D8) kmol/s
NA calc =(D17+(B16/B8+F16/F8+D16/F8)*B19)/(D16/B8-D16/F8) kmol/s
chk NA-Nacalc =B20-B10
100000 chk =100000*B21 Goal seek to zero
NC =-B19-B20 change B10

393
SPE 3rd Solution Manual Chapter 16
New Problems and new solutions are listed as new immediately after the solution number. These new
problems are: 16.A1, 16.A7, 16.A8, 16.C2 16C4, 16.C5, 16.D3, 16.D9, 16.D16 to 16.D22, 16.G1-
16.G3, 16.H1-16.H2. Chapter 16 was chapter 15 in the 2nd edition. Problems from that edition
have the same problem number, but the chapter number is now 16 (e.g., problem 15.D6 is now
16.D6).
x
16.C3. (was problem 15C3 in the 2nd edition.) Equilibrium y , Operating y = x,
1 1 x
y out
dy
n OG *
. Substitute in for y* and let x = y (total reflux operating line)
y in
y y
dy dy
n OG
y y y y2 y2
y
1 1 y 1 1 y
y out y out
1 dy 1 1 dy
n OG
1 y in
y y 1 1 y in
y 1

1 y out 1 y in 1 y out 1
n OG n 1 n
1 y out y in y in 1
which becomes Eq. (16-81).

16.C4. New problem in 3rd edition. With extract dispersed,


y IN y M,OUT
E MD
y IN y*M,OUT

y M,OUT
Since y IN 0, E MD *
where y*M,OUT m x M,OUT
y M,OUT

Mixer mass balance with y IN 0, Fx IN S yM,OUT F x M,OUT

F x IN S y M ,OUT
Solving for x M,OUT : x M ,OUT
F
F x IN S y M ,OUT
Then, y*M ,OUT m
F
y M ,OUT y M ,OUT
E M ,D *
y M ,OUT m F x IN S y M ,OUT
F
E M ,D m x IN
y M ,OUT
mS
1 E MD
F

395
External MB y IN 0 F x IN S yS,OUT F x S,OUT

yS,IN yS,OUT
E S,D *
Equil : y*S,OUT m x S,OUT
yS,IN y S,OUT

S
Substituting: y*S,OUT m x IN yS,OUT
F

y S,IN y S,OUT y S,IN 1 E SD m E SD x IN


E S,D y S,OUT
S mS
y S,IN m x IN y S,OUT 1 E SD
F F

E MD m x IN
y S,IN y M ,OUT
Substitute in mS and do some algebra to obtain,
1 E MD
F

mS
E MD E SD 1 E MD 1
F
y S,OUT m x IN
mS mS
1 E MD 1 E SD
F F

The definition of the total stage efficiency is,


x D IN x D ,S,OUT y IN y S,OUT y S,OUT
E total,D * *
with y IN 0
x D IN x D ,S,OUT y IN y S,OUT y *S,OUT

S
Equilibrium: y*S,OUT m x IN yS,OUT . Substitute into definition.
F

y S,OUT y S,OUT x IN
E total,D
S S y S,OUT
m x IN y S,OUT m 1
F F x IN

Which after substituting in yS,OUT and doing some algebra, becomes.

mS
E MD E SD 1 E MD 1
F
E total
m SE MD m S E MD mS mS
1 1 E MD E SD 1 E MD 1
F F F F

16.D1. (was problem 15.D1 in the 2nd edition) The corrected value of H G,E 1.41 ft is given in
Example 16-2. From the results of Examples 16-1 and 16-2 and from Eq. (16-38),

396
0.15 0.15
2.61 2.61
H L,E,cor 0.83 0.85 ft
H L,E,Initial
2.2 2.2
mV
Eq. (16-27a) H OG H L H G where V L 8 5
L
The value of m (the slope of the equilibrium curve) varies throughout the enriching section.
From the McCabe-Thiele diagram used to prepare Figure 16-4A values of m were found from
y 0.442 y* .63 to y x D 0.8 y* 0.82 .

y 0.442 0.5315 0.621 0.7105 0.8


y* 0.63 0.598 0.66 .727 .822
m 0.441 0.406 0.449 .5144 .745
H OG 2.01 1.96 2.02 2.11 2.42
y* - y 0.188 0.0665 0.039 0.0165 0.022
y* y
1 5.32 15.04 25.641 60.606 45.45

1
Arithmetic Average: H OG 2.01 1.96 2.02 2.11 2.42 2.10
5
15
Geometric Average: H OG 2.01 1.96 2.02 2.11 2.42 2.10
No difference!

To find n OG do Simpson’s rule in 2 parts because of the unusual shape of 1 y* y vs. y.


0.179
n OG ,1 5.32 4 15.04 25.641 2.718
6
0.179
n OG ,2 25.641 4 60.606 45.45 9.353
6
n OG n OG,1 n OG,2 12.072 , h E H OG n OG 25.35
This is reasonably close to the 26.1 ft estimated in Example 16-1.

16.D2. (was problem 15.D2 in the 2nd edition) a. The physical properties and use of Figure 10-25
calculated in Example 16-2 are unchanged. Now F = 48.
12 12
F 2 20
G flood G flood 2 1.16 0.749 lb ft 2
F 1 48
14 14
F 1 48
Dia 1 Dia 2 4.68 5.83 ft
F 2 20
b. In Eq. (16-37) 2 141, 1 55, Dcol 2 same .
Estimate h p ~ 10 ft (we know it will be less than before), SC,V is unchanged, terms in
denominator are unchanged.
13
13
1 hp 1 10 55 10
H G ,E 1 13
H G ,E 2 1.33 0.40 ft
2 hp 2 10 141 22

397
In Eq. (16-38), 1 0.045 , 2 0.07 , h p 1 10 , h p 2 22 .
Cf ,L , and SC,L unchanged.
0.15 0.15
1 hp 1 0.045 10
H L,E 1 H L,E 2 0.83 0.47 ft
2 hp 2 0.07 22

16.D3. New problem in 3rd edition. In the enriching section,


HETP H OG ln(mV / L) / [(mV / L ) 1] and H OG H L mV / L H G
With H L 0.827, H G 1.33, m / ( L / V ) 0.63 / (5 / 8), H OG 2.16, and HETP = 2.15 (from
Example 16-2). With the same mass transfer coefficients, but m = 0.577,
H OG [0.577 / (5 / 8)]0.83 1.33 2.10 and HETP 2.10 ln(0.9232) / (.9232 1) 2.19 ft

A 24.4% increase in both mass transfer coefficients gives H L 1.244(0.83) 1.033 and
HG 1.244(1.33) 1.65 .
For m = 0.63, H OG [0.63 / (5 / 8)]1.033 1.65 2.69 and
HETP 2.69 ln[0.63 / (5 / 8)] / [0.63 / (5 / 8) 1] 2.68 .
With the same mass transfer coefficients, but with m = 0.577, H OG 2.61 and HETP = 2.71.

A 24.4% decrease in both mass transfer coefficients gives:


For m = 0.63, H L 0.63, H G 1.01, H OG 1.65, HETP 1.64
For m = 0.577, H L 0.63, H G 1.01, H OG 1.59, HETP 1.65 .

Clearly, the variation in mass transfer coefficients results in a large range for HETP (from 1.64 to
2.69 feet for m = 0.63) while the small change in m had little effect. To be safe the larger value
of HETP = 2.69 would be used. This is a safety factor of 1.20. Note that Bolles and Fair (1982)
recommend a safety factor of up to 1.70.

16.D4. (was problem 15.D4 in the 2nd edition) Since we have total reboiler, ys,in 0.04
q .6
Feed line: q .6, 1.5 . Plot this and operating lines (see figure).
q-1 .4
L L L L D 0.9
Top: y x 1 x D where 0.474
V V V 1 L D 1.9
L
y intercept 1 xD 0.484
D
Bottom goes through y x xB 0.04 .

398
dy
nG , where y S,out y E,in .605, y E,out .92
y AI y A
For enriching section, from Eq. (16-16) draw line of slope
L HG 1.3
0.474 .770 to get y I and x I . For stripping section slope is
V HL 0.8
L HG 1.3
2.568 4.17
V HL .8
From figure generate following table.

yA yAI yAI-yA 1/(yAi-yA)


Stripping: .04 .13 0.09 11.111
.3225 .46 .1375 7.2727
.605 .63 .025 40.00
Enriching: .605 .62 .015 66.66
.7625 .80 .0375 26.66
.92 .95 .03 33.333
.565
Simpson’s rule, n G ,S 11.11 4 7.2727 40 7.55 , h strip H G n G,S 9.8 ft.
6
.315
n G ,E 66.66 4 26.66 33.33 10.8 , h E HG n G 1.3 10.8 14.1 ft
6

16.D5. (was problem 15.D5 in the 2nd edition) a) Total Reflux. y out 0.956, yin 0.65
1 .956 1 .65 .65 1
Eq. (16-81), n OG n n
1 2.26 .65 1 .956 .956 1

399
6
n OG 4.0257, H OG 1.4904
4.0257
b) Finite reflux. Plot op. line. Find y & y* (see graph). Use Simpson’s rule in 2 parts.

y y* y* y 1
y* y
0.783 0.842 0.059 16.949 n OG1
0.8615 0.783
16.949 4 17.467 19.608 1.3924
0.82225 0.8795 0.05725 17.467 6
0.8615 .9125 0.051 19.608
0.94 0.8615 1.89166
.90075 .943 0.04225 23.6686 n OG 2 19.608 4 23.6686 30.303
6 n OG 3.2841
0.94 .973 0.033 30.303

c) Changes in L/V in equation connecting HETP and H OG

400
401
16.D6. (was problem 15.D6 in the 2nd edition) a. n OG 11.11 3.56 14.67
H OG h n OG 7.47 14.67 0.509 m
b. From McCabe-Thiele diagram we find the following

y 0.016 0.066 0.116 .494 .872 .922 .972


y* .0267 0.1067 0.1815 .623 .9201 .9523 .9832
y* - y 0.017 0.0407 0.0655 .129 0.0481 .0303 .0112
1/(y* - y) 93.46 24.57 15.27 7.752 20.79 33.00 89.29

Do integration with Simpson’s rule in three parts.


.1 .756
n OG 93.46 4 24.57 15.27 15.27 7.752 4 20.79
6 6
.1
20.79 4 33.0 89.29 15.97
6
The difference is because of inaccuracies in Simpson’s rule.

16.D7. (was problem 15.D7 in the 2nd edition) Bottoms x A partial reboiler 0.1 Distillate
xA .9
From equilibrium data in 4.D7. @ x A 0.1 , equil. y 0.262 yin With total condenser,
y out xD 0.9
yA x A
Find average . From data in 4.D7., y* 0.929 when x 0.9.
1 yA 1 xA
0.262 0.1 .929 0.9
Bottom 3.195 , dist 1.454
0.738 0.9 .071 0.1
12
Geometric Avg., avg 3.195 1.454 2.155
1 0.9 1 0.262 0.262 1
Eq. (16-81), n OG n n 4.80
1 2.155 0.262 1 0.9 0.9 1
H OG h n OG 0.42 m

16.D8. (was problem 15.D8 in the 2nd edition) m H Ptot 22500 855 26.3

402
L 26.3 .0011 L
max 28.93 , act 26.04 . Basis V = 1, then L = 26.04.
V .0011 .0001 V
L
y out x in x out 26.04 .001 0.02604
V
L 26.04
x *A ,out 0, 0.9895
mV 26.316
Use Colburn Eq. (16-34b),
1 0.0011 0
n OL n 0.010488 .9895 9.51
0.010488 0.0001 0
h 0.84 9.51 7.99 m
Can check with Eq. (16-63) and get same n OL

16.D9. New problem in 3rd edition.


For example, if nO ED 1, we have for the perfectly mixed model,
ED nO
EMD 1/ 2
1 nO ED
For the plug flow model,
EMD 1 exp( nO ED ) 1 exp( 1) 0.632
For same value of nO ED the plug flow model always predicts a higher stage efficiency.

16.D10. (was problem 15.D10 in the 2 nd edition) For both countercurrent and cocurrent
y in
dy
n OG *
. For both cases y* 0, and y y* y , then
y out y y
y in
dy y in 0.01
n OG n n 4.6
y out
y y out 0.0001
Note that n OG is same because of irreversible reaction.
c. Flow rates enter into solution only as a check that at least the stoichiometric amount of sulfuric
acid is available to neutralize the ammonia.

16.D11. (was problem 15.D11 in the 2nd edition) From Eq. (16-72) with irreversible reaction,
y A in 50.0 ppm
n OG n n 8.517 for both cocurrent and countercurrent.
y A out 0.01 ppm

16.D12. a. (was problem 15.D12 in the 2nd edition) m H p tot 2.7 1.1 2.4545, y mx
L .013 .00004
2.4453 (see Figure)
G min .0053
L y in y out
15 L G min
36.679, x out 0.0003533
G L G
Use Eq. (16-57), yin 0.013, y*in m x out 0.0008672, yout 0.00004, y*out 0

403
y y* 0.0121328, y y* .00004
in out

.01296 .0121328
n OG n 6.1246 , h 4.59 ft
0.0120928 .00004

b. Cocurrent. Operating and equilibrium lines shown in figure. Lowest y out is at intersection point
= 0.00081. For y out 0.00085 ,
y in y out .013 .00085
x out 0.00033125, y*Aout mx out 0.000813
L V 36.679
m 2.4545
0.0669184 . Use Eq. (16-20),
L V 36.679
1 0.013 0.000813
n OG 1.0669184 0.0669184 5.496
1.0669184 0.00085 0.000813
h 1.98 ft

16.D13. (was problem 15.D13 in 2nd edition) a. Same conditions as Example 16-3. Assume same H OL .
If operation is possible, find n OG & y out .
Dilute – Use Eq. 16-70. m = 30.36 (Example 16-3)
y*out m x out b 30.36 0.001 0.03036
mV 30.36 1 mV y in y *out mV
2.024 , n OG n 1 *
L 15 mV L y out y L
1 out
L
Mass bal. Lx in Vyin Lx out Vy out
L L
y out x in y in x out 15 0 0.03082 15 0.001 0.01582
V V
1 0.03082 0.03036 mV
n OG n 3.024
3.024 0.01582 0.03036 L
Not possible, term inside brackets is negative.
b) Same conditions as Example 16-3 except x out 0.002 . Assume same H OL . If operation is
possible, then

404
y*out m x out 30.36 .002 0.06072
y out 15 0 0.03082 15 0.002 0.00082
1 0.03082 0.06072
n OG n 3.024 2.024 still not possible
3.024 0.00082 0.06072
c) Same conditions, except L V 40 and x out 0.0003
*
y out 30.36 0.0003 0.0091 , mV L 30.36 40 0.759
y out 15 0 0.03082 40 0.0003 0.01882
1 0.03082 0.0091
n OG n 1.759 0.759 0.656
1.759 0.01882 0.0091

16.D14. (was problem 15.D14 in the 2nd edition) a. Use equilibrium data shown in Figure 4-16,
x 0.8, m 0.415; x 0.16, m 1.5
Mixed: E MV 1 exp K y aA active h V [Eq. (16-77)] where V A c 30 and b 2 12 .
V
Then: K ya n 1 E MV
h A active
30
x 0.8, K y a 1 2 n 1 .77 330.62 where 1 2 0.8
2 12
30
x 0.16, K y a n 1 .69 263.5
.8 2 12
1 m 1
Eq. (16-6a),
K ya k xa k ya
1 0.415 1 1 1.5 1
x 0.8, , x 0.16,
330.62 k xa k ya 263.5 k xa k ya
Solving simultaneously, k x a 1408.19 and k y a 366.317
b. x 0.01, m 6.06. Then from Eq. (16-6a), K y a 142.18 & E MV 0.468 from Eq. 16-77.

16-D15. (was problem 15.D1 in the 2 nd edition) Assume feed to Example 16-4 is sat’d liquid, z = 0.5, &
2.5 .5
separation complete x D ~ 1, x B ~ 0 . x F z 0.5, y f 0.7143
1 1.5 .5
L 1 0.7143 L 0.7143 0
xD 1, 0.5714 , x B 0, 1.4286
Vmin 1 0.5 V max
0.5 0
L L
At x .5, y intersection op lines x 1 xD .8 .5 .2 1.0 0.6
V V
L 0.6 0
1.2
V actual
0.5 0
Calculate at x 0.1, 0.3, 0.7 in example , 0.9

405
x Sect L/V L
m 2
Eq. 15 80
1 1 x mV
0.1 Strip 1.2 2.5 0.6348
2
1.8904
1 .15
0.3 Strip 1.2 2.5 1.0092
2
1.1891
1 .45
0.7 Enrich .8 2.5 1.3447
2
0.5949
1 1.05
0.9 Enrich .8 2.5 1.767
2
0.453
1 1.35
L E MV
E mV const. 0.97. Calculate Ept. Solve Eq. (16-78) E pt n 1
mV L mV
V
and then K y a from Eq. (16-76a), K y a n 1 E pt
A active h
x E pt K ya
0.1 0.971 133.42
0.6348 n 1 0.5891
.6348
0.3 0.971 171.02
1.0092 n 1 0.6802
1.0092
0.7 0.971 196.9
1.3447 n 1 0.731
1.3447
0.9 0.971 222.65
1.767 n 1 0.7733
1.767

16.D16. New problem in 3rd edition. Using Simpson’s rule the new value for A1= 7.18, the new value
for A2 = 10.85 and the new total area = 18.03. Then the calculated height of the enriching section
(0.4054 m)(18.03) = 7.31 m compared to the previous result of 7.95 m. This is an error of 8.1 %.
Thus, a rather small error in mole fractions becomes a larger error.

16.D17. New problem in 3rd edition. Relative errors in k G a 24.4% .


Same relative errors in H G and H L .

In the enrichment section the slope of the mass transfer line is

L HG 5 0.4054 24.4%
Slope
V HL 8 0.253 24.4%

406
a) If H L correct but H G varies by 24.4%, range of slope is from 1.242 to 0.761.
Value at HG 0.4054 is slope = 1.0015. At top:

If equilibrium line straight from azeotrope to point x = 0.7472, y = 0.7815, then can fit
this portion of equilibrium as,

y 0.7668x 0.2085

MT line y sx b. Since y x 0.8, b 0.8 1 s

If s 1.242, b 1.7936

If s 1.0015, b 1.6012

If s 0.761, b 1.4088

All calculations at y A 0.8. y I at intersection equilibrium, y 0.7668x 0.2085 and


y b
mass transfer line, y sx b x
s

407
yI b
Substitute in for x, yI 0.7668 0.2085
s
Solve for y I
0.7668b
0.2085
yI s
.7668
1
s

1
yI yI y yI y
s 1.242, b 1.7936 : 0.81356 0.013565 73.72
s = 1.0015, b = 1.6012: 0.812426 0.01246 80.48*
s = 0.761, b = 1.4038: 0.80842 0.008419 118.78

*83.3 in Example 16.1 since numbers rounded first. Amount of error depends on
distance between equilibrium and operating lines. Less error if closer, but more impact
on 1 y AI y I .
Assume same relative errors:
1 80.48 73.72
H L no error, H G higher, error 8.4%
y AI yA 80.48
1 118.78 80.48
H L no error, H G lower, error 47.6%
y AI yA 80.48
Assume error in H is same every point. Thus enriching area can be a lot different than
calculated. But if H G is down by 24.4%, area is up by 47.6% so there is some
cancellation of error.

b. If H L & H G both vary could have s 1.634 and b 2.1072 . Thus, y I 0.81493,
1
yI y 0.493, = 66.97
yI y

66.97
Area 19.6 16.31. Since
80.48
H GE 1.24(0.4054) 0.503 m, z H G,E n G,E 8.20 m which is a 3.2 % increase.

16.D18. New problem in 3rd edition.


a. Equilibrium is y = mx. Value of m is unknown, but Cextract = mconc units Craffinate with

kmol _ Benzoic / m 3extract


mconc _ units 20.8 . We need m in
kmol _ Benzoic / m 3raffinate

408
mol Benzoic / mol extract
mmole fraction units . The resulting conversion is,
mol Benzoic / mol raffinate
MWextract raffinate
mmole _ fraction _ units mconc _ units ( )( )
extract MWraffinate
Since the system is dilute, extract properties are essentially the same as pure solvent and
raffinate properties are essentially the same as diluent.

m = (20.8)(92.14/865)(1000/18) = 123.1

Note that y = xD and EMy = EMD.

EMy = (yin-yout)/(yin-yout*) where yout* = m xout = (123.1)(1.99E-06) = .000245

EMy = (0-.000230)/(0-.000245) = 0.939

b. From Eq. (16-76b), nO-ED = EMy/(1-EMy) = 0.939/(1 – 0.939) = 15.393

KO a Vmixer
nO ED
ED
, Vmixer h D2 / 4 (0.75) 0.75 2 / 4 0.331m3
QD D MWD
KO ED a nO ED QD D / (Vmixer MWD )
(15.393)(.0012)(865) / [(0.331)(92.14)]
0.524 kmol / s m 2 mol frac disp m 2dispersed / m 3total volume
Note that K O ED a is larger than in Example 16-5 because the residence time
Vmixer
t res 37.45s is shorter than the 361 s in Example 16-5. Thus, this problem
(Q D Q C ) d
requires much more vigorous mixing.

c. Differential Model E MD 1 exp n OED

exp n OED 1 E MD 0.061

n OED n 0.061 2.797 n OED 2.797

KO ED a nO ED QD D / (Vmixer MWD )
(2.7969)(.0012)(865) / [(0.331)(92.14)]
0.0952 kmol / s m 2 mol frac disp m 2dispersed / m 3 total volume

d. Use of mixed models: If use K O ED a 0.524 from mixed staged model, then
KO ED a Vmixer
nO ED 15.393 and with differential model
QD D MWD

409
E MD 1 exp n OED 1 exp( 15.393) .9999998 . Obviously, mixing models gives
wrong results.

16.D19. New problem in 3rd edition.


C D in C D out
a. E MD ,Conc *
with C*D,out mCD CC,out
C D in C D ,out

CD,in 0.000 , CD,out 0.00023 , CC,out 0.00536

C*D,out mCD CC,out 0.0481 CC,out 0.0002578

.00023
E MD .892
.0002578
nO E MD,Conc 0.892
b. E MD,Conc ED
nO ED 8.26
1 nO ED 1 E MD,Conc 0.108
KO ED a nO ED QD D / (Vmixer MWD )
(8.26)(.0012)(865) / [(0.331)(92.14)]
0.2811 kmol / s m 2 kmol/m 3 m 2 dispersed / m 3 total volume

c. Differential Model:
E MD,Conc 1 exp n OED → exp n OED 1 E MD,Conc 0.108

n OED n .108 2.2256 n OED 2.2256

KO ED a nO ED QD D / (Vmixer MWD )
(2.2256)(.0012)(865) / [(0.331)(92.14)]

0.0757 kmol / s m 2 kmol/m 3 m 2 dispersed / m 3 total volume

d. E MD,Conc 1 exp n OED 1 exp 8.26 0.99974


16.D.20. New Problem in 3rd Edition.
a. Eq. (16-92). Terms: d 0.167, Dbenzoic-water 2.2 10-9 C
4 3 2 . 6 8
CD A C

(from Example 16-6)


2 2
di 0.2070 16.6667
5.8632
g 9.807
Note N in rps

410
di 0.2070
820.13
dp 0.0002524
dp 0.0002524
0.00030487
d tank 0.8279
D d pg 865 0.0002524 9.807
96.447
0.0222
9
2.2 10 5 12 2
kC 0.00001237 432.68 820.13
0.0002524
12 5 4 1
x 0.00030487 96.447 12
0.014801
0.167
Significantly higher. But Note: d 0.06, so correlation may not be valid. Same
1 1 1
With same k d 0.001905.
K LD kD m CD k C
1 1 1
1930.2
K LD 0.001905 1
0.014801
20.8
1
K LD 0.0005181 & K LD a 2.0586 s
K OED a 865 2.0568 92.14 19.13
19.31
E MD 0.951, which agrees with guess for residence time.
20.31
1
b. Resistance (dispersed) = 524.9
0.001905
1
Resistance (continuous) 1404.5
0.000712
Sum of resistances 1929.4 Gives K LD 1 1929.4 0.0005181
% resistance from dispersed 524.9 1929.4 100 27.2%
This is a significant contribution because of significantly higher predicted
continuous phase mass transfer coefficient.
c. If k D is ignored , K LD k C m CD 0.014801 20.8 0.000712
Note that this is significantly too high.
d. To be safe, use lower estimate of E MD . Note that d is really too large for use of
Eq. (16-92).

16.D.21. New problem in 3rd edition. In settler velocity approaches zero


( u t 0.00172 in Example 13-5, but d p 0.000204 not 0.000150 assumed). Thus
u t ~ 0.00495 m s and Eq. (16-88a) is reasonable approximation.
Sh C k C d Dbenzoic-water 2.0 where in settler d 0.0002524

411
9 5
kC 2.0 2.2 10 0.0002524 1.7433 10
5 7
Using K LD ~ k C mCD 1.7433 10 1 20.8 8.381 10 ms
If the interface in the settler is at the centerline, then the volume of aqueous phase is
1
D S2 4 L S . From Example 13-5, DS 1.023m and LS 4DS . Then Volume
2
aqueous phase 1.682 m3 . The water residence time is
1.682 m3 0.006 m3 s 280.3s. Entering total velocity is 0.0072 m3 s and

d 0.167 leaving mixer. A drop that starts at the bottom of the water phase travels a
vertical distance of DS 2 0.5115 m to reach the interface. This requires
0.5115 m 0.00495 m s 103.2s.
Assume equal distribution of drops. Everything (half the drops) above interface are
collected very quickly. All drops collected in 103.2 280.3 0.368 fraction of settler.
If average over this fraction is 0.5 d,IN , then D,avg 0.5 0.167 0.368 0.031.
6 6 0.031
a d
737 and K LD a settler
~ 0.000618
dp 0.0002524
K OED a settler D K LD a settler
MWD 865 0.000618 92.14 0.0058
Vsettler Q D 0.0058 3.364 0.0012 0.0058
n OED,settler 1.7314.
865 92.14 865 92.14
Since settler is not well mixed, E settler,D 1 exp 1.734 0.823
This is high because of long residence time.

16.D.22. New problem in 3rd edition.


y M,OUT yS,OUT y IN y M ,OUT y M ,OUT
a) E SD *
0.82, E MD *
0.794
y M,OUT y S,OUT y IN y M ,OUT y*M ,OUT
y*M,OUT mx raf ,OUT .
From Example 16.5 in mole fraction units m 123.1.
Mixer mass balance with y IN 0 (Example 16.5) is Fx F Sy M,OUT Fx C,OUT
3
In Example 13.5 feed is 0.006 m s and solvent is 0.0012 m3 s

S 0.0012 m3 s 865 kg m3 92.14 kg kmol 0.01127 kmol s


F 0.006 m3 s 998 kg m3 18.02 kg kmol 0.3323kmol s
0.01127
x M ,OUT x feed S F y OUT 0.00026 y OUT
0.3323
Then substitute into E MD
y M ,OUT
0.794
123.1 0.00026 0.03390 y M ,OUT

412
Solving, we obtain y M,OUT 0.00589.

x M,OUT y M,OUT m E MD 0.00589 123.1 0.794 6.027E 5.


For settler, y*S,OUT m x S,OUT
Settler M.B. is, Sy M,OUT Fx M,OUT SyS,OUT Fx S,OUT
S
x S,OUT x M ,OUT y S,OUT y M ,OUT
F
x S,OUT 6.027E 5 0.03390 yS,OUT 0.00589
Substitute into equation for E SD

0.00589 yS,OUT
0.82
0.00589 123.1 6.027E 5 0.00019967 0.03390y S,OUT
Solving, we obtain yS,OUT 0.006174.
From Mass Balance,

x S,OUT 6.027E 5 0.03390 0.006174 0.00589 5.0653E 5


*
y S,OUT m x S,OUT 123.1 5.0653E 5 0.006235
yS,OUT 0.006174
E Total,D *
0.99015
y S,OUT 0.006235
mS 0.01127
b) From 16.C4 123.1 4.17495
F 0.3323
E Total,D
0.794 0.82 1 0.794 4.17495 1
1 4.1794 0.794 1 4.17495 0.82 4.17495 0.794 0.82 1 0.794 4.17495 1
.99003

16.G1 and 16.G2. New problems in Chapter 16. Aspen Plus runs showed that N =13 (Aspen Plus
notation) with feed on Nfeed = 11 (optimum location) gave ethanol mole fractions of xD = 0.7990
and xB = 0.020298. These values are within the specified tolerances. The stripping section starts
with the vapor leaving the reboiler (yin,strip = yreb = 0.18709) and ends at the intersection of the two
operating lines. This last value can be determined by calculating the points on the operating lines
(xn, yn+1). For example n = 1, x1 and y2 are on the enriching section operating line. When the
slope changes from 0.61 in the top to 2.07 in the bottom, the intersection point has been passed.
This occurs for yout,strip = y11 = 0.44631.

The mole fractions of ethanol in the liquid and vapor leaving each stage (Aspen stage notation is
used) are:

413
Stage x y Equilibrium: Calculation m
1 0.79904 0.81824
2 0.77157 0.79904 The equilibrium parameter m is the average slope of the
3 0.74561 0.78189 equilibrium curve from x (calculated at y) to xI.
4 0.71971 0.76569 At the reboiler y = 0.18709 for x = 0.020289, and y* =
5 0.69245 0.74957 0.44631 for xoper =0.10033. Then yavg = 0.3167.
6 0.66203 0.73263 The slope can be determined by taking the chord from
7 0.62575 0.71375 x = 0.04 (y*= 0.29209) to x = .05 (y*= 0.33018).
8 0.57882 0.69129 m = (0.33018 – 0.29209)/0.01 = 3.809.
9 0.51135 0.66233 The equilibrium values are from Analysis in Aspen Plus.
10 0.40050 0.62090 At yout,strip = 0.44631, xoper = 0.22553, y*= 0.55339, and
11 0.22553 0.55339 yavg = 0.49985. m = (0.50405 – 0.49482)/(0.15 – 0.14)
12 0.10033 0.44631 or m = 0.923.
13 0.020289 0.18709

To use Simpson’s rule for the first integral in Eq. (16-22a) we also need an average m for the y
and y* values calculated at the average between yin,strip and yout,stip, which is y = 0.3167. At this y,
xop = 0.1659 (determined from the stripping section operating line) and y*=0.5174. The average
between y and y* = 0.41705. m = (0.42921 – 0.41012)/(0.09 – 0.08) = 1.909. The second integral
in Eq. (16-22a) is the usual estimation from Simpson’s rule of the nOG integral,

y A ,out

HG dy A / ( y A * y A ) = (0.2835/6)(0.44631 – 0.18709)[1/(0.44631 – 0.18709) + 4/(0.5174 - 0.3167)


y A ,in

+ 1/(0.55339 – 0.44631)] = 0.4057 m

The first integral in Eq. (16-22a) can also be estimated from Simpson’s rule,

y A ,out

( H L / ( L / V )) mdy A / ( y A * y A ) = (0.1067/2.032){[(0.44631 – 0.18709)/6]×


y A ,in

[3.8577(3.809) + 4(4.983)(1.909) + 9.3388(0.923)]} = 0.1393

Then h = .4057 + 0.1393 = 0.545 m, which is somewhat more than the 0.507 m calculated in
Example 16-1. Note that the Aspen Plus calculation does not assume CMO whereas the
calculation in Example 16-1 did, but the calculation here required an assumption of how to
calculate m. Thus, it is difficult to say which is more accurate.

16.G2. New problem in 3rd edition.


Enriching section yin,enrich = yout,strip = 0.44631, and yout,enrich = xD =x1 = 0.79904. At yin,enrich =
yout,strip = 0.44631, the calculation of m is the same as done previously for yout,strip: xoper =
0.22553, y*= 0.55339, and yavg = 0.49985; m = (0.50405 – 0.49482)/(0.15 – 0.14) = 0.923.
At yout,enrich = xD =x1 = 0.79904, y* = 0.81824, and yavg = .80864; m = (0.81180 – 0.80481)/0.01 =
0.699.

In Example 16-1 the enriching section integration was done with Simpson’s rule in two parts.
From yin,enrich to y = 0.725 and from y = 0.725 to yout,enrich. For the calculation here we will use y =
0.71375, which is in the table of data from Aspen Plus, as a convenient break mole fraction to do
the integration in two parts. For y = 0.71375, xoper = 0.66203, y*= 0.73263, and to find m, yavg =
0.72319. m = (0.72623 – 0.72101)/(0.01) = 0.522.

414
The average y from yin,enrich = 0.44631 to y = 0.71375 is 0.58003, xoper = 0.44423, y* = 0.63659,
and to find m, yavg = 0.60831. m = (0.61020 – 0.6067)/(0.01) = 0.350.

For the 2nd integral in Eq. (16-22a) integrated from yin,enrich = 0.44631 to y = 0.71375 we obtain,

y A ,out

HG dy A / ( y A * y A ) = (0.4054/6)[.71375 - .44631][1/(.55339-.44631) + 4/(.63659 -.58003) +


y A ,in

1/(.73263-.71375)] = 2.404 m

Same integral integrated from y = 0.71375 to yout,enrich = 0.79904. The average y from y = 0.71375
to yout,enrich = 0.79904 is 0.75640, xoper = 0.73068, y* = 0.77246, and to find m, yavg = 0.76443. m
= (0.76587 – 0.75985)/(0.01) = 0.602.

=(0.4054/6)[.79904 - .71375][ 1/(.73263-.71375) + 4/(.77246 -.75640) + 1/(.81824 - .79904)] = 2.041

Total for integral 2 = 4.445 m

For first integral from yin,enrich = 0.44631 to y = 0.71375 we obtain,

y A ,out

( H L / ( L / V )) mdy A / ( y A * y A ) = (0.253/0.625)[(0.71375 – 0.44631)/6][(0.923)(9.388) +


y A ,in

4(0.350)(62.2665) + (0.522)(52.966)] = 1.102

Same integral integrated from y = 0.71375 to yout,enrich = 0.79904,

y A ,out

( H L / ( L / V )) mdy A / ( y A * y A ) = (0.253/0.625)[(0.79904 - 0.71375)/6][(0.522)(52.966) +


y A ,in

4(.602)(18.2815) + (.699)(52.0833)] = 0.870

Total for integral 1 = 1.972 m

Total height of enriching section = 4.445 + 1.972 = 6.417 m


Total height of packing = 0.545 (from 16.G1) + 6.417 = 6.962 m.
This is less than the total of 8.457 m calculated in Example 16-1. However, it does agree
reasonably well with the number of stages (11) in the column since,
11 × HETP (estimated = 0.655 m in Example 16-2) = 7.208.

Both this result and Example 16-1 require calculating a small difference and then taking the
inverse of this number. This type of calculation can cause very significant errors. The graphical
calculation was based on accurate experimental measurements of the equilibrium data, and this
data is probably more accurate than the NRTL correlation used in the computer. On the other
hand, calculation errors are probably larger in the graphical than in the computer calculation.

Both calculations depend significantly on the accuracy of the mass transfer data (HL and HG),
which can easily have errors greater than 20%, which can cause even larger errors in calculation
of NTU and HTU or in HETP (see problem 16.D16) for the staged calculation.

415
16.G3. New problem in 3rd edition.
Part b. Equilibrium stage optimum. (L/D)min = 5.1, L/D = 6.12, N = 33, NF = 14, 1 section, yC4,dist
= 0.008556, xC3,bot = 0.005706, Dia = 1.804 m on plate 32, minimum diameter = 1.565m on tray
14.

Part c. VPLUG optimum. N = 33, NF = 14, yC4,dist = 0.007953, xC3,bot = 0.005302, Dia = 1.812 m
on plate 32, DC backup/tray spacing = 0.391, weir loading = 0.021m2/s. Note that this separation
is better than the equilibrium result.

Part d. MIXED optimum. N = 49, NF = 22, yC4,dist = 0.007651, xC3,bot = 0.00510, Dia = 1.808 m on
plate 48, DC backup/tray spacing = 0.390, weir loading = 0.021m2/s. Note that this separation
require significantly more stages than both the equilibrium-stage and the plug flow results.

16.H1 and 16H.2. New problems in 3rd edition. Fit for the ethanol-water VLE was done in Appendix B of
Chapter 2. The spread sheet for both problems is given below with a y value chosen in the
enriching section (ignore the stripping section operating line values). Overall mass balances to
find D and B were done with Eqs. (3-3) and (3-1). L and V were determined at the total
condenser and flows in the stripping section were determined at the feed stage with the calculated
value of q. L bar = L + q F. The intersection point of the two operating lines was determined
from Eq. (4-38). yreb is the start of the stripping section and is the y value in equilibrium (yeq)
with y = x = xB. In the stripping section the value of 1/(yeq – y) was determined at the start of the
stripping section (yreb), at the end of the stripping section (yintersection), and at the average of these
two values. Then Simpson’s rule was used to calculate nOG,strip = 1.57787 from Eq. (16-24b). To
determine HOG from Eq. (16-27a) an average slope m of the equilibrium curve is required. The
slope of the chord from the equilibrium y at the intersection point of the two operating lines to the
reboiler vapor that is in equilibrium with xB, mstrip ( y eq ( xint er sec tion) y reboiler) / ( xint ersec tion x B)
= 1.95, and HOG,strip = 1.265. The resulting height of the stripping section, 1.998 feet, is somewhat
more than the 1.66 feet determined in Example 16-1.

In the enriching section a similar procedure was used except the integration to find nOG,enrich was
done in two parts. The average slope of the equilibrium curve was determined from,
menrich ( y1 yequilibrium( xint ersec tion)) / ( x D xint ersec tion) = 0.4558 with y1 xD . The resulting
value of HOG,enrich = 1.935. The resulting height of the enriching section, 29.35 feet, is somewhat
more than the 26.1 feet determined in Example 16-1 and more than the 25.35 feet determined in
problem 16.D1.

Note: in both 16.H1 and 16.H2 the average slope of the equilibrium curve m has to be calculated
with fairly large chords, not by taking the derivative of the 6th order polynomial fit to the VLE.
The reason is that the fit oscillates around the experimental data points and the slopes will
fluctuate greatly.

416
417
SPE 3rd Edition Solution Manual Chapter 17.

New Problems and new solutions are listed as new immediately after the solution number. These new
problems are: 17A2, 17A7-17A9, 17B2, 17C4,17D3, 17D8, 17D10-17D13, 17D15b-h, 17D16-
17D18, 17E1, 17H2-17H7. Chapter 17 was chapter 16 in the 2nd edition. Problems from that
edition have the same problem number, but the chapter number is now 17 (e.g., problem 16.D6 is
now 17.D6). Figures in the solutions to these old problems still have the designation of chapter
16.

17.A2. New problem in 3rd edition. Change the value of thetatot in the spreadsheet until the area matches
the desired value. This can be done as a manual trial and error or a loop can be added to the spreadsheet.

17.A7. New problem in 3rd edition. Part a. Increase the stirrer speed.
If no gel, increasing stirring increases mass transfer coefficient k which reduces M and hence xw is
reduced. This reduces xp increasing retention R.
Part b. Decrease the stirrer speed. This reduces k, increases M and xw. When xw > xgel, a gel forms and
R increases (probably to l.0) .

17.A8. New problem in 3rd edition. Since there is a gel the retention of the low molecular weight
compound also increases.

17.A9. New problem in 3rd edition. Do not invest. Osmotic pressure can often be ignored in UF because
with large molecules with high molecular weight the mole fraction is always low even if the weight
fraction is high. With low mole fraction the osmotic pressure is low. If there is a concentrated salt with a
low molecular weight the mole fraction will be high and the osmotic pressure cannot be ignored.

17.B1. Look at Suk, D.E. and Matsuura, T. (2006) ‘Membrane-based hybrid processes: a review’, Sep.
Sci. Technol. Vol. 41, pp.595–626 for additional processes.

17.B.2. New problem in 3rd edition. One possible approach is as follows: Increase stirring to increase the
mass transfer coefficient and reduce the wall concentration to prevent gel formation. Then use a permeate
in series cascade with recycle of the retentate from the second module in series back to the feed of the
first module. The low molecular weight product is the permeate from the second module. The
intermediate molecular weight polypeptide product is the retentate from the first module.

10 cc STP cm
17.D1. PCO 2 15 10 tm 1 10 6 m
cm 2 s cm Hg
pr pH 12 atm 12 76.0 912.0 cm Hg
10
PCH4 0.48 10 pP pL 0.2 atm 15.2 cm Hg
AB PCO2 PCH4 31.25 ˆ CO ˆ CH 1.0, p p p r 0.016666
2 4

a) Generate RT curve from Eq. 17-6a.


pp
yp 1 1 yP 1
pr y P 1.5042 0.5042 y P
yr
AB AB 1 yP 31.25-30.25 y P

418
RT Curve
yP yr Op. Eq., FP FIN 0.32

0 0 1 FP FIN y IN
yP y out
0.1 0.00515 FP FIN FP FIN
0.20 0.01114 yP y out y IN 0.15 CO 2 mole frac
1 0.32
0.30 0.01830 Slope 2.125
0.32
0.40 0.02721 Plot two arbitrary points:
0.5 0.03882 y out 0, y P .15 .32 0.46875
0.6 0.05504 y out 0.10, y P 2.125 .10 0.46875 0.25625
0.7 0.08000
0.8 0.12492
0.9 0.2349
1.0 1.0 Answer (from graph) yP 0.402, y out 0.0276 CO 2 conc.
PCO2
J CO2 pr yr pP yP (17-2b)
t ms
10
15 10 cc STP cm cc STP
J CO2 4 2
912 cm Hg 0.0276 15.2 cm Hg 0.402 0.0002859
1 10 cm cm s cm Hg cm 2 s
FP gmole 1 hr gmol
FP 1000 0.088888
FIN h 3600 s s
cc STP L 1.0 gmole
J CO 2 0.0002859 2
1.2764 E 8 gmole s cm 2
cm s 1000 cc 22.4 LSTP
FP y P,CO 2
Area 2.80 10 6 cm 2
J CO 2
FP
FP Fin 0.32 kgmole/hr , Fout Fin FP 1 0.32 0.68 kgmole/hr.
Fin
b. Fin,1 1 kgmole/hr
1 1
FP1 FP,part a 0.32 0.16 FP 2 , Fout1 Fin2 FP1 1 0.16 0.84 Fin ,2
2 2
Stage 1. FP1 Fin1 0.16 1.00 0.16 , yin ,1 0.15
RT curve is unchanged!
1 .16 0.84
Op. Line: Slope 5.25
0.16 0.16
Find arbitrary points to plot line:
0.15
If y out ,1 0, y p 0.9375 (off graph).
0.16

419
If y out,1 0.04, y p 5.25 0.04 0.9375 0.7275
If y out,1 0.08, y p 5.25 0.08 0.9375 0.5175
Answer (from graph): y P,1 0.625, y out,1 0.0595 yin,2
FP 2 0.16 y in2 0.0595
Stage 2 0.1905, 0.3123
Fin 2 0.84 FP2 Fin2 0.1905
1 0.1905
Slope 4.2500 . Plot curve
0.1905
Answer: y P2 0.250, yout 2 0.015 (see graph)
Stage 1:
15 10 10 cm 3 STP
J CO 2 912 0.0595 15.2 0.625 0.0006715
1 10 4 cm 2s
1L 1 mol
JCO 2 J CO 2 2.9976 E 8
1000 cc 22.4 cm 2s
mol 1 h
FP1 0.16 1000 0.04444 mol s
h 3600s
FP1 0.625
Area 1 92.67m 2
JCO 2 ,1

15 1010 cm 3 STP
Stage 2: J CO 2 ,2 4
912 0.015 13.2 0.250 0.0001482
1 10 cm 2s
1L 1 mol
JCO 2 ,2 J CO 2 3
6.6161 E 8
1000 cm 22.4 cm 2s
lh
FP 2 0.1905 840 mol h 0.04445 mol s
3600 s
FP 2 y P 2
Area 2 1, 680, 000 cm 2 168 m 2
JCO 2 ,2
It is interesting to compare parts a and b.

Part a: 1 stage Area 280m 2


y out,CO 2 0.0276 or 97.24% CH 4
y P,CO 2 0.402
Part b: 2 stage Total Area 260.67m 2
y out 0.015 or 98.5% CH 4
y P1 0.625 y P2 0.250

420
1 FP Fin y in
17.D2. a. yP y out , yP y out y in 0.2 CO2 , FP Fin .3
FP Fin FP Fin
.7 0.2
Slope 2.333 , When y out 0, y P 0.6667
.3 0.3

421
y in 0.2
When yP 0, y out 0.286
1 FP Fin .7
RT curve is same as in Problem 17.D1.
Draw op line. From graph: y P,CO2 0.53, y r y out,CO2 0.06
PA p r y r pP yP
b. J CO2
t ms

10
15 10 cm 3 STP cm 76 cm Hg
J CO 2 60 atm 0.06 3.3 0.53 atm
1 10 4 cm cm 2s cm Hg atm
cm 3 STP FP mol
J CO 2 0.002148 2
, FP Fin 0.6 .
cm s Fin s
0.6 mol s 0.53 1000 cm 3 STP
A 3
3, 254, 000 cm 2
1 mol cm STP L STP
0.002148
22.4 L STP cm 2s
Or 325.4 m 2 . Very sensitive to y P & y r values.
FP
Can also calculate J J CO 2 J CH 4 . A
ˆJ
Check: Fin FP Fout , 2 0.6 Fout Fout 1.4
Fin x in FP y P Fout y out , 0 .4 2 0.2 0.6 0.53 1.4 0.016 .402 , OK

422
17.D3.
New problem in 3rd edition Since no concentration polarization x w xr x out ,
K solv
J solv pr pp a xr xp
t ms
J solv K solv
Solve for pr pp a xr xp , 33.29, J solv 415.4
K solv t ms t ms
xp
R 1 xp 1 R x out , R 0.9804
x out
1 xF
and M.B. xp , 0.22, x F 0.0077

xF 0.0077 0.22
Then, x out 0.0098175
1 0.78
1 R 1 0.9804
0.22
xp 1 R x out 0.0001924
415.4
Then pr 1.1 15.446 0.0098175 0.0001924 13.72 atm.
33.29
17.D4. Partially new problem in 3rd edition.

423
xw 4.625 / 997000 g/m 3
a. M exp ( J solv / solv ) k exp 5
1.069
xr 6.94 10
1 xP Mxr xP
xP pr pP Mxr xP a
1 3.6 E 4 1.069 0.054 3.6 E 4 1
2.29
3.6 E 4 75 2 1.069 0.054 3.6 E 4 59.895 atm
K solv J solv 4.625 g
0.0665 2
t ms pr pp a Mxr xP 73 1.0689 .054 3.6 E 4 59.895 m s atm
KA K solv
Since K solv K A , K A K solv and
t ms t ms
g
0.0665
KA m s atm 0.029 g 2

t ms 2.29 atm 1 m 2s
c. Write Eq. (17-37a) for old and new situations – Divide new by old. Obtain
0.75
new
k new k old Everything else divides out. Since rpm ,
old
.75
2000
k new k old 0.000117m / s
1000
3
K solv J solv 1.5 10 g 1
17.D5. a. 2
1.47 10 5 g cm 2 s atm
t ms pr pP cm s 102 atm
1 FP Fin x in
RT eq., R 1 x P x out , Op eq., x P x out
FP Fin FP Fin
Solve simultaneously & obtain Eq. (17-26), which with M = 1 is
1 R x in 0.003 0.05 xP
xP 0.000272 , x out 0.091
1 R 1 0.997 0.45 1 R
1 xP xr xP
x P pr pP xr xP a
1 0.000272 0.091 0.000272 1
3.44 atm
0.000272 102 0.091 0.000272 59.895
b. Plot the RT curve and operating line
xP xP a 1 pr pP 1 1 x in
xr , xP xr
M 1 a 1 xP
See graph. Intersection occurs at x r 0.0585, x P 0.000752
K solv
J solv pr pP a M xr xP
t ms

424
425
5
1.47 10 g g
J solv 2
78 atm 59.895 atm 3 0.0585-7.52 10 -4 0.000991
cm s atm cm 2 s
FP 1 x P 2 2
A , FP Fin 5 kg s 3.33 kg s
J solv 3 3
4
3.33 kg s 1 7.52 10 1000 g kg
A 4 2
3.36 10 6 cm 2 336 m 2
9.91 10 g cm s
17.D6. Eq. (17-45) can be written as J solv / solv k n xg n xr
Plot J solvent vs n x r . Slope = k and intercept = k n x g
cm wt frac
J solv / solv xr n xr
min dextran
0.052 0.012 4.423
0.037 0.03 3.507
0.026 0.06 2.813
0.0134 0.135 2.00
See Figure. Plot J solv / solv J solv versus ln xr
Intercept 0.0185
n xg 1.159, which is x g 0.314 .
Slope 0.01596

K solv J solv 69.25 g


17.D7. Stirred cell data: cP g L, 23.1 2
t ms p 3.0 m s bar
L
J solv J solv / solv 23.1 / (997 g / L ) 0.0232 2
m s bar

426
J A ~ J solv c P 0.0232 0.30 0.00696 g (m 2 s) . Also, J A M c cout 1 R c J solv
JA 0.00696
With Mc 1, 1 R c 0.03 → R c 0.97
c out J solv 10 (0.0232)
Spiral Wound: JA J so l v c P M c cout 1 R c J solv
cP 1.0
Solving for M c 4.1660
c out 1 R c 8 0.030
K solv
J solv p 23.1 3.5 80.8g / (m 2s)
t ms
Since osmotic pressure is ignored, M c does not effect solvent flux in UF.

Pp
yp He H 2 1 1 yp 1
rd
Pr
17.D8. New problem in 3 edition From 17-6b, y rHe
1 y P He
10
23.7 10
He H 2 PHe PH 2 0.2610 , p p p r 0.2
90.8 10 10
.739 0.2 .975 1 0.021397
y rHe 0.025 .07656
.261 .739 0.25 .279475
Pp
y PH H 2 He 1 1 y PH 1
2
Pr 2 0.975 3.8314 1 .2 .025 1
Check y rH .9234
2
H 2 He H 2 He 1 y PH 3.8314 2.8314 .975
2

y rHe 1 y rH 0.07656 OK.


2

y FHe y rout He 0.05 0.07656


Then 0.5152
y PHe y r ,He 0.025 0.07656
b. Use solution in Eqs. (17-9) to (17-10e)
pp .75
a 1 .2 .261 1 2.3648
1 pr .25
pp y F,He 1 .05 1
b 1 .739 .2 3 1.4874
pr 1 1 1 .25 .25

c y F,He (1 ) .261 .05 .25 0.0522

b b2 4ac 1.4874 2.212359 .49377 1.4874 1.645


yP
2a 2 2.3648 4.7296
.15763
Must use minus sign to have positive y p . y PHe 0.00333
4.7296

427
y F,He .05
y r,He y PHe 3 .00333 0.10001
1 1 .25
pp pp
c. Solve RT eq. (17-6b) for y p : y r yr 1 yp 1 y p2 1 1 yp
pr pr
pp pp
0 1 y 2p 1 yr 1 1 yp yr
pr pr
c
b
a

pp
a 1 .739 .2 0.1478
pr
pp
b 1 1 yr 1 .739 .2 1 .06 .739 .80786
pr
c yr .06 .261 0.01566
b b2 4ac .80786 .65264
yp 0.004842
2a .2956
y FHe y rHe 0.05 0.06
Use + sign for positive y p , 0.1813
y PHe y rHe 0.004842 0.06
d. For Part A FIN 100m3 STP / h. y He,IN 0.05, y H2 IN .95
y rH2 .9234, y rHe 0.07656 y p,H 2 0.025, y p,H 2 0.975 .5152
PH 2 A
Use 17-5a written as FP y PH pr yr pp yp
2
tm s
FP FIN 51.52 m3 STP h

51.52m 3 STP 1000000 cm 3 1h cm 3 STP


FP 14311.11
h m3 3600 s s

cm 3 STP
FP y PH tm s 14311.11 .975 1.0 10 4 cm
A 2 s
PH 2 Pr y r,H 2 Pp y P,H 2 10 cm 3 STP cm
90.8 10 380 .9234 76 .975 cm Hg
cm 2s cm Hg
A 555,186 cm 2 55.52 m 2
17.D9. a. Plot the data on a semilog plot in the form of J solv / solv J solv L / (m 2 h)

428
xr
J solv k n . k n xg k n xr. Intercept k n xg
xg
L m
From graph, slope k 18.3 and k 18.3 2
5.08
m h s
2 2
When x r 0, J solv / solv =J solv 82.9 L m h 23.0g / (m s) k n xg
23.0 g m 2s
n xg 4.53 xg 92.8%
5.08 m / s

The value of xg is very sensitive.

b. There is only one point further out on the ℓn axis. Any error in point is greatly amplified in the
least squares regression. Hence, another point in this region would be most useful. The higher
the concentration, the better.

17.D10. a) New problem in 3rd edition Solve RT eq. and op. eq. simultaneously
2700 pp 0.3 .5
3.375 , a 1 2.375 1.6116
800 1 pr 0.7 2
pp yN 1 .5 .3 .25 1
b 1 2.375 3.8884
pr 1 1 1 2 .7 .7 .7
c yin 1 3.375 .25 .7 1.20536
2
3.8884 3.8884 4 1.6116 1.20536
yp , use minus sign to have yp between
2 1.6116
0 and 1..

429
3.8884 15.1196 4 1.6116 1.205
yp 0.365295
2 1.6116
y IN .3 .25
From op. eq.: y r yp .365295 0.2006
1 1 .7 .7
PA A ˆ A
b) Since F̂p y p,A pr yr pp yp
t ms
F̂p y p,A t ms
A ˆ
. Since F 1 mol s, Fˆp FˆIN 0.3 mol s
IN
PA ˆ A p r y r pp yp
mol
0.3 0.365295 1.2 10 4 cm
s
A 3
10 cm STP cm 1 1.0 L STP
2700 10 2
cm s cm Hg L STP 10 3 cm 3 STP
22.4
mol
76 cm Hg
2.0 .2006 .5 .365295 atm
atm
A 6.569 104 cm 2

F̂p y p,A t ms
c) A , Fˆ p Fˆ IN 0.4 mol s
PA ˆ A p r y r pp y2
mol
0.4 0.325 1.2 10 4 cm
s
A
1
3
cm STP cm 22.4L STP 1.0 L STP 76 cm Hg
2700 1010 2
2.0 0.175 0.5 0.325 atm
cm s cm Hg mol 10 3 cm 3STP atm

A 9.0823 104 cm 2
x IN 0.001
17.D11. New problem in 3rd edition. xp 0 since R 0 1.0, x out 0.0016667
1 .6
Gelling occurs at a solvent flux of J solv 5200 L/(m 2 day ) which is
L g day g
J solv J solv solv 5200 2
997 60.0
m day L 86400 s m2s
60.00g / (m 2 s) 997000g / m 3
Then x gel x out exp (J solv / solv ) k 0.0016667 exp 0.01334
2.89 10 5 m / s

PHe
17.D12. New problem in 3rd edition p p / p r 1.0 4.5 . He H 2 0.261. Use Eq. (17-6b),
PH 2

430
p 1
y p ,He He H 2 1 1 yp 1 0.1 .261 1 1 .1 1
pr 4.5
y r ,He 0.25446
1 yp .261 .261 1 .1
y FHe y rHE .2 .254
Eq. (17-7c) 0.353
y p He y rHe .1 .254

17.D13. New problem in 3rd edition .035 NaCl x in , cut .55 , perfectly mixed
1 R xn
Rejection xp (17-27)
1 R
a) If x p 0.00050 & 0.55, what value R required. Find R (including concentration
polarization effect). From Eq (17-27), x p xpR x IN Rx IN which gives
x IN xp 0.035 0.0005
R x IN xp x IN xp → R 0.9935
x IN xp 0.035 .55 .0005
x IN xp 0.035 0.001
b) If xp 0.0016, 0.55 , R 0.9869
x IN xp 0.035 .55 .001
c) If R° (inherent rejection coefficient with M = 1) for part b is R 0.992, what was value of
M that gave R 0.9869
M CaseB MA 1
R Case B 1 1 R CaseA . Let A be highly stirred .
M CaseA RA R
M CaseA 1 R CaseB 1.0 1 0.9869
M CaseB 1.6375
1 R CaseA 1 0.992

wB xw 43 x w
17.D14. RT curve: y w
1 wB 1 xw 1 42 x w
wB 43 (mole frac). Since x w ,IN 0.10, only need RT curve below 0.10. Create table and plot
xw yw
0.10 0.8269
0.08 0.78299
0.05 0.6935
0.03 0.571
0.01 0.3028
0.0025 0.0973
0.001 0.0413

cal kcal 74.12 g


B 141.6 10.5 kcal mol
g 1000 cal mol
Feed 0.9 10.5 0.1 9.72 10.42 kcal mol

431
1 1
C PB 0.625 74.12 0.046 , C PW 1.0 18.016 0.018
1000 1000
CP,in 0.9 0.046 0.1 0.018 0.0435 kcal mol C

MW feed xW , F MWW xB , F MWB 0.1(18.016) 0.9(74.12) 68.51


a) Assume y P 0.5 to calculate λp 0.5 10.5 0.5 9.72 10.11
C PL,in 0.0435
and Tin Tout where Tin Tout 30 then, 30 0.129. This is a
P
10.11
1
molar ratio. Slope Op line 6.75 , and op line goes through point (mole fractions)

x in 0.10
x out 0, y P 0.775. Plot operating line. From graph, y P 0.57, x out 0.031
0.129
(mole fraction water). This value of yp is reasonably close to our assumption.

Fp / FF ( Fp MW p ) / ( FF MWF ) ( MW p / MWF )

432
MWp y p ,W MWW y p , B MWB = 0.57(18.016) + 0.43 (74.12) = 42.13
.129(42.13 / 68.51) 0.0791 in (lb/h)/(lb/h).
Area Permeate Rate Flux ( 0.0791 100 lb h) 0.2 lb h ft 2 39.53 ft 2
b) Cut 0.08 x in,w 0.10
1 cut 0.92
Slope 11.5. Find y P 0.68 from graph.
cut 0.08
Then P 0.32 10.5 0.68 9.72 9.97
0.08
Tin Tout P 30 9.97 kcal mol 48.3 C
C PL,in 0.0435kcal / (mol o C)

c) x out 0.05, x P 0.6935 (From RT table or graph).


x in x out 0.10 0.05
FP Fin 0.0777
yP x out 0.6935 0.05
P 0.3065 10.5 0.6935 9.72 9.959

433
0.0777
Tin Tout 30 C 9.959 47.8 C
C PL,in 0.0435
17.D15. Parts b to h are new in 3rd edition RT curve: xP 1 R M xr 0.
Mass balance perfectly mixed Fin x r ,in FP x P Fout x r ,out
Fin
Since xP 0, x r,out x r,in
Fout
Fout Fin FP FP Fin 1
1 0.8 . Then
Fin Fin Fin Fout 0.8
1
x r ,out 0.10 0.125 , Fout 0.8Fin 80 kg h
0.8
Alternate graphical solution gives same result.

6.5 Op. line


1 FP Fin x r ,in
-4 xP xr xP x r ,out
xP FP Fin FP Fin
x r,in 0.10 0.8
RT curve Slope 4
0.125 0.2 x r,in 0.10
xr x r,out When x r,out 0, x P 0.5
FP Fin 0.2

b. Area = Fp / Jsolv = (20kg/h)(1 L)/0.997 kg)(24h/day)/ (2500 L/m2 day) = 0.193 m2.

c. Gel formation occurs when x w = 0.5 and xw = M xout = 0.125 M. M = 0.5/0.125 = 4.0

d. Gel formation occurs when xw = 0.5 and xw = M xout = M xF / (1 – θ’) = 1.2 (0.1)/(1 – θ’)
Then 1 – θ’ = 0.12/0.5 = 0.24 and θ’ = 0.76.

e. Gel formation occurs when x w = 0.5 and xw = M xout = 1.2 xF / (1 – θ’) = 1.2 xF / (1 – 0.2).
Obtain xF = 0.333.
xr,out = xF / (1 – θ’) = 0.3333/0.8 = 0.416

f. We have xr,out = xF /(1 – θ’) = 0.20/0.75 = 0.26667. M = xgel / xr,out = 0.5/0.26667 = 1.875.
First occurs when Jsolv = 2500 = k ln (M). Obtain k = 3977 L/(m2 day) = 4.603×10-5 m/s.

g. M = 1.875 and k = 3977. Since we change the pressures, J changes which will change M. However
with constant stirring k is constant. First, assume no gel and calculate J and M.
pr pp K Jsolv 2500 L (m 2 kg) L
J solv K solv , solv 2083.33 2
t ms t ms p r pp 2.2 1.0 bar m day bar

Then, without gel, J solv 2083.33 3.4 1.0 5000 L m 2 d ay


From Eq. (17-34) M = exp (Jsolv / k) = 3.516.

434
Mx F 3.516 0.2
0.2. Then, x w 0.878, gel forms.
1 .8
xF 0.2
With a gel, previous work is incorrect. Set R = 1.0, x p = 0, x r 0.25,
1 .8
And from Eq. (17-45), Jsolv = k ln (xgel/xr) = 3977 ln (0.5/0.25) = 2756.6 L/(m2 day)
Note: The same answer is obtained in parts g and h if convert to J´solv and use k in m/s.
0.2
h. Gel forms since it did previously, k 3977, 0.26, x F 0.2, x r 0.27027
.74
x gel .5 L 0.5
J solv k n k n 3977 2
n 2446.6 L (m 2 d ay)
xr xr m d ay 0.27027
17.D16. a) New problem in 3rd edition Case B, R 0B 0.976, M B 1.0, p p,B 1.1, p r,B 12.06
Case C, R C .939, p p,C 1.1, p r,C 15.2

1 RC pr pp 1.0 .061 14.1


C
For Experiment C. M C MB 3.27
1 RB pr pp .024 10.96
B

x IN 0.0093 0.26
b) x out 0.01230
1 .74
1 RC 1 .939
.26
xp 1 R C x out 0.0007505
K solv
J solv pr pp a M C xout xp
tms
1.387 152 1.1 15.446[ 3.27 .01230 0.0007505] 18.71 g m 2s

17.D17. New Problem in 3rd edition.


a) log a log P b , P 0.0001, 42 , P 10000, 1.9
log 42 a log 0.0001 b 1.6232 4a b
log 1.9 a log 10, 000 b 0.27875 4a b
1.9020 2b
b 0.95100 , a .16805
Equation: log 0.16805 logP 0.951

b) If 8, log 0.90309

log .951
log P
0.16805 0.16805
log P 0.28509 PO2 1.93 Barrers

435
17.D.18. New problem in 3rd edition. Ideal Gas: Vol% = Mole %
10
PN2 250, PCO2 2700, PHe 300, PHe 550 10
3

3
10 2 cm
FP FIN 0.4 m3 s. FIN 1.0 m s 3
10 6 cm 3 s , FP .4 106 cm3 s
m
76 cm Hg
Part A pr 2.5 atm 190 cm Hg pp 76 cm Hg
atm
Part B pr 76 cm Hg pp 0.4 76 30.4 cm Hg
.00254 cm
t ms 0.8 mil 0.002032 cm
mil
Pi t ms p r
Eq. (17-11d) K m,i Need to guess value of FP A or of y P ,
FP A Pi t ms p p
Since CO 2 has highest permeability, CO 2 will be concentrated; thus, guess y p,CO2 yCO2,IN 0.40
y IN ,CO 2
Then y r ,CO 2 y p,CO 2 ,guess where 0.4 and y IN,CO2 0.4.
1 1
Then, FP A guess PCO 2 t ms y p,CO 2 p r y r.CO 2 p p y p,CO 2

Use FP A in Eq. (17-11d) to calculate all K mi


y IN,i
Then check y r,i 1.0 ?
1 K mi 1
Put in Spread Sheet. Can use Goal Seek to force y r,i 1.0 as change y P,CO2 .
Results: a. y P,N2 .15037, y P,CO2 .54446, yP,He .3351, yP,H 2 .27154, .999885
y r,N2 .3164, y r,CO2 .3037, y r,He .06099, y r,H2 .3189 1.0000766
b. Same answers for mole fractions since p r p p is same.

306 2008 HW 8 Problem 2

yF
yF N2 0.25 yF CO2 0.4 He 0.05 yF H2 0.3
F,cm3/s 1000000 tm, cm 0.002032 theta 0.4
pr, cm Hg 190 pp, cm Hg 76
P N2 0.000000025 P CO2 0.00000027
P He 0.00000003 P H2 0.000000055
Fp 400000
change yp toget sum=1
yp CO2 0.544455884 yr CO2 0.303696077
Fp/A 0.003983763 A, cm 2 100407575.7
A, m 2 10040.75757
K N2 0.475237299 y r N2 0.316417678
K CO2 1.792765611 yr CO2 0.303696077

436
K He 0.549397235 yr He 0.060993544
K H2 0.851323363 yr H2 0.318969314
Goal seek Sum 1.000076613
yp N2 0.150373483
yp CO2 0.544455884
yp He 0.033509684
yp H2 0.271546029
sum 0.99988508

17.E1. New problem in 3rd edition For dilute systems J solvent J total solution , FIN Fp Fout , Basis: FIN 1.0
Fp xF x out 0.022 0.056
Transfer Eq. (17-7c) 0.6106
FIN xp x out 0.00032 0.056
xp 0.00032
R 1 1 0.9943 , J s u c rose J solventx p J solvent solution x p
x out 0.056 Permeate

permate 0.997 0.4 x p 0.997 0.4 0.00032 0.99713 kg L


Initial assumption is OK.
J solv J solv / solv (3.923 g / m 2 s) / (997 g / L) 0.003935 L / ( m 2 s)
J sucrose (3.923g / m 2 s)(0.00032) 0.00126g / (m 2s)
K water 1 xp Mxr xp
b) Eq. (17-27) water solute
K sucrose x p pr pp Mxr xp a
1 .00032 0.056 0.00032 g water
w s 3.131
0.00032 60.0 1.1 0.056 0.00032 59.895 g sucrose
K water J solv
From Eq. (17-16c),
tms pr pp a Mxr xp
K water 3.923
0.0706 g m 2 s atm
t ms 60 1.1 59.895 1.0 .056 0.00032
K sucrose J sucrose 0.00126 g sucrose
Eq. (17-18) 0.0226
tms M xr xp 0.056 0.00032 m 2s wt frac
c. Solution 1. R 1 M 1 R 1 2.1 1 0.9943 0.98803
Then RT equation is xp 1 R x out 0.01197 x out
1 xF .39 0.022
Operating Equation is (17-23) xp x out x out
.61 0.61
xp 0.6393 x out 0.036066
Solve RT & operating equation simultaneously. x out 0.05537 , x p 0.000663

437
xp
Check R 1 0.98803 0K
x out
K water
J water pr pp a M xr xp
t ms
= 0.0706 60 1.1 59.895[ 2.1 0.05537 0.000663] 3.67g/(m 2s)
J sucrose J water x p / (1 x p )

J sucrose 3.67 0.000663 / (1 .000663) 0.00243 g (m2 s)


K sucrose g
Alternatively, J sucrose Mx r xp 0.0226[ 2.1 (0.05537) 0.000663] 0.00261
t ms m 2s
6.9% different
xp xp 3.140 59.895 0.997 1 3.140 .997 60 1.1 1
Solution 2. RT Eq. (17-21), x r
2.1 1 3.140 59.895 .997 1 xp

x p 186.5 x p 185.391
Simplifies to, xr
2.1 391.66 x p
Linearize x r @ x p 0.0003 0.02509 . Note xp = 0.0003 is an arbitrary point.
xp 0.0003
Slope = 0.01196
xr 0.02509
Then linear form of RT equation is x p 0.01196 x r or x p 0.01196 x out
Solve simultaneouly with Operating Equation
1 xF
xp x out 0.6393 x out 0.036066

x out 0.05538 , x p 0.0006623 . Very close to value obtained with retention analysis.

FP C PL,in
17.E2. (was 16.D11 in 2nd ed.) Eq. (17-59b): Tin Tout , Tin Tout 85 25 60
Fin P

Stage 1. Assume y P ~ .05 water, 0.95 ethanol


P 0.95 w 0.05 E 2290.3 kJ kg (See Example 17-9)
For Feed CPL,in 0.1 CPL,w 55 C 0.9 CPL,E
kJ kJ
0.1 4.1915 0.9 2.7595 2.903
kg K kg K
Where average temperature from 25 to 85 is 55ºC and C P values are from Perry’s 7th, pp. 2-
306 and pp. 2-237.

FP1 2.903
60 0.0760 and FP1 0.0760 100 7.60 kg hr .
Fin 2290.3

438
Op. line intersects y P x out x IN 0.10 (water wt. frac.)
1 1 0.760
Slope 12.16
0.0760
0.1
Op. Eq. is, y P 12.16 x out 12.16 x out 1.32
0.0760
If y P 1, x out 0.32 12.16 0.0263
Plot Op. Line on Figure 16-17a and find intersection:
y P1 0.66, x out1 x 2,in 0.055 (water values)
Fout1 Fin 2 100 7.60 92.40 kg h

Stage 1 Trial 2. Since yp ≠ yp, assumed, do a second trial.

yP 0.66 water, 0.34 E, P 0.66 w 0.34 E 0.66 2359 0.34 985 1892 kJ kg
FP1 2.903 60
0.0921
F1in 1892
FP1 0.0921 100 9.21 kg h , Fout1 100 9.21 91.79 kg h
1 0.0921 0.1
Slope 9.86 , y P 9.86 x out 9.86 x out 1.086
0.0921 0.0921
0.086
If y P 1, x out 0.00872
9.86
Plot operating line and determine (from graph) y P1 0.64, x out1 x in 2 0.05 water. This
value of yp is close to the assumed value of 0.66. Can proceed to stage 2.

Stage 2: Estimate y P 0.50 (water), P 0.50 w 0.50 E 1672


For x in 2 0.050, CPL,in 0.05 4.1915 0.95 2.903 2.967
FP2 2.967
2 60 0.1065, Fin 2 .1065 90.66 9.64 kg h
Fin 2 1672
1 0.1065
Fout 2 90.66 9.64 8.10 kg hr , Slope 8.40
0.1065
0.050
yP 8.40 x out 8.40 x out 0.4421
0.1065
If x out 0, y P 0.4421 . Draw op. line. Intersection gives y P 0.34
Fin 2 x in 2 FP2 y P2 90.66 0.050 9.64 0.34
For x out 2 use MB. x out 2 ,
Fout 2 81.00
x out 2 ,w 0.0155 or x out,2 ,ETOH 0.9845.
This is a close as we can get graphically.
9.21 0.64 9.64 0.34
Mixed Permeate: y p,mix 0.487 wt frac water
9.21 9.64

439
kg 1000 g
FP
h kg
Area , J from Fig. 16-17b based on x out
J g h m2
9.34 1000 g h
Stage 1 J 0.8333 g h m 2 , A1 11, 208 m2
0.8333
9.64 1000
Stage 2 J 0.208 g h m 2 , A 2 46, 346 m2
0.208
Other flow patterns will reduce area. Area is large because of low flux caused by low ethanol
permeation rate.

x x 16.6 .3
17.F1. RT eqn., y , x Benz 0.3, 16.6, y 0.87676
1 1 x 1 15.6 .3
18.3 .2
x Benz 0.2, 18.3, y 0.8266
1 17.3 .2
6.66 .1
x Benz 0.1, 6.66, y 0.4253 . Plot RT equation.
1 5.66 .1
1 .9
Operating equation Slope 9 . Plot on graph. Find y PBenz ~ 0.844, x out,Benz ~ 0.238
.1

440
P y Pbenz benz 1 y Pbenz iP 0.844 94.27 1 0.844 164 105.15 cal g

CPL,in x Pbenxw CPLbenz 1 x benz,in CPL ,iP 0.3 0.423 .7 0.73 0.6379 cal g C

441
P
0.1 105.15
Tin Tout 50 C 66.48 C
C PL,in 0.6379

17.H1. (was 16.G1 in 2nd edition)


This is set up for Area being the unknown and cut being known. Problem 17.H1
Fr,in 10000.000000 yin,A 0.2500 cut=Fp/Fin 0.2500
tmem,cm 0.002540 pr,cm Hg 300.0000 pp,cm Hg 30.0000
yin,B 0.5500 P,A 0.0000000200
Fptot 2500.000000 yin,C 0.2000 P,B 0.0000000050
Fr,out 7500.000000 P,C 0.0000000025

Guess values of A or equivalently Fp/A until sum y,r and sum u,p are = 1.00
Fp/A 0.0007059 (this is final result)
KA 2.507328 KB 0.7720 KC 0.4015
sum x eq
y,r,A 0.181576 y,p,A 0.455271198
y,r,B 0.583244 y,p,B 0.450268647
y,r,C 0.235190 y,p,C 0.094429331 Area, cm2 3541578.1
sum y,r 1.000010 sum y,p 9.999692E-01

These results agree very well with Geankoplis’ results.

17.H.2. New problem in 3rd edition Part a)


y p 0.5243, y r,out 0.0610, A 3, 200,152 cm 2
b) y p,avg 0.6193, y r,out 0.0203, A 2, 636,196 cm 2

17.H3. New problem in 3rd edition


Counter –current. Shows final guess for theta.
Fin,
cm3/s 100000 yin 0.209 thetatot 0.714 PB/tms 0.00175
PA/tms 0.003905 pr, cm Hg 114 pp, cm Hg 76
M 15 N 100 yroutguess 0.2 erroracc 0.0000001
df 0.9

j=N-i+1 100 99 98 97 96 95 94
Fr 28600 29314 30028 30742 31456 32170 32884
yp 0.173174301 0.1738615 0.174547 0.1752323 0.175916 0.1765981 0.17727918
yr 0.144051968 0.1447613 0.14547 0.1461768 0.146883 0.147588 0.14829195
Area 9710.750234 9664.4075 9618.879 9574.1438 9530.183 9486.978 9444.50995
Fp 714 1428 2142 2856 3570 4284 4998
Fp/Fr,j-1 0.024356963 0.0475556 0.069677 0.0907935 0.110973 0.1302761 0.14875885
yp 0.235015561
Areatot 824015.8215
yincalc 0.208999973
Fincalc 100000
Massbal 9.09495E-13
yrout 0.144051968

442
17.H4. New problem in 3rd edition The spread sheet equations are shown below for part b. Part a agreed
with problem 17.D14. Part b answers: yp,W = 0.412, θ = 0.2122, xout,W = 0.0160, θ’= 0.158,
Area = 79.0 ft2. Note that if the starting guess for yp,W is too high, Goal Seek will converge
on an answer with yp,W > 1, which is obviously not physically possible.

17.H5. This problem is very similar to Example 17.7. It is easiest to solve on a spreadsheet, which is
shown below. The results are shown in the spreadsheet.
New problem in 3rd edition

443
444
17.H6. New problem in 3rd edition The spreadsheet is similar to that for problem 17.H5 and is shown
below,

17.H.7. The same spread sheet that was used in problems 17.H5 is used.

445
446
SPE 3rd Edition Solution Manual Chapter 18.

New Problems and new solutions are listed as new immediately after the solution number. These new
problems are: 18.A3, 18.A16, 18.B4, 18.C4, 18.C14, 18.D3, 18.D8, 18D9, 18D14, 18D15,
18D18, 18D21, 18D24, 18D25, 18.D29, 18.D30, 18.F1, 18.H1-18.H2. Chapter 18 was chapter
17 in the 2nd edition. Most problems from that edition have the same problem number, but the
chapter number is now 18 (e.g., problem 17.D6 is now 18.D6).

18.A1. 1c; 2 b; 3a
18.A.2. 1c; 2a; 3b
18.A.3. New problem in 3rd edition. One barrier is lack of knowledge. Most chemical engineers are not
trained in use of adsorption, chromatography, and ion exchange. Thus, they do not think of these
processes as a potential solution. A second barrier is the simulation tools are not as developed and widely
available as the simulation tools for equilibrium staged separations such as distillation.
18.A4. Regeneration is too difficult.
18.A5. In the SMB the mass transfer zone between the two solutes stays inside the cascade. In a
chromatograph the MTZ exits the column and must either be completely separated,
which requires a significant amount of desorbent, or recycled appropriately.
18.A7. d
18.A.8. New problem in 3rd edition. The LUB approach assumes constant pattern behavior. Linear
systems do not have constant pattern behavior.

18.A9. d
18.A10. e

18.B.4. New problem in 3rd edition. There are obviously many possibilities. One is to develop sorption
processes that use an energy separation processes (e.g., pressure or temperature) to produce purge or
desorbent from the feed so that a separate purge or desorbent does not have to be added.

18.C1. T e 1 e P1 1 e f cry P2

Vavailable e 1 e P4 1 f cry 1 e f cry P2 K di Vcol.

P 1 f cry clay 1 P1 f cry cry 1 P2 P1 1 f cry f f cry f P2

B 1 e P e f (same as 18-3b)
rd
18.C4. New problem in 3 edition.

Amount in mobile phase = e (Vol. Col. Segment) c e Δz a c Δx cT


Amount in pores = 0 (no pores)
Amount exchanged Δz A c cRT Δy K DE
No 1 e term because c RT is equivalent/L
ε e Δz A c Δx c T v int er
Obtain, u ion
ε e Δz A c c T Δx Δz A c c RT Δy K DE
v int er
Simplify to, u ion (18-44)
c RT y
1 K DE
ecT x

445
CA 1 z uAt
18.C7. 1 erf 12
C AF 2 4E eff t u A
v inter
Sketch of break through:

erf (a) .9 a 1.164

95%
t final

5% tw
t st
t1
erf (a) 0.90 a 1.164

L u A t st L u A t final
At 5% point, 1.164 12
and at 95%, 1.164 12
t st u A uA
4E 4E t final
v v
12
12 uA
Or 2.328 E t st L u A t st
v
12 2 12 u 1A2
let x1 t , uAx
st 1 2.328 E x1 L 0.
v1 2

E 1 2 u 1A2 2 E u At
2.328 2.328 4Lu A
v1 2 v
x1
2u A
E 1 2 u 1A2 2.328 2 E u A
2.328 4L u A
12 v1 2 v
Let x2 t , then x 2
fin
2u A

By definition, t MTZ t final t st x 22 x12


Use sign for both (has to be to have positive times).

446
2 2 2
2 2 2.328 E 1 2 u 1A2 2.328 E 1 2 u 1A2 2.328 E uA 4L u A 2.328 E uA
4u A x 2 12
2 12
4L u A
v v v v

2 2 2
2 2 2.328E 1 2 u 1A2 2.328E 1 2 u 1A2 2.328 E uA 2.328 E uA
4u A x 1 12
2 12
4L u A 4L u A
v v v v

2
2.328E 1 2 u 1A2 2.328 E uA
4 4L u A
2 2 v1 2 v
t MTZ x 2 x 1
4u 2A
2
2.328 E
If 4L very reasonable since E is usually small ,
v
12
2 2.328 E
t MTZ L
v1 2 u A

18C9. New problem in 3rd edition. In – Out = Accumulation


For Figure 18-7B, In t vinter A c CT,after x i,after
Out t vinter A cCT,before x i,before
Accumulation LA c yi,after yi,before CRT LAc x i,after CT,after x i,before CT,before
Note that C RT is constant. After dividing both sides by t A c , mass balance is
v int er x i ,afterC T ,after x i ,beforeC T ,before

L L
C RT y i ,after y i ,before
x i ,afterC T ,after x i ,beforeC T ,before
t t
For Figure 18-7B with a total ion wave,
L
u total ion v int er
t
The first and third terms in the mass balance cancel each other. Thus,
L
C RT y i,after y i,before 0
t
Which requires, yi,after yi,before

18.C10.

447
1
A
v1 v2 v A ,prod u A ,i C A ,i L i u A1 M 1A u port
2
F v2 v3 vF u BL C B,i Vi u B2 M 2 B u port
3 v3 v4 v B,prod u A3 M 3A u port
B v4 v1 vD u B4 M 4 B u port
4

CA
CB v2 M 2B u port CA v2 M 2B u port (1)
CB
C A v3 M 3A u port CA v 2 vF M 3A u port (2)
CA M 2B
Subtract eq. (2) from (1), CA vF M 3A u port
CB
vF
u port
M 2B M 3A
CB CA
Then vD v4 v1 where v 4 u B4 CB M 4B u port CB
v1 u A1 CA M1A u port CA

M 4B u port M 1A u port M 4B M 1A vf
Thus vD
CB CA CB CA M 2B M 3A
CB CA
D vD M 4B M 1A M 4B M 3A
F vF CB CA CB CA
D 1 1 1 1
If all Mi 1.0, 1.0
F CB CA CB CA

pA 1 1
18.D1. Rearrange: pA . Plot p A q A vs. p A
qA q MAX q MAX K A

448
296 K 480 K
p/q p p/q p
135.863 275.788 1786.943 637.7598
278.679 1137.645 1709.129 1296.2036
478.666 2413.145 1974.657 2378.6716
696.073 3757.6116 2309.538 3709.3486
939.619 5239.9722 2778.150 5329.6030
1116.143 6274.1772 3011.134 6246.5981
1189.735 6687.8589 3122.979 6687.8589

At 296 K At 480 K
1 1
Intercept 80 Intercept 1380
q max K A q max K A
1 1
Slope 0.163636 , q max 6.1125 Slope 0.260606 , q max 3.8372
q max q max
1 q max 0.163636 1 q max 0.260606
KA 0.00204545 KA 0.00018884
1 q max K A 80 1 q max K A 1380

449
L soln 1 kg L
18.D2. a = 22 liter soln/kg ads = 22 0.022
kg ads 1000 g g ads
L 1 mol L
b = 375 liter soln/g mole anthracene = 375 2.104
mol 178.22 g g ads
q max K A,c C A L
q , thus, K A,C b 2.104
1 K Ac C A g anth
a 0.022 g anth
q max 0.10456
K A,C 2.104 g ads.

18.D.3. New problem in 3rd edition.


v inter
Part a uj , vinter e v super 40
1 e
1 e 1 p
1 p Kd s K ij
e e

40 40
uj , u AN 5.671 cm min
1 0.69 1782K j 1 0.69 1782 0.00301

Time AN = L/uAN = 25/5.671 = 4.408 min


40
b. u DN 5.474, time L u S,DN 4.576
1 0.69 1782 0.00316
c. u u AN u DN 2 5.567
4Ru
From (18-83) N1 2 104.33 N 10885
u S,AN u S,AN
HETP L N 0.002297 cm
2 2
From (18-81), width at half height 5.54 peak max N
d. To find width in time units, peak max is in time units = retention
time L u S,AN 4.40864 min , width 0.09946 min

t 0.425 width 1/2 height 0.042271 min

v sup er 10.0 cm
18.D4. v int er 23.26
e 0.43 min
v int er
a) u s (18-15c)
1 e p Kd 1 e 1 p s
1 Kx T
e e f

23.26
us 0 0.6027 cm min
0.57 .48 1.0 0.57 0.52 2100
1 17.46
0.43 0.43 684
t br 200 cm 0.6027 cm min 331.8 min

450
b) Assume wall heat capacity is small:
v int er
u th
1 e 1 e 1 p s C ps
1 p
e e F C pf
23.26 cm
u th 5.911
.57 .52 2100 2000 min
1.636
.43 684 1841
t th,br 200 cm 5.911 cm min 33.84 min

23.26
c) K x 1.23 g g @80 C , u s 80 5.4868 cm min
1.636 2.603
t br 200 cm 5.4868 36.45 min , see figure.
cm 0.0011 g tol
M.B.in 10.0 A c cm 2 331.8 min 0.684 g cm 3
min g fluid
out 10.0 A c 0.684 33.84 0.0011 36.45 33.84 C conc
297.96 .0011
Simplifying: 331.8 33.84 .0011 2.611 Cconc → C conc 0.1255 wt frac.
2.611
1 1
1 1
C 80 us 0 u th
Alternate: Eq. (18-24) 0.6027 5.911 113.92
C 0 1 1 1 1
u s 80 u th 5.4868 5.911

C 80 0.0011 113.92 0.1253 wt frac . A very considerable amount of concentration occurs.

451
80º, C = 0

z
usol (80ºC)
uth
0.0011
C=0
t
0.0011 Cconc
0.0011 331.8
min

33.84

36.45 min

0.1255
Cout

0.0011
0
33.84 36.45 time, min

18.D5. vsuper 20 cm min vint er vsuper e 20 0.4 50 cm min


For step input w. unfavorable isotherm, get a diffuse wave.
v int er
Langmuir formula: u s
1 e 1 e 1 p a
1 Kd p s 2
e e 1 bc
But now b 0.46 0
50 50
us
.6 .6 .46 1.124 kg 1.2 0.93067
1 1.01 .54 2
1.81 2
.4 .4 liter 1 0.46 c 1 0.46 c

c,g/l us, cm/min tout = L/us, min


0 18.2437 2.741 min
0.25 16.676 2.998
0.50 14.794 3.3797
0.75 12.565 3.979
1.00 9.997 5.002
1.25 7.1813 6.9625
1.50 4.3499 11.4944

452
f C p f v int er
18.D6. a) u th
1 e
1 e 1 p W
1 p f C pf C ps s C pw
e e eAc

If wall effects are negligible,


0.684 2240 30
u th 12.61 cm min
.57 .57
1 .5 .684 2240 .5 920 1.80
.43 .43

b) t thermal,br 50 cm 12.61 cm min 3.965 min

v int er 30
u s 300K 3.0964 cm min
1 1 1 .57 .57
e
Kd
e p
K xy 1 .5 1.0 .5 12.109
p .43 .43
e e

u s 350K 6.5298 K xy 4.423 in same eqn.


c) t br 300K 50 3.0964 min . Exits at c F 0.010 .

453
At t = 20, start hot, t br,hot 50 12.61 20 23.965 min
Feed is concentrated.

C 350 1 1 1 1 1 1 1 1
3.2989
C 300 u s 300 u th u s 350 u th 3.0964 12.61 6.5298 12.61

C 350 0.010 3.2989 0.032989 g L . This continues until breakthrough at


t 20 L u s 350 K 20 50 6.5298 27.6572 minutes

0.032989
18.D6.

g/L

0.010

t 16.1478 23.965 27.6572

18.D7. vint er vsuper e 15 0.434 34.56 cm min


v int er
a) At 4ºC: u s (A)
1 e
1 e 1 p
1 p Kd s K 4
e e

34.56
us 4 C 0.3715 cm min
0.566 0.57 1.0 0.566 0.43
1 1820 0.08943
0.434 0.434
tr L us 60 cm 0.3715 cm min 161.49 min . Concentration out is zero from t = 0 to t =
161.49 minutes. Then exits at C F 0.01 .

454
v int er
b) u th
1 e p
1 e 1 p C ps s WC pw
1.
e e C pf f e A c C pf f

34.56
u th 17.293 cm min ,
0.566 0.43 0.25 1820
1.743
0.434 1.00 1000
t br,th L u th 60.0 17.293 3.4696 min +1200

Eq. A but with K(60º)


34.56
u s 60 C 0.720258 cm min
0.566 0.43
1.743 1820 0.045305
0.434
t br,conc 60 L u s 60 60.0 0.720258 83.3035 min +1200

C=0

60º
60
60º

z 0º 60º
C=0
uth

Elution time: 0
CF C high 83.3
3.4696 C=0

1 1 1 1
c high 60 cF
us 4 u th u s 60 u th

chigh 0.01 2.6918 0.05783 1.38839 0.05783 0.019796 kmol m3

18.D.8. New problem in 3rd edition. Example 18-3: vinter,F 18.60 and yinter,purge 25.58 cm min .
25.58
u s vinter,F, 0 C 0.3799 cm min , u s v inter,purge,0 C 0.3799 0.5225 cm min
18.60
u th vinter,purge, 6.466 cm min , u s vinter,purge,80 C 4.343 cm min
18.60
u s v inter,F, 80 C 4.343 3.158 cm min
25.58
If t purge hot purge time and t F is cold feed time, with
t purge t hot wave breakthrough vinter,purge 18.56 min (from Example 18-3) then breakthrough
equation is u s v Inter,F, 0 C t F u s v Inter,purge,0 C t thermal,breakthro
ugh 120 cm

455
120 0.5225 18.56
tF 290.35 min
0.3799
The next feed input at 290.35 + 18.56 = 308.91 min. This starts a cold thermal wave at
v Inter,F , u th v Inter,F 4.701 cm min which breaks through in another 25.53 min for total time
to cold breakthrough of 308.91 + 25.53 = 334.44 min.
The solute is hot, first at v Inter,purge u s 80 , v Inter,purge 4.343 cm min and then
18.60
u s 80 ,v inter,F 4.343 3.158 cm min after 18.56 minutes. Next solute step is
25.58
u s vinter,purge, 80 18.56 u s,F 80 , vinter,F t 120
t 120 4.343 18.56 3.158 12.47 min
Exit Time Solute tF t purge t 290.35 18.56 12.47 321.38 min.
Since Exit Time Solute 334.44 breakthrough cold wave, the solute is at 80°C the
entire time.

Solute exits from 290.35+18.56=308.91 min to 321.38 minutes = 12.47 minutes & it exits at
superficial velocity of 8.0 cm min .
Mass Balance
All solute in = Solute out
t F vsuper A c c IN t out vsuper A c c out,AVG
tF 290.35
c out ,AVG c IN 0.0009 wt frac 0.02096 wt frac.
t out 12.47
This is same as peak concentration in Example 18-3, but greater than x out,avg 0.00748. To
have same concentrations need to recycle the material exiting at feed concentration in counter-
flow system. NOTE: Counter flow system has advantages of not contaminating the product end
of the column and typically has less spreading of the zone.

18.D.9. New problem in 3rd edition. a. e vinter vSuper 0.05 m s. vinter 0.5 0.43 0.1163 m s
e v inter
u s,feed,M 0.01712 m s
e 1 e p Kd 1 e 1 p s RTK A,p
from Eq. (18-27) is same as Example 18-4, M 0.2128
M
Pressurization Step
Feed end (for pressurization) 0.75m (Measured from closed product end)
0.2128
4.0 atm
z after 0.75 m 0.5584 m
1.0 atm
which is 0.75 0.5584 0.1916 m from feed and
0.2128 1
4.0
y M after 0.003 0.001007
1.0
Feed Step
u sfeed 0.01712 m s 7 sec 0.11984 m 0.1916 m for pressurization step
= 0.3114 m. Does not breakthrough in first cycle. From 0 to 0.11984 m, concentration is y F .
456
Blowdown.
Measuring from closed top, z before 0.75 0.3114 0.4386 m
0.2128
1.0
z after 0.4386
0.5890 m
4.0
The far end of the feed wave does not get removed from the bed. The close wave
0.11984 or 0.75 0.11984 0.6302 m from closed end has
0.2128
1.0
z after 0.6302 0.8463 m, so it all exits. The mole fraction of this portion is
4.0
0.2128 1.0
1.0
y after y feed 0.003 2.9781 0.00893
4.0
Part of the feed that was pressurized also exits during blowdown.
0.2128
1.0
This z after 0.75 z before
1.3431 z before z before 0.5584 m from closed
4.0
(product) end. This is 0.75 0.5584 0.196 m from feed end. This gas entered at an unknown
pressure between p L 1.0 and p H 4.0. Can calculate this pressure from Eq. (18-28c)
1 1 0.2128
z after 0.5584
p before p after 4.0 1.00003 atm
z before 0.75
0.2128 1
4.0
y after,press 0.003 0.001007 After Pressurization Step.
1.00003
This gas is depressurized to 1.0 & exits column
0.2128 1
1
y after,BD 0.001007 0.00300 or essentially the feed composition.
4
Exit from Col

0.008933
y
0
.0030

time

p after
Part b. Want z after blowdown 0.75, then z before z after
p before
0.2128
1.0
z before 0.75 0.5584
4.0
from closed end, which is 0.75 0.5584 0.1916 m from feed end. Want the feed to end at this
point. During constant pressure feed step, feed travels u s,feed t F 0.01712 t F . Then for
pressurization step z after (from feed end) 0.1916 0.1712 t F . From closed end this is

457
p after
0.5584 0.1712 t F z after z before
p before
0.2128
4
0.5584 0.1712 t F 0.75 0.5584 or t F 0.
1
Thus, need a purge step if have feed step at constant pressure for complete cleanout.
1
0.4 0.2128
18.D10. a) pt.10 : z after 0.4, A 0.2128, p before 3.0 1.05128 atm
0.5
0.2128 1
3.0
y after 0.002 0.000876
1.015128
0.4 m
Travels, 25.126s 1.0s for blow-down 26.126s
0.01592 m s
b) Start with Arbitrary point at t = 1 sec (end repress) z after 0.48 (.02 from feed end)
1
0.2128 1
.48 .2128 3.0
p before 3.0 2.4763 atm , y after 0.002 0.00172
.5 2.4763
Dist. Traveled @ t = 30s: 0.02 + 0.01592 × 29s = 0.48168 m
For blow-down: distance from closed end = 0.01832 cm
.2128 .2128 1.0
0.5 0.5
z after 0.01832 0.026824 , y after ,BD 0.00172 0.007048
3.0 3.0
Purge: u M,purge 0.01751 m s . Exits bottom column during purge (point 11)
0.5-0.026824
(distance traveled)/upurge 31 s + 58.023s
0.01751

18.D11. If repressurize with product, bed remains clean.


Feed step is same as to point 3 (at 0.462 m from feed end) on Figure 18-13.
Blowdown then pt. 4 (0.056 m from top) and purge exits at pt. 8 (56.36s)
Product gas is cleaner (y = 0), but there is lower productivity – less feed per cycle.
See Figure.

458
BD
3
4
y=0 y=0

y = 0.0082

18.D12. a) The clean bed receiving feed has a shockwave for Langmuir isotherm.
2
v sup er 320 cm 3 min A c , where A c r2 4 cm 50.2654 cm 2
vsup er 6.366 cm min , vinter vsup er e 6.366 10.434 14.669 cm min
v int er
u sh
1 e p Kd 1 e 1 p q
1 s
e e c
q q after q before
where c before 0, q before 0
c c after c before
0.190 50
c after 50 mol m 3 , q after 1.1446 mol kg
1 0.146 50

14.669
u sh 0.5843 cm min
0.566 0.57 1.0 0.566 0.43 1.1446
1 1820
0.434 0.434 50
t br L u sh 50 cm 0.5843 cm min 85.579 min
Outlet concentration is zero until t br then becomes 50.
Concentrated solution eluted by dilute soln. Gives diffuse wave for Langmuir isotherm.
v
u s u diffuse
1 e 1 e 1 p a
1 p K d p 2
e e 1 bc
14.669 14.669
us
0.566 0.57 1.0 0.566 0.43 1820 0.19 193.92
1 2
1.74336 2
0.434 0.434 1 0.146 c 1 0.146c
Create Table.

459
c 50, u s 3.218, t L us 50 3.218 15.537 min
c 0, u s 0.07497 cm min , t L us 666.93 min
c 40, u s 2.491, t 20.071 min
c 30, u s 1.737, t 28.779 min
c 15, u s 0.7052, t 70.898
c 5, u s 0.2205, t 226.80

18.D13. A de xtran, B fructose


CA v1 u A1 M1u port (1)
CB v 2 u B2 M 2 u port (2)
C A v3 u A3 M 3 u port (3)
CB v 4 u B4 M 4 u port (4)
1000 cm 3 min
Vol Feed v F,sup er
v2 v3 v F , v F,super , vF 1.9894 cm 3 min
D2 40 2
e

4 4
CB vF
Solve eqs. (2) and (3) simultaneously, u port
CB
M2 M3
CA

1 1 1 1
CA 0.7435 , C B 0.4914
1 e .6 1 e .6
1 KA 1 0.23 1 KB 1 .69
e .4 e .4
0.4914 1.9894 cm min
u port 3.03175 cm min L t sw
0.4914
0.99 1.01
0.7435
L 60
t sw 19.791 min
u port 3.03175

M1 0.97 D2
v1 u port 3.03175 3.955 cm min : V1,sup er v1 e 1988.176 cm 3 min
CA 0.7435 4

M2 0.99
v2 u port 3.03175 6.1079 cm min ; V2,sup er 3070.15 cm 3 min
CB 0.4914
M 3 u port 1.01 3.03175
v3 4.1184; V3,sup er 2070.14 cm 3 min
CA 0.7435
M 4 u port 1.03 3.03175
v4 6.3547; V4,sup er 3194.19 cm 3 min
CB 0.4914
Recycle flow V1,sup er 1988.176 cm3 min
460
D
D V4,sup er V1,sup er 3194.19 1988.176 1206.0 cm3 min , 1.2060
F
M4 M1
CB
VD D V4 V1 M4 M 1 u port CB CA
Check:
VF F VF CB CA vF CB
M2 M3
CA
1.03 .97
.4914
D .4914 .7435
1.2060 , OK
F .4914
.99 1.01
.7435
Raffinate Product V2 V1 3070.15 1988.18 1081.97 cm 3 min
Extract Product V4 V3 3194.19 2070.14 1124.05 cm3 min
18.D.14. New problem in 3rd edition. From Eq. (18-40c) K K H KK Li KH Li

Anderson’s data: KK H 2.9 1.3 2.2308


DeChow’s data: K K H 2.63 1.26 2.0873
For the shockwave Eq. (18-46) holds for K+
Since resin is initially in H+ form, x K,before CK,before CT 0 and y K,before CR ,K CRT 0.
x K,after CK,after CT 1.0
y K,after CR ,K,after CRT 1.0
v inter
a) u sh ,K
1 C RT y K ,after y K ,before
1 K DE ,K
e CT x K ,after x K ,before
25 0.42 L
u sh ,K 1.115 cm min, t sh
44.84 min
1 2.2 1 0 u sh
1 1.0
0.42 0.1 1 0
Same for both sets of data since K K H does not enter into equation when initial and feed
contain only one ion.
b) C t 1.0, u Sh,K 9.542 cm min , t sh 5.24 min
c) Ct 1.0, x K,before 0.2, x K,after 0.85. y K values depend on equilibrium parameter.
K KH x K
yK
1 K KH 1 x K
2.2308 0.2
Anderson’s Data: y K ,before 0.3580
1 2.2308 1 0.2
2.2308 0.85
y K ,after 0.9267
1 2.2308 1 0.85

461
25 0.42
u sh 10.662 , t K L u sh 4.69 min
1 2.2 0.9267 0.3580
1 1.0
0.42 1.0 0.85 0.2

DeChow’s data:
2.0873 0.2
y K ,before 0.3148
1 2.0873 1 0.2
2.0873 0.85
y K ,after 0.9220
1 2.0873 1 0.85
25 0.42
u sh 10.100 , t K L u sh 4.95 min
1 2.2 0.9220 0.3148
1 1.0
0.42 1.0 0.85 0.2
4.69 4.95
% difference 100 5.55%
4.69
d) There is a difference if either initial or feed contains both ions. System with higher
K K H had higher shock velocity.
v
18.D15. New problem in 3rd edition. Part a. u sh ,i
1 c RT y i,after y i,before
1 K DE
e cT x i,after x i,before
x i,before y i,before 0
For both Na & K ,
x i,after y i,after 1.0
25 0.42
Thus same u sh , u sh 5.186 cm min
1 2.2
1
0.42 0.5
t center L u sh 50 5.186 9.64 min
v
u i,sh
Part b. 1 c RT y i,after y i,before
1 K DE
e cT x i,after x i,before

For both Na+ and K+: xbefore = 0.4 and xafter = 0.9. For Na+
K Na H xNa (2.0 / 1.3)(0.4)
y Na ,before 0.506
1 ( K Na H 1) xNa 1 [(2.0 / 1.3) 1](0.4)

K Na H x Na (2.0 / 1.3)(0.9)
y Na , after 0.933
1 ( K Na H 1) xNa 1 [(2.0 / 1.3) 1](0.9)

462
v (25 / 0.42)
u sh ,Na 5.98
1 c RT y Na ,after y Na ,before 1(2.2)(1.0) 0.933 0.506
1 K DE 1
(0.42)(0.5) 0.9 0.4
e cT x Na ,after x Na ,before
t Na L / ush , Na 50 / 5.98 8.36 min .

For K+ we obtain,

KK H xK (2.9 / 1.3)(0.4)
y K ,before 0.598
1 (KK H 1) xK 1 [(2.9 / 1.3) 1](0.4)

KK H xK (2.9 / 1.3)(0.9)
y K , after 0.953
1 (KK H 1) xK 1 [(2.9 / 1.3) 1](0.9)
v (25 / 0.42)
u sh ,K 7.054
1 c RT y K ,after y K ,before 1(2.2)(1.0) 0.953 0.598
1 K DE 1
(0.42)(0.5) 0.9 0.4
e cT x K ,after x K ,before

tK L / ush , K 50 / 7.054 7.09 min .

v
Part c. u
1 c RT dy
1 K DE
e cT dx
dy Na K Na H
K Na Li KH Li
2 2
dx Na 1 K Na 1 x Na 1 K Na KH 1 x Na
H Li Li

dy Na 2.0 1.3 25 0.42


2
0.955, u Na 5.409
dx Na 1 2.0 1.3 1 .5 1 2.2
1 0.955
.42 0.5
t Na L u shNA 9.244 min
dy K KK H
KK Li KH Li
2 2
dx K 1 KK 1 xK 1 KK KH 1 xK
H Li Li

dy 2.9 1.3 25 0.42


2
0.855, u K 5.979
dx 1 2.9 1.3 1 .5 1 2.2
1 0.855
.42 0.5
Part d. x Na 0 xK 0
dy Na
K Na H 2.0 1.3 1.538, u Na 3.477, t Na 14.38 min
dx Na x Na 0

dy K
KK H 2.9 1.3 2.231, uK 2.442, tK 20.47 min
dx K xK 0

Part e. x Na .9 xK .9
463
dy Na 1.538
2
0.698, u 7.159, t Na 6.984 min
dx Na 1 0.538 .9
Na

dy K 2.231
2
0.502, uK 9.5075, tK 5.259 min
dx K 1 1.231 .9
Part f. The velocities and hence the derivatives are equal. Thus,

dy Na K Na Li KH Li dy K KK Li KH Li
2 2
dx Na 1 K Na KH 1 x Na dx K 1 KK KH 1 xK
Li Li Li Li

With xNa = xK. The result from a spreadsheet is x = 0.35056

18.D16. vint er vsup er e 15 0.40 37.5 cm s


MW p 28.9 g mol 50 kPa 1.0 kg
f 3
0.5832 kg m 3
RT m kPa 1000 g
0.008314
298 K
mol K
q kg toluene kg carbon
is in . Then, shockwave velocity is
c kg toluene kg air
v int er
u sh
1 e p Kd 1 e 1 p s q
1
e e f y
37.5 cm s
u sh
0.6 0.65 1.0 0.6 0.35 1500 kg m 3 q 2 q1
1
0.4 0.4 0.5832 kg m 3 y 2 y1
37.5 cm s
u sh
q2 q1
1.975 1350.308
y2 y1
2000 .0005
For u sh ,1 : y1 0, q1 0, y 2 0.0005, q 2 0.47619
1 2200 0.0005
37.5
u sh ,1 0.00002916 cm s 0.104976 cm h
0.47619
1.975 1350.308
0.0005
u sh 2 : y1 0.0005, q1 0.47619
2000 0.0015
y2 0.0015, q 2 0.69767
1 2200 0.0015
37.5
u sh 2 0.00012539 cm s 0.451393 cm h
0.69767 0.47619
1.975 1350.308
0.0015 0.0005
At L min : u sh1t L min u sh 2 t 10 h where t is in hours.
464
u sh 2 10 0.451393 10
Solve for t 13.03 h
u sh 2 u sh1
0.451393 0.104976
cm
L min u sh1 t 0.104976 13.03 h 1.368 cm
h
Thus, for any column of partial length we will see a single shockwave exit the column.
v sup er 21.0
18.D17. v int er 52.5 cm s
e 0.4
pV n RT
MW n MW p 28.9 50
v 0.5383 kg m 3
V RT 1000 g kg 0.008314 298
Since Cinit C F , Get 2 diffuse waves
v int er q 2000
us where 2
1 1 e 1 p q y 1 2200y
e s
1 p Kd
e e f y
52.5 52.5 cm s
us
.6 0.6 0.35 1500 q 1.975+1350.23 q
1 0.65 1.0
0.4 0.4 0.583 y y
y q u s cm s t L u s 25 u s
y
0.0010 195.31 0.0001991 125,581s = 34.8835 h
0.00075 284.799 0.0001365 183,117.6s = 50.866 h
0.00050 453.515 0.00008573 291,596.6s = 80.999 h
2nd wave (0.00050) - add 20 hours 100.999 h
0.00025 832.466 0.00004671 535,250.5 = 148.681 + 20 = 168.681 h
0.00 2000 0.00001914 1285937.96 = 357.205 + 20 = 377.205 h

us y 0.001 0.00075 0.00025

z
us y 0

us y 0.005
0 t
2
us y 0.0005

465
80.999
34.88
0.001 50.866
0
c 0.00075
0.0005
·
100.99
·
168.88

0.00025

t 377.2

18.D.18.
. New problem in 3rd edition.
Part a. u S,G 11.12 S cm min is calculated in Example 18-9.

20
u S,F 8.416 , u uS,G u S,F 2 9.771
0.61 1.0 0.88
1 0
0.39
2
From Eq. (18-93), N 4Ru u S,G u S,F 229.465
1 2N E eff
From Eq. (18-78a) N L v E eff L
2 v

2 229.465 5.0 cm 2 min


L 114.73 cm
20 cm min
Part b. tG L u S,G 114.73 11.25 10.20 min
tF L u S,F 114.73 8.416 13.63 min
1/ 2 1/ 2
L 1 1
Part c. Eq. (18-80a), t , t ,G 10.20 min 0.673 min
uS N 229.465
1/ 2
1
t ,F 13.63 min 0.900 min
229.465

2.93 x Ag
18.D19. K AgK K Ag Li KK Li 8.5 2.9 2.93 , y Ag
1 1.93 x Ag
a) Ion wave: u total con vint er vsuper e 3.0 0.4 7.5 cm min
Breakthrough of ion wave, 50 cm 7.5 cm min 6.667 min

466
v int er
b) Shock wave, u sh
1 C RT y Ag after y Ag before
1 KE
e CT x Ag after x Ag before
before: x Ag y Ag 0 . after: x Ag y Ag 1.0 .
7.5 cm min L 50 cm
u sh 1.4516 cm min , t sh 34.44 min
1 2.0 1.0 1.0 u sh 1.4516 cm min
1
0.4 1.2 1.0
v int er v int er
c) Diffuse wave: u s
1 C RT dy 1 C RT K Ag -K
1 KE 1 KE
e CT dx 2
e CT 1 K Ag 1 x Ag
K

7.5 cm min 7.5


u s,Ag
1 2.0 2.93 12.208
1 1.0 2
1 2
0.4 1.2 1 1.93 x 1 1.93 x Ag
Ag

7.5 cm L
x Ag 1.0, u s 3.0965 , t out 16.147 min
12.208 min us
1 2
2.93
7.5 50
x Ag 0.5, u s 1.8021 , t out 27.745 min
12.208 1.8021
1
3.86
7.5 L 50
x Ag 0, u s 0.5678 cm min , t out 88.0555 min
13.208 us 0.5678
From spreadsheet:
xAg u_dif,Ag t_dif,Ag
1 3.097163211 16.14380534
0.9 2.852615804 17.5277722
0.8 2.598940969 19.23860549
0.7 2.337763116 21.38796684
0.6 2.071284133 24.13961427
0.5 1.802351071 27.74154314
0.4 1.53450084 32.58388572
0.3 1.27196632 39.3091764
0.2 1.019637556 49.0370324
0.1 0.782936124 63.86217015
0 0.567638906 88.08416667

K Ca 5.2
18.D20. Was 18D23 in 2nd edition. Table 18-5, K CaK Li
2 2
0.6183
K K Li 2.9
a.) c RT 2.0 eq L , c T 0.02 eq L , x Ca 0.8 at t 0.
K CaK C RT 0.6183 2.0
61.83 shockwave .
CT 0.02
Column: L 75 cm, vsuper 20 cm min , vinter 20 .4 50 cm min
467
p 0, e 0.4, K E 1.0
v int er
Feed: u sh , before: x Ca 0, y Ca 0 , after: x Ca 0.8
C RT K E y
1
e CT x
from Eq. (18-43) y Ca 0.971965
50 75
u sh 0.16407 cm min , t br 457.1 min
1 2.0 0.971965 u sh
1 1.0
0.4 .02 0.8
75
b.) Regenerate: at 500 min → Ion wave at vint er 50 cm min takes 1.5 min.
50
K Ca C RT
New 1.2366 y Ca unchanged. Use Eq. (18-43) with new value K Ca C RT CT .
CT
x Ca 0.9689677 . Obtain diffuse wave.
3
v imter dy Ca 1.2336 1 y Ca 1 x Ca
u diffuse where
C RT K E dy Ca dx Ca
3
1 x Ca 1 y Ca
1
C T e dx Ca
(Wankat, 1990, Eq. (9-25b)).
dy Ca 50
At x Ca 0, 1.2366, u Ca 6.96088 cm min
dx Ca 1.0 2.0
1 1.2366
0.4 1.0
75 cm
t out 10.7745 min (slow wave)
6.96088
At x Ca 0.96897, and y Ca 0.971965
3
dy Ca 1.2336 0.028035 1.96897
3
0.908386
dx Ca 0.03103 1.971965
50 75
u Ca 9.022 cm min , t out 8.312898 min (fast wave)
2.0 1.0 9.022
1 0.908386
1.0 0.4
At x Ca 0.5, y Ca 0.534927
3
dy Ca 1.2336 0.465073 1.5
3
0.97013
dx Ca .5 1.534927
50 75
u Ca 8.546 cm min , t out 8.776 min . (in-between)
2.0 1.0 8.546
1 0.97013
1.0 0.4

18.D.21. New problem in 3rd edition.


vF 1
u port , C Tol 0.132234
M2 M3 1 1 e 1 p
e
Cy C Tol 1 p Kd K Tol 300
e e

468
K Tol 300 K 0.0061 e 2175.2696 300 8.5972 , K xy 300 K 0.0105 e 2115.1052 300 12.1092
1
C xy 0.10017
1.63627 0.68930 12.1092
vF
u port 0.6479 v F 0.6479 cm min , L u port t SW 64.79 cm
.95 1.05
.10007 .132234
0.95 .6479
v2 M 2 u port C xy 6.1447 cm min , v3 v 2 v F 5.14476
0.10017
0.6479
v1 M1 u port C Tol .95 4.6547
0.132234
1.05 0.6479
v4 M 4 u port C xy 6.7914
0.10017
v Tol prod v2 v1 6.1447 4.6547 1.4900
v xy,prod v4 v3 6.7914 5.14476 1.6466
vD v4 v1 6.7914 4.6547 2.1367, D F 2.1367
Check: vOut v tol prod v xy,prod 3.1366 , v total in vF vD 3.1367 OK.
dp 2 1 dp 1
18.D22. E eff ED u s2 K , where
6 k M,c 6 k m,c k m,c a p

20 cm 3 min v sup er
v sup er 2
6.366 cm min , v 15.915 cm min
cm
v 15.915
us 7.821 cm min
1 e 2.035
1 0 K
e
2
cm 2
7.821 0.6 0.69
min
E eff 0.15 1
8.063 cm 2 min
5.52 min 0.4

C 1 z ust
Eq. (17-69) X 1 erf 12
for step up
CF 2 4 E eff u s t
v int er
200 7.821 t
Argument of erf, a 12
15.849 t

469
1 L u s t 8u s
Step down: X L, t 8 1 erf 12
2 4 E eff u s t 8
v int er
323.04 7.821 t
Argument of erf , a 12
15.849 t 126.792
Total Solution X X
L 123.03
If t 25.573 min a 0, a 12
7.358
us 279.6
1 1
X and X 1 1.000 0 , c cF 0.50 or c 25.0 (for smaller t, can ignore X )
2 2
L 8u s 63.96
If t 33.575, a 0, X 1 2, a 12
2.773, erf .998
us 532.13
1 c 1
X 1.998 0.999 For higher t, X = 1.0, 0.999 0.499 c 24.975 .
2 cF 2
31.2765
Peak at 25.575 4 29.575, a 12
1.44473, erf a 0.95847
468.663
1
X 1.95847 0.979236
2
31.2835 1
a 12
1.69189, erf a 0.983186 , X 1 .983186 0.0084
341.89 2
c
0.979235 0.0084 0.970835 , c 50 0.970835 48.54
cF
See also Problem 18.G1.

18.D23. Was 18D24 in 2nd edition. Cout Cinitial Cinitial CF1 X z, t 17.5 CF1X z, t 28

18.D.24. New problem in 3rd edition.


vF 1
u port , C Tol (300K ) 0.132234
M2 M3 1 1 e 1 p
e
C C 1 p Kd K Tol 300
xy Tol
e e

K Tol 300 K 0.0061 e 2175.2696 300 8.5972 , K xy 300 K 0.0105 e 2115.1052 300 12.1092

1
C xy (300K) 0.10017
1.63627 0.68930 12.1092

470
vF
u port 1.4812 v F 1.4812 cm min , L u port t SW 148.12 cm
.90 1.10
.10007 .132234
0.90 1.4812
v2 M 2 u port C xy (300K) 13.3084 cm min , v3 v2 vF 12.3084
0.10017
1.4812
v1 M1 u port C Tol (300K) .90 10.0812
0.132234

vTol prod v2 v1 13.3084 10.0812 3.2272

1.10 1.4812
v4 M 4 u port C xy (300) 16.2655
0.10017
v xy,prod v4 v3 16.2655 12.3084 3.9571
vD v4 v1 16.2655 10.0812 6.1843, D F 6.1843
Check: vOut v tol prod v xy,prod 7.1843 , v total in vF vD 7.1843 OK.

18.D.25. New problem in 3rd edition. Zones 2 & 3 are same as in 18.D.24 since at 300 K
u port 0.6479 v F 0.6479 cm min , L u port t SW 64.79 cm
v2 6.1447 , v3 5.14476
v1 M1 u port CTol 273 K and v 4 M 4 u port C xy 350 K
M1 0.5 and M 4 2.0 (reciprocal values).
K Tol 273K 0.0061 exp 2175.2695 273 17.612
K xy 350K 0.0105 exp 2115.1052 350 4.423
1
C Tol 273K 0.07259
1.63627 0.68930 17.612
1
C xy 350K 0.2135
1.63627 0.68930 4.423
v1 0.5 0.6479 0.07259 4.4627 , v 4 2.0 0.6479 0.2135 6.0707
vTol prod v2 v1 1.6820 , v xy prod v4 v3 0.9260 , v D v4 v1 1.608
D / F 1.608

2
4Ru s v 1.0
18.D26. a) N , R 1.5 , u s A 0.15385
u sA u s,B 1 s
6.5
1 KA

1.0
u sD 0.147059 , u 0.15045
1 5.8

471
2
4 1.5 0.15045
Need N 17689 , L 0.05 N 884.45 cm.
0.0067873
L 884.45
b) t R ,A 5748.88 95.813 min
uA 0.15385
12 12
L 1 1
t ,A 95.813 min 0.7204 min
uA N 17689
2 2
CA t tR t 95.813
exp 2
exp 2
p
C A ,max 2 t 2 0.7204

t,min 90 92 94 95 95.813 96 97
CA CA,max 7.27E-15 8.3E-7 0.0421 0.52898 1.00 0.9669 0.2573

CA
0.33 X A L, t 1.0 0.33 X A L, t 0.4t F
18.D27. CF
1 .55 X A L, t .8t F 0.55 0 X A L, t tF

25.0 cm
18.D28. a) u p L t center 0.706 cm min , L MTZ,lab u pt MTZ 0.706 2.8 1.9774 cm
35.4 min
2
t MTZ, LS d 2p ,LS D eff 1.0
b) Large-Scale system 2
69.44
t MTZ,lab d 2p ,lab D eff 0.12
Independent of velocity t MTZ,LS 69.44 2.8 194.44 min
v super
u p ,LS e
LS 12 4 4
→ u p,Ls 0.706 0.941 cm min
u p ,lab v super 9 3 3

e lab
lab

L MTZ,larg e scale u p t MTZ 0.941 cm min 194.44 min 183.03 cm


For frac. bed use = 0.80 & symmetrical pattern,
0.5 L MTZ 0.5 183.03 t MTZ
L 457.6 cm 4.576 m , t br t center
1 Frac bed use 1 .8 2
457.6 194.44
t center L up 486.27 min , t br 486.27 389.05 min .
0.941 2
This is length of feed time if column is completely regenerated.

K Ca Li 5.2
18D.29. K CaK 2 2
0.6183
K K Li 2.9
a.) c RT 2.5 eq L , cT 0.03 eq L , x Ca 0.7 at t 0.

472
K CaK C RT 0.6183 2.5
51.525 shockwave .
CT 0.03
Column: L 90 cm, vsuper 25 cm min , vinter 25 / .39 64.10 cm min
p 0, e 0.39, K E 1.0
v int er
Feed: u sh , before: x Ca 0, y Ca 0 , after: x Ca 0.7
C RT K E y
1
e CT x
y Ca K Ca K C RT x Ca
from Equilibrium, 2
400.75
(1 y Ca ) CT (1 x Ca ) 2

Solve this for unknown y value. I used a spreadsheet.


yCa 0.95128
64.1 90
u sh 0.22000 cm min , t br 409.10 min
1 2.5 0.95128 u sh
1 1.0
0.39 .03 0.7
90
b.) Regenerate: Ion wave at vint er 35.0 / 0.39 89.74 cm min takes 1.003 min.
89.74
K CaK C RT (0.6183)(2.5)
New 1.4057 y Ca 0.95128 unchanged. Use equilibrium with
CT 1.1
y Ca K Ca K C RT x Ca
old y and with new value K Ca C RT CT 400.75 to
, (1 y Ca ) 2
CT (1 x Ca ) 2
find x Ca 0.94251 . Obtain diffuse wave.
3
v imter dy Ca 1.4057 1 y Ca 1 x Ca
u diffuse where
C RT K E dy Ca dx Ca
3
1 x Ca 1 y Ca
1
C T e dx Ca
(Wankat, 1990, Eq. (9-25b)).
dy Ca 89.74
At x Ca 0, 1.4057, u Ca 9.7631 cm min
dx Ca 1.0 2.5
1 1.4057
0.39 1.1

As an alternative can do numerical calculation of derivative. At x = 0, y = 0. x = 0.001, y = 0.001404 and


y / x (0.001404 0) / (0.001 0) 1.404 , which is reasonably close.

90 cm
t out 9.22 min (slow wave)
9.7631
At x Ca 0.94251, and y Ca 0.95128
3
dy Ca 1.4057 0.04872 1.94251
3
0.85169
dx Ca 0.05749 1.95128

473
89.74 90
u Ca 15.049 cm min , t out 5.981min (fast wave)
2.5 1.0 15.049
1 0.85169
1.1 0.39
From equilibrium, at the arbitrary value x Ca 0.5, y Ca 0.55544
3
dy Ca 1.4057 1 0.55544 1.5
3
0.95282
dx Ca .5 1.55544
89.74 90
u Ca 13.695 cm min , t out 6.572 min .
2.5 1.0 13.695
1 0.95282
1.1 0.39

This is in-between the other two waves.

K CaK C RT (0.6183)(2.5)
c. To not have a diffuse wave must have 1.0
CT CT
This requires CT > 1.546.

18.D30. New Problem in 3rd edition. K K H KK Li KH Li

DeChow’s data: K K H 2.63 1.26 2.0873


a.) This will be a shock wave since K+ is more concentrated in the feed to the column than it
is initially and KK-H > 1.

v inter
u sh ,K
1 C RT y K ,after y K ,before
1 K E ,K
e CT x K ,after x K ,before

Ct 1.0, x K,before 0.2, x K,after 0.85. y K values depend on equilibrium parameter.


K KH x K
yK
1 K KH 1 x K

2.0873 0.2
y K ,before 0.3148
1 2.0873 1 0.2
2.0873 0.85
y K ,after 0.9220
1 2.0873 1 0.85
25 0.42
u sh 10.100 cm/min,
1 2.2 0.9220 0.3148
1 1.0
0.42 1.0 0.85 0.2
tK L u sh 49.5 min
All three times are the same for the shock wave.
474
b.) This will be a diffuse wave since K+ is less concentrated in the feed to the column than it
is initially and KK-H > 1.

v inter 25 / 0.42
u diffuse,K
1 C RT dy K 2.2(1) dy K
1 K E,K 1
e CT dx K (0.42)(1.0) dx K

dy K K KH 2.0873
2
1.543
At xK = 0.15, dxK (1 ( K KH 1) xK ) [1 (1.0873)(0.15)]2

25 / 0.42 59.524 59.524


u diffuse,K 6.554cm / min
2.2(1) dy K dy 1 5.238(1.543)
1 1 5.238 K
(0.42)(1.0) dx K dx K

Thus, at xK = 0.15, tK = L/udiffuse,K = 500/6.554 = 76.29 min. Then at xK = 0.5 we obtain

dy K K KH 2.0873
2
0.876
dxK (1 ( K KH 1) xK ) [1 (1.0873)(0.5)]2
59.524 59.524
u diffuse,K 10.65cm / min
dy 1 5.238(0.876)
1 5.238 K
dx K

Thus, at xK = 0.5, tK = L/udiffuse,K = 500/10.65 = 46.94 min. Then at xK = 0.8 we obtain

dy K K KH
0.5970
dxK (1 ( K KH 1) xK ) 2

59.524 59.524
u diffuse,K 14.42cm / min
dy 1 5.238(0.597)
1 5.238 K
dx K

tK = L/udiffuse,K = 500/14.42 = 34.67 min.

18.D31. New problem in 3rd edition. a. vSuper 10 vint er 10 .4 25, e 0.4, L 30.0

475
2.9
c RT 2.4, cT 1.10, KK Na 1.45
2.0
v int er 25
u sh ,K
1 c RT y K ,after y K ,before 1 2.4 1 0
1 K DE 1 1.0
.4 1.1 1 0
e cT x K ,after x K ,before
u sh,K 3.783 cm min, t center,exp ected 7.75 min . , t center ,measured 7.31 min
7.31 7.75
% error 100 6.00%
7.31
b. u sh,exp t L t center 30 7.31 4.10 cm min .
c. L MTZ u sh t MTZ 4.10 7.57 7.06 2.093 cm
Frac. bed use (symmetric wave) 1 0.5 L MTZ L 0.965
d 2p v Super D eff 16 d 2p 200 D eff
l arg e scale
L MTZ l arg e scale L MTZLab L MTZ,Lab
Lab
d.
d 2p v Super D eff d 2p 100 D eff
Lab Lab

With same beads assume no change in D eff .


L MTZ,larg e scale 16 2 2.093 cm 66.98 cm
frac bed use 1 0.5 L MTZ L 1 0.5 66.98 200 0.833
Breakthrough start time t center 0.5 t MTZ L u sh 0.5 L MTZ u sh
v inter,large scale
u sh u sh ,lab ,exp tl 8.2
v inter,lab scale
Breakthrough start time [200 0.5(66.98)] / 8.20 20.31min

18.F1. New problem in 3rd edition. Constraints: w L 860 ft 2 T max 500 C 932 F
wLh 2580 ft 3 , h=2580/860=3ft., p0 6 atm 88.14 psia 73.14 psig

End
View Weight vessel
Seider etal,
Di ts L 0.8 D i t s s
Eq.16 59

Seider etal. (2004), Eq. (16.61)

476
2
Pd exp 0.60608 0.91615 n p0 0.0015655 n p0 96.66 psig
Wall thickness
Pd D i 96.66 D i
(Eq. 16.60) tp 3.7057E 3 D i
2SE 1.2 Pd 2 13,100 1 1.2 96.66
where S 13.100 psi p. 529 with SA-387B steel, E = 1.0

s 490 lbm ft 3 0.284 lbm in 3 p. 529 (Seider et al, 2004)


0.5h 1.5 3 1
Relate w to h.: cos cos 3 D
r r D
0.5w w w 1 3
90 cos cos 90 cos
r D D D
860
Weight D 3.7057E 3 D 1
0.8D 3.7057 E 3 D 490
D cos 90 cos 3D
1
D in ft. (In Spreadsheet A cos cos
In Spreadsheet, angle is in radius 90 2
D Weight Width L
3.5 33655 1.80 477.0
3.7 31358 2.17 397.1
3.8 30735.9 2.33 368.7
3.9 30325. 2.49 345.1
4 30070.31 2.65 325
4.1 29933 2.7947 307.7
4.2 29889 2.939 292.6
4.3 29918 3.08 279.2
6 35103 5.196 165
8 44837 7.42 115.96
10 56197 9.54 90.15
12 68940 11.62 74.02
14 83144 13.67 62.88
Goal seek L = 60
D = 14.64 ft Weight = 88052.75 Width = 14.333 L = 60 ts 0.054266 ft
From Seider et al, p. 527: Cp Fm C v CPL in mid 2000 (MS = 1103)
2
(Eq. 16.53) horizontal Cv exp 8.717 0.2330 n w 0.04333 n w
0.20294
(Eq. 16.55) CpL 1580 D 2724
From p. 531, Fm 1.2 for low-alloy steel, C v 118,323 → Cp 144, 711 in mid 2000
Installed Cost: Calc C p with Fm 1.0 in 2000 Cp 118323 2724 121047
Bare module factor, FBm 3.05 for horizontal
CBm Cp Fm 1.0, 2000 3.05 1.0 1.0 1.2 1 $393, 400
Absorbent: p. 553 Cp $60 ft 3 , Cp 60 2580 $154,800

477
18.G1. Was 17G1 in 2nd edition. Figures are labeled 17G1.

478
479
480
18.G2. Was 17G2 in 2nd edition.
a.) With QDS with 50 nodes find t center 4.52 min
t MTZ 6.0 3.13 2.87 min

18.G3. Was 17G3 in 2nd edition.

a) D F 1.0. D 141.55 E R F.
Find CA 0.343 and CB 0.219
vF
Eq. (17.31a) u port
M 2B M 3A
CB CA
F 141.55 cm 3 min
vF 2 2
4.5057 cm min
e Dc 4 0.4 10 4
4.5057
u port 2.7295 cm min
1 1
0.219 0.343
t sw L u port 50 2.7295 18.32 min
CA vint er u A1 M1u port 2.7295 cm min
2.7295
v1,int er 7.9577 cm min
0.343
v1,sup er 0.4 v1,int er 3.18308 cm min
2
Recycle Rate 3.18308 10 4 250 cm 3 min
Obtained raffinate = 96.6% and extract = 94.3%.
b) One approach is to keep a symmetric cycle. Then D = 283.1 and
E F
E R 212.325
2
Flow optimizer can be used to give t sw ~ 9.1 and Recycle rate ~ 500. Depending on
values obtain raffinate and extract > 97%.

18.G4. Was 17G4 in 2nd edition. Figure below is labeled 17G4.

18G5. Was 17G5 in 2nd edition. Figure below is labeled 17G5.

481
482
483
18.G6. Was 17G6 in 2nd edition.

a. k m,a p 1.5 min1 , L 25.0 cm

484
20.2 ml min
v sup er 6.366 cm min
2.0 m 2 4

6.366 cm min
Eq. (18-66) 25.0 cm < 4.5 19.1
1.5 min -1
Satisfied, but close. Thus some bypasses but most undergoes equilibration.

18.G7. Was 17G7 in 2nd edition. Figure is labeled 17.G7.

485
18H1. New problem in 3rd edition. Spreadsheets with numbers and formulas shown.

486
487
18.H.2. New problem in 3rd edition. The spreadsheets are shown on the next pages. They are based on
the previous, but includes both a step up and a step down. Because of the quirk in Excel not
allowing negative arguments, it was set up with multiple solution paths. The correct solution
occurs when there are numbers.

Time, min 15 20 22.5 25 25.5726 27.5 30 33.575 35 37.5 40

C 0 .0134 1.798 24.96 25.0 42.32 48.52 24.97 11.13 1.114 .040

488
489
490

S-ar putea să vă placă și